Sunteți pe pagina 1din 302

1)

0/1
You are seeing a 64-year old female who is presenting with painless bright red rectal
bleeding for the last two days. She notes bright red blood coating the stool and on the
toilet paper when she wipes. He vitals are normal and her physical exam shows a soft non
tender abdomen. Stool is brown with red streaks. What is the most likely cause of
bleeding in this patient and the most common overall cause of lower GI bleeding in
adults?
(Select 1)(1pts)

Inflammatory Bowel Disease

Diverticulosis

Neoplasm

Angiodysplasia

You didn't make any selections

The answer is B. Diverticulosis results in 60% of cases of lower GI bleeding followed by


IBD(13%), benign anorectal disease (11%), neoplasia (10%) and angiodysplasia (<5%).
The causes of lower gastrointestinal bleeding (LGIB) may be grouped into several
categories (table 1): From Uptodate
 Anatomic (diverticulosis)
 Vascular (angiodysplasia, ischemic, radiation-induced)
 Inflammatory (inflammatory bowel disease, infectious)
 Neoplastic
In most series, diverticulosis is the most common source of LGIB, accounting for
approximately 15 to 55 percent of cases [9,10]. Angiodysplasia may be the most
frequent cause in patients over the age of 65 years [11,12], though more recent data
suggest that angiodysplasia may be a less common cause of LGIB than once thought.
Hemorrhoids are the most common cause of rectal bleeding in patients under the age of
50 years [13]. However, hemorrhoidal bleeding is usually minor. Similarly, bloody
diarrhea due to inflammatory causes can sometimes be distinguished from other causes of
LGIB because of the clinical setting. In general, anatomic and vascular causes of
bleeding present with painless, large-volume blood loss, whereas inflammatory sources
are associated with diarrhea and abdominal pain [14].
In a review of several large studies that included 1559 patients with acute hematochezia,
the following bleeding sources were identified [10]:
 Diverticulosis – 5 to 42 percent
 Ischemia – 6 to 18 percent
 Anorectal (hemorrhoids, anal fissures, rectal ulcers) – 6 to 16 percent
 Neoplasia (polyps and cancers) – 3 to 11 percent
 Angiodysplasia – 0 to 3 percent
 Postpolypectomy – 0 to 13 percent
 Inflammatory bowel disease – 2 to 4 percent
 Radiation colitis – 1 to 3 percent
 Other colitis (infectious, antibiotic associated, colitis of unclear etiology)
– 3 to 29 percent
 Small bowel/upper GI bleed – 3 to 13 percent
 Other causes – 1 to 9 percent
 Unknown cause – 6 to 23 percent
The vast majority of colonic diverticula are actually false diverticula (pseudodiverticula)
that contain only serosa and mucosa. They occur at weak points in the colonic wall where
the vasa recta penetrate the muscularis to supply the mucosa; as the diverticulum
expands, these vessels are displaced. A 1976 anatomic study of colonic specimens from
patients with diverticular bleeding used angiography to demonstrate that in all cases, the
vasa recta overlying the diverticulum ruptured into the lumen of the diverticulum, not
into the peritoneum.100

It has been estimated that approximately 17% of patients with colonic diverticulosis
experience bleeding, which may range from minor to severe and life-threatening.
Endoscopic treatment of diverticular hemorrhage can be difficult because of the high
bleeding rate and the location of the bleeding point within the diverticulum. In 2000, one
group of investigators reported their experience with endoscopic therapy for severe
hematochezia and diverticulosis in a prospective series of 121 patients.101 In this series,
none of the patients treated endoscopically with epinephrine injections, bipolar
coagulation, or both required surgery and none experienced recurrent bleeding episodes.
A 2001 study from another group, however, reported high rates of recurrent bleeding
episodes in both the early and the late posttreatment periods.102 In the absence of
prospective, randomized trials, it is difficult to draw definitive conclusions about the
utility of endoscopic therapy in treating diverticular hemorrhage.

Fortunately, most diverticular hemorrhages stop spontaneously. In one series, surgery


was unlikely to be necessary if fewer than four units of packed RBCs were transfused in a
24-hour period, whereas 60% of patients receiving more than four units of packed RBCs
in a 24-hour period required surgical intervention.103 Semielective surgical therapy is
usually offered after a second diverticular bleeding episode because once a second such
episode has occurred, the risk that a third will follow exceeds 50%. In a series of 83
conservatively managed cases of diverticular disease, the predicted yearly recurrence
rates were 9% at 1 year, 10% at 2 years, 19% at 3 years, and 25% at 4 years.104

In general, patients who require more than four units of blood in a 24-hour period to
remain hemodynamically stable, who have not stopped bleeding after 72 hours, or who
experience rebleeding within 1 week after an initial episode should undergo surgery.

2014. Scientific American Surgery. Hamilton, Ontario & Philadelphia, PA. Decker
Intellectual Properties Inc. ISSN 2368-2744. STAT!Ref Online Electronic Medical
Library. http://online.statref.com/Document.aspx?fxId=61&docId=848. 10/22/2014
7:03:07 PM CDT (UTC -05:00).

Feedback on your answer


Collapse
2)
0/1
You are seeing a 45-year old obese male with type II diabetes who presents with acute
onset left lower quadrant abdominal pain, vomiting, pain with defecation, and
fever. Physical exam is normal except for his abdominal exam that shows left lower
quadrant tenderness without signs of peritonitis. His labs are remarkable for a WBC of
16,000 mm3. Which of the following choices is most likely to reduce repeat incidences of
this patients condition?
(Select 1)(1pts)

Routine daily stool softners or laxatives

Biannual colonoscopy

High fiber diet

Avoidance of nuts and legumes

You didn't make any selections


The answer is C, High fiber diet. Environmental and lifestyle factors are important risk
factors for diverticular disease. Out of the following options only increasing fiber intake
has been shown to reduce episodes of diverticulitis. The role of fiber in the development
of diverticulosis is unclear. Several early studies suggested that low dietary fiber
predisposes to the development of diverticular disease, but other studies have been
conflicting [3,30-39]. Fiber also does not reduce symptoms in patients with
symptomatic uncomplicated diverticular disease [37,38,40,41]. However, dietary fiber
and a vegetarian diet may reduce the incidence of symptomatic diverticular disease by
decreasing intestinal inflammation and altering the intestinal microbiota [4,31,42].
In a cohort study that included over 47,000 men, after adjustment for age, energy-
adjusted total fat intake, and physical activity, total dietary fiber intake was noted to be
inversely associated with the risk of symptomatic diverticular disease (RR 0.58 highest
quintile versus lowest quintile for fiber intake) [31].
Fat and red meat — In the same cohort study, the risk of diverticular disease was
significantly increased with diets that were low in fiber and were high in total fat or red
meat as compared with diets that were low in both fiber and total fat or red meat (RR
2.35 and 3.32, respectively) [31].
Seeds and nuts — Nut, corn, and popcorn consumption arenot associated with an
increase in risk of diverticulosis, diverticulitis or diverticular bleeding. In a large
observational study (The Health Professionals Follow-up Study) that included 47,228
men between the ages of 40 and 75 years, there was an inverse association between the
amount of nut and popcorn consumption and the risk of diverticulitis (HR nuts 0.8, 95%
CI 0.63-1.01; HR popcorn 0.72, 95% CI 0.56-0.92) [43]. In addition, no association was
found between consumption of corn and diverticulitis or between nut, popcorn, or corn
consumption and diverticular bleeding or uncomplicated diverticulosis.
Physical activity — It is unclear if lack of vigorous exercise is a risk factor for
diverticular disease. However, vigorous physical activity appears to reduce the risk of
diverticulitis and diverticular bleeding. In a prospective study of approximately 48,000
men aged 40 to 75 who were free of known colonic disease at baseline, the risk of
developing symptomatic diverticular disease was inversely related to overall physical
activity (RR 0.63 for highest versus lowest extremes) after adjustment for age and dietary
fat and fiber [44]. Most of the decrease in risk with exercise was associated with
vigorous activity such as jogging and running. Men in the lowest quintile for both dietary
fiber and physical activity had an increased risk of symptomatic diverticular disease as
compared with men in the highest quintile for both (RR 2.56, 95% CI 1.36-4.82).
Obesity — Obesity has been associated with an increase in risk of both diverticulitis
and diverticular bleeding. In a large, prospective cohort study of 47,228 male health
professionals, there were 801 incident cases of diverticulitis and 383 cases of diverticular
bleeding during 18 years of follow-up [45]. The risk of diverticulitis and diverticular
bleeding was significantly higher in those with the highest quintile of waist
circumference as compared with the lowest (RR diverticulitis 1.56, 95% CI 1.18-2.07;
RR diverticular bleeding 1.96, 95% CI 1.30-2.97).
Other — Current smokers appear to be at increased risk for perforated diverticulitis and
a diverticular abscess as compared with nonsmokers (OR 1.89, 95% CI 1.15-3.10) [46].
Caffeine and alcohol are not associated with an increased risk for symptomatic
diverticular disease. Read more
Feedback on your answer
Collapse
3)
0/1
You are seeing a 58-year-old female who is presenting with two days of a low-grade
fever and increasing abdominal cramps that are exacerbated by bowel movements. She
began a course of augmentin 2 days ago that her PCP gave her but has had no relief of
her symptoms and is getting worse. She has not had nausea, vomiting, urinary symptoms,
or bloody stools. She has a past medical history of hypertension.

Her vitals show a temperature is 100.6°F, pulse is 75/min, and blood pressure is 150/80
mm Hg. Her lungs are clear to auscultation. Cardiac examination shows no
abnormalities. Her abdomen is soft, and there is tenderness to palpation of the left lower
quadrant with guarding but no rebound. Bowel sounds are normal. Her stool is brown,
and test for occult blood is negative. Her hemoglobin concentration is 14.5 g/dL,
leukocyte count is 15,800/mm3, and platelet count is 280,000/mm3; serum studies and
urinalysis show no abnormalities. Which of the following is the most appropriate next
step in diagnosis?
(Select 1)(1pts)

Examination of the stool for ova and parasites

Test of the stool for Clostridium difficile toxin

Xray Abdomen Series

CT scan of the abdomen/pelvis with IV and oral contrast

You didn't make any selections


The answer is D, CT scan of the abdomen with IV and oral contrast. This patient is being
treated for diverticulitis and is not getting better. She needs a CT scan to evaluate for a
possible abscess. Read more

Feedback on your answer


Collapse
4)
0/1
You are seeing a 49 -ear old obese male who presents with two days of worsening left
sided abdominal pain. He denies fever or chills but does note painful bowel movements
that have streaks of blood. On exam you note moderate left lower quadrant tenderness to
palpation without rebound or guarding. His labs show a white blood cell count of
14,000mm3 and his CT scan of his abdomen/pelvis shows acute uncomplicated sigmoid
diverticulitis. Nonoperative treatment with oral antibiotics will resolve acute
uncomplicated diverticulitis in what percent of patients?
(Select 1)(1pts)

<25%

50%

80%

>95%

You didn't make any selections


The best answer is B, 80%. Although most patients with acute diverticulitis can be treated
medically, approximately 15-20% will require surgery. Uncomplicated diverticulitis may
be managed as an outpatient (dietary modification and oral antibiotics) for those without
appreciable fever, excessive vomiting, or marked peritonitis, as long as there is the
opportunity for follow-up. The patient should be able to take liquids and antibiotics by
mouth. Hospitalization for treatment (dietary modification and intravenous antibiotics) is
usually best if the above conditions are not met, or if the patient fails to improve with
outpatient therapy. Antibiotics should be selected to treat the most common bacteria
found in the colon: gram-negative rods and anaerobic bacteria. Single and multiple
antibiotic regimens are equally effective, as long as both groups of organisms are
covered. Nonoperative treatment will resolve acute diverticulitis in 80
percent of patients, but approximately one-third will have a recurrent attack often
within one year. approximately 20% of patients with diverticulitis require surgical
treatment.30 Most surgical procedures are reserved for patients who experience recurrent
episodes of acute diverticulitis that necessitate treatment (inpatient or out-patient) or who
have complicated diverticulitis. The most common indication for elective resection is
recurrent attacks—that is, several episodes of acute diverticulitis documented by studies
such as CT. Rerecurrences may be more common than recurrences. In 2000, a task force
of the American Society of Colon and Rectal Surgeons recommended sigmoid resection
after two attacks of diverticulitis. A subsequent cost analysis using a Markov model
suggested that cost savings could be achieved if resection was done after three
attacks.33 There is a growing tendency to question arbitrary guidelines for surgical
management of recurrent attacks, with the exception of certain groups, such as
immunocompromised patients. Current practice guidelines state that the recommendation
to perform elective sigmoid resection after recovery from uncomplicated acute
diverticulitis should be made on a case-by-case basis. The decision-making process
should be influenced by the age and medical condition of the patient, the frequency and
severity of attacks, and the presence of symptoms after the acute attack. Elective
resection is generally recommended after an episode of complicated
diverticulitis.36 Efforts are made to time surgical treatment so that it takes place during a
quiescent period 8 to 10 weeks after the last attack. Barium enema or colonoscopy may
be employed to evaluate the diverticular disease and rule out carcinoma. The bowel can
then be prepared mechanically and with antibiotics (e.g., oral neomycin and
metronidazole on the day before operation).
2014. Scientific American Surgery. Hamilton, Ontario & Philadelphia, PA. Decker
Intellectual Properties Inc. ISSN 2368-2744. STAT!Ref Online Electronic Medical
Library. http://online.statref.com/Document.aspx?fxId=61&docId=1368. 10/20/2014
12:41:34 PM CDT (UTC -05:00).

Feedback on your answer


Collapse
5)
0/1
You are seeing a 64-year old white male who is presenting with left lower abdominal
pain since last night. His WBC is 14,000mm3 and his CT scan of the abdomen and pelvis
of the abdomen/pelvis shows acute uncomplicated sigmoid diverticulitis. What bacteria is
usually the target in the treatment of acute diverticulitis?
(Select 1)(1pts)
Gram positive cocci

Gram positive rods

Gram negative cocci

Gram negative rods

You didn't make any selections

The answer is D, gram negative Rods. The most common choices would be either
augmentin as single coverage or cipro plus flagyl. Conservative treatment of acute
uncomplicated diverticulitis is successful in 70 to 100 percent of patients. Uncomplicated
diverticulitis may be managed as an outpatient (dietary modification and oral antibiotics)
for those without appreciable fever, excessive vomiting, or marked peritonitis, as long as
there is the opportunity for follow-up. The patient should be able to take liquids and
antibiotics by mouth. Hospitalization for treatment (dietary modification and intravenous
antibiotics) is usually best if the above conditions are not met, or if the patient fails to
improve with outpatient therapy. Antibiotics should be selected to treat the most common
bacteria found in the colon: gram-negative rods and anaerobic bacteria. 13 Single and
multiple antibiotic regimens are equally effective, as long as both groups of organisms
are covered. Nonoperative treatment will resolve acute diverticulitis in 85 percent of
patients, but approximately one-third will have a recurrent attack often within one
year. Read More

Feedback on your answer


Collapse
Unit 5
6)
0/1
You are seeing a 78-year old female who is post op day 1 following a colon resection
following a bout of recurrent diverticulitis with perforation. She has a past medical
history of cirrhosis from hepatitis C. On exam you note chronic lower extremity swelling
and 2+ pitting edema. Her albumin level was 2.7 g/dL. You are wanting to give her a liter
of normal saline (NS) suspecting dehydration as she has not eaten anything in 48 hours
and her heart rate has increased to around 105 beats/min. Out of the 1 Liter given of NS,
what volume administered stays in the intravascular space?
(Select 1)(1pts)

50-100 cc

250-330cc
500-650 cc

750-800cc

You didn't make any selections


The answer is B, 250-330cc. Crystalloid solutions are water-based solutions to which
electrolytes (and, sometimes, organic molecules such as dextrose) have been added. The
crystalloid solutions used for resuscitation are generally isotonic to blood plasma and
include such common examples as 0.9% sodium chloride, lactated Ringer solution, and
Plasma-Lyte (Baxter Healthcare, Round Lake, IL). The choice to use one solution over
another is usually inconsequential, but there are a few notable exceptions. For example,
in the setting of renal dysfunction, there is a risk of hyperkalemia when potassium-
containing solutions such as lactated Ringer solution and Plasma-Lyte are used. As
another example, the administration of large volumes of 0.9% sodium chloride, which
has a pH of 5.0 and a chloride content of 154 mmol/L, can lead to hyperchloremic
metabolic acidosis. Regardless of which crystalloid solution is used, large volumes may
have to be infused to achieve a significant increase in the circulating intravascular
volume. Only one third to one quarter (250 to 330 mL/L) of the fluid administered stays
in the intravascular space; the rest migrates by osmosis into the interstitial tissues,
producing edema and potential impairment of tissue perfusion (the latter is a theoretical
consequence whose existence has not yet been directly demonstrated). Read more
Feedback on your answer
Collapse
7)
0/1
Same-day surgery is appropriate for patients who (1) have few or no comorbid medical
conditions and (2) are undergoing a procedure that involves short-duration anesthesia or
local anesthesia plus sedation and that carries a low likelihood of urgent complications.
Operations commonly performed on a same-day basis include which of the following?
(Select 1)(1pts)

inguinal or umbilical hernia repair

simple laparoscopic cholecystectomy

breast biopsy

small subcutaneous procedures

All of the above

None of the above

You didn't make any selections


The answer is all of the above. Same-day surgery is appropriate for patients who (1) have
few or no comorbid medical conditions and (2) are undergoing a procedure that involves
short-duration anesthesia or local anesthesia plus sedation and that carries a low
likelihood of urgent complications. Operations commonly performed on a same-day basis
include inguinal or umbilical hernia repair, simple laparoscopic cholecystectomy, breast
biopsy, and small subcutaneous procedures. The growth in the performance of minor and
same-day procedures has led to the development of various types of short-stay units or
wards. The level of care provided by a short-stay ward is generally equivalent to that
provided by a regular nursing ward; however, the anticipated duration of care is
substantially shorter, typically ranging from several hours to a maximum of 48 hours.
Short-stay wards also undergo some modifications to facilitate the use of streamlined
teaching protocols designed to prepare patients for home care. Many hospitals now have
short-stay units, as do some independent surgery centers. Read more
Feedback on your answer
Collapse
8)
0/1
Which of the following is NOT true regarding post operative placement of foley
catheters?
(1pts)

As many as 40% of all hospital infections are UTIs

Only 10% of hospital acquired UTIs are associated with urinary catheters

After many procedures, patients initially find it extremely difficult or


impossible to mobilize for urination, and a urinary catheter may be quite
helpful during this time

In the setting of bladder or genitourinary surgery, they are often


employed to decompress the system so that it will heal more readily

You didn't make any selections


Answer B is false. Urinary catheters can serve a large variety of purposes. In the setting
of bladder or genitourinary surgery, they are often employed to decompress the system so
that it will heal more readily. After general surgical procedures—and many other surgical
procedures as well—they are used to provide accurate measurements of volume output
and thus, indirectly, to give some indication of the patient's overall volume and
resuscitation status. Furthermore, after many procedures, patients initially find it
extremely difficult or impossible to mobilize for urination, and a urinary catheter may be
quite helpful during this time.
Their utility and importance notwithstanding, urinary catheters are
associated with the development of nosocomial urinary tract infections
(UTIs). As many as 40% of all hospital infections are UTIs, and
80 to 90% of these UTIs are associated with urinary
catheters.Accordingly, when catheterization is no longer deemed
necessary, prompt removal is indicated. As a rule, orders specifically
pertaining to urinary catheter care are few, typically including gravity
drainage, flushing to maintain patency (if warranted), and removal
when appropriate. At times, irrigation is employed after urologic
procedures or for the management of certain infectious agents. Read
more
Feedback on your answer
Collapse
9)
0/1
Initial wound management after an operative procedure generally entails placement of a
sterile dressing to cover the incision. The traditional recommendation has been to keep
this dressing in place and dry for what period of time?

(Select 1)(1pts)

12 hours

24 hours

48 hours

72 hours

All of the above

None of the above

You didn't make any selections

The answer is C, 48 hours. Initial wound management after an operative procedure


generally entails placement of a sterile dressing to cover the incision. The traditional
recommendation has been to keep this dressing in place and dry for the first 48 hours
after operation; because epithelialization is known to take place within approximately this
period, the assumption is that this measure will reduce the risk of wound infection.
Although most surgeons still follow this practice, especially in general surgical cases,
supporting data from randomized clinical studies are lacking. In addition, several small
studies that evaluated early showering with closed surgical incisions found no increases
in the rate of infection or dehiscence.8,9 It should be kept in mind, however, that these
small studies looked primarily at soft tissue and other minor skin incisions that did not
involve fascia. Thus, even though the traditional approach to initial dressing management
is not strongly supported, the data currently available are insufficient to indicate that it
should be changed.
Wet-to-dry dressings are used in a variety of settings. In a surgical context, they are most
often applied to a wound that cannot be closed primarily as a consequence of
contamination or inability to approximate the skin edges. Wet-to-dry dressings provide a
moist environment that promotes granulation and wound closure by secondary intention.
Moreover, their removal and replacement cause debridement of excess exudate or
unhealthy superficial tissue. Postoperative orders should specify the frequency of
dressing changes and the solution used to provide dampness. For most clean open
incisions, twice-daily dressing changes using normal saline solution represent the most
common approach. If there is excess wound exudate to be debrided, dressing changes
may be performed more frequently. If there is particular concern about wound
contamination or superficial colonization of organisms, substitution of dilute Dakin
solution for normal saline may be considered. Read more

Feedback on your answer


Collapse
10)
0/1
Which of the following statements regarding peri and post-operative MI is true?
(Select 1)(1pts)

plaque rupture is the most common cause of MI

The majority of cardiac ischemic events occur after the 4th day

95% of patients who experience this complication will present with classic
symptoms

Perioperative beta blockade for patients at risk for MI is now routine as


this practice yields significant risk reductions in terms of both cardiac
morbidity and mortality

You didn't make any selections


Answer D is the only true statement. Approximately one third of patients who undergo
noncardiac surgery in the United States have some degree of coronary artery disease and
thus are at increased risk for perioperative MI. The incidence of coronary artery disease is
even higher in certain subpopulations, such as patients who undergo major vascular
procedures.40,41 In the perioperative setting, however, the pathophysiology of coronary
ischemia is different from that in nonsurgical settings, where plaque rupture is the most
common cause of MI. Approximately 50% of all MIs occurring in surgical patients are
caused by increased myocardial oxygen demand in the face of inadequate supply
resulting from factors such as fluid shifts, physiologic stress, hypotension, and the effects
of anesthesia. The majority of cardiac ischemic events occur in the first 4 days of the
postoperative period.
Perioperative beta blockade for patients at risk for MI is now routine. Multiple trials and
meta-analyses have demonstrated that this practice yields significant risk reductions in
terms of both cardiac morbidity and mortality42,43 and that these risk reductions are
achieved regardless of the type of surgery being performed. Al though there has been
some variation in the protocols used by these trials and the results reported, there is
general agreement that beta blockade should be initiated preoperatively, delivered at the
time of surgery, and continued postoperatively for up to 1 week.
Diagnosis of postoperative MI is complicated by the fact that as many as 95% of patients
who experience this complication may not present with classic symptoms (e.g., chest
pain). Identification of MI may be further hindered by the ECG changes brought on by
the stress of the perioperative period (including dysrhythmias). Ultimately, the most
useful signal of an ischemic cardiac event in the postoperative period is a rise in the
levels of cardiac enzymes, particularly troponin I. Accordingly, cardiac enzyme activity
should be assessed whenever there is a high index of suspicion for MI or a patient is
considered to be at significant perioperative risk for MI.
Treatment of postoperative MI focuses on correcting any factors contributing to or
exacerbating the situation that led to the event (e.g., hypovolemia or hypotension).
Typically, although antiplatelet agents (e.g., aspirin) are sometimes given, thrombolytic
therapy is avoided because of concerns about postoperative bleeding. Acute percutaneous
coronary intervention is also associated with an increased risk of bleeding but has
nonetheless been used successfully in the peri-operative setting and is recommended by
some physicians.44 Beta blockade is often advocated as a means of treating postoperative
MI, although it is probably more effective when used both preoperatively and
perioperatively as a means of preventing MI.Read more
Feedback on your answer
Collapse
Unit 3
11)
0/1
Treatment for all patients with acute nonperforated appendicitis must include:

(Select 1)(1pts)

Witholding analgesia

Immediate antibiotics

Blood cultures

Surgery within 12 hours

All of the above

You didn't make any selections


The answer is B, Immediate antibiotics. The treatment of acute appendicitis in the ED
consists of 3 major aspects: 1) supportive care, 2) antibiotic therapy, and 3) definitive
treatment as decided upon by a surgical consultation. Supportive care is to be tailored to
the patient’s symptoms. Most often, patients with appendicitis should receive IV fluids
(lactated Ringer or normal saline), should receive symptomatic treatment (eg, antiemetics
and analgesia), and should remain NPO. Choice of analgesia will depend on the personal
preference of the provider; however, IV opiates are often considered first-line treatment
in possible appendicitis cases.120 Multiple prospective randomized trials and a Cochrane
Database review on analgesia in patients with acute abdominal pain have all concluded
that opiates improve patients’ cooperation, relieve their pain, and do not mask the
physical signs of appendicitis. Unless contraindicated, it is unacceptable to withhold
analgesia from patients presenting with acute abdominal pain.121-124 In the rare case of
acute appendicitis causing septic shock, treatment is guided by the Surviving Sepsis
Campaign International Guidelines of Severe Sepsis and Septic Shock.
Administer antibiotics promptly upon making the diagnosis of appendicitis.126 A
Cochrane database review of 45 studies involving 9576 patients found that antibiotic
administration in appendicitis reduces the incidence of both postoperative wound
infection and intra-abdominal abscess formation when compared to placebo.127
Interestingly, these studies demonstrated no difference in the postoperative histologic
findings of appendicitis in the antibiotic versus placebo groups.
Recommendations on definitive management for acute appendicitis depend on the
presence or absence of complications. Treatment for nonperforated appendicitis with a
well-circumscribed abscess is nonoperative and consists of IV antibiotics and
percutaneous drainage. The management recommendations for nonperforated
appendicitis without abscess or perforated appendicitis is urgent operative intervention.
Conservative management for appendicitis without abscess has become a controversial
topic recently. Conservative therapy consists of hospitalization for IV antibiotics rather
than emergent surgical intervention. Although Dr. Coldrey published a protocol in 1956
involving exclusive antibiotic treatment of appendicitis lasting more than 24 hours with
great personal success, only in the last 20 years has the idea of conservative treatment
been highlighted by multiple prospective trials and meta-analyses. The benefit of
conservative treatment is rooted in the fact that there is significant perioperative
morbidity associated with an appendectomy. 28,135 On average, the rate of
complications from appendectomies is approximately 4.6% and includes small bowel
obstruction, wound infection, abscess formation, and adhesions. Read more
Feedback on your answer
Collapse
12)
0/1
Which sign on physical examination is MOST predictive in diagnosing appendicitis?

(Select 1)(1pts)
Presence of a Rovsing sign

RLQ tenderness and rigidity

Presence of a psoas sign

Temperature above 38.3°C (101°F)

Presence of a obturator sign

You didn't make any selections

The answer is B, RLQ tenderness and rigidity. Signs on physical examination that are
most predictive of appendicitis are RLQ tenderness and rigidity. Signs that demonstrated
a small effect to discriminate appendicitis were other peritoneal signs (rebound
tenderness, guarding, percussion tenderness), temperature above 38.3°C (101°F), and the
presence of a psoas sign.11,44,45 Signs that do not help in diagnosing appendicitis
include tenderness on rectal examination, increased local skin temperature of RLQ, and
Rovsing sign (ie, pain in the RLQ upon palpation of the LLQ).Read more

Feedback on your answer


Collapse
13)
0/1
Neoplastic lesions of the appendix are found in as many as 5% of specimens obtained
with routine appendectomy for acute appendicitis. Most are benign. Preoperative
detection of such conditions is rare, and intraoperative diagnosis is made in fewer than
50% of cases. Which of the following statement regarding neoplastic lesions of the
appendix is true?
(Select 1)(1pts)

Metastasis to the appendix is common

Adenocarcinoma is the most common type represting >80%

The incidence of malignancy in the appendix is 1.35%.

If the tumor is less than 8 cm in diameter, is located within the body or


the tip of the appendix, and has not metastasized, appendectomy is the
treatment of choice.

All of the above

None of the above


You didn't make any selections
Answer C is the only true statement. Neoplastic lesions of the appendix are found in as
many as 5% of specimens obtained with routine appendectomy for acute appendicitis.18-
21 Most are benign. Preoperative detection of such conditions is rare, and intraoperative
diagnosis is made in fewer than 50% of cases. Appendectomy alone may be curative for
appendiceal mucocele, localized pseudomyxoma peritonei, most appendiceal carcinoids,
and other benign tumors. Definitive management of an appendiceal mass unexpectedly
encountered during exploration for clinically suspected acute appendicitis depends on
whether the tumor is carcinoid, its size and location, the presence or absence of metastatic
disease, and histologic and immunohistochemical findings.
Benign neoplasms of the appendix include mucosal hyper-plasia or metaplasia,
leiomyomas, neuromas, lipomas, angiomas, and other rare lesions. Appendiceal
adenomas tend to be diffuse and to have a predominant villous character. Mucus-
producing cystadenomas predispose to appendiceal mucocele, sometimes accompanied
by localized pseudomyxoma peritonei. These lesions are rarely symptomatic and are
often encountered incidentally during operation; however, they may also be clinically
manifested as acute appendicitis, torsion, intussusception, ureteral obstruction, or another
acute condition. If the base of the appendix is free of disease, appendectomy alone is
sufficient treatment.
Malignant tumors of the appendix primarily consist of carcinoids and adenocarcinomas;
all together, they account for 0.5% of all gastrointestinal malignancies. The incidence of
malignancy in the appendix is 1.35%. Metastasis to the appendix is rare. Carcinoids are
substantially more common than adenocarcinomas in the appendix: as many as 80% of
all appendiceal masses are carcinoid tumors. Overall, carcinoid tumors are found in 0.5%
of all appendiceal specimens, and appendiceal carcinoid tumors account for 18.9% of all
carcinoid lesions.23 These tumors are predominantly of neural cellular origin and have a
better prognosis than all other intestinal carcinoid tumors, which typically are of mucosal
cellular origin. If the tumor is less than 2 cm in diameter, is located within the body or the
tip of the appendix, and has not metastasized, appendectomy is the treatment of choice. If
the lesion is at the base of the appendix, is larger than 2 cm in diameter, or has
metastasized, right hemicolectomy is indicated. In addition, secondary right
hemicolectomy is indicated if the tumor is invasive, if mucin production is noted, or if the
tumor is found to be of mucosal cellular origin at the final pathologic examination.24,25
Patients with metastatic appendiceal carcinoid tumors appear to have a far better
prognosis than those with other types of metastatic cancers.24 Therefore, hepatic
debulking for symptomatic control is indicated and justified in cases of liver metastasis.
Figure 8. Laparoscopic appendectomy. The mesoappendix having been divided (a), the
base of the appendix is cleared circumferentially and divided with a gastrointestinal
anastomosis stapler (b). Read more
Feedback on your answer
Collapse
14)
0/1
You are seeing a 32-year-old white female who is currently 16 weeks'
pregnant and is presenting with right lower quadrant pain that started
abruptly last night along with nausea, vomiting, and a low grade fever.
Which one of the following imaging studies would be most appropriate
for initial evaluation of this patient?
(Select 1)(1pts)

CT of the abdomen

MRI of the abdomen

Ultrasonography of the abdomen and Right lower quadrant

A small bowel series

You didn't make any selections


The answer is C, Ultrasonography of the abdomen and Right lower quadrant. CT has
demonstrated superiority over transabdominal ultrasonography for identifying
appendicitis, associated abscess, and alternative diagnoses. However, ultrasonography is
indicated for the evaluation of women who are pregnant and women in whom there is a
high degree of suspicion for gynecologic disease. Ref: Paulson EK, Kalady MF, Pappas
TN: Suspected appendicitis. N Engl J Med 2003;348(3):236-241.
Feedback on your answer
Collapse
15)
0/1
You are seeing a 31-year old male who presents with 24 hours of progressively
worsening lower abdominal pain, vomiting and chills. He denies past medical history and
denies prior surgery.

On exam his heart rate is 95 bpm, BP is 129/80, oral temp 100.2 F and O2 sat is 100% on
room air. Upon palpation he has right lower quadrant pain when you palpated his left
lower quadrant. His labs show a WBC of 9,700mm3 with no bands.

True or False: A normal white blood cell count effectively rules out appendicitis in this
patient.
(1pts)
You didn't answer this question
False: Appropriate diagnostic testing varies based on the clinical situation. A complete
blood count is appropriate if infection or blood loss is suspected. One study of patients 15
to 83 years of age with suspected appendicitis found that a white blood cell count greater
than 10,000 per mm3 (10 × 10 9 per L) was 77 percent sensitive and 63 percent specific
for the diagnosis (LR+ = 2.1, LR– = 0.37). Thus, nearly one in four patients with
appendicitis does not have an elevated white blood cell count. Read More
Feedback on your answer
Collapse
Unit 1
16)
0/1
Nasogastric (NG) tubes are commonly placed after gastrointestinal (GI) operations, most
frequently for drainage of gastric secretions when an ileus is anticipated or offloading of
the upper GI tract when a fresh anastomosis is located close by. Although NG tubes have
often been placed routinely after abdominal surgery, the current literature cites a number
of reasons why routine use is inadvisable and selective use is preferable. Which of the
following has been found to be true when NG tubes are not routinely placed or when they
are removed within 24 hours after operation?

(Select 1)( 1pts extra credit)

significantly earlier return of bowel function

a trend toward fewer pulmonary complications

enhanced patient comfort

decreased nausea

all of the above are true

none of the above are true

You didn't make any selections


The answer is E, all of the above. Nasogastric (NG) tubes are commonly placed after
gastrointestinal (GI) operations, most frequently for drainage of gastric secretions when
an ileus is anticipated or offloading of the upper GI tract when a fresh anastomosis is
located close by. Although NG tubes have often been placed routinely after abdominal
surgery, the current literature cites a number of reasons why routine use is inadvisable
and selective use is preferable. For example, significantly earlier return of bowel
function, a trend toward fewer pulmonary complications, and enhanced patient comfort
and decreased nausea are reported when NG tubes are not routinely placed or when they
are removed within 24 hours after operation.2
When postoperative placement of an NG tube is considered appropriate, an order from a
physician is required, along with direction regarding the method of drainage. Sometimes
NG tubes are placed to low continuous suction; more often, however, they are placed to
low intermittent suction to eliminate the chance of continuous suction against a visceral
wall and to promote generalized drainage. If large volumes of secretions are not
expected, continuous gravity may be used instead of suction. A key concern with NG
tubes is maintenance of the patency of both the main port and the sump port. Should
either port become blocked, the tube will be rendered ineffective. This concern should be
discussed with the nursing staff. At times, it may be necessary to issue specific orders to
ensure that this concern is appropriately addressed. As a rule, surgical nurses are well
acquainted with tube maintenance; however, if thick secretions are expected, orders for
routine flushing may be indicated. When the tube is no longer indicated, its removal may
be ordered.Read more

Feedback on your answer


Collapse
17)
0/1
You are seeing a 59-year-old male for preoperative evaluation before an elective right hip
replacement. in the morning. His medical history includes hypertension treated with
norvasc, lisinopril, and hydrochlorothiazide, prior cholecystectomy and dental
extractions. He usually drinks 1 to 2 glasses of wine with dinner, does not smoke, and
does not use illicit drugs. Results of laboratory studies performed last month, including
serum electrolyte levels, creatinine level, and lipid profile, were normal.
On physical examination, pulse rate is 72/min, and blood pressure is 141/85 mm Hg. The
remainder of the examination is normal. Which of the following blood pressure
medications is recommended to be continued on the morning of the surgery?

(Select 1)( 1pts extra credit)

Norvasc

Lisinopril

Hydrochlorothiazide

All of the medications should be stopped

All of the medications can be continued without risk

You didn't make any selections


The answer is A, Norvasc can be continued on the morning of the surgery. It is
recommended to continue β-blockade, clonidine, or calcium-channel blockade (Norvasc),
including on the day of surgery. Discontinue angiotensin-converting enzyme inhibitors,
angiotensin-receptor blockers, and diuretics on the morning of surgery, if possible,
because of the potential risk for perioperative hypotension or hypokalemia. Read more
Feedback on your answer
Collapse
18)
0/1
Abdominal pain is traditionally divided into 3 categories. What are the 2 basic causes of
abdominal pain of the 3 categories?

( 1pts extra credit)


Visceral and Autonomic

Visceral and Parietal

Parietal and Somatic

Referred and Somatic

You didn't make any selections


Abdominal pain is traditionally divided into 3 categories: visceral, parietal and
referred. In general visceral (autonomic) and parietal (somatic) are the 2 basic causes of
abdominal pain. Referred pain can be considered separately as a cortical misperception of
either visceral or parietal afferent stumuli. The two are often not discrete as visceral pain
often blends with parietal pain as a pathological process evolves. Tintinalli's Emergency
Medicine: A Comprehensive Study Guide, 7th edition, Tintinalli et al. Read more
on Acute Abdominal Pain Power Point PDF
Feedback on your answer
Collapse
19)
0/1
You are seeing a 54-year-old male in the hospital following a laparotomy three days ago
for an intestinal obstruction secondary to adhesions. He has a past medical history of type
II diabetes, hypothyroidism, and hypertension. This morning he is complaining of pain
around his surgical wound. His vitals show a fever of 101°F, heart rate of 115 beats/min,
and his most recent blood glucose level was 300 mg/dL. On examination his
wound shows abundant cloudy gray discharge, the edges are dusky and friable, and there
is minimal crepitus. Sensation also appears to be decreased at the edges of the wound.
Which of the following is the best management for this patient?

( 1pts extra credit)

Start Intravenous ceftriaxone and vancomycin

Negative pressure wound therapy

Urgent surgical exploration

Wound culture first and await cultures before starting antibiotics

You didn't make any selections


The answer is C, Urgent surgical exploration. The patient presents with signs and
symptoms suggestive of necrotizing surgical infection. Symptoms include intense pain in
the wound, fever/hypotension/tachycardia, decreased sensitivity and edges of the wound,
cloudy gray discharge, tense edema outside involved skin, subcutaneous gas with
crepitus.
Necrotizing infections more commonly occur in diabetics and are usually caused by mix
gram-positive and gram-negative the room. These infections are considered emergencies
if they involve the fascial plane and developing the fasciitis. The most important step in
the management of this condition is really surgical exploration to assess the extent of the
process and debride the necrotizing tissues. Read more
Feedback on your answer
Collapse
20)
0/1
According to multiple sources, what is the most common diagnosis of abdominal pain in
the adult?

( 1pts extra credit)

Acute appendicitis

Acute Cholecystitis

Acute Pancreatitis

Non Diagnostic Abdominal Pain

You didn't make any selections

The answer is D, Non Diagnostic Abdominal Pain. This is in contrast to the most
common surgical cause of abdominal pain in the adult, which is appendicitis. The value
of detailed epidemiologic knowledge notwithstanding, it is worthwhile to keep in mind
the truism that common things are common. Regarding which things are common, the
most extensive information currently available comes from the ongoing survey begun in
1977 by the Research Committee of the OMGE. As of the last progress report on this
survey, which was published in 1988, more than 200 physicians at 26 centers in 17
countries had accumulated data on 10,320 patients with acute abdominal pain
[see Table 3].23 The most common diagnosis in these patients was nonspecific
abdominal pain (NSAP)—that is, the retrospective diagnosis of exclusion in which no
cause for the pain can be identified.24,25 NSAP accounted for 34% of all patients seen;
the four most common diagnoses accounted for more than 75%. The most common
surgical diagnosis in the OMGE survey was acute appendicitis, followed by acute
cholecystitis, small bowel obstruction, and gynecologic disorders. Relatively few patients
had perforated peptic ulcer, a finding that confirms the current downward trend in the
incidence of this condition. Cancer was found to be a significant cause of acute
abdominal pain. There was little variation in the geographic distribution of surgical
causes of acute abdominal pain (i.e., conditions necessitating operation) among
developed countries. In patients who required surgery, the most common causes were
acute appendicitis (42.6%), acute cholecystitis (14.7%), small bowel obstruction (6.2%),
perforated peptic ulcer (3.7%), and acute pancreatitis (4.5%).23 The OMGE survey's
finding that NSAP was the most common diagnosis in patients with acute abdominal pain
has been confirmed by several srudies12,13,25; the finding that acute appendicitis,
cholecystitis, and intestinal obstruction were the three most common diagnoses in
patients with acute abdominal pain who require operation has also been amply confirmed
[see Table 3].1,12,13

Table 3. Frequency of Specific Diagnoses in Patients with Acute Abdominal Pain

Frequency in Individual Studies (% of Patients)

Wilso Brew
n et er et
OMGE23( Irvin13( de
Diagno al (N
82 al12(N Hawthorn83
N= N= Dombal1(
sis = = (N = 496)
10,320) 1,190) N = 552)
1,196 1,000
) )

Nonspecifi
c
34.0 45.6 34.9 41.3 50.5 36.0
abdominal
pain

Acute
appendicit 28.1 15.6 16.8 4.3 26.3 14.9
is

Acute
cholecysti 9.7 5.8 5.1 2.5 7.6 5.9
tis

Small
bowel
4.1 2.6 14.8 2.5 3.6 8.6
obstructio
n

Acute
gynecolog 4.0 4.0 1.1 8.5 — —
ic disease

Acute
pancreatiti 2.9 1.3 2.4 — 2.9 2.1
s
Urologic
2.9 4.7 5.9 11.4 — 12.8
disorders

Perforated
pepticulce 2.5 2.3 2.5 2.0 3.1 —
r

Cancer 1.5 — 3.0 — — —

Diverticula
1.5 1.1 3.9 — 2.0 3.0
r disease

Dyspepsia 1.4 7.6 1.4 1.4 — —

Gastroenter
— — 0.3 6.9 — 5.1
itis

Inflammato
ry bowel — — 0.8 — — 2.1
disease

Mesenteric
— 3.6 — — — 1.5
adenitis

Gastritis — 2.1 — 1.4 — —

Constipatio
— 2.4 — 2.3 — —
n

Amebic
hepatic 1.2 — 1.9 — — —
abscess

Miscellane
6.3 1.3 5.2 15.5 4.0 8.0
ous

OMGE = World Organization of Gastroenterology.


2014. Scientific American Surgery. Hamilton, Ontario & Philadelphia, PA. Decker
Intellectual Properties Inc. ISSN 2368-2744. STAT!Ref Online Electronic Medical
Library. http://online.statref.com/Document.aspx?fxId=61&docId=803. 10/16/2014
1:15:18 PM CDT (UTC -05:00).

Feedback on your answer


Collapse
Unit 2
21)
0/1
What is the risk for serious medical complications from surgery in healthy patients?
(Select 1)(1pts)

<0.1%

<0.5-1%

2-3%

5-6%

You didn't make any selections


The answer is A, <0.1%. Healthy patients have significant physiologic reserves and
tolerate major and minor surgical procedures well. The risk for serious medical
complications from surgery is less than 0.1% overall in healthy patients. When evaluating
healthy patients before surgery, physicians should use a focused history and physical
examination to predict the risk for serious medical complications. Researchers collected
information at 16 and 72 hours and 30 days for 38 598 patients undergoing 45 090
consecutive procedures and found that only 33 (0.08%) had major morbidity or mortality.
Of 4 deaths that occurred within 30 days of the procedure, 2 were due to myocardial
infarction and 2 were due to automobile accidents unlikely to be related to surgery. More
than one third of major morbidity occurred 48 hours or more after surgery. Read more
Feedback on your answer
Collapse
22)
0/1
Where is the surgical amphitheaters known as "The Ether Dome" located?
(Select 1)(1pts)

Beaumont Hospital in Dublin, Ireland

Guy's Hospital in London, England

Massachusetts General Hospital

Padua Hospital in Padua, Italy

You didn't make any selections


The answer is C, Massachusetts General Hospital. The configuration and design of the
modern OR were derived from the anatomic amphitheater of late-Renaissance Italy.
Following the publication of Andreas Vesalius's De Fabrica Humani Corporis in 1543,
the study of human anatomy and dissection emerged as central elements in medical and
particularly in surgical education. This innovation reflected an estrangement from the
previous galenic curriculum, the beginnings of the rejection of scholasticism as the
dominating heuristic in medical investigation, and a sentinel movement toward
empiricism and what we now recognize as the beginnings of modern science. The first
documented dissecting amphitheater, constructed after the classical Greek style of tiered
seats or stands surrounding the stage, was built in Padua in the mid-16th century.1 Others
in Europe soon followed. Although early surgeons generally operated in the patient's
home or on the battlefield, and in hospitals only infrequently, by the first decade of the
18th century, the anatomic amphitheater could serve as an operating theater. In 1707,
Pierre Dionis, a Parisian anatomist and surgeon, included an etching of his surgical
amphitheater in his textbook of surgery. In England, at about the same time, the new
hospitals built to replace the Catholic institutions destroyed by Henry VIII and serve the
needs of the growing urban areas included operating theaters. Guy's Hospital built its
theater on the top floor so that it could be lit by skylights.2-4 A magnificently preserved
example of premodern design can be seen in Dublin, at the Royal College of Surgeons, in
the anatomic building and dissecting rooms that remain in use today. The effective
deployment of natural lighting is remarkable and instructive. American examples of 19th
century surgical amphitheaters also survive. The Ether Dome at the Massachusetts
General Hospital, which derived its name from the effective demonstration of ether as an
anesthetic, may be the best known, but there are others of equal historical
significance. Read more
Feedback on your answer
Collapse
23)
0/1
Which of the following statements regarding the pre-operative care of patients is false?
(Select 1)(1pts)

When a preoperative antibiotic is indicated, a single dose of therapeutic


strength, administered shortly before incision, usually suffices

Cooling patients during colorectal surgery have been shown to reduce


infection rates

Supplemental perioperative oxygen (i.e., an FIO2 of 80% instead of 30%)


significantly overcomes the decrease in phagocytosis and bacterial killing
usually associated with anesthesia and surgery (and significantly reduces
postoperative nausea and vomiting)

Intraoperative transfusion of packed red blood cells was an independent


risk factor for perioperative infection

You didn't make any selections


Answer B is the only false statement. A 1996 study showed that warming patients during
colorectal surgery reduced infection rates.173 This finding was confirmed in a
subsequent observational cohort study that reported a significantly increased incidence of
SSI with hypothermia.174 A randomized controlled trial of 421 patients, published in
2001, determined that warming patients before short-duration clean procedures (breast,
varicose vein, or hernia) reduced infection (p = .001) and reduced wound scores (p =
.007) in this setting as well.175 The safest and most effective way of protecting patients
from hypothermia is to use forced-air warmers with specialized blankets placed over the
upper or lower body. Alternatives include placing a warming water mattress under the
patient and draping the patient with an aluminized blanket. Second-line therapy involves
warming all IV fluids. Any irrigation fluids used in a surgical procedure should be at or
slightly above body temperature before use. Radiant heating devices placed above the
operative field may be especially useful during operations on infants. Use of a warmer on
the inhalation side of the anesthetic gas circuit can also help maintain the patient's body
temperature during an operation.
The association between blood transfusion and increased perioperative infection rates is
well documented. In a 1997 study, geriatric hip fracture patients undergoing surgical
repair who received blood transfusion had significantly higher rates of perioperative
infection than those who did not (27% versus 15%), and this effect was confirmed on
multivariate analysis.177 Another study yielded similar findings in colorectal cancer: the
relative risk of infection was 1.6 for transfusion of one to three units of blood and 3.6 for
transfusion of more than three units.178 A large prospective cohort study published in
2002 evaluated the association between anemia, blood transfusion, and perioperative
infection.179 Regression analysis confirmed that intraoperative transfusion of packed red
blood cells was an independent risk factor for perioperative infection (odds ratio 1.06;
95% confidence interval 1.01 to 1.11; p > .0001). Furthermore, transfusion of more than
four units of packed red blood cells was associated with a ninefold increased risk of
infection (95% confidence interval 5.74 to 15.00; p < .0001).
In brief, the principles of optimal surgical antimicrobial prophylaxis include (1)
appropriate choice of an antimicrobial agent, (2) proper timing of antibiotic
administration before incision, and (3) limited duration of antibiotic administration after
operation. SSIs are established several hours after contamination.161 There is a well-
recognized "golden period" during which prophylactic antibiotics can be effective.
Administration of antibiotics before contamination reduces the risk of infection but
subsequently proves to be of little prophylactic value.162 The selective use of short-
duration, narrow-spectrum antibiotic agents chosen to reflect (1) the patient's individual
risk factors and (2) the recognized epidemiology of SSI, modified for local conditions
and experience, should be considered where deemed appropriate [see Table 9 for the
usual pathogens isolated from SSIs].163 When a preoperative antibiotic is indicated, a
single dose of therapeutic strength, administered shortly before incision, usually
suffices.164 The routine dose may have to be increased to achieve adequate serum levels
if the patient is morbidly obese.165 A second dose is indicated if the procedure is longer
than two half-lives of the drug or if extensive blood loss occurs. Continuation of
prophylaxis beyond 24 hours is not recommended. The provision of prophylactic
antibiotics, where indicated, has been accepted as a measure of surgical quality aimed at a
reduction in SSIs.166,167 A strong economic case has been made for infection control,
including prophylactic antibiotics, benefiting the hospital and the patient.168 The
perioperative nursing team has a central role, together with the anesthesiologist, in
monitoring the administration of preoperative prophylactic antibiotics.
Destruction by oxidation or by oxidative killing is the body's most important defense
against surgical pathogens. This defensive response depends on adequate levels of
oxygen tension in contaminated tissue. An easy method of improving oxygen tension in
adequately perfused tissue is to increase the FiO2. Supplemental perioperative oxygen
(i.e., an FIO2 of 80% instead of 30%) significantly overcomes the decrease in
phagocytosis and bacterial killing usually associated with anesthesia and surgery (and
significantly reduces postoperative nausea and vomiting). Low oxygen tension in wound
tissue has been found to be a good predictor of SSI risk.176 With that said, raised oxygen
tension is also an important risk factor in the outbreak of surgical fires. A balance must
be achieved between the adequacy of PO2 in the tissue and the restriction of O2 flow in
head and neck surgery and in other high-risk fire settings. Read more
Feedback on your answer
Collapse
24)
0/1
A 72-year-old African-American male comes to your office for surgical clearance for an
elective left hemicolectomy for recurrent diverticulitis. His past medical history includes
an inferior-wall myocardial infarction approximately 12 months ago. Since then he had a
stress test that was normal 6 months ago. Currently, the patient feels well, walks while
playing nine holes of golf weekly, and is able to walk up a flight of stairs without chest
pain or significant dyspnea. Findings are normal on a physical examination.
Which one of the following would be most appropriate for this patient prior to surgery?
(1pts)

A 12-lead resting EKG

Another Nuclear Stress test

A dobutamine stress echocardiogram

A persantine stressed nuclear tracer study (technetium or thallium)

You didn't make any selections


The answer is A, A 12-lead resting EKG. The current recommendations from the
American College of Cardiology and the American Heart Association on preoperative
clearance for noncardiac surgery state that preoperative intervention is rarely needed to
lower surgical risk. Patients who are not currently experiencing unstable coronary
syndrome, severe valvular disease, uncompensated congestive heart failure, or a
significant arrhythmia are not considered at high risk, and should be evaluated for most
surgery primarily on the basis of their functional status. If these patients are capable of
moderate activity (greater than 4 METs) without cardiac symptoms, they can be cleared
with no stress testing or coronary angiography for an elective minor or intermediate-risk
operation such as the one this patient is to undergo. A resting 12-lead EKG is
recommended for males over 45, females over 55, and patients with diabetes, symptoms
of chest pain, or a previous history of cardiac disease. Ref: ACC/AHA guideline update
for perioperative cardiovascular evaluation for noncardiac surgery--Executive summary:
A report of the American College of Cardiology/American Heart Association Task Force
on Practice Guidelines (Committee to Update the 1996 Guidelines on Perioperative
Cardiovascular Evaluation for Noncardiac Surgery). Circulation 2002;105(10):1257-
1267. Read more here
Feedback on your answer
Collapse
25)
0/1
With regards to positioning patients in the operating room, which of the following
statements if FALSE?
(Select 1)(1pts)

The lithotomy position is used in approximately 9% of cases and is the


next most commonly used position

Approximately 80% of surgical procedures are performed with patients in


the supine position.

Two major preventable consequences of malpositioning are neuropathy or


plexopathy and skin burns or ulceration.

Ulnar neuropathy and brachial plexopathy are the two most common
complications in this position and constitute around 90% of claims.

Damage to the obturator, sciatic, lateral femoral cutaneous, and peroneal


nerves from being placed in lithotomy position account for 5% of medical
liability claims for nerve damage

You didn't make any selections


Answer D is the only false statement as Ulnar neuropathy and brachial plexopathy are
the two most common complications in this position and constitute 28% and 20% of
claims, respectively. The risks associated with malpositioning deserve further comment.
The skin over any bony prominence is subject to injury from excessive pressure. The
fragile skin of infants and older patients may be injured by dragging rather than lifting
into position. The American Society of Anesthesiologists (ASA) practice advisory on the
prevention of perioperative peripheral neuropathies recommends that when practical,
patients should be placed in the intended position before induction of anesthesia to test
for comfort.78 Uncomfortable positions should be modified. It is essential to listen to the
patient. This is especially important with older patients or patients with degenerative
spine and joint disease or skeletal instability. Approximately 80% of surgical procedures
are performed with patients in the supine position. Two major preventable consequences
of malpositioning are neuropathy or plexopathy and skin burns or ulceration. Ulnar
neuropathy and brachial plexopathy are the two most common complications in this
position and constitute 28% and 20% of claims, respectively, in recent closed claims
studies.79-84 Padding of the precondylar groove of the humerus and tucking of the arms
or, at the least, restriction of abduction to less than 90° help prevent this problem.
The lithotomy position is used in approximately 9% of cases and is the next most
commonly used position.80,82 Damage to the obturator, sciatic, lateral femoral
cutaneous, and peroneal nerves has been observed. These injuries account for 5% of
medical liability claims for nerve damage in the Closed Claims Data Base. They
represent a small but not insignificant risk.79 Other patient positions have also been
associated with malpositioning injuries. As would be expected, longer procedures in
sicker or mal-nourished patients are more likely to result in complications of
positioning. Read more

0/1
There is a high risk of pathogen transfer during surgery. This is a risk from which both
the patient and the surgical team must be protected. The risk can be reduced by using
protective barriers, such as surgical gloves. A 2002 Cochrane Review concluded that
wearing two pairs of latex gloves can further reduce the risk of contamination to what %?
(Select 1)(1pts)

20%

15%

10%

5%

You didn't make any selections


The answer is D, There is a high risk of pathogen transfer during surgery. This is a risk
from which both the patient and the surgical team must be protected. The risk can be
reduced by using protective barriers, such as surgical gloves. A 2002 Cochrane Review
concluded that wearing two pairs of latex gloves significantly reduced the risk of hand
exposure. Wearing two pairs of surgical gloves rather than a single pair provides an
additional barrier and further reduces the risk of contamination. The rate of single-glove
perforations is approximately 15%. The rate of inner glove perforation when two pairs of
gloves are worn is approximately 5%. Read more
Feedback on your answer
Collapse
2)
0/1
A 72-year-old African-American male comes to your office for surgical clearance for an
elective left hemicolectomy for recurrent diverticulitis. His past medical history includes
an inferior-wall myocardial infarction approximately 12 months ago. Since then he had a
stress test that was normal 6 months ago. Currently, the patient feels well, walks while
playing nine holes of golf weekly, and is able to walk up a flight of stairs without chest
pain or significant dyspnea. Findings are normal on a physical examination.
Which one of the following would be most appropriate for this patient prior to surgery?
(1pts)

A 12-lead resting EKG

Another Nuclear Stress test

A dobutamine stress echocardiogram

A persantine stressed nuclear tracer study (technetium or thallium)

You didn't make any selections


The answer is A, A 12-lead resting EKG. The current recommendations from the
American College of Cardiology and the American Heart Association on preoperative
clearance for noncardiac surgery state that preoperative intervention is rarely needed to
lower surgical risk. Patients who are not currently experiencing unstable coronary
syndrome, severe valvular disease, uncompensated congestive heart failure, or a
significant arrhythmia are not considered at high risk, and should be evaluated for most
surgery primarily on the basis of their functional status. If these patients are capable of
moderate activity (greater than 4 METs) without cardiac symptoms, they can be cleared
with no stress testing or coronary angiography for an elective minor or intermediate-risk
operation such as the one this patient is to undergo. A resting 12-lead EKG is
recommended for males over 45, females over 55, and patients with diabetes, symptoms
of chest pain, or a previous history of cardiac disease. Ref: ACC/AHA guideline update
for perioperative cardiovascular evaluation for noncardiac surgery--Executive summary:
A report of the American College of Cardiology/American Heart Association Task Force
on Practice Guidelines (Committee to Update the 1996 Guidelines on Perioperative
Cardiovascular Evaluation for Noncardiac Surgery). Circulation 2002;105(10):1257-
1267. Read more here
Feedback on your answer
Collapse
3)
0/1
Where is the surgical amphitheaters known as "The Ether Dome" located?
(Select 1)(1pts)

Beaumont Hospital in Dublin, Ireland

Guy's Hospital in London, England

Massachusetts General Hospital

Padua Hospital in Padua, Italy

You didn't make any selections


The answer is C, Massachusetts General Hospital. The configuration and design of the
modern OR were derived from the anatomic amphitheater of late-Renaissance Italy.
Following the publication of Andreas Vesalius's De Fabrica Humani Corporis in 1543,
the study of human anatomy and dissection emerged as central elements in medical and
particularly in surgical education. This innovation reflected an estrangement from the
previous galenic curriculum, the beginnings of the rejection of scholasticism as the
dominating heuristic in medical investigation, and a sentinel movement toward
empiricism and what we now recognize as the beginnings of modern science. The first
documented dissecting amphitheater, constructed after the classical Greek style of tiered
seats or stands surrounding the stage, was built in Padua in the mid-16th century.1 Others
in Europe soon followed. Although early surgeons generally operated in the patient's
home or on the battlefield, and in hospitals only infrequently, by the first decade of the
18th century, the anatomic amphitheater could serve as an operating theater. In 1707,
Pierre Dionis, a Parisian anatomist and surgeon, included an etching of his surgical
amphitheater in his textbook of surgery. In England, at about the same time, the new
hospitals built to replace the Catholic institutions destroyed by Henry VIII and serve the
needs of the growing urban areas included operating theaters. Guy's Hospital built its
theater on the top floor so that it could be lit by skylights.2-4 A magnificently preserved
example of premodern design can be seen in Dublin, at the Royal College of Surgeons, in
the anatomic building and dissecting rooms that remain in use today. The effective
deployment of natural lighting is remarkable and instructive. American examples of 19th
century surgical amphitheaters also survive. The Ether Dome at the Massachusetts
General Hospital, which derived its name from the effective demonstration of ether as an
anesthetic, may be the best known, but there are others of equal historical
significance. Read more
Feedback on your answer
Collapse
4)
0/1
What is the risk for serious medical complications from surgery in healthy patients?
(Select 1)(1pts)

<0.1%

<0.5-1%

2-3%

5-6%

You didn't make any selections


The answer is A, <0.1%. Healthy patients have significant physiologic reserves and
tolerate major and minor surgical procedures well. The risk for serious medical
complications from surgery is less than 0.1% overall in healthy patients. When evaluating
healthy patients before surgery, physicians should use a focused history and physical
examination to predict the risk for serious medical complications. Researchers collected
information at 16 and 72 hours and 30 days for 38 598 patients undergoing 45 090
consecutive procedures and found that only 33 (0.08%) had major morbidity or mortality.
Of 4 deaths that occurred within 30 days of the procedure, 2 were due to myocardial
infarction and 2 were due to automobile accidents unlikely to be related to surgery. More
than one third of major morbidity occurred 48 hours or more after surgery. Read more
Feedback on your answer
Collapse
5)
0/1
You are consulted to a 38-year-old black male for evaluation of his preoperative
cardiovascular risk. He has a 20-year history of type 1 diabetes mellitus and hypertension
and is undergoing preoperative evaluation for renal transplantation secondary to
uncontrolled hypertension and diabetic nephropathy. His current blood pressure is 142/85
mm Hg. He currently smokes one half of a pack of cigarettes daily. His
electrocardiogram is consistent with left ventricular hypertrophy and left atrial
enlargement. Which of the following is the most appropriate recommendation at this
time?
(1pts)

No further evaluation is needed, he is cleared for surgery without further


testing

Pharmacologic stress nuclear study

24-hour electrocardiographic holter monitor

Coronary angiography

You didn't make any selections

The answer is B, Pharmacologic stress nuclear study. In general, the indications for
noninvasive cardiac testing are the same for patients undergoing surgery as for those who
are not. All symptomatic patients should be evaluated for cardiac ischemia, usually with
cardiac stress testing, even if their symptoms are atypical. Patients undergoing a low- or
intermediate risk surgery who have at most minor clinical risk predictors (advanced age,
abnormal ECG, rhythm other than sinus, low functional capacity, history of stroke, or
uncontrolled hyper tension) may proceed to surgery without preoperative cardiac testing.
Likewise, patients with intermediate clinical risk predictors (mild angina, diabetes
mellitus, compensated heart failure, previous myocardial infarction, or renal
insufficiency) who have good exercise capacity also do not need further preoperative
cardiac testing before a low- or intermediate-risk procedure. However, in the absence of
high-quality data, some experts recommend that those with intermediate clinical
predictors who have poor functional status and are undergoing an intermediate-risk
procedure, as well as those with intermediate clinical predictors and good functional
status who are undergoing a high-risk procedure, should have noninvasive testing
performed before surgery.
Coronary revascularization is rarely needed just to get a patient through surgery, but it
may be warranted in selected patients with CAD undergoing noncardiac surgery if they
would have required the procedure anyway. However, the only large randomized trial on
the subject showed no benefit for revascularization (40). Avoid preoperative coronary
sten ting to avoid the risk for perioperative stent thrombosis. Patients with recently placed
coronary stents are at risk for perioperative stent thrombosis. For bare-metal stents, the
risk diminishes 4 to 6 weeks after implantation; for drug-eluting stents, an elevated risk
for rethrombosis persists for at least 12 months after placement because of delayed
endothelialization. Because these patients require antiplatelet treatment to mitigate this
risk, purely elective procedures should be delayed accordingly, and the interval without
antiplatelet therapy should be minimized when surgery is required within the window of
vulnerability. Read more

Feedback on your answer


Collapse
Unit 4
6)
0/1
You are seeing a 49 -ear old obese male who presents with two days of worsening left
sided abdominal pain. He denies fever or chills but does note painful bowel movements
that have streaks of blood. On exam you note moderate left lower quadrant tenderness to
palpation without rebound or guarding. His labs show a white blood cell count of
14,000mm3 and his CT scan of his abdomen/pelvis shows acute uncomplicated sigmoid
diverticulitis. Nonoperative treatment with oral antibiotics will resolve acute
uncomplicated diverticulitis in what percent of patients?
(Select 1)(1pts)

<25%

50%

80%

>95%

You didn't make any selections


The best answer is B, 80%. Although most patients with acute diverticulitis can be treated
medically, approximately 15-20% will require surgery. Uncomplicated diverticulitis may
be managed as an outpatient (dietary modification and oral antibiotics) for those without
appreciable fever, excessive vomiting, or marked peritonitis, as long as there is the
opportunity for follow-up. The patient should be able to take liquids and antibiotics by
mouth. Hospitalization for treatment (dietary modification and intravenous antibiotics) is
usually best if the above conditions are not met, or if the patient fails to improve with
outpatient therapy. Antibiotics should be selected to treat the most common bacteria
found in the colon: gram-negative rods and anaerobic bacteria. Single and multiple
antibiotic regimens are equally effective, as long as both groups of organisms are
covered. Nonoperative treatment will resolve acute diverticulitis in 80
percent of patients, but approximately one-third will have a recurrent attack often
within one year. approximately 20% of patients with diverticulitis require surgical
treatment.30 Most surgical procedures are reserved for patients who experience recurrent
episodes of acute diverticulitis that necessitate treatment (inpatient or out-patient) or who
have complicated diverticulitis. The most common indication for elective resection is
recurrent attacks—that is, several episodes of acute diverticulitis documented by studies
such as CT. Rerecurrences may be more common than recurrences. In 2000, a task force
of the American Society of Colon and Rectal Surgeons recommended sigmoid resection
after two attacks of diverticulitis. A subsequent cost analysis using a Markov model
suggested that cost savings could be achieved if resection was done after three
attacks.33 There is a growing tendency to question arbitrary guidelines for surgical
management of recurrent attacks, with the exception of certain groups, such as
immunocompromised patients. Current practice guidelines state that the recommendation
to perform elective sigmoid resection after recovery from uncomplicated acute
diverticulitis should be made on a case-by-case basis. The decision-making process
should be influenced by the age and medical condition of the patient, the frequency and
severity of attacks, and the presence of symptoms after the acute attack. Elective
resection is generally recommended after an episode of complicated
diverticulitis.36 Efforts are made to time surgical treatment so that it takes place during a
quiescent period 8 to 10 weeks after the last attack. Barium enema or colonoscopy may
be employed to evaluate the diverticular disease and rule out carcinoma. The bowel can
then be prepared mechanically and with antibiotics (e.g., oral neomycin and
metronidazole on the day before operation).
2014. Scientific American Surgery. Hamilton, Ontario & Philadelphia, PA. Decker
Intellectual Properties Inc. ISSN 2368-2744. STAT!Ref Online Electronic Medical
Library. http://online.statref.com/Document.aspx?fxId=61&docId=1368. 10/20/2014
12:41:34 PM CDT (UTC -05:00).

Feedback on your answer


Collapse
7)
0/1
What percentage of diveriticulosis affects the sigmoid colon?
(Select 1)(1pts)

50%
65%

75%

95%

You didn't make any selections


The answer is D. The prevalence of diverticulosis is age-dependent, increasing from less
than 20 percent at age 40 to 60 percent by age 60. Approximately 95 percent of patients
with diverticula have sigmoid diverticula. Diverticula are limited to the sigmoid colon in
65 percent of patients; in 24 percent of patients diverticula predominantly involve the
sigmoid, but are also present in other parts of the colon (image 2); in 7 percent of patients
diverticula are equally distributed throughout the colon; and in 4 percent diverticula are
limited to a segment proximal to the sigmoid colon. Approximately 4 to 15 percent of
patients with diverticulosis develop diverticulitis [7,20-22]. The incidence of
diverticulitis increases with age. The mean age at admission for acute diverticulitis is 63
years [23]. While the incidence of acute diverticulitis is lower in younger individuals,
approximately 16 percent of admissions for acute diverticulitis are in patients under 45
years of age [24]. In contrast to Asia, diverticular disease is predominantly left-sided in
western countries, with right-sided diverticulitis being present in only 1.5 percent of cases
[25]. Thirty-five percent of patients with sigmoid diverticulosis also have disease in the
more proximal colon. Diverticula are rare below the pelvic peritoneal reflection.
Prevalence correlates with age; approximately 30 percent of the population has acquired
diverticular change by age 60 years,whereas almost 60 percent of those aged 80 years and
older are affected. Ten to 25 percent of patients with diverticulosis will develop
diverticulitis. Read More
Feedback on your answer
Collapse
8)
0/1
You are seeing a 58-year-old female who is presenting with two days of a low-grade
fever and increasing abdominal cramps that are exacerbated by bowel movements. She
began a course of augmentin 2 days ago that her PCP gave her but has had no relief of
her symptoms and is getting worse. She has not had nausea, vomiting, urinary symptoms,
or bloody stools. She has a past medical history of hypertension.

Her vitals show a temperature is 100.6°F, pulse is 75/min, and blood pressure is 150/80
mm Hg. Her lungs are clear to auscultation. Cardiac examination shows no
abnormalities. Her abdomen is soft, and there is tenderness to palpation of the left lower
quadrant with guarding but no rebound. Bowel sounds are normal. Her stool is brown,
and test for occult blood is negative. Her hemoglobin concentration is 14.5 g/dL,
leukocyte count is 15,800/mm3, and platelet count is 280,000/mm3; serum studies and
urinalysis show no abnormalities. Which of the following is the most appropriate next
step in diagnosis?
(Select 1)(1pts)
Examination of the stool for ova and parasites

Test of the stool for Clostridium difficile toxin

Xray Abdomen Series

CT scan of the abdomen/pelvis with IV and oral contrast

You didn't make any selections


The answer is D, CT scan of the abdomen with IV and oral contrast. This patient is being
treated for diverticulitis and is not getting better. She needs a CT scan to evaluate for a
possible abscess. Read more

Feedback on your answer


Collapse
9)
0/1
You are seeing a 64-year old female who is presenting with painless bright red rectal
bleeding for the last two days. She notes bright red blood coating the stool and on the
toilet paper when she wipes. He vitals are normal and her physical exam shows a soft non
tender abdomen. Stool is brown with red streaks. What is the most likely cause of
bleeding in this patient and the most common overall cause of lower GI bleeding in
adults?
(Select 1)(1pts)

Inflammatory Bowel Disease

Diverticulosis

Neoplasm

Angiodysplasia

You didn't make any selections

The answer is B. Diverticulosis results in 60% of cases of lower GI bleeding followed by


IBD(13%), benign anorectal disease (11%), neoplasia (10%) and angiodysplasia (<5%).
The causes of lower gastrointestinal bleeding (LGIB) may be grouped into several
categories (table 1): From Uptodate
 Anatomic (diverticulosis)
 Vascular (angiodysplasia, ischemic, radiation-induced)
 Inflammatory (inflammatory bowel disease, infectious)
 Neoplastic
In most series, diverticulosis is the most common source of LGIB, accounting for
approximately 15 to 55 percent of cases [9,10]. Angiodysplasia may be the most
frequent cause in patients over the age of 65 years [11,12], though more recent data
suggest that angiodysplasia may be a less common cause of LGIB than once thought.
Hemorrhoids are the most common cause of rectal bleeding in patients under the age of
50 years [13]. However, hemorrhoidal bleeding is usually minor. Similarly, bloody
diarrhea due to inflammatory causes can sometimes be distinguished from other causes of
LGIB because of the clinical setting. In general, anatomic and vascular causes of
bleeding present with painless, large-volume blood loss, whereas inflammatory sources
are associated with diarrhea and abdominal pain [14].
In a review of several large studies that included 1559 patients with acute hematochezia,
the following bleeding sources were identified [10]:
 Diverticulosis – 5 to 42 percent
 Ischemia – 6 to 18 percent
 Anorectal (hemorrhoids, anal fissures, rectal ulcers) – 6 to 16 percent
 Neoplasia (polyps and cancers) – 3 to 11 percent
 Angiodysplasia – 0 to 3 percent
 Postpolypectomy – 0 to 13 percent
 Inflammatory bowel disease – 2 to 4 percent
 Radiation colitis – 1 to 3 percent
 Other colitis (infectious, antibiotic associated, colitis of unclear etiology)
– 3 to 29 percent
 Small bowel/upper GI bleed – 3 to 13 percent
 Other causes – 1 to 9 percent
 Unknown cause – 6 to 23 percent
The vast majority of colonic diverticula are actually false diverticula (pseudodiverticula)
that contain only serosa and mucosa. They occur at weak points in the colonic wall where
the vasa recta penetrate the muscularis to supply the mucosa; as the diverticulum
expands, these vessels are displaced. A 1976 anatomic study of colonic specimens from
patients with diverticular bleeding used angiography to demonstrate that in all cases, the
vasa recta overlying the diverticulum ruptured into the lumen of the diverticulum, not
into the peritoneum.100

It has been estimated that approximately 17% of patients with colonic diverticulosis
experience bleeding, which may range from minor to severe and life-threatening.
Endoscopic treatment of diverticular hemorrhage can be difficult because of the high
bleeding rate and the location of the bleeding point within the diverticulum. In 2000, one
group of investigators reported their experience with endoscopic therapy for severe
hematochezia and diverticulosis in a prospective series of 121 patients.101 In this series,
none of the patients treated endoscopically with epinephrine injections, bipolar
coagulation, or both required surgery and none experienced recurrent bleeding episodes.
A 2001 study from another group, however, reported high rates of recurrent bleeding
episodes in both the early and the late posttreatment periods.102 In the absence of
prospective, randomized trials, it is difficult to draw definitive conclusions about the
utility of endoscopic therapy in treating diverticular hemorrhage.

Fortunately, most diverticular hemorrhages stop spontaneously. In one series, surgery


was unlikely to be necessary if fewer than four units of packed RBCs were transfused in a
24-hour period, whereas 60% of patients receiving more than four units of packed RBCs
in a 24-hour period required surgical intervention.103 Semielective surgical therapy is
usually offered after a second diverticular bleeding episode because once a second such
episode has occurred, the risk that a third will follow exceeds 50%. In a series of 83
conservatively managed cases of diverticular disease, the predicted yearly recurrence
rates were 9% at 1 year, 10% at 2 years, 19% at 3 years, and 25% at 4 years.104

In general, patients who require more than four units of blood in a 24-hour period to
remain hemodynamically stable, who have not stopped bleeding after 72 hours, or who
experience rebleeding within 1 week after an initial episode should undergo surgery.

2014. Scientific American Surgery. Hamilton, Ontario & Philadelphia, PA. Decker
Intellectual Properties Inc. ISSN 2368-2744. STAT!Ref Online Electronic Medical
Library. http://online.statref.com/Document.aspx?fxId=61&docId=848. 10/22/2014
7:03:07 PM CDT (UTC -05:00).

Feedback on your answer


Collapse
10)
0/1
You are seeing a 45-year old obese male with type II diabetes who presents with acute
onset left lower quadrant abdominal pain, vomiting, pain with defecation, and
fever. Physical exam is normal except for his abdominal exam that shows left lower
quadrant tenderness without signs of peritonitis. His labs are remarkable for a WBC of
16,000 mm3. Which of the following choices is most likely to reduce repeat incidences of
this patients condition?
(Select 1)(1pts)

Routine daily stool softners or laxatives

Biannual colonoscopy

High fiber diet

Avoidance of nuts and legumes

You didn't make any selections


The answer is C, High fiber diet. Environmental and lifestyle factors are important risk
factors for diverticular disease. Out of the following options only increasing fiber intake
has been shown to reduce episodes of diverticulitis. The role of fiber in the development
of diverticulosis is unclear. Several early studies suggested that low dietary fiber
predisposes to the development of diverticular disease, but other studies have been
conflicting [3,30-39]. Fiber also does not reduce symptoms in patients with
symptomatic uncomplicated diverticular disease [37,38,40,41]. However, dietary fiber
and a vegetarian diet may reduce the incidence of symptomatic diverticular disease by
decreasing intestinal inflammation and altering the intestinal microbiota [4,31,42].
In a cohort study that included over 47,000 men, after adjustment for age, energy-
adjusted total fat intake, and physical activity, total dietary fiber intake was noted to be
inversely associated with the risk of symptomatic diverticular disease (RR 0.58 highest
quintile versus lowest quintile for fiber intake) [31].
Fat and red meat — In the same cohort study, the risk of diverticular disease was
significantly increased with diets that were low in fiber and were high in total fat or red
meat as compared with diets that were low in both fiber and total fat or red meat (RR
2.35 and 3.32, respectively) [31].
Seeds and nuts — Nut, corn, and popcorn consumption arenot associated with an
increase in risk of diverticulosis, diverticulitis or diverticular bleeding. In a large
observational study (The Health Professionals Follow-up Study) that included 47,228
men between the ages of 40 and 75 years, there was an inverse association between the
amount of nut and popcorn consumption and the risk of diverticulitis (HR nuts 0.8, 95%
CI 0.63-1.01; HR popcorn 0.72, 95% CI 0.56-0.92) [43]. In addition, no association was
found between consumption of corn and diverticulitis or between nut, popcorn, or corn
consumption and diverticular bleeding or uncomplicated diverticulosis.
Physical activity — It is unclear if lack of vigorous exercise is a risk factor for
diverticular disease. However, vigorous physical activity appears to reduce the risk of
diverticulitis and diverticular bleeding. In a prospective study of approximately 48,000
men aged 40 to 75 who were free of known colonic disease at baseline, the risk of
developing symptomatic diverticular disease was inversely related to overall physical
activity (RR 0.63 for highest versus lowest extremes) after adjustment for age and dietary
fat and fiber [44]. Most of the decrease in risk with exercise was associated with
vigorous activity such as jogging and running. Men in the lowest quintile for both dietary
fiber and physical activity had an increased risk of symptomatic diverticular disease as
compared with men in the highest quintile for both (RR 2.56, 95% CI 1.36-4.82).
Obesity — Obesity has been associated with an increase in risk of both diverticulitis
and diverticular bleeding. In a large, prospective cohort study of 47,228 male health
professionals, there were 801 incident cases of diverticulitis and 383 cases of diverticular
bleeding during 18 years of follow-up [45]. The risk of diverticulitis and diverticular
bleeding was significantly higher in those with the highest quintile of waist
circumference as compared with the lowest (RR diverticulitis 1.56, 95% CI 1.18-2.07;
RR diverticular bleeding 1.96, 95% CI 1.30-2.97).
Other — Current smokers appear to be at increased risk for perforated diverticulitis and
a diverticular abscess as compared with nonsmokers (OR 1.89, 95% CI 1.15-3.10) [46].
Caffeine and alcohol are not associated with an increased risk for symptomatic
diverticular disease. Read more
Feedback on your answer
Collapse
Unit 5
11)
0/1
Which of the following is NOT true regarding post operative placement of foley
catheters?
(1pts)

As many as 40% of all hospital infections are UTIs

Only 10% of hospital acquired UTIs are associated with urinary catheters

After many procedures, patients initially find it extremely difficult or


impossible to mobilize for urination, and a urinary catheter may be quite
helpful during this time

In the setting of bladder or genitourinary surgery, they are often


employed to decompress the system so that it will heal more readily

You didn't make any selections


Answer B is false. Urinary catheters can serve a large variety of purposes. In the setting
of bladder or genitourinary surgery, they are often employed to decompress the system so
that it will heal more readily. After general surgical procedures—and many other surgical
procedures as well—they are used to provide accurate measurements of volume output
and thus, indirectly, to give some indication of the patient's overall volume and
resuscitation status. Furthermore, after many procedures, patients initially find it
extremely difficult or impossible to mobilize for urination, and a urinary catheter may be
quite helpful during this time.
Their utility and importance notwithstanding, urinary catheters are
associated with the development of nosocomial urinary tract infections
(UTIs). As many as 40% of all hospital infections are UTIs, and
80 to 90% of these UTIs are associated with urinary
catheters.Accordingly, when catheterization is no longer deemed
necessary, prompt removal is indicated. As a rule, orders specifically
pertaining to urinary catheter care are few, typically including gravity
drainage, flushing to maintain patency (if warranted), and removal
when appropriate. At times, irrigation is employed after urologic
procedures or for the management of certain infectious agents. Read
more
Feedback on your answer
Collapse
12)
0/1
Urinary catheters can serve a large variety of purposes. In the setting of bladder or
genitourinary surgery, they are often employed to decompress the system so that it will
heal more readily. After general surgical procedures—and many other surgical
procedures as well—they are used to provide accurate measurements of volume output
and thus, indirectly, to give some indication of the patient's overall volume and
resuscitation status.
Their utility and importance notwithstanding, urinary catheters are associated with the
development of nosocomial urinary tract infections (UTIs). As many as 40% of all
hospital infections are UTIs. What % of these UTI's are associated with urinary catheters?
(Select 1)(1pts)

20%

40%

60%

80%

You didn't make any selections


The answer is D, 80%. Urinary catheters can serve a large variety of purposes. In the
setting of bladder or genitourinary surgery, they are often employed to decompress the
system so that it will heal more readily. After general surgical procedures—and many
other surgical procedures as well—they are used to provide accurate measurements of
volume output and thus, indirectly, to give some indication of the patient's overall volume
and resuscitation status. Furthermore, after many procedures, patients initially find it
extremely difficult or impossible to mobilize for urination, and a urinary catheter may be
quite helpful during this time. Their utility and importance notwithstanding, urinary
catheters are associated with the development of nosocomial urinary tract infections
(UTIs). As many as 40% of all hospital infections are UTIs, and 80 to 90% of these UTIs
are associated with urinary catheters [see Postoperative Complications,below].
Accordingly, when catheterization is no longer deemed necessary, prompt removal is
indicated. As a rule, orders specifically pertaining to urinary catheter care are few,
typically including gravity drainage, flushing to maintain patency (if warranted), and
removal when appropriate. At times, irrigation is employed after urologic procedures or
for the management of certain infectious agents. Read more
Feedback on your answer
Collapse
13)
0/1
Initial wound management after an operative procedure generally entails placement of a
sterile dressing to cover the incision. The traditional recommendation has been to keep
this dressing in place and dry for what period of time?

(Select 1)(1pts)

12 hours

24 hours

48 hours

72 hours

All of the above

None of the above

You didn't make any selections

The answer is C, 48 hours. Initial wound management after an operative procedure


generally entails placement of a sterile dressing to cover the incision. The traditional
recommendation has been to keep this dressing in place and dry for the first 48 hours
after operation; because epithelialization is known to take place within approximately this
period, the assumption is that this measure will reduce the risk of wound infection.
Although most surgeons still follow this practice, especially in general surgical cases,
supporting data from randomized clinical studies are lacking. In addition, several small
studies that evaluated early showering with closed surgical incisions found no increases
in the rate of infection or dehiscence.8,9 It should be kept in mind, however, that these
small studies looked primarily at soft tissue and other minor skin incisions that did not
involve fascia. Thus, even though the traditional approach to initial dressing management
is not strongly supported, the data currently available are insufficient to indicate that it
should be changed.
Wet-to-dry dressings are used in a variety of settings. In a surgical context, they are most
often applied to a wound that cannot be closed primarily as a consequence of
contamination or inability to approximate the skin edges. Wet-to-dry dressings provide a
moist environment that promotes granulation and wound closure by secondary intention.
Moreover, their removal and replacement cause debridement of excess exudate or
unhealthy superficial tissue. Postoperative orders should specify the frequency of
dressing changes and the solution used to provide dampness. For most clean open
incisions, twice-daily dressing changes using normal saline solution represent the most
common approach. If there is excess wound exudate to be debrided, dressing changes
may be performed more frequently. If there is particular concern about wound
contamination or superficial colonization of organisms, substitution of dilute Dakin
solution for normal saline may be considered. Read more

Feedback on your answer


Collapse
14)
0/1
Same-day surgery is appropriate for patients who (1) have few or no comorbid medical
conditions and (2) are undergoing a procedure that involves short-duration anesthesia or
local anesthesia plus sedation and that carries a low likelihood of urgent complications.
Operations commonly performed on a same-day basis include which of the following?
(Select 1)(1pts)

inguinal or umbilical hernia repair

simple laparoscopic cholecystectomy

breast biopsy

small subcutaneous procedures

All of the above

None of the above

You didn't make any selections


The answer is all of the above. Same-day surgery is appropriate for patients who (1) have
few or no comorbid medical conditions and (2) are undergoing a procedure that involves
short-duration anesthesia or local anesthesia plus sedation and that carries a low
likelihood of urgent complications. Operations commonly performed on a same-day basis
include inguinal or umbilical hernia repair, simple laparoscopic cholecystectomy, breast
biopsy, and small subcutaneous procedures. The growth in the performance of minor and
same-day procedures has led to the development of various types of short-stay units or
wards. The level of care provided by a short-stay ward is generally equivalent to that
provided by a regular nursing ward; however, the anticipated duration of care is
substantially shorter, typically ranging from several hours to a maximum of 48 hours.
Short-stay wards also undergo some modifications to facilitate the use of streamlined
teaching protocols designed to prepare patients for home care. Many hospitals now have
short-stay units, as do some independent surgery centers. Read more
Feedback on your answer
Collapse
15)
0/1
Multiple randomized clinical trials have now demonstrated that routine use of drains after
elective operations—including appendectomies and colorectal, hepatic, thyroid, and
parathyroid procedures-- reduces which of the following?
(Select 1)(1pts)

anastomotic complications

infection rates

seroma formation

bleeding

All of the above

None of the above

You didn't make any selections


The answer is C, Drains and tubes are placed in a wide variety of locations for a number
of different purposes—in particular, drainage of purulent materials, serum, or blood from
body cavities. Several types are commonly used, including soft gravity drains (e.g.,
Penrose), closedsuction drains (e.g., Hemovac, Jackson-Pratt, and Blake), and sump
drains, which draw air into one lumen and extract fluid via a companion lumen.
Traditionally, surgeons have often made the decision to place a drain on the basis of their
surgical training and practice habits rather than of any firm evidence that drainage is
warranted. Multiple randomized clinical trials have now demonstrated that routine use of
drains after elective operations—including appendectomies and colorectal, hepatic,
thyroid, and parathyroid procedures—does not prevent anastomotic and other
complications (although it does reduce seroma formation). Consequently, it is
recommended that drains, like NG tubes, be employed selectively.3- 5Once a drain is in
place, specific orders must be issued for its maintenance. These include use of gravity or
suction (and the means by which suction is to be provided if ordered), management and
measurement of output, stripping, and care around the drain exit site. Read more
Feedback on your answer
Collapse
Unit 1
16)
0/1
You are rounding on a 46-year old female who post-op day 1 following
a cholecystectomy. She has no other past medical history, takes no medications and she is
a one pack per day smoker for the last 20 years. Upon reviewing her 6 am vital signs, you
notice that her O2 saturations were declining all night from 99% to currently 90% on
room air. The rest of her vitals show an oral temperature of 98.1 F, respirations 16/min,
BP 125/90, and heart rate is 85 beats/min. You order an ABG which shows a pH of 7.44,
pO2 65 mmhg and CO2 of 35 mmhg. What is the most likely explanation of these
findings?

(Select 1)( 1pts extra credit)

Aspiration

Atelectasis

Pulmonary Embolism

Pneumonia

ARDS

You didn't make any selections


The answer is B, Atelectasis is one of the most common postoperative pulmonary
complications, particularly following abdominal and thoracoabdominal
procedures. Postoperative atelectasis can be asymptomatic or it may manifest as
increased work of breathing and hypoxemia. The onset of hypoxemia due to
postoperative atelectasis tends to occur after the patient has left the post-anesthesia care
unit. It typically becomes most severe during the second postoperative night and
continues through the fourth or fifth postoperative night [5,6].
Hypoxemia that develops earlier (ie, in the post-anesthesia care unit) should prompt the
consideration of postoperative complications other than atelectasis, such as
hypoventilation due to residual anesthetic effects and upper airway obstruction due to
airway tissue edema. The latter may be due to the accumulation of pharyngeal secretions,
prolapse of the tongue posteriorly, or tongue edema due to either surgical manipulation or
an allergic reaction. Read more
A-a O2 Gradient = [ (FiO2) * (Atmospheric Pressure - H2O Pressure) - (PaCO2/0.8) ] -
PaO2 from ABG
Normal Gradient Estimate = (Age/4) + 4
The 5 Causes of Hypoxemia, #1-3 have an elevated A-a Gradient:
1. V/Q Mismatch (ex: PNA, CHF, ARDS, atelectasis, etc)
2. Shunt (ex: PFO, ASD, PE, pulmonary AVMs)
3. Alveolar Hypoventilation (ex: interstitial lung dz, environmental lung dz, PCP
PNA)
4. Hypoventilation (ex: COPD, CNS d/o, neuromuscular dz, etc)
5. Low FiO2 (ex: high altitude)

Feedback on your answer


Collapse
17)
0/1
You are seeing a 74-year old female who presents with acute onset of left lower
abdominal pain along with fever and vomiting. She has a prior history of diverticular
disease as well as type II diabetes. You are suspicious for diverticulitis and order a CT
abdomen/pelvis with IV and PO contrast to confirm. Which of
the following incorrectly defines acute complicated diverticulitis?
(Select 1)( 1pts extra credit)

Divericulitis with abscess

Divericulitis with fistula

Divericulitis with colon obstruction

Divericulitis with elevated WBC and bandemia

All options correctly define complicated diverticulitis

You didn't make any selections


Answer D incorrectly define complicated diverticulitis. For the purposes of this
discussion, complicated diverticulitis is defined as acute diverticulitis accompanied by
abscess, fistula, obstruction, or free intra-abdominal perforation. An elevated WBC with
bandemia does not define this.
Some cases of diverticulitis are classified as complicated, meaning that the disease
process has progressed to obstruction, abscess or fistula formation, or free perforation.
Complicated diverticulitis may be particularly challenging to manage, especially because
patients may have no known history of diverticular disease. Lower gastrointestinal (GI)
bleeding is also a complication of diverticular disease in 30 to 50% of cases; in fact,
diverticula are the most common colonic cause of lower GI bleeding. When diverticular
hemorrhage occurs [see 5:5 Lower Gastrointestinal Bleeding], it is usually
associated with diverticulosis rather than with diverticulitis. Approximately 50% of
diverticular bleeding originates in the right colon, despite the low incidence of diverticula
in this segment of the colon. Patients tend to be elderly and to have cardiovascular
disease and hypertension. Regular intake of NSAIDs may increase the risk of this
complication. Although patients may lose 1 to 2 units of blood, the bleeding usually
ceases spontaneously,13 and expeditious operative treatment generally is not necessary.
The most common form of complicated diverticulitis involves the development of a
pericolic abscess, typically signaled by high fever, chills, and lassitude. Such abscesses
may be small and localized or may extend to more distant sites (e.g., the pelvis). They
may be categorized according to the Hinchey classification of diverticular
perforations,14 in which stage I refers to a localized pericolic abscess and stage II to a
larger mesenteric abscess spreading toward the pelvis On rare occasions, an abscess
forms in the retroperitoneal tissues, subsequently extending to distant sites such as the
thigh or the flank. The location of the abscess can be defined precisely by means of
computed tomography (CT) with contrast.
Some abscesses rupture into adjacent tissues or viscera, resulting in the formation of
fistulas. The fistulas most commonly seen in this setting (50 to 65% of cases) are
colovesical fistulas. This complication is less common in women because of the
protection afforded by the uterus. Symptoms of colovesical fistulas tend to involve the
urinary tract (e.g., pneumaturia, hematuria, and urinary frequency). Fecaluria is
diagnostic of colovesical or enterovesical fistulas. Colovaginal fistulas (which account
for 25% of all diverticular fistulas) are usually seen in women who have undergone
hysterectomies. The diseased colon is adherent to the vaginal cuff. Most commonly,
patients complain of a foul vaginal discharge; however, some patients present with stool
emanating from the vagina.

About 10% of colon obstructions are attributable to diverticulitis. Acute diverticulitis can
cause colonic edema and a functional obstruction that usually resolves with antibiotic
infusion and bowel rest. Stricture formation is more common, usually occurring as a
consequence of recurrent attacks of diverticulitis. Circumferential pericolic fibrosis is
noted, and marked angulation of the pelvic colon with adherence to the pelvic sidewall
may be seen. Patients complain of constipation and narrowed stools. Colonoscopy can be
difficult and potentially dangerous in this setting. Differentiating a diverticular stricture
from carcinoma may be impossible by any means short of resection.

The term malignant diverticulitis has been employed to describe an extreme form of
sigmoid diverticulitis that is characterized by an extensive phlegmon and inflammatory
reaction extending below the peritoneal reflection, with a tendency toward obstruction
and fistula formation.15 Malignant diverticulitis is seen in fewer than 5% of patients older
than 50 years who are operated on for diverticulitis.15 The process is reminiscent of
Crohn disease, and CT scans demonstrate extensive inflammation. In this setting, a
staged resection might be preferable to attempting a primary resection through the pelvic
phlegmon. The degree of pelvic inflammation may subside significantly after
diversion.15
A dangerous but rare complication of acute diverticulitis (occurring in 1 to 2% of cases)
is free perforation,16 which includes both perforation of a diverticular abscess throughout
the abdomen leading to generalized peritonitis (purulent peritonitis; Hinchey stage III)
and free spillage of stool thorough an open diverticulum into the peritoneal cavity (fecal
peritonitis; Hinchey stage IV). The incidence of free perforations may be increasing, at
least in the southwestern United States. The overall mortality in this group is between 20
and 30%, that for purulent peritonitis is approximately 13%, and that for fecal peritonitis
is about 43%.

2014. Scientific American Surgery. Hamilton, Ontario & Philadelphia, PA. Decker
Intellectual Properties Inc. ISSN 2368-2744. STAT!Ref Online Electronic Medical
Library. http://online.statref.com/Document.aspx?fxId=61&docId=1368. 10/22/2014
12:22:12 PM CDT (UTC -05:00).
Feedback on your answer
Collapse
18)
0/1
You are consulted to see a 58-year old white male who in the ER acute diverticulitis. He
has a past medical history of type II diabetes and 3 prior episodes of diverticulitis in the
past without complication. His vitals show his heart rate 110 beats/min, oral temperature
101.5 F, blood pressure 141/81, respirations 18/min. Physical exam shows abdominal
guarding and diffuse tenderness to palpation in all 4 quadrants. Review of his CT scan
shows stranding in the sigmoid colon along with a 4 cm abscess. What is the best
treatment modality for this patient?
(Select 1)( 1pts extra credit)

Admit for IV Ceftriaxone and Flagyl only

Percutaneous drainage with IR

Surgery with a traditional Hartmann procedure

PO Ceftriaxone and Flagyl and DC home with follow up in 48 hours

You didn't make any selections


The answer is C. This patient by exam has acute complicated diverticulitis with an
abscess of 4 cm and peritonitis and will require surgery. Urgent sigmoid colectomy is
required for patients with diffuse peritonitis or for those who fail nonoperative
management of acute diverticulitis. Level of Evidence: III; Grade of Recommendation:
B. If a patient presents with severe or diffuse peritonitis, emergency colon resection is
necessary. Also, if sepsis does not improve with inpatient conservative treatment of acute
diverticulitis or after percutaneous drainage, surgery is indicated. Immunosuppressed or
immunocompromised patients are more likely to present with perforation or fail medical
management, so a lower threshold for urgent or elective surgery should apply to them.
After emergency sigmoid resection, anastomosis might be performed, depending on the
status of the patient and the severity of intra-abdominal contamination (Hinchey
classification). A traditional Hartmann procedure is commonly performed (sigmoid
colectomy, end sigmoid or descending colostomy, and closure of the rectal stump);
however, the later second-stage operation to close this colostomy can be technically
difficult. Alternatives to a Hartmann procedure may be primary anastomosis with or
without intraoperative colonic lavage, or resection and anastomosis with temporary
diverting ileostomy. The precise role and relative safety of primary anastomosis,
especially without proximal diversion remains unsettled.
Abscesses occur in 16 percent of patients with acute diverticulitis without peritonitis [11]
and in 31 to 56 percent of those requiring surgery for diverticulitis [12]. Prior to
interventional radiologic techniques, abscesses mandated operative intervention, often
with two-stage procedures. Percutaneous drainage permits elective single-stage surgery in
60 to 80 percent of patients [13-17]; furthermore, in selected patients with
contraindications to surgery, catheter drainage may be sufficient to relieve symptoms
[16]. However, low quality evidence suggests that conservative management of
complicated diverticulitis with abscess formation may result in chronic or recurrent
diverticular symptoms, and is associated with a high probability of resective surgery. A
systematic review identified 22 studies involving a total of 1051 patients with acute
diverticulitis with abscess formation (modified Hinchey grades IB and II) [13].
Percutaneous abscess drainage (diameter >3 cm) was successful in 49 percent of patients
and antibiotic therapy in 14 percent of patients. Recurrence rates were high at 39 percent
among those awaiting elective resection and 18 percent in the nonsurgical group, for an
overall recurrence rate of 28 percent. Overall, only 28 percent avoided surgery and had
no recurrence during follow-up.
Drainage is usually performed through the anterior abdominal wall; abscesses deep in the
pelvis or obscured by other organs may be accessed transgluteally, or through the rectum
or vagina, although this approach is rarely performed [18,19]. The catheter is left in
place until drainage is less than 10 mL in 24 hours; this may take as long as 30 days [16].
Catheter sinograms during this period can show persistent communication between fistula
and bowel, and permit assessment of resolution of the abscess cavity. Surgical
intervention is mandatory if improvement does not occur.
Until recently, all abscesses have been considered to represent complicated diverticulitis,
and surgery has typically been recommended once resolution has occurred. However,
with continued improvements in CT scan technology, smaller abscesses can be identified.
Some of these are so small (ie, less than 3 cm) that they are not amenable to CT-guided
drainage [20-22]. In addition to size limitations, the presence of small bowel loops
adjacent to the collection may preclude drainage. Abscess recurrence and/or fistula
formation after percutaneous drainage may also complicate the patient's course and
compromise the surgical approach. Accordingly, there has been interest in treating small
abscesses with systemic antibiotics alone.
In a retrospective study of 22 patients with 23 diverticular abscesses less than 3 cm in
diameter and no evidence of free air, all resolved the acute episode with antibiotic
treatment alone [20]. In subsequent follow up, 36 percent required eventual surgery, 41
percent reported full resolution of symptoms, and 23 percent had mild recurrent or
persistent symptoms that did not warrant surgery. Limitations of the study are the small
sample size and the lack of proof that these collections were actually abscesses as
opposed to localized reactive peritoneal fluid collections.
Patients with acute diverticulitis should be evaluated by a surgeon. CT scan findings
should be reviewed with the interventional radiologist to determine if any collections are
amenable to percutaneous drainage. Indications for immediate surgery include free
intraperitoneal air. A patient with a fluid collection less than 3 cm and not amenable to
catheter drainage can be treated with intravenous antibiotics and monitored carefully for
any worsening of symptoms. Patients with worsening symptoms require surgical
intervention.
For patients who are treated by CT-guided percutaneous drainage, surgery is indicated if
the drain does not improve symptoms in 24 to 48 hours. For patients successfully treated
by percutaneous drainage, a clinical assessment is performed to determine if the patient
can be safely observed, or if operative intervention is recommended. Read more
Feedback on your answer
Collapse
19)
0/1

You are seeing a 64-year old morbidly obese male in clinic for a pre operative evaluation
prior to a scheduled colectomy next week for a suspicious colon lesion that was found on
colonoscopy. His past medical history is significant for type II diabetes mellitus and
hypertension. He has no formal diagnosis of obstructive sleep apnea per history.
His exam shows a morbidly obese male in no acute distress. Blood pressure is 145/91,
heart rate is 78 beats/min, respiratory rate 14/min and oral temperature is 98.5 F.
Of the option below which of the following screening tests prior to surgery is
recommended for this patient?

( 1pts extra credit)

Chest X-ray

Obstructive sleep apnea

TSH

α-fetoprotein (AFP)

You didn't make any selections

The answeris B. Obstructive sleep apnea (OSA) has rapidly become the most prevalent
sleep disorder over the last 20+ years. At present, estimates suggest that it affects 9% of
women and 24% of men in the United States. Additionally, estimates also suggest that as
many as 82% of men and 92% of women affected do not carry the formal diagnosis of
OSA. Despite the large percentage of the population affected, no well-vetted specific
guidelines are present in the current literature. That being said, there is a wealth of
discussion of the issue from which several more general guidelines may be drawn.
Preoperative screening for OSA is the first and most critical step in management. As a
majority of affected patients have not been formally diagnosed, this allows the operative
team the opportunity to detect the greatest number of patients at risk and to adjust their
operative plan of care accordingly. Most commonly, screening is done by anesthesiology
in the preanesthesia setting using one of a number of available screening tools. In
addition to this, it is important for the surgeon to also consider the potential presence of
OSA as this may significantly affect the choice of surgical setting and postoperative
setting planned.53 Although, again, there are no set guidelines provided in the literature
dictating criteria for outpatient versus inpatient surgical settings, for patients with known
or suspected OSA requiring significant sedation or general anesthesia, using a setting
with access to difficult airway carts, advanced respiratory care, and the option of a
prolonged postanesthesia recovery unit (PACU) observation is recommended.54
The two greatest issues in postoperative management are the level of monitoring
provided and the use of sedatives and narcotic analgesics. The choice of
surgical/postoperative setting is best made after consideration of known or potential
OSA, any associated comorbidities, and the type of surgery to be performed. Patients
with severe OSA generally require inpatient monitoring postoperatively. Outside of
known severe OSA, the time observed in the PACU may also provide further information
on which to base ongoing care or discharge.
51 Any patient who currently uses a home continuous positive airway pressure (CPAP)
or bilevel positive airway pressure (BiPAP) device should use this in the immediate
postoperative period.55 In the inpatient setting, the use of continuous pulse oximetry and
telemetry should be considered, and supplemental oxygen should be used with
caution.53,55
Anesthetics and sedatives depress the central nervous system, inhibit respiration, depress
skeletal muscle tone, and relax the upper airway.
Opiate drugs can depress respiratory drive and rate, decrease tidal volume, and lead to
decreased sensitivity to arousal. Thus, the use of sedatives postoperatively should be
avoided or used with great caution. Narcotic agents need to be titrated to adequate pain
relief but should be used only when nonnarcotic agents are ineffective. Additionally, the
need for opiates may be significantly decreased with the use of nonnarcotic agents such
as acetaminophen, centrally acting agents such as tramadol hydrochloride, NSAIDs, or
topical anesthetics when appropriate.53
The prevalence of OSA is only likely to increase given current trends in associated
comorbidities such as obesity. Further prospective data are needed for more definitive
future recommendations, particularly regarding postoperative monitoring and admission
status. Read more

Feedback on your answer


Collapse
20)
0/1
What classic sign is described as right lower quadrant pain when the left lower quadrant
is palpated?
(Select 1)( 1pts extra credit)

Murphy's Sign

Rovsing's Sign

Obturator Sign

Iliopsoas Sign

You didn't make any selections


The answer is B. Rovsing's Sign is described as RLQ pain when the LLQ is percussed in
acute appendicitis

Murphy's Sign is described as inspiratory arrest while continuous pressure is maintained


in the RUQ indicative of acute cholecystitis.

Obturator Sign is described when the patient develops hypogastric pain when the hip and
knee is flexed and this hip is internally and externally rotated. This sign reflects
appendiceal inflammation adjacent to the internal obturator muscle/

Iliopsoas Sign is described as pain with hyperextension of the right leg while the patient
is lying on his left side while the knee is flexed. This is more commonly seen with
retrocecal appendicitis. Read More
Feedback on your answer
Collapse
Unit 3
21)
0/1
You are seeing a 24-year old male who presents with fever, right lower quadrant pain
(RLQ), and vomiting over the last 12 hours. On exam he has severe RLQ tenderness
without rebound. His current oral temp is 102.1 F, heart rate 110 beats/min, RR 16, blood
pressure 134/90. His WBC comes back at 18,000mm3 with 10% bands. What is the most
common underlying cause of the disease process in this patient?
(Select 1)(1pts)

Fecalith

Lymphoid Hyperplasia

Foreign Bodies

Tumors
You didn't make any selections

The answer is B. Lymphoid Hyperplasia. The most common causes of appendicits are as
follows Lymphoid Hyperplasia 60% , Fecalith 35% , Foreign Bodies 4%,
Tumors 1% The incidence of appendicitis gradually rises from birth, peaks in the late
teen years, and gradually declines in the geriatric years. The mean age when appendicitis
occurs in the pediatric population is 6-10 years. Lymphoid hyperplasia is observed more
often among infants and adults and is responsible for the increased incidence of
appendicitis in these age groups. Younger children have a higher rate of perforation, with
reported rates of 50-85%. The median age at appendectomy is 22 years. Although rare,
neonatal and even prenatal appendicitis have been reported. Clinicians must maintain a
high index of suspicion in all age groups. Read more from Medscape

Feedback on your answer


Collapse
22)
0/1
You are seeing a 31-year old male who presents with 24 hours of progressively
worsening lower abdominal pain, vomiting and chills. He denies past medical history and
denies prior surgery.

On exam his heart rate is 95 bpm, BP is 129/80, oral temp 100.2 F and O2 sat is 100% on
room air. Upon palpation he has right lower quadrant pain when you palpated his left
lower quadrant. His labs show a WBC of 9,700mm3 with no bands.

True or False: A normal white blood cell count effectively rules out appendicitis in this
patient.
(1pts)
You didn't answer this question
False: Appropriate diagnostic testing varies based on the clinical situation. A complete
blood count is appropriate if infection or blood loss is suspected. One study of patients 15
to 83 years of age with suspected appendicitis found that a white blood cell count greater
than 10,000 per mm3 (10 × 10 9 per L) was 77 percent sensitive and 63 percent specific
for the diagnosis (LR+ = 2.1, LR– = 0.37). Thus, nearly one in four patients with
appendicitis does not have an elevated white blood cell count. Read More
Feedback on your answer
Collapse
23)
0/1
Treatment for all patients with acute nonperforated appendicitis must include:

(Select 1)(1pts)

Witholding analgesia
Immediate antibiotics

Blood cultures

Surgery within 12 hours

All of the above

You didn't make any selections


The answer is B, Immediate antibiotics. The treatment of acute appendicitis in the ED
consists of 3 major aspects: 1) supportive care, 2) antibiotic therapy, and 3) definitive
treatment as decided upon by a surgical consultation. Supportive care is to be tailored to
the patient’s symptoms. Most often, patients with appendicitis should receive IV fluids
(lactated Ringer or normal saline), should receive symptomatic treatment (eg, antiemetics
and analgesia), and should remain NPO. Choice of analgesia will depend on the personal
preference of the provider; however, IV opiates are often considered first-line treatment
in possible appendicitis cases.120 Multiple prospective randomized trials and a Cochrane
Database review on analgesia in patients with acute abdominal pain have all concluded
that opiates improve patients’ cooperation, relieve their pain, and do not mask the
physical signs of appendicitis. Unless contraindicated, it is unacceptable to withhold
analgesia from patients presenting with acute abdominal pain.121-124 In the rare case of
acute appendicitis causing septic shock, treatment is guided by the Surviving Sepsis
Campaign International Guidelines of Severe Sepsis and Septic Shock.
Administer antibiotics promptly upon making the diagnosis of appendicitis.126 A
Cochrane database review of 45 studies involving 9576 patients found that antibiotic
administration in appendicitis reduces the incidence of both postoperative wound
infection and intra-abdominal abscess formation when compared to placebo.127
Interestingly, these studies demonstrated no difference in the postoperative histologic
findings of appendicitis in the antibiotic versus placebo groups.
Recommendations on definitive management for acute appendicitis depend on the
presence or absence of complications. Treatment for nonperforated appendicitis with a
well-circumscribed abscess is nonoperative and consists of IV antibiotics and
percutaneous drainage. The management recommendations for nonperforated
appendicitis without abscess or perforated appendicitis is urgent operative intervention.
Conservative management for appendicitis without abscess has become a controversial
topic recently. Conservative therapy consists of hospitalization for IV antibiotics rather
than emergent surgical intervention. Although Dr. Coldrey published a protocol in 1956
involving exclusive antibiotic treatment of appendicitis lasting more than 24 hours with
great personal success, only in the last 20 years has the idea of conservative treatment
been highlighted by multiple prospective trials and meta-analyses. The benefit of
conservative treatment is rooted in the fact that there is significant perioperative
morbidity associated with an appendectomy. 28,135 On average, the rate of
complications from appendectomies is approximately 4.6% and includes small bowel
obstruction, wound infection, abscess formation, and adhesions. Read more
Feedback on your answer
Collapse
24)
0/1
You are seeing a 32-year-old white female who is currently 16 weeks'
pregnant and is presenting with right lower quadrant pain that started
abruptly last night along with nausea, vomiting, and a low grade fever.
Which one of the following imaging studies would be most appropriate
for initial evaluation of this patient?
(Select 1)(1pts)

CT of the abdomen

MRI of the abdomen

Ultrasonography of the abdomen and Right lower quadrant

A small bowel series

You didn't make any selections


The answer is C, Ultrasonography of the abdomen and Right lower quadrant. CT has
demonstrated superiority over transabdominal ultrasonography for identifying
appendicitis, associated abscess, and alternative diagnoses. However, ultrasonography is
indicated for the evaluation of women who are pregnant and women in whom there is a
high degree of suspicion for gynecologic disease. Ref: Paulson EK, Kalady MF, Pappas
TN: Suspected appendicitis. N Engl J Med 2003;348(3):236-241.
Feedback on your answer
Collapse
25)
0/1
Neoplastic lesions of the appendix are found in as many as 5% of specimens obtained
with routine appendectomy for acute appendicitis. Most are benign. Preoperative
detection of such conditions is rare, and intraoperative diagnosis is made in fewer than
50% of cases. Which of the following statement regarding neoplastic lesions of the
appendix is true?
(Select 1)(1pts)

Metastasis to the appendix is common

Adenocarcinoma is the most common type represting >80%

The incidence of malignancy in the appendix is 1.35%.


If the tumor is less than 8 cm in diameter, is located within the body or
the tip of the appendix, and has not metastasized, appendectomy is the
treatment of choice.

All of the above

None of the above

You didn't make any selections


Answer C is the only true statement. Neoplastic lesions of the appendix are found in as
many as 5% of specimens obtained with routine appendectomy for acute appendicitis.18-
21 Most are benign. Preoperative detection of such conditions is rare, and intraoperative
diagnosis is made in fewer than 50% of cases. Appendectomy alone may be curative for
appendiceal mucocele, localized pseudomyxoma peritonei, most appendiceal carcinoids,
and other benign tumors. Definitive management of an appendiceal mass unexpectedly
encountered during exploration for clinically suspected acute appendicitis depends on
whether the tumor is carcinoid, its size and location, the presence or absence of metastatic
disease, and histologic and immunohistochemical findings.
Benign neoplasms of the appendix include mucosal hyper-plasia or metaplasia,
leiomyomas, neuromas, lipomas, angiomas, and other rare lesions. Appendiceal
adenomas tend to be diffuse and to have a predominant villous character. Mucus-
producing cystadenomas predispose to appendiceal mucocele, sometimes accompanied
by localized pseudomyxoma peritonei. These lesions are rarely symptomatic and are
often encountered incidentally during operation; however, they may also be clinically
manifested as acute appendicitis, torsion, intussusception, ureteral obstruction, or another
acute condition. If the base of the appendix is free of disease, appendectomy alone is
sufficient treatment.
Malignant tumors of the appendix primarily consist of carcinoids and adenocarcinomas;
all together, they account for 0.5% of all gastrointestinal malignancies. The incidence of
malignancy in the appendix is 1.35%. Metastasis to the appendix is rare. Carcinoids are
substantially more common than adenocarcinomas in the appendix: as many as 80% of
all appendiceal masses are carcinoid tumors. Overall, carcinoid tumors are found in 0.5%
of all appendiceal specimens, and appendiceal carcinoid tumors account for 18.9% of all
carcinoid lesions.23 These tumors are predominantly of neural cellular origin and have a
better prognosis than all other intestinal carcinoid tumors, which typically are of mucosal
cellular origin. If the tumor is less than 2 cm in diameter, is located within the body or the
tip of the appendix, and has not metastasized, appendectomy is the treatment of choice. If
the lesion is at the base of the appendix, is larger than 2 cm in diameter, or has
metastasized, right hemicolectomy is indicated. In addition, secondary right
hemicolectomy is indicated if the tumor is invasive, if mucin production is noted, or if the
tumor is found to be of mucosal cellular origin at the final pathologic examination.24,25
Patients with metastatic appendiceal carcinoid tumors appear to have a far better
prognosis than those with other types of metastatic cancers.24 Therefore, hepatic
debulking for symptomatic control is indicated and justified in cases of liver metastasis.
Figure 8. Laparoscopic appendectomy. The mesoappendix having been divided (a), the
base of the appendix is cleared circumferentially and divided with a gastrointestinal
anastomosis stapler (b). Read more

Unit 4
1)
0/1
You are seeing a 49 -ear old obese male who presents with two days of worsening left
sided abdominal pain. He denies fever or chills but does note painful bowel movements
that have streaks of blood. On exam you note moderate left lower quadrant tenderness to
palpation without rebound or guarding. His labs show a white blood cell count of
14,000mm3 and his CT scan of his abdomen/pelvis shows acute uncomplicated sigmoid
diverticulitis. Nonoperative treatment with oral antibiotics will resolve acute
uncomplicated diverticulitis in what percent of patients?
(Select 1)(1pts)

<25%

50%

80%

>95%

You didn't make any selections


The best answer is B, 80%. Although most patients with acute diverticulitis can be treated
medically, approximately 15-20% will require surgery. Uncomplicated diverticulitis may
be managed as an outpatient (dietary modification and oral antibiotics) for those without
appreciable fever, excessive vomiting, or marked peritonitis, as long as there is the
opportunity for follow-up. The patient should be able to take liquids and antibiotics by
mouth. Hospitalization for treatment (dietary modification and intravenous antibiotics) is
usually best if the above conditions are not met, or if the patient fails to improve with
outpatient therapy. Antibiotics should be selected to treat the most common bacteria
found in the colon: gram-negative rods and anaerobic bacteria. Single and multiple
antibiotic regimens are equally effective, as long as both groups of organisms are
covered. Nonoperative treatment will resolve acute diverticulitis in 80
percent of patients, but approximately one-third will have a recurrent attack often
within one year. approximately 20% of patients with diverticulitis require surgical
treatment.30 Most surgical procedures are reserved for patients who experience recurrent
episodes of acute diverticulitis that necessitate treatment (inpatient or out-patient) or who
have complicated diverticulitis. The most common indication for elective resection is
recurrent attacks—that is, several episodes of acute diverticulitis documented by studies
such as CT. Rerecurrences may be more common than recurrences. In 2000, a task force
of the American Society of Colon and Rectal Surgeons recommended sigmoid resection
after two attacks of diverticulitis. A subsequent cost analysis using a Markov model
suggested that cost savings could be achieved if resection was done after three
attacks.33 There is a growing tendency to question arbitrary guidelines for surgical
management of recurrent attacks, with the exception of certain groups, such as
immunocompromised patients. Current practice guidelines state that the recommendation
to perform elective sigmoid resection after recovery from uncomplicated acute
diverticulitis should be made on a case-by-case basis. The decision-making process
should be influenced by the age and medical condition of the patient, the frequency and
severity of attacks, and the presence of symptoms after the acute attack. Elective
resection is generally recommended after an episode of complicated
diverticulitis.36 Efforts are made to time surgical treatment so that it takes place during a
quiescent period 8 to 10 weeks after the last attack. Barium enema or colonoscopy may
be employed to evaluate the diverticular disease and rule out carcinoma. The bowel can
then be prepared mechanically and with antibiotics (e.g., oral neomycin and
metronidazole on the day before operation).
2014. Scientific American Surgery. Hamilton, Ontario & Philadelphia, PA. Decker
Intellectual Properties Inc. ISSN 2368-2744. STAT!Ref Online Electronic Medical
Library. http://online.statref.com/Document.aspx?fxId=61&docId=1368. 10/20/2014
12:41:34 PM CDT (UTC -05:00).

Feedback on your answer


Collapse
2)
0/1
You are seeing a 64-year old white male who is presenting with left lower abdominal
pain since last night. His WBC is 14,000mm3 and his CT scan of the abdomen and pelvis
of the abdomen/pelvis shows acute uncomplicated sigmoid diverticulitis. What bacteria is
usually the target in the treatment of acute diverticulitis?
(Select 1)(1pts)

Gram positive cocci

Gram positive rods

Gram negative cocci

Gram negative rods

You didn't make any selections

The answer is D, gram negative Rods. The most common choices would be either
augmentin as single coverage or cipro plus flagyl. Conservative treatment of acute
uncomplicated diverticulitis is successful in 70 to 100 percent of patients. Uncomplicated
diverticulitis may be managed as an outpatient (dietary modification and oral antibiotics)
for those without appreciable fever, excessive vomiting, or marked peritonitis, as long as
there is the opportunity for follow-up. The patient should be able to take liquids and
antibiotics by mouth. Hospitalization for treatment (dietary modification and intravenous
antibiotics) is usually best if the above conditions are not met, or if the patient fails to
improve with outpatient therapy. Antibiotics should be selected to treat the most common
bacteria found in the colon: gram-negative rods and anaerobic bacteria. 13 Single and
multiple antibiotic regimens are equally effective, as long as both groups of organisms
are covered. Nonoperative treatment will resolve acute diverticulitis in 85 percent of
patients, but approximately one-third will have a recurrent attack often within one
year. Read More

Feedback on your answer


Collapse
3)
0/1
You are seeing a 58-year-old female who is presenting with two days of a low-grade
fever and increasing abdominal cramps that are exacerbated by bowel movements. She
began a course of augmentin 2 days ago that her PCP gave her but has had no relief of
her symptoms and is getting worse. She has not had nausea, vomiting, urinary symptoms,
or bloody stools. She has a past medical history of hypertension.

Her vitals show a temperature is 100.6°F, pulse is 75/min, and blood pressure is 150/80
mm Hg. Her lungs are clear to auscultation. Cardiac examination shows no
abnormalities. Her abdomen is soft, and there is tenderness to palpation of the left lower
quadrant with guarding but no rebound. Bowel sounds are normal. Her stool is brown,
and test for occult blood is negative. Her hemoglobin concentration is 14.5 g/dL,
leukocyte count is 15,800/mm3, and platelet count is 280,000/mm3; serum studies and
urinalysis show no abnormalities. Which of the following is the most appropriate next
step in diagnosis?
(Select 1)(1pts)

Examination of the stool for ova and parasites

Test of the stool for Clostridium difficile toxin

Xray Abdomen Series

CT scan of the abdomen/pelvis with IV and oral contrast

You didn't make any selections


The answer is D, CT scan of the abdomen with IV and oral contrast. This patient is being
treated for diverticulitis and is not getting better. She needs a CT scan to evaluate for a
possible abscess. Read more
Feedback on your answer
Collapse
4)
0/1
You are seeing a 45-year old obese male with type II diabetes who presents with acute
onset left lower quadrant abdominal pain, vomiting, pain with defecation, and
fever. Physical exam is normal except for his abdominal exam that shows left lower
quadrant tenderness without signs of peritonitis. His labs are remarkable for a WBC of
16,000 mm3. Which of the following choices is most likely to reduce repeat incidences of
this patients condition?
(Select 1)(1pts)

Routine daily stool softners or laxatives

Biannual colonoscopy

High fiber diet

Avoidance of nuts and legumes

You didn't make any selections


The answer is C, High fiber diet. Environmental and lifestyle factors are important risk
factors for diverticular disease. Out of the following options only increasing fiber intake
has been shown to reduce episodes of diverticulitis. The role of fiber in the development
of diverticulosis is unclear. Several early studies suggested that low dietary fiber
predisposes to the development of diverticular disease, but other studies have been
conflicting [3,30-39]. Fiber also does not reduce symptoms in patients with
symptomatic uncomplicated diverticular disease [37,38,40,41]. However, dietary fiber
and a vegetarian diet may reduce the incidence of symptomatic diverticular disease by
decreasing intestinal inflammation and altering the intestinal microbiota [4,31,42].
In a cohort study that included over 47,000 men, after adjustment for age, energy-
adjusted total fat intake, and physical activity, total dietary fiber intake was noted to be
inversely associated with the risk of symptomatic diverticular disease (RR 0.58 highest
quintile versus lowest quintile for fiber intake) [31].
Fat and red meat — In the same cohort study, the risk of diverticular disease was
significantly increased with diets that were low in fiber and were high in total fat or red
meat as compared with diets that were low in both fiber and total fat or red meat (RR
2.35 and 3.32, respectively) [31].
Seeds and nuts — Nut, corn, and popcorn consumption arenot associated with an
increase in risk of diverticulosis, diverticulitis or diverticular bleeding. In a large
observational study (The Health Professionals Follow-up Study) that included 47,228
men between the ages of 40 and 75 years, there was an inverse association between the
amount of nut and popcorn consumption and the risk of diverticulitis (HR nuts 0.8, 95%
CI 0.63-1.01; HR popcorn 0.72, 95% CI 0.56-0.92) [43]. In addition, no association was
found between consumption of corn and diverticulitis or between nut, popcorn, or corn
consumption and diverticular bleeding or uncomplicated diverticulosis.
Physical activity — It is unclear if lack of vigorous exercise is a risk factor for
diverticular disease. However, vigorous physical activity appears to reduce the risk of
diverticulitis and diverticular bleeding. In a prospective study of approximately 48,000
men aged 40 to 75 who were free of known colonic disease at baseline, the risk of
developing symptomatic diverticular disease was inversely related to overall physical
activity (RR 0.63 for highest versus lowest extremes) after adjustment for age and dietary
fat and fiber [44]. Most of the decrease in risk with exercise was associated with
vigorous activity such as jogging and running. Men in the lowest quintile for both dietary
fiber and physical activity had an increased risk of symptomatic diverticular disease as
compared with men in the highest quintile for both (RR 2.56, 95% CI 1.36-4.82).
Obesity — Obesity has been associated with an increase in risk of both diverticulitis
and diverticular bleeding. In a large, prospective cohort study of 47,228 male health
professionals, there were 801 incident cases of diverticulitis and 383 cases of diverticular
bleeding during 18 years of follow-up [45]. The risk of diverticulitis and diverticular
bleeding was significantly higher in those with the highest quintile of waist
circumference as compared with the lowest (RR diverticulitis 1.56, 95% CI 1.18-2.07;
RR diverticular bleeding 1.96, 95% CI 1.30-2.97).
Other — Current smokers appear to be at increased risk for perforated diverticulitis and
a diverticular abscess as compared with nonsmokers (OR 1.89, 95% CI 1.15-3.10) [46].
Caffeine and alcohol are not associated with an increased risk for symptomatic
diverticular disease. Read more
Feedback on your answer
Collapse
5)
0/1
You are seeing a 64-year old female who is presenting with painless bright red rectal
bleeding for the last two days. She notes bright red blood coating the stool and on the
toilet paper when she wipes. He vitals are normal and her physical exam shows a soft non
tender abdomen. Stool is brown with red streaks. What is the most likely cause of
bleeding in this patient and the most common overall cause of lower GI bleeding in
adults?
(Select 1)(1pts)

Inflammatory Bowel Disease

Diverticulosis

Neoplasm
Angiodysplasia

You didn't make any selections

The answer is B. Diverticulosis results in 60% of cases of lower GI bleeding followed by


IBD(13%), benign anorectal disease (11%), neoplasia (10%) and angiodysplasia (<5%).
The causes of lower gastrointestinal bleeding (LGIB) may be grouped into several
categories (table 1): From Uptodate
 Anatomic (diverticulosis)
 Vascular (angiodysplasia, ischemic, radiation-induced)
 Inflammatory (inflammatory bowel disease, infectious)
 Neoplastic
In most series, diverticulosis is the most common source of LGIB, accounting for
approximately 15 to 55 percent of cases [9,10]. Angiodysplasia may be the most
frequent cause in patients over the age of 65 years [11,12], though more recent data
suggest that angiodysplasia may be a less common cause of LGIB than once thought.
Hemorrhoids are the most common cause of rectal bleeding in patients under the age of
50 years [13]. However, hemorrhoidal bleeding is usually minor. Similarly, bloody
diarrhea due to inflammatory causes can sometimes be distinguished from other causes of
LGIB because of the clinical setting. In general, anatomic and vascular causes of
bleeding present with painless, large-volume blood loss, whereas inflammatory sources
are associated with diarrhea and abdominal pain [14].
In a review of several large studies that included 1559 patients with acute hematochezia,
the following bleeding sources were identified [10]:
 Diverticulosis – 5 to 42 percent
 Ischemia – 6 to 18 percent
 Anorectal (hemorrhoids, anal fissures, rectal ulcers) – 6 to 16 percent
 Neoplasia (polyps and cancers) – 3 to 11 percent
 Angiodysplasia – 0 to 3 percent
 Postpolypectomy – 0 to 13 percent
 Inflammatory bowel disease – 2 to 4 percent
 Radiation colitis – 1 to 3 percent
 Other colitis (infectious, antibiotic associated, colitis of unclear etiology)
– 3 to 29 percent
 Small bowel/upper GI bleed – 3 to 13 percent
 Other causes – 1 to 9 percent
 Unknown cause – 6 to 23 percent
The vast majority of colonic diverticula are actually false diverticula (pseudodiverticula)
that contain only serosa and mucosa. They occur at weak points in the colonic wall where
the vasa recta penetrate the muscularis to supply the mucosa; as the diverticulum
expands, these vessels are displaced. A 1976 anatomic study of colonic specimens from
patients with diverticular bleeding used angiography to demonstrate that in all cases, the
vasa recta overlying the diverticulum ruptured into the lumen of the diverticulum, not
into the peritoneum.100

It has been estimated that approximately 17% of patients with colonic diverticulosis
experience bleeding, which may range from minor to severe and life-threatening.
Endoscopic treatment of diverticular hemorrhage can be difficult because of the high
bleeding rate and the location of the bleeding point within the diverticulum. In 2000, one
group of investigators reported their experience with endoscopic therapy for severe
hematochezia and diverticulosis in a prospective series of 121 patients.101 In this series,
none of the patients treated endoscopically with epinephrine injections, bipolar
coagulation, or both required surgery and none experienced recurrent bleeding episodes.
A 2001 study from another group, however, reported high rates of recurrent bleeding
episodes in both the early and the late posttreatment periods.102 In the absence of
prospective, randomized trials, it is difficult to draw definitive conclusions about the
utility of endoscopic therapy in treating diverticular hemorrhage.

Fortunately, most diverticular hemorrhages stop spontaneously. In one series, surgery


was unlikely to be necessary if fewer than four units of packed RBCs were transfused in a
24-hour period, whereas 60% of patients receiving more than four units of packed RBCs
in a 24-hour period required surgical intervention.103 Semielective surgical therapy is
usually offered after a second diverticular bleeding episode because once a second such
episode has occurred, the risk that a third will follow exceeds 50%. In a series of 83
conservatively managed cases of diverticular disease, the predicted yearly recurrence
rates were 9% at 1 year, 10% at 2 years, 19% at 3 years, and 25% at 4 years.104

In general, patients who require more than four units of blood in a 24-hour period to
remain hemodynamically stable, who have not stopped bleeding after 72 hours, or who
experience rebleeding within 1 week after an initial episode should undergo surgery.

2014. Scientific American Surgery. Hamilton, Ontario & Philadelphia, PA. Decker
Intellectual Properties Inc. ISSN 2368-2744. STAT!Ref Online Electronic Medical
Library. http://online.statref.com/Document.aspx?fxId=61&docId=848. 10/22/2014
7:03:07 PM CDT (UTC -05:00).

Feedback on your answer


Collapse
Unit 2
6)
0/1
Which of the following statements regarding the pre-operative care of patients is false?
(Select 1)(1pts)

When a preoperative antibiotic is indicated, a single dose of therapeutic


strength, administered shortly before incision, usually suffices
Cooling patients during colorectal surgery have been shown to reduce
infection rates

Supplemental perioperative oxygen (i.e., an FIO2 of 80% instead of 30%)


significantly overcomes the decrease in phagocytosis and bacterial killing
usually associated with anesthesia and surgery (and significantly reduces
postoperative nausea and vomiting)

Intraoperative transfusion of packed red blood cells was an independent


risk factor for perioperative infection

You didn't make any selections


Answer B is the only false statement. A 1996 study showed that warming patients during
colorectal surgery reduced infection rates.173 This finding was confirmed in a
subsequent observational cohort study that reported a significantly increased incidence of
SSI with hypothermia.174 A randomized controlled trial of 421 patients, published in
2001, determined that warming patients before short-duration clean procedures (breast,
varicose vein, or hernia) reduced infection (p = .001) and reduced wound scores (p =
.007) in this setting as well.175 The safest and most effective way of protecting patients
from hypothermia is to use forced-air warmers with specialized blankets placed over the
upper or lower body. Alternatives include placing a warming water mattress under the
patient and draping the patient with an aluminized blanket. Second-line therapy involves
warming all IV fluids. Any irrigation fluids used in a surgical procedure should be at or
slightly above body temperature before use. Radiant heating devices placed above the
operative field may be especially useful during operations on infants. Use of a warmer on
the inhalation side of the anesthetic gas circuit can also help maintain the patient's body
temperature during an operation.
The association between blood transfusion and increased perioperative infection rates is
well documented. In a 1997 study, geriatric hip fracture patients undergoing surgical
repair who received blood transfusion had significantly higher rates of perioperative
infection than those who did not (27% versus 15%), and this effect was confirmed on
multivariate analysis.177 Another study yielded similar findings in colorectal cancer: the
relative risk of infection was 1.6 for transfusion of one to three units of blood and 3.6 for
transfusion of more than three units.178 A large prospective cohort study published in
2002 evaluated the association between anemia, blood transfusion, and perioperative
infection.179 Regression analysis confirmed that intraoperative transfusion of packed red
blood cells was an independent risk factor for perioperative infection (odds ratio 1.06;
95% confidence interval 1.01 to 1.11; p > .0001). Furthermore, transfusion of more than
four units of packed red blood cells was associated with a ninefold increased risk of
infection (95% confidence interval 5.74 to 15.00; p < .0001).
In brief, the principles of optimal surgical antimicrobial prophylaxis include (1)
appropriate choice of an antimicrobial agent, (2) proper timing of antibiotic
administration before incision, and (3) limited duration of antibiotic administration after
operation. SSIs are established several hours after contamination.161 There is a well-
recognized "golden period" during which prophylactic antibiotics can be effective.
Administration of antibiotics before contamination reduces the risk of infection but
subsequently proves to be of little prophylactic value.162 The selective use of short-
duration, narrow-spectrum antibiotic agents chosen to reflect (1) the patient's individual
risk factors and (2) the recognized epidemiology of SSI, modified for local conditions
and experience, should be considered where deemed appropriate [see Table 9 for the
usual pathogens isolated from SSIs].163 When a preoperative antibiotic is indicated, a
single dose of therapeutic strength, administered shortly before incision, usually
suffices.164 The routine dose may have to be increased to achieve adequate serum levels
if the patient is morbidly obese.165 A second dose is indicated if the procedure is longer
than two half-lives of the drug or if extensive blood loss occurs. Continuation of
prophylaxis beyond 24 hours is not recommended. The provision of prophylactic
antibiotics, where indicated, has been accepted as a measure of surgical quality aimed at a
reduction in SSIs.166,167 A strong economic case has been made for infection control,
including prophylactic antibiotics, benefiting the hospital and the patient.168 The
perioperative nursing team has a central role, together with the anesthesiologist, in
monitoring the administration of preoperative prophylactic antibiotics.
Destruction by oxidation or by oxidative killing is the body's most important defense
against surgical pathogens. This defensive response depends on adequate levels of
oxygen tension in contaminated tissue. An easy method of improving oxygen tension in
adequately perfused tissue is to increase the FiO2. Supplemental perioperative oxygen
(i.e., an FIO2 of 80% instead of 30%) significantly overcomes the decrease in
phagocytosis and bacterial killing usually associated with anesthesia and surgery (and
significantly reduces postoperative nausea and vomiting). Low oxygen tension in wound
tissue has been found to be a good predictor of SSI risk.176 With that said, raised oxygen
tension is also an important risk factor in the outbreak of surgical fires. A balance must
be achieved between the adequacy of PO2 in the tissue and the restriction of O2 flow in
head and neck surgery and in other high-risk fire settings. Read more
Feedback on your answer
Collapse
7)
0/1
Where is the surgical amphitheaters known as "The Ether Dome" located?
(Select 1)(1pts)

Beaumont Hospital in Dublin, Ireland

Guy's Hospital in London, England

Massachusetts General Hospital

Padua Hospital in Padua, Italy

You didn't make any selections


The answer is C, Massachusetts General Hospital. The configuration and design of the
modern OR were derived from the anatomic amphitheater of late-Renaissance Italy.
Following the publication of Andreas Vesalius's De Fabrica Humani Corporis in 1543,
the study of human anatomy and dissection emerged as central elements in medical and
particularly in surgical education. This innovation reflected an estrangement from the
previous galenic curriculum, the beginnings of the rejection of scholasticism as the
dominating heuristic in medical investigation, and a sentinel movement toward
empiricism and what we now recognize as the beginnings of modern science. The first
documented dissecting amphitheater, constructed after the classical Greek style of tiered
seats or stands surrounding the stage, was built in Padua in the mid-16th century.1 Others
in Europe soon followed. Although early surgeons generally operated in the patient's
home or on the battlefield, and in hospitals only infrequently, by the first decade of the
18th century, the anatomic amphitheater could serve as an operating theater. In 1707,
Pierre Dionis, a Parisian anatomist and surgeon, included an etching of his surgical
amphitheater in his textbook of surgery. In England, at about the same time, the new
hospitals built to replace the Catholic institutions destroyed by Henry VIII and serve the
needs of the growing urban areas included operating theaters. Guy's Hospital built its
theater on the top floor so that it could be lit by skylights.2-4 A magnificently preserved
example of premodern design can be seen in Dublin, at the Royal College of Surgeons, in
the anatomic building and dissecting rooms that remain in use today. The effective
deployment of natural lighting is remarkable and instructive. American examples of 19th
century surgical amphitheaters also survive. The Ether Dome at the Massachusetts
General Hospital, which derived its name from the effective demonstration of ether as an
anesthetic, may be the best known, but there are others of equal historical
significance. Read more
Feedback on your answer
Collapse
8)
0/1
With regards to positioning patients in the operating room, which of the following
statements if FALSE?
(Select 1)(1pts)

The lithotomy position is used in approximately 9% of cases and is the


next most commonly used position

Approximately 80% of surgical procedures are performed with patients in


the supine position.

Two major preventable consequences of malpositioning are neuropathy or


plexopathy and skin burns or ulceration.

Ulnar neuropathy and brachial plexopathy are the two most common
complications in this position and constitute around 90% of claims.

Damage to the obturator, sciatic, lateral femoral cutaneous, and peroneal


nerves from being placed in lithotomy position account for 5% of medical
liability claims for nerve damage

You didn't make any selections


Answer D is the only false statement as Ulnar neuropathy and brachial plexopathy are
the two most common complications in this position and constitute 28% and 20% of
claims, respectively. The risks associated with malpositioning deserve further comment.
The skin over any bony prominence is subject to injury from excessive pressure. The
fragile skin of infants and older patients may be injured by dragging rather than lifting
into position. The American Society of Anesthesiologists (ASA) practice advisory on the
prevention of perioperative peripheral neuropathies recommends that when practical,
patients should be placed in the intended position before induction of anesthesia to test
for comfort.78 Uncomfortable positions should be modified. It is essential to listen to the
patient. This is especially important with older patients or patients with degenerative
spine and joint disease or skeletal instability. Approximately 80% of surgical procedures
are performed with patients in the supine position. Two major preventable consequences
of malpositioning are neuropathy or plexopathy and skin burns or ulceration. Ulnar
neuropathy and brachial plexopathy are the two most common complications in this
position and constitute 28% and 20% of claims, respectively, in recent closed claims
studies.79-84 Padding of the precondylar groove of the humerus and tucking of the arms
or, at the least, restriction of abduction to less than 90° help prevent this problem.
The lithotomy position is used in approximately 9% of cases and is the next most
commonly used position.80,82 Damage to the obturator, sciatic, lateral femoral
cutaneous, and peroneal nerves has been observed. These injuries account for 5% of
medical liability claims for nerve damage in the Closed Claims Data Base. They
represent a small but not insignificant risk.79 Other patient positions have also been
associated with malpositioning injuries. As would be expected, longer procedures in
sicker or mal-nourished patients are more likely to result in complications of
positioning. Read more
Feedback on your answer
Collapse
9)
0/1
You are consulted to a 38-year-old black male for evaluation of his preoperative
cardiovascular risk. He has a 20-year history of type 1 diabetes mellitus and hypertension
and is undergoing preoperative evaluation for renal transplantation secondary to
uncontrolled hypertension and diabetic nephropathy. His current blood pressure is 142/85
mm Hg. He currently smokes one half of a pack of cigarettes daily. His
electrocardiogram is consistent with left ventricular hypertrophy and left atrial
enlargement. Which of the following is the most appropriate recommendation at this
time?
(1pts)

No further evaluation is needed, he is cleared for surgery without further


testing

Pharmacologic stress nuclear study


24-hour electrocardiographic holter monitor

Coronary angiography

You didn't make any selections

The answer is B, Pharmacologic stress nuclear study. In general, the indications for
noninvasive cardiac testing are the same for patients undergoing surgery as for those who
are not. All symptomatic patients should be evaluated for cardiac ischemia, usually with
cardiac stress testing, even if their symptoms are atypical. Patients undergoing a low- or
intermediate risk surgery who have at most minor clinical risk predictors (advanced age,
abnormal ECG, rhythm other than sinus, low functional capacity, history of stroke, or
uncontrolled hyper tension) may proceed to surgery without preoperative cardiac testing.
Likewise, patients with intermediate clinical risk predictors (mild angina, diabetes
mellitus, compensated heart failure, previous myocardial infarction, or renal
insufficiency) who have good exercise capacity also do not need further preoperative
cardiac testing before a low- or intermediate-risk procedure. However, in the absence of
high-quality data, some experts recommend that those with intermediate clinical
predictors who have poor functional status and are undergoing an intermediate-risk
procedure, as well as those with intermediate clinical predictors and good functional
status who are undergoing a high-risk procedure, should have noninvasive testing
performed before surgery.
Coronary revascularization is rarely needed just to get a patient through surgery, but it
may be warranted in selected patients with CAD undergoing noncardiac surgery if they
would have required the procedure anyway. However, the only large randomized trial on
the subject showed no benefit for revascularization (40). Avoid preoperative coronary
sten ting to avoid the risk for perioperative stent thrombosis. Patients with recently placed
coronary stents are at risk for perioperative stent thrombosis. For bare-metal stents, the
risk diminishes 4 to 6 weeks after implantation; for drug-eluting stents, an elevated risk
for rethrombosis persists for at least 12 months after placement because of delayed
endothelialization. Because these patients require antiplatelet treatment to mitigate this
risk, purely elective procedures should be delayed accordingly, and the interval without
antiplatelet therapy should be minimized when surgery is required within the window of
vulnerability. Read more

Feedback on your answer


Collapse
10)
0/1
A 72-year-old African-American male comes to your office for surgical clearance for an
elective left hemicolectomy for recurrent diverticulitis. His past medical history includes
an inferior-wall myocardial infarction approximately 12 months ago. Since then he had a
stress test that was normal 6 months ago. Currently, the patient feels well, walks while
playing nine holes of golf weekly, and is able to walk up a flight of stairs without chest
pain or significant dyspnea. Findings are normal on a physical examination.
Which one of the following would be most appropriate for this patient prior to surgery?
(1pts)

A 12-lead resting EKG

Another Nuclear Stress test

A dobutamine stress echocardiogram

A persantine stressed nuclear tracer study (technetium or thallium)

You didn't make any selections


The answer is A, A 12-lead resting EKG. The current recommendations from the
American College of Cardiology and the American Heart Association on preoperative
clearance for noncardiac surgery state that preoperative intervention is rarely needed to
lower surgical risk. Patients who are not currently experiencing unstable coronary
syndrome, severe valvular disease, uncompensated congestive heart failure, or a
significant arrhythmia are not considered at high risk, and should be evaluated for most
surgery primarily on the basis of their functional status. If these patients are capable of
moderate activity (greater than 4 METs) without cardiac symptoms, they can be cleared
with no stress testing or coronary angiography for an elective minor or intermediate-risk
operation such as the one this patient is to undergo. A resting 12-lead EKG is
recommended for males over 45, females over 55, and patients with diabetes, symptoms
of chest pain, or a previous history of cardiac disease. Ref: ACC/AHA guideline update
for perioperative cardiovascular evaluation for noncardiac surgery--Executive summary:
A report of the American College of Cardiology/American Heart Association Task Force
on Practice Guidelines (Committee to Update the 1996 Guidelines on Perioperative
Cardiovascular Evaluation for Noncardiac Surgery). Circulation 2002;105(10):1257-
1267. Read more here
Feedback on your answer
Collapse
Unit 1
11)
0/1
Abdominal pain is traditionally divided into 3 categories. What are the 2 basic causes of
abdominal pain of the 3 categories?

( 1pts extra credit)

Visceral and Autonomic

Visceral and Parietal


Parietal and Somatic

Referred and Somatic

You didn't make any selections


Abdominal pain is traditionally divided into 3 categories: visceral, parietal and
referred. In general visceral (autonomic) and parietal (somatic) are the 2 basic causes of
abdominal pain. Referred pain can be considered separately as a cortical misperception of
either visceral or parietal afferent stumuli. The two are often not discrete as visceral pain
often blends with parietal pain as a pathological process evolves. Tintinalli's Emergency
Medicine: A Comprehensive Study Guide, 7th edition, Tintinalli et al. Read more
on Acute Abdominal Pain Power Point PDF
Feedback on your answer
Collapse
12)
0/1
Which one of the following is most predictive of increased perioperative cardiovascular
events associated with noncardiac surgery in the elderly?

( 1pts extra credit)

Age > 80 years

Left bundle-branch block on ECG

Atrial fibrillation at a rate of 80 beats/min

Renal insufficiency (creatinine 2.9 mg/dL)

You didn't make any selections


The answer is D. Renal insufficiency (creatinine 2.9 mg/dL) is considered intermediate
risk, all other options are considered low risk. Clinical predictors of increased
perioperative cardiovascular risk for elderly patients include major risk factors such as
unstable coronary syndrome (acute or recent myocardial infarction, unstable angina),
decompensated congestive heart failure, significant arrhythmia (high-grade AV block,
symptomatic ventricular arrhythmia, supraventricular arrhythmias with uncontrolled
ventricular rate), and severe valvular disease. Intermediate predictors are mild angina,
previous myocardial infarction, compensated congestive heart failure, diabetes mellitus,
and renal insufficiency. Minor predictors are advanced age, an abnormal EKG, left
ventricular hypertrophy, left bundle-branch block, ST and T-wave abnormalities, rhythm
other than sinus, low functional capacity, history of stroke, and uncontrolled
hypertension. Ref: Schroeder BM: Updated guidelines for perioperative cardiovascular
evaluation for noncardiac surgery. Am Fam Physician 2002;66(6):1096-1107.
Feedback on your answer
Collapse
13)
0/1
You are seeing a 75-year old elderly male for a pre-operative consult for a hip
replacement after he fell breaking his right hip. The last time you saw him he weighed
about 150 lbs. He has no prior cardiac history but he does have terminal lung cancer and
needs palliative surgery but his recent fragility and weight loss has changed his risk. He
notes losing 30 lbs in the last 6 weeks and his last serum albumin was 2.4 g/dl. What is
his overall operative risk?

( 1pts extra credit)

Risk scores shall not apply to him given his cancer is terminal anyway

Low Risk as he does not have any cardiac history

Moderately high but can improve his nutrional status with TPN

Very high but can be improved with 7 days of intensive nutritional support
via his gut

You didn't make any selections


The answer is E. This patient has severe nutritional depletion which is identified by a loss
of 20% of body weight in a few months or a serum albumin less than 3 as well as a serum
transferrin less than 200mg/dl. This however does not preclude surgery as intensive gut
nutrition from 4-10 days (depending on time to surgery) can greatly improve nutritional
status. Read more on the Preoperative Evaluation
Feedback on your answer
Collapse
14)
0/1
Following abdominal surgery, how many days does a postoperative ileus usually last?

( 1pts extra credit)

1 day

3 days

6 days

9 days
You didn't make any selections

The answer is B. Ileus, or the cessation of bowel function and transit in the absence of
mechanical obstruction, is a common postoperative phenomenon particularly after
abdominal surgery and occurs in virtually all patients undergoing bowel resection. Its
causes are multifactorial, including an overall increase in sympathetic tone, the
inflammatory response to bowel as a result of direct manipulation, and the well-known
effects of narcotic analgesics on GI motility. Although the typical duration of
postoperative ileus averages around 3 days, in up to 32% of patients, return of bowel
function may be particularly delayed.Read more

Feedback on your answer


Collapse
15)
0/1
You are seeing a 74-year old male who is post op following colectomy for ischemic
colitis. He has a past medical history of hypertension and type II diabetes. During the
surgery he received 2 units of PRBCs and 1 L of normal saline. The nurses say that he
developed a fever of 101.5 F about 45 minutes after the surgery. His BP is 135/67, HR
90, RR 16. His abdominal wound looks intact without redness and his lungs are clear. He
has a foley catheter and a right subclavian triple lumen catheter that were both placed in
pre operative holding. What is the most likely source of his fever?

( 1pts extra credit)

Urinary tract infection

Pneumonia

Catheter associated infection

Wound infection

Transfusion reaction

Pulmonary embolism

Malignant hyperthermia

You didn't make any selections


The answer is E. The most common transfusion reaction is a febrile, nonhemolytic
transfusion reaction (FNHTR). The clinical manifestations of this reaction include fever,
often a chill, and sometimes mild dyspnea within one to six hours after transfusion of red
cells or platelets. FNHTRs are benign, causing no lasting sequelae, but are uncomfortable
and sometimes frightening to the patient. Furthermore, since fever, with or without a
chill, also may be the sign of a severe, acute hemolytic transfusion reaction (see 'Acute
hemolytic reactions' below) or infection, FNHTRs cannot be ignored. More From
UptoDate
Feedback on your answer
Collapse
Unit 3
16)
0/1
You are seeing a 7-year old boy in the ER accompanied by his mother. She notes that he
has been complaining of a fever aand lower abdominal pain for the last 24 hours. She
also notes that he has not eaten anything since yesterday morning as he vomits everything
he attempts to eat. His vitals show a temp is 101.2 F, HR 120, BP 98/70, RR 18 and O2
sat of 99% on room air. On exam he has moderate periumbilical tenderness to light
palpation as well as in his right lower quadrant. His lab work is pending and the surgeon
on call tells you to get some radiology study for confirmation. What is the recommended
initial imaging modality for this patient?
(Select 1)(1pts)

CT scan abdomen/pelvis with IV and PO contrast only

Ultrasound of the Right lower quadrant

MRI abdomen/pelvis

CT scan abdomen/pelvis without IV and PO contrast only

You didn't make any selections

The answer is B, Ultrasound with its lack of ionizing radiation should be the investigation
of choice in young patients, and is effective in competent hands in identifying abnormal
appendixes, especially in thin patients. However, the identification of a normal appendix
is more problematic, and in many instances, appendicitis cannot be ruled out.
The technique used is known as graded compression, and uses the linear probe over
the site of maximal thickness, with gradual increasing pressure exerted to displace normal
overlying bowel gas.
Findings supportive of the diagnosis of appendicitis include 4:
 aperistaltic, noncompressible, dilated appendix ( >6mm outer diameter)
 distinct appendiceal wall layers
 target appearance (axial section)
 appendicolith
 periappendiceal fluid collection
 echogenic prominent pericaecal fat
 confirming that the structure visualised is the appendix is clearly essential
and requires demonstration of it being blind ending and arising from the
base of the caecum
Read more

Feedback on your answer


Collapse
17)
0/1
You are seeing a 31-year old male who presents with 24 hours of progressively
worsening lower abdominal pain, vomiting and chills. He denies past medical history and
denies prior surgery.

On exam his heart rate is 95 bpm, BP is 129/80, oral temp 100.2 F and O2 sat is 100% on
room air. Upon palpation he has right lower quadrant pain when you palpated his left
lower quadrant. His labs show a WBC of 9,700mm3 with no bands.

True or False: A normal white blood cell count effectively rules out appendicitis in this
patient.
(1pts)
You didn't answer this question
False: Appropriate diagnostic testing varies based on the clinical situation. A complete
blood count is appropriate if infection or blood loss is suspected. One study of patients 15
to 83 years of age with suspected appendicitis found that a white blood cell count greater
than 10,000 per mm3 (10 × 10 9 per L) was 77 percent sensitive and 63 percent specific
for the diagnosis (LR+ = 2.1, LR– = 0.37). Thus, nearly one in four patients with
appendicitis does not have an elevated white blood cell count. Read More
Feedback on your answer
Collapse
18)
0/1
Treatment for all patients with acute nonperforated appendicitis must include:

(Select 1)(1pts)

Witholding analgesia

Immediate antibiotics

Blood cultures

Surgery within 12 hours

All of the above


You didn't make any selections
The answer is B, Immediate antibiotics. The treatment of acute appendicitis in the ED
consists of 3 major aspects: 1) supportive care, 2) antibiotic therapy, and 3) definitive
treatment as decided upon by a surgical consultation. Supportive care is to be tailored to
the patient’s symptoms. Most often, patients with appendicitis should receive IV fluids
(lactated Ringer or normal saline), should receive symptomatic treatment (eg, antiemetics
and analgesia), and should remain NPO. Choice of analgesia will depend on the personal
preference of the provider; however, IV opiates are often considered first-line treatment
in possible appendicitis cases.120 Multiple prospective randomized trials and a Cochrane
Database review on analgesia in patients with acute abdominal pain have all concluded
that opiates improve patients’ cooperation, relieve their pain, and do not mask the
physical signs of appendicitis. Unless contraindicated, it is unacceptable to withhold
analgesia from patients presenting with acute abdominal pain.121-124 In the rare case of
acute appendicitis causing septic shock, treatment is guided by the Surviving Sepsis
Campaign International Guidelines of Severe Sepsis and Septic Shock.
Administer antibiotics promptly upon making the diagnosis of appendicitis.126 A
Cochrane database review of 45 studies involving 9576 patients found that antibiotic
administration in appendicitis reduces the incidence of both postoperative wound
infection and intra-abdominal abscess formation when compared to placebo.127
Interestingly, these studies demonstrated no difference in the postoperative histologic
findings of appendicitis in the antibiotic versus placebo groups.
Recommendations on definitive management for acute appendicitis depend on the
presence or absence of complications. Treatment for nonperforated appendicitis with a
well-circumscribed abscess is nonoperative and consists of IV antibiotics and
percutaneous drainage. The management recommendations for nonperforated
appendicitis without abscess or perforated appendicitis is urgent operative intervention.
Conservative management for appendicitis without abscess has become a controversial
topic recently. Conservative therapy consists of hospitalization for IV antibiotics rather
than emergent surgical intervention. Although Dr. Coldrey published a protocol in 1956
involving exclusive antibiotic treatment of appendicitis lasting more than 24 hours with
great personal success, only in the last 20 years has the idea of conservative treatment
been highlighted by multiple prospective trials and meta-analyses. The benefit of
conservative treatment is rooted in the fact that there is significant perioperative
morbidity associated with an appendectomy. 28,135 On average, the rate of
complications from appendectomies is approximately 4.6% and includes small bowel
obstruction, wound infection, abscess formation, and adhesions. Read more
Feedback on your answer
Collapse
19)
0/1
You are seeing a 24-year old male who presents with fever, right lower quadrant pain
(RLQ), and vomiting over the last 12 hours. On exam he has severe RLQ tenderness
without rebound. His current oral temp is 102.1 F, heart rate 110 beats/min, RR 16, blood
pressure 134/90. His WBC comes back at 18,000mm3 with 10% bands. What is the most
common underlying cause of the disease process in this patient?
(Select 1)(1pts)
Fecalith

Lymphoid Hyperplasia

Foreign Bodies

Tumors

You didn't make any selections

The answer is B. Lymphoid Hyperplasia. The most common causes of appendicits are as
follows Lymphoid Hyperplasia 60% , Fecalith 35% , Foreign Bodies 4%,
Tumors 1% The incidence of appendicitis gradually rises from birth, peaks in the late
teen years, and gradually declines in the geriatric years. The mean age when appendicitis
occurs in the pediatric population is 6-10 years. Lymphoid hyperplasia is observed more
often among infants and adults and is responsible for the increased incidence of
appendicitis in these age groups. Younger children have a higher rate of perforation, with
reported rates of 50-85%. The median age at appendectomy is 22 years. Although rare,
neonatal and even prenatal appendicitis have been reported. Clinicians must maintain a
high index of suspicion in all age groups. Read more from Medscape

Feedback on your answer


Collapse
20)
0/1
Which sign on physical examination is MOST predictive in diagnosing appendicitis?

(Select 1)(1pts)

Presence of a Rovsing sign

RLQ tenderness and rigidity

Presence of a psoas sign

Temperature above 38.3°C (101°F)

Presence of a obturator sign

You didn't make any selections


The answer is B, RLQ tenderness and rigidity. Signs on physical examination that are
most predictive of appendicitis are RLQ tenderness and rigidity. Signs that demonstrated
a small effect to discriminate appendicitis were other peritoneal signs (rebound
tenderness, guarding, percussion tenderness), temperature above 38.3°C (101°F), and the
presence of a psoas sign.11,44,45 Signs that do not help in diagnosing appendicitis
include tenderness on rectal examination, increased local skin temperature of RLQ, and
Rovsing sign (ie, pain in the RLQ upon palpation of the LLQ).Read more

Feedback on your answer


Collapse
Unit 5
21)
0/1
Urinary catheters can serve a large variety of purposes. In the setting of bladder or
genitourinary surgery, they are often employed to decompress the system so that it will
heal more readily. After general surgical procedures—and many other surgical
procedures as well—they are used to provide accurate measurements of volume output
and thus, indirectly, to give some indication of the patient's overall volume and
resuscitation status.
Their utility and importance notwithstanding, urinary catheters are associated with the
development of nosocomial urinary tract infections (UTIs). As many as 40% of all
hospital infections are UTIs. What % of these UTI's are associated with urinary catheters?
(Select 1)(1pts)

20%

40%

60%

80%

You didn't make any selections


The answer is D, 80%. Urinary catheters can serve a large variety of purposes. In the
setting of bladder or genitourinary surgery, they are often employed to decompress the
system so that it will heal more readily. After general surgical procedures—and many
other surgical procedures as well—they are used to provide accurate measurements of
volume output and thus, indirectly, to give some indication of the patient's overall volume
and resuscitation status. Furthermore, after many procedures, patients initially find it
extremely difficult or impossible to mobilize for urination, and a urinary catheter may be
quite helpful during this time. Their utility and importance notwithstanding, urinary
catheters are associated with the development of nosocomial urinary tract infections
(UTIs). As many as 40% of all hospital infections are UTIs, and 80 to 90% of these UTIs
are associated with urinary catheters [see Postoperative Complications,below].
Accordingly, when catheterization is no longer deemed necessary, prompt removal is
indicated. As a rule, orders specifically pertaining to urinary catheter care are few,
typically including gravity drainage, flushing to maintain patency (if warranted), and
removal when appropriate. At times, irrigation is employed after urologic procedures or
for the management of certain infectious agents. Read more
Feedback on your answer
Collapse
22)
0/1
Which of the following statements regarding peri and post-operative MI is true?
(Select 1)(1pts)

plaque rupture is the most common cause of MI

The majority of cardiac ischemic events occur after the 4th day

95% of patients who experience this complication will present with classic
symptoms

Perioperative beta blockade for patients at risk for MI is now routine as


this practice yields significant risk reductions in terms of both cardiac
morbidity and mortality

You didn't make any selections


Answer D is the only true statement. Approximately one third of patients who undergo
noncardiac surgery in the United States have some degree of coronary artery disease and
thus are at increased risk for perioperative MI. The incidence of coronary artery disease is
even higher in certain subpopulations, such as patients who undergo major vascular
procedures.40,41 In the perioperative setting, however, the pathophysiology of coronary
ischemia is different from that in nonsurgical settings, where plaque rupture is the most
common cause of MI. Approximately 50% of all MIs occurring in surgical patients are
caused by increased myocardial oxygen demand in the face of inadequate supply
resulting from factors such as fluid shifts, physiologic stress, hypotension, and the effects
of anesthesia. The majority of cardiac ischemic events occur in the first 4 days of the
postoperative period.
Perioperative beta blockade for patients at risk for MI is now routine. Multiple trials and
meta-analyses have demonstrated that this practice yields significant risk reductions in
terms of both cardiac morbidity and mortality42,43 and that these risk reductions are
achieved regardless of the type of surgery being performed. Al though there has been
some variation in the protocols used by these trials and the results reported, there is
general agreement that beta blockade should be initiated preoperatively, delivered at the
time of surgery, and continued postoperatively for up to 1 week.
Diagnosis of postoperative MI is complicated by the fact that as many as 95% of patients
who experience this complication may not present with classic symptoms (e.g., chest
pain). Identification of MI may be further hindered by the ECG changes brought on by
the stress of the perioperative period (including dysrhythmias). Ultimately, the most
useful signal of an ischemic cardiac event in the postoperative period is a rise in the
levels of cardiac enzymes, particularly troponin I. Accordingly, cardiac enzyme activity
should be assessed whenever there is a high index of suspicion for MI or a patient is
considered to be at significant perioperative risk for MI.
Treatment of postoperative MI focuses on correcting any factors contributing to or
exacerbating the situation that led to the event (e.g., hypovolemia or hypotension).
Typically, although antiplatelet agents (e.g., aspirin) are sometimes given, thrombolytic
therapy is avoided because of concerns about postoperative bleeding. Acute percutaneous
coronary intervention is also associated with an increased risk of bleeding but has
nonetheless been used successfully in the peri-operative setting and is recommended by
some physicians.44 Beta blockade is often advocated as a means of treating postoperative
MI, although it is probably more effective when used both preoperatively and
perioperatively as a means of preventing MI.Read more
Feedback on your answer
Collapse
23)
0/1
Same-day surgery is appropriate for patients who (1) have few or no comorbid medical
conditions and (2) are undergoing a procedure that involves short-duration anesthesia or
local anesthesia plus sedation and that carries a low likelihood of urgent complications.
Operations commonly performed on a same-day basis include which of the following?
(Select 1)(1pts)

inguinal or umbilical hernia repair

simple laparoscopic cholecystectomy

breast biopsy

small subcutaneous procedures

All of the above

None of the above

You didn't make any selections


The answer is all of the above. Same-day surgery is appropriate for patients who (1) have
few or no comorbid medical conditions and (2) are undergoing a procedure that involves
short-duration anesthesia or local anesthesia plus sedation and that carries a low
likelihood of urgent complications. Operations commonly performed on a same-day basis
include inguinal or umbilical hernia repair, simple laparoscopic cholecystectomy, breast
biopsy, and small subcutaneous procedures. The growth in the performance of minor and
same-day procedures has led to the development of various types of short-stay units or
wards. The level of care provided by a short-stay ward is generally equivalent to that
provided by a regular nursing ward; however, the anticipated duration of care is
substantially shorter, typically ranging from several hours to a maximum of 48 hours.
Short-stay wards also undergo some modifications to facilitate the use of streamlined
teaching protocols designed to prepare patients for home care. Many hospitals now have
short-stay units, as do some independent surgery centers. Read more
Feedback on your answer
Collapse
24)
0/1
You are seeing a 78-year old female who is post op day 1 following a colon resection
following a bout of recurrent diverticulitis with perforation. She has a past medical
history of cirrhosis from hepatitis C. On exam you note chronic lower extremity swelling
and 2+ pitting edema. Her albumin level was 2.7 g/dL. You are wanting to give her a liter
of normal saline (NS) suspecting dehydration as she has not eaten anything in 48 hours
and her heart rate has increased to around 105 beats/min. Out of the 1 Liter given of NS,
what volume administered stays in the intravascular space?
(Select 1)(1pts)

50-100 cc

250-330cc

500-650 cc

750-800cc

You didn't make any selections


The answer is B, 250-330cc. Crystalloid solutions are water-based solutions to which
electrolytes (and, sometimes, organic molecules such as dextrose) have been added. The
crystalloid solutions used for resuscitation are generally isotonic to blood plasma and
include such common examples as 0.9% sodium chloride, lactated Ringer solution, and
Plasma-Lyte (Baxter Healthcare, Round Lake, IL). The choice to use one solution over
another is usually inconsequential, but there are a few notable exceptions. For example,
in the setting of renal dysfunction, there is a risk of hyperkalemia when potassium-
containing solutions such as lactated Ringer solution and Plasma-Lyte are used. As
another example, the administration of large volumes of 0.9% sodium chloride, which
has a pH of 5.0 and a chloride content of 154 mmol/L, can lead to hyperchloremic
metabolic acidosis. Regardless of which crystalloid solution is used, large volumes may
have to be infused to achieve a significant increase in the circulating intravascular
volume. Only one third to one quarter (250 to 330 mL/L) of the fluid administered stays
in the intravascular space; the rest migrates by osmosis into the interstitial tissues,
producing edema and potential impairment of tissue perfusion (the latter is a theoretical
consequence whose existence has not yet been directly demonstrated). Read more
Feedback on your answer
Collapse
25)
0/1
The most common method of monitoring patients' nutritional status with nutritional
supplementation is to measure the serum albumin and prealbumin (transthyretin)
concentrations. What is the half life of prealbumin?
(Select 1)(1pts)

1-2 days

3-5 days

8-10 days

14-20 days

You didn't make any selections


The answer is A, 1-2 days. Patients who require assistance with nutritional intake should
be monitored to determine whether the interventions being carried out are having the
desired effect. The most common method of monitoring patients' nutritional status with
nutritional supplementation is to measure the serum albumin and prealbumin
(transthyretin) concentrations. Albumin has a half-life of approximately 14 to 20 days
and thus serves as a marker of longer-term nutritional status. A value lower than 2.2 g/dL
is generally considered to represent severe malnutrition, but even somewhat higher values
(< 3.0 g/dL) have been associated with poorer outcomes after elective surgery. Although
the serum albumin concentration is a commonly used marker, it is not always a reliable
one. Because of albumin's relatively long half-life, the serum concentration does not
reflect the patient's more recent nutritional status. In addition, the measured concentration
can change quickly in response to the infusion of exogenous albumin or to the
development of dehydration, sepsis, and liver disease despite adequate nutrition. Pre
albumin is a separate serum protein that has a halflife of approximately 24 to 48 hours
and thus can serve as a marker of current and more recent nutritional status. Like the
albumin concentration, the prealbumin concentration can be affected by liver and renal
disease. Overall, however, it is more immediately reliable in following the effects of
nutritional intervention. Read more

n particular, drainage of purulent materials, serum, or blood from body cavities. Several
types are commonly used, including soft gravity drains (e.g., Penrose), closedsuction
drains (e.g., Hemovac, Jackson-Pratt, and Blake), and sump drains, which draw air into
one lumen and extract fluid via a companion lumen. Traditionally, surgeons have often
made the decision to place a drain on the basis of their surgical training and practice
habits rather than of any firm evidence that drainage is warranted. Multiple randomized
clinical trials have now demonstrated that routine use of drains after elective operations—
including appendectomies and colorectal, hepatic, thyroid, and parathyroid procedures—
does not prevent anastomotic and other complications (although it does reduce seroma
formation). Consequently, it is recommended that drains, like NG tubes, be employed
selectively.3- 5Once a drain is in place, specific orders must be issued for its
maintenance. These include use of gravity or suction (and the means by which suction is
to be provided if ordered), management and measurement of output, stripping, and care
around the drain exit site. Read more
Feedback on your answer
Collapse
2)
0/1
Same-day surgery is appropriate for patients who (1) have few or no comorbid medical
conditions and (2) are undergoing a procedure that involves short-duration anesthesia or
local anesthesia plus sedation and that carries a low likelihood of urgent complications.
Operations commonly performed on a same-day basis include which of the following?
(Select 1)(1pts)

inguinal or umbilical hernia repair

simple laparoscopic cholecystectomy

breast biopsy

small subcutaneous procedures

All of the above

None of the above

You didn't make any selections


The answer is all of the above. Same-day surgery is appropriate for patients who (1) have
few or no comorbid medical conditions and (2) are undergoing a procedure that involves
short-duration anesthesia or local anesthesia plus sedation and that carries a low
likelihood of urgent complications. Operations commonly performed on a same-day basis
include inguinal or umbilical hernia repair, simple laparoscopic cholecystectomy, breast
biopsy, and small subcutaneous procedures. The growth in the performance of minor and
same-day procedures has led to the development of various types of short-stay units or
wards. The level of care provided by a short-stay ward is generally equivalent to that
provided by a regular nursing ward; however, the anticipated duration of care is
substantially shorter, typically ranging from several hours to a maximum of 48 hours.
Short-stay wards also undergo some modifications to facilitate the use of streamlined
teaching protocols designed to prepare patients for home care. Many hospitals now have
short-stay units, as do some independent surgery centers. Read more
Feedback on your answer
Collapse
3)
0/1
Which of the following is NOT true regarding post operative placement of foley
catheters?
(1pts)

As many as 40% of all hospital infections are UTIs

Only 10% of hospital acquired UTIs are associated with urinary catheters

After many procedures, patients initially find it extremely difficult or


impossible to mobilize for urination, and a urinary catheter may be quite
helpful during this time

In the setting of bladder or genitourinary surgery, they are often


employed to decompress the system so that it will heal more readily

You didn't make any selections


Answer B is false. Urinary catheters can serve a large variety of purposes. In the setting
of bladder or genitourinary surgery, they are often employed to decompress the system so
that it will heal more readily. After general surgical procedures—and many other surgical
procedures as well—they are used to provide accurate measurements of volume output
and thus, indirectly, to give some indication of the patient's overall volume and
resuscitation status. Furthermore, after many procedures, patients initially find it
extremely difficult or impossible to mobilize for urination, and a urinary catheter may be
quite helpful during this time.
Their utility and importance notwithstanding, urinary catheters are
associated with the development of nosocomial urinary tract infections
(UTIs). As many as 40% of all hospital infections are UTIs, and
80 to 90% of these UTIs are associated with urinary
catheters.Accordingly, when catheterization is no longer deemed
necessary, prompt removal is indicated. As a rule, orders specifically
pertaining to urinary catheter care are few, typically including gravity
drainage, flushing to maintain patency (if warranted), and removal
when appropriate. At times, irrigation is employed after urologic
procedures or for the management of certain infectious agents. Read
more
Feedback on your answer
Collapse
4)
0/1
Urinary catheters can serve a large variety of purposes. In the setting of bladder or
genitourinary surgery, they are often employed to decompress the system so that it will
heal more readily. After general surgical procedures—and many other surgical
procedures as well—they are used to provide accurate measurements of volume output
and thus, indirectly, to give some indication of the patient's overall volume and
resuscitation status.
Their utility and importance notwithstanding, urinary catheters are associated with the
development of nosocomial urinary tract infections (UTIs). As many as 40% of all
hospital infections are UTIs. What % of these UTI's are associated with urinary catheters?
(Select 1)(1pts)

20%

40%

60%

80%

You didn't make any selections


The answer is D, 80%. Urinary catheters can serve a large variety of purposes. In the
setting of bladder or genitourinary surgery, they are often employed to decompress the
system so that it will heal more readily. After general surgical procedures—and many
other surgical procedures as well—they are used to provide accurate measurements of
volume output and thus, indirectly, to give some indication of the patient's overall volume
and resuscitation status. Furthermore, after many procedures, patients initially find it
extremely difficult or impossible to mobilize for urination, and a urinary catheter may be
quite helpful during this time. Their utility and importance notwithstanding, urinary
catheters are associated with the development of nosocomial urinary tract infections
(UTIs). As many as 40% of all hospital infections are UTIs, and 80 to 90% of these UTIs
are associated with urinary catheters [see Postoperative Complications,below].
Accordingly, when catheterization is no longer deemed necessary, prompt removal is
indicated. As a rule, orders specifically pertaining to urinary catheter care are few,
typically including gravity drainage, flushing to maintain patency (if warranted), and
removal when appropriate. At times, irrigation is employed after urologic procedures or
for the management of certain infectious agents. Read more
Feedback on your answer
Collapse
5)
0/1
The most common method of monitoring patients' nutritional status with nutritional
supplementation is to measure the serum albumin and prealbumin (transthyretin)
concentrations. What is the half life of prealbumin?
(Select 1)(1pts)

1-2 days

3-5 days
8-10 days

14-20 days

You didn't make any selections


The answer is A, 1-2 days. Patients who require assistance with nutritional intake should
be monitored to determine whether the interventions being carried out are having the
desired effect. The most common method of monitoring patients' nutritional status with
nutritional supplementation is to measure the serum albumin and prealbumin
(transthyretin) concentrations. Albumin has a half-life of approximately 14 to 20 days
and thus serves as a marker of longer-term nutritional status. A value lower than 2.2 g/dL
is generally considered to represent severe malnutrition, but even somewhat higher values
(< 3.0 g/dL) have been associated with poorer outcomes after elective surgery. Although
the serum albumin concentration is a commonly used marker, it is not always a reliable
one. Because of albumin's relatively long half-life, the serum concentration does not
reflect the patient's more recent nutritional status. In addition, the measured concentration
can change quickly in response to the infusion of exogenous albumin or to the
development of dehydration, sepsis, and liver disease despite adequate nutrition. Pre
albumin is a separate serum protein that has a halflife of approximately 24 to 48 hours
and thus can serve as a marker of current and more recent nutritional status. Like the
albumin concentration, the prealbumin concentration can be affected by liver and renal
disease. Overall, however, it is more immediately reliable in following the effects of
nutritional intervention. Read more
Feedback on your answer
Collapse
Unit 2
6)
0/1
You are consulted to a 38-year-old black male for evaluation of his preoperative
cardiovascular risk. He has a 20-year history of type 1 diabetes mellitus and hypertension
and is undergoing preoperative evaluation for renal transplantation secondary to
uncontrolled hypertension and diabetic nephropathy. His current blood pressure is 142/85
mm Hg. He currently smokes one half of a pack of cigarettes daily. His
electrocardiogram is consistent with left ventricular hypertrophy and left atrial
enlargement. Which of the following is the most appropriate recommendation at this
time?
(1pts)

No further evaluation is needed, he is cleared for surgery without further


testing

Pharmacologic stress nuclear study

24-hour electrocardiographic holter monitor


Coronary angiography

You didn't make any selections

The answer is B, Pharmacologic stress nuclear study. In general, the indications for
noninvasive cardiac testing are the same for patients undergoing surgery as for those who
are not. All symptomatic patients should be evaluated for cardiac ischemia, usually with
cardiac stress testing, even if their symptoms are atypical. Patients undergoing a low- or
intermediate risk surgery who have at most minor clinical risk predictors (advanced age,
abnormal ECG, rhythm other than sinus, low functional capacity, history of stroke, or
uncontrolled hyper tension) may proceed to surgery without preoperative cardiac testing.
Likewise, patients with intermediate clinical risk predictors (mild angina, diabetes
mellitus, compensated heart failure, previous myocardial infarction, or renal
insufficiency) who have good exercise capacity also do not need further preoperative
cardiac testing before a low- or intermediate-risk procedure. However, in the absence of
high-quality data, some experts recommend that those with intermediate clinical
predictors who have poor functional status and are undergoing an intermediate-risk
procedure, as well as those with intermediate clinical predictors and good functional
status who are undergoing a high-risk procedure, should have noninvasive testing
performed before surgery.
Coronary revascularization is rarely needed just to get a patient through surgery, but it
may be warranted in selected patients with CAD undergoing noncardiac surgery if they
would have required the procedure anyway. However, the only large randomized trial on
the subject showed no benefit for revascularization (40). Avoid preoperative coronary
sten ting to avoid the risk for perioperative stent thrombosis. Patients with recently placed
coronary stents are at risk for perioperative stent thrombosis. For bare-metal stents, the
risk diminishes 4 to 6 weeks after implantation; for drug-eluting stents, an elevated risk
for rethrombosis persists for at least 12 months after placement because of delayed
endothelialization. Because these patients require antiplatelet treatment to mitigate this
risk, purely elective procedures should be delayed accordingly, and the interval without
antiplatelet therapy should be minimized when surgery is required within the window of
vulnerability. Read more

Feedback on your answer


Collapse
7)
0/1
With regards to positioning patients in the operating room, which of the following
statements if FALSE?
(Select 1)(1pts)

The lithotomy position is used in approximately 9% of cases and is the


next most commonly used position
Approximately 80% of surgical procedures are performed with patients in
the supine position.

Two major preventable consequences of malpositioning are neuropathy or


plexopathy and skin burns or ulceration.

Ulnar neuropathy and brachial plexopathy are the two most common
complications in this position and constitute around 90% of claims.

Damage to the obturator, sciatic, lateral femoral cutaneous, and peroneal


nerves from being placed in lithotomy position account for 5% of medical
liability claims for nerve damage

You didn't make any selections


Answer D is the only false statement as Ulnar neuropathy and brachial plexopathy are
the two most common complications in this position and constitute 28% and 20% of
claims, respectively. The risks associated with malpositioning deserve further comment.
The skin over any bony prominence is subject to injury from excessive pressure. The
fragile skin of infants and older patients may be injured by dragging rather than lifting
into position. The American Society of Anesthesiologists (ASA) practice advisory on the
prevention of perioperative peripheral neuropathies recommends that when practical,
patients should be placed in the intended position before induction of anesthesia to test
for comfort.78 Uncomfortable positions should be modified. It is essential to listen to the
patient. This is especially important with older patients or patients with degenerative
spine and joint disease or skeletal instability. Approximately 80% of surgical procedures
are performed with patients in the supine position. Two major preventable consequences
of malpositioning are neuropathy or plexopathy and skin burns or ulceration. Ulnar
neuropathy and brachial plexopathy are the two most common complications in this
position and constitute 28% and 20% of claims, respectively, in recent closed claims
studies.79-84 Padding of the precondylar groove of the humerus and tucking of the arms
or, at the least, restriction of abduction to less than 90° help prevent this problem.
The lithotomy position is used in approximately 9% of cases and is the next most
commonly used position.80,82 Damage to the obturator, sciatic, lateral femoral
cutaneous, and peroneal nerves has been observed. These injuries account for 5% of
medical liability claims for nerve damage in the Closed Claims Data Base. They
represent a small but not insignificant risk.79 Other patient positions have also been
associated with malpositioning injuries. As would be expected, longer procedures in
sicker or mal-nourished patients are more likely to result in complications of
positioning. Read more
Feedback on your answer
Collapse
8)
0/1
Which of the following statements regarding the pre-operative care of patients is false?
(Select 1)(1pts)
When a preoperative antibiotic is indicated, a single dose of therapeutic
strength, administered shortly before incision, usually suffices

Cooling patients during colorectal surgery have been shown to reduce


infection rates

Supplemental perioperative oxygen (i.e., an FIO2 of 80% instead of 30%)


significantly overcomes the decrease in phagocytosis and bacterial killing
usually associated with anesthesia and surgery (and significantly reduces
postoperative nausea and vomiting)

Intraoperative transfusion of packed red blood cells was an independent


risk factor for perioperative infection

You didn't make any selections


Answer B is the only false statement. A 1996 study showed that warming patients during
colorectal surgery reduced infection rates.173 This finding was confirmed in a
subsequent observational cohort study that reported a significantly increased incidence of
SSI with hypothermia.174 A randomized controlled trial of 421 patients, published in
2001, determined that warming patients before short-duration clean procedures (breast,
varicose vein, or hernia) reduced infection (p = .001) and reduced wound scores (p =
.007) in this setting as well.175 The safest and most effective way of protecting patients
from hypothermia is to use forced-air warmers with specialized blankets placed over the
upper or lower body. Alternatives include placing a warming water mattress under the
patient and draping the patient with an aluminized blanket. Second-line therapy involves
warming all IV fluids. Any irrigation fluids used in a surgical procedure should be at or
slightly above body temperature before use. Radiant heating devices placed above the
operative field may be especially useful during operations on infants. Use of a warmer on
the inhalation side of the anesthetic gas circuit can also help maintain the patient's body
temperature during an operation.
The association between blood transfusion and increased perioperative infection rates is
well documented. In a 1997 study, geriatric hip fracture patients undergoing surgical
repair who received blood transfusion had significantly higher rates of perioperative
infection than those who did not (27% versus 15%), and this effect was confirmed on
multivariate analysis.177 Another study yielded similar findings in colorectal cancer: the
relative risk of infection was 1.6 for transfusion of one to three units of blood and 3.6 for
transfusion of more than three units.178 A large prospective cohort study published in
2002 evaluated the association between anemia, blood transfusion, and perioperative
infection.179 Regression analysis confirmed that intraoperative transfusion of packed red
blood cells was an independent risk factor for perioperative infection (odds ratio 1.06;
95% confidence interval 1.01 to 1.11; p > .0001). Furthermore, transfusion of more than
four units of packed red blood cells was associated with a ninefold increased risk of
infection (95% confidence interval 5.74 to 15.00; p < .0001).
In brief, the principles of optimal surgical antimicrobial prophylaxis include (1)
appropriate choice of an antimicrobial agent, (2) proper timing of antibiotic
administration before incision, and (3) limited duration of antibiotic administration after
operation. SSIs are established several hours after contamination.161 There is a well-
recognized "golden period" during which prophylactic antibiotics can be effective.
Administration of antibiotics before contamination reduces the risk of infection but
subsequently proves to be of little prophylactic value.162 The selective use of short-
duration, narrow-spectrum antibiotic agents chosen to reflect (1) the patient's individual
risk factors and (2) the recognized epidemiology of SSI, modified for local conditions
and experience, should be considered where deemed appropriate [see Table 9 for the
usual pathogens isolated from SSIs].163 When a preoperative antibiotic is indicated, a
single dose of therapeutic strength, administered shortly before incision, usually
suffices.164 The routine dose may have to be increased to achieve adequate serum levels
if the patient is morbidly obese.165 A second dose is indicated if the procedure is longer
than two half-lives of the drug or if extensive blood loss occurs. Continuation of
prophylaxis beyond 24 hours is not recommended. The provision of prophylactic
antibiotics, where indicated, has been accepted as a measure of surgical quality aimed at a
reduction in SSIs.166,167 A strong economic case has been made for infection control,
including prophylactic antibiotics, benefiting the hospital and the patient.168 The
perioperative nursing team has a central role, together with the anesthesiologist, in
monitoring the administration of preoperative prophylactic antibiotics.
Destruction by oxidation or by oxidative killing is the body's most important defense
against surgical pathogens. This defensive response depends on adequate levels of
oxygen tension in contaminated tissue. An easy method of improving oxygen tension in
adequately perfused tissue is to increase the FiO2. Supplemental perioperative oxygen
(i.e., an FIO2 of 80% instead of 30%) significantly overcomes the decrease in
phagocytosis and bacterial killing usually associated with anesthesia and surgery (and
significantly reduces postoperative nausea and vomiting). Low oxygen tension in wound
tissue has been found to be a good predictor of SSI risk.176 With that said, raised oxygen
tension is also an important risk factor in the outbreak of surgical fires. A balance must
be achieved between the adequacy of PO2 in the tissue and the restriction of O2 flow in
head and neck surgery and in other high-risk fire settings. Read more
Feedback on your answer
Collapse
9)
0/1
There is a high risk of pathogen transfer during surgery. This is a risk from which both
the patient and the surgical team must be protected. The risk can be reduced by using
protective barriers, such as surgical gloves. A 2002 Cochrane Review concluded that
wearing two pairs of latex gloves can further reduce the risk of contamination to what %?
(Select 1)(1pts)

20%

15%

10%
5%

You didn't make any selections


The answer is D, There is a high risk of pathogen transfer during surgery. This is a risk
from which both the patient and the surgical team must be protected. The risk can be
reduced by using protective barriers, such as surgical gloves. A 2002 Cochrane Review
concluded that wearing two pairs of latex gloves significantly reduced the risk of hand
exposure. Wearing two pairs of surgical gloves rather than a single pair provides an
additional barrier and further reduces the risk of contamination. The rate of single-glove
perforations is approximately 15%. The rate of inner glove perforation when two pairs of
gloves are worn is approximately 5%. Read more
Feedback on your answer
Collapse
10)
0/1
A 72-year-old African-American male comes to your office for surgical clearance for an
elective left hemicolectomy for recurrent diverticulitis. His past medical history includes
an inferior-wall myocardial infarction approximately 12 months ago. Since then he had a
stress test that was normal 6 months ago. Currently, the patient feels well, walks while
playing nine holes of golf weekly, and is able to walk up a flight of stairs without chest
pain or significant dyspnea. Findings are normal on a physical examination.
Which one of the following would be most appropriate for this patient prior to surgery?
(1pts)

A 12-lead resting EKG

Another Nuclear Stress test

A dobutamine stress echocardiogram

A persantine stressed nuclear tracer study (technetium or thallium)

You didn't make any selections


The answer is A, A 12-lead resting EKG. The current recommendations from the
American College of Cardiology and the American Heart Association on preoperative
clearance for noncardiac surgery state that preoperative intervention is rarely needed to
lower surgical risk. Patients who are not currently experiencing unstable coronary
syndrome, severe valvular disease, uncompensated congestive heart failure, or a
significant arrhythmia are not considered at high risk, and should be evaluated for most
surgery primarily on the basis of their functional status. If these patients are capable of
moderate activity (greater than 4 METs) without cardiac symptoms, they can be cleared
with no stress testing or coronary angiography for an elective minor or intermediate-risk
operation such as the one this patient is to undergo. A resting 12-lead EKG is
recommended for males over 45, females over 55, and patients with diabetes, symptoms
of chest pain, or a previous history of cardiac disease. Ref: ACC/AHA guideline update
for perioperative cardiovascular evaluation for noncardiac surgery--Executive summary:
A report of the American College of Cardiology/American Heart Association Task Force
on Practice Guidelines (Committee to Update the 1996 Guidelines on Perioperative
Cardiovascular Evaluation for Noncardiac Surgery). Circulation 2002;105(10):1257-
1267. Read more here
Feedback on your answer
Collapse
Unit 1
11)
0/1
Abdominal pain is traditionally divided into 3 categories. What are the 2 basic causes of
abdominal pain of the 3 categories?

( 1pts extra credit)

Visceral and Autonomic

Visceral and Parietal

Parietal and Somatic

Referred and Somatic

You didn't make any selections


Abdominal pain is traditionally divided into 3 categories: visceral, parietal and
referred. In general visceral (autonomic) and parietal (somatic) are the 2 basic causes of
abdominal pain. Referred pain can be considered separately as a cortical misperception of
either visceral or parietal afferent stumuli. The two are often not discrete as visceral pain
often blends with parietal pain as a pathological process evolves. Tintinalli's Emergency
Medicine: A Comprehensive Study Guide, 7th edition, Tintinalli et al. Read more
on Acute Abdominal Pain Power Point PDF
Feedback on your answer
Collapse
12)
0/1
You are seeing a 48-year old obese black male who presents with acute onset of left
lower quadrant abdominal pain since this morning. He has a no prior history of
diverticulitis but has type II diabetes and takes metformin. His vitals show a heart rate of
95 bpm, blood pressure of 165/81, respirations 18/min, oral temperature 100.6
F. WBC 14,500mm3 without bands. CT scan with IV and PO contrast of his
abdomen/pelvis shows acute sigmoid diverticulitis with an abscess measuring 1.0
cm. True or False: The recommended treatment for this patient is either surgery
or percutaneous drainage as most abscesses of less than 2 cm will not improve with IV
antibiotics.
( 1pts extra credit)
You didn't answer this question
This statement is False: Approximately 15% of patients with acute diverticulitis will
develop a pericolonic or intramesenteric abscess. For these patients, hospitalization and
intravenous antibiotics are indicated. Abscesses <2 cm in diameter may
resolve without further intervention . Patients with larger abscesses are
candidates for percutaneous catheter drainage; the majority of patients can avoid an
emergency operation and a multistaged approach involving a stoma by using this
intervention.
From another reference Scientific American Surgery, they use <5cm:
If fever and leukocytosis persist despite antibiotic therapy, the presence of an abscess
should be suspected. Small (< 5 cm) abscesses may respond to antibiotic infusion and
bowel rest. Larger abscesses that are localized and isolated may be accessible to
percutaneous drainage .27 Generally, this technique is reserved for abscesses greater than
5 cm in diameter in low-risk patients who are not immunocompromised. It often leads to
resolution of sepsis and the resulting symptoms and signs (e.g., abdominal pain and
tenderness and leukocytosis), usually within 72 hours, thereby facilitating subsequent
elective surgical resection of the colon. In addition, percutaneous drainage offers cost
advantages in that it reduces the number of operative procedures required and shortens
hospital stay.28Patients with severe comorbidities at times can be managed with drainage
alone.29

Access to a pelvic collection may be difficult to obtain, and the drainage procedure
typically must be done with the patient in a prone or lateral position. If the catheter
drainage amounts to more than 500 mL/day after the first 24 hours, a fistula should be
suspected. Before the catheter is removed, a CT scan is done with injection of contrast
material through the tube to determine whether the cavity has collapsed. If this approach
fails (as it usually does in patients with multiple or multiloculated abscesses), an
expeditious operation may be necessary.17 An initial surgical procedure is required in
about 20% of cases.

2014. Scientific American Surgery. Hamilton, Ontario & Philadelphia, PA. Decker
Intellectual Properties Inc. ISSN 2368-2744. STAT!Ref Online Electronic Medical
Library. http://online.statref.com/Document.aspx?fxId=61&docId=1368. 10/22/2014
12:22:12 PM CDT (UTC -05:00).
Access to a pelvic collection may be difficult to obtain, and the drainage procedure
typically must be done with the patient in a prone or lateral position. If the catheter
drainage amounts to more than 500 mL/day after the first 24 hours, a fistula should be
suspected. Before the catheter is removed, a CT scan is done with injection of contrast
material through the tube to determine whether the cavity has collapsed. If this approach
fails (as it usually does in patients with multiple or multiloculated abscesses), an
expeditious operation may be necessary.17 An initial surgical procedure is required in
about 20% of cases.

Feedback on your answer


Collapse
13)
0/1
You have admitted a 46-year old male for acute diverticulitis of his descending colon and
started him on antibiotics and pain medicine as needed. He weighs 100 kg, he has no past
medical history and takes no medications. You plan on starting him on maintenance
fluids of normal saline as you have made him NPO. Which of the following is the best
answer choice?

(Select 1)( 1pts extra credit)

40 ml/hr

80 ml/hr

140 ml/hr

220 ml/hr

None of the above

You didn't make any selections

The answer is C, 140 ml/hr. Maintenance fluids provide required daily amounts of free
water and electrolytes (e.g., sodium, potassium, and chloride) to balance expected daily
losses and maintain homeostasis. A basic rule of thumb used by many practitioners to
calculate the infusion rate for maintenance IV fluids is the so-called 4, 2, 1 rule: in a 100
kg patient
• 4 mL/kg/hr for the first 10 kg of body weight (40 ml/hr) +
• 2 mL/kg/hr for the next 10 kg of body weight (20 ml/hr) and +
• 1 mL/kg/hr for every 1 kg of body weight above 20 kg (80 ml/hr)
= 40+20+80=140 ml/hr
Generally accepted maintenance requirements include 30 to 35 mL/kg/day for free water,
1.5 mEq/kg/day for chloride, 1 mEq/kg/day for sodium, and 1 mEq/kg/day for potassium.
In the setting of starvation or poor oral intake, dextrose 5% is often added to maintenance
fluids to inhibit muscle breakdown. In regular practice, however, these specific values are
not commonly used; more often, a rough estimate is made of expected daily fluid
requirements, and solutions are ordered in accordance with this estimate. Although this
practice is unlikely to cause noticeable harm in the majority of postoperative patients,
there are situations where inaccurate calculations can lead to dehydration and volume
overload. Three studies from the early 2000s evaluated patients undergoing elective
colorectal surgery with the aim of determining whether providing higher volumes of fluid
perioperatively had an impact on outcome.In all three, the data supported the use of
smaller fluid volumes perioperatively, which was shown to result in earlier return of gut
function after operation, shorter hospital. Read more

Feedback on your answer


Collapse
14)
0/1
Multiple randomized clinical trials have now demonstrated that routine use of drains
(such as Penrose, Hemovac, Jackson-Pratt, and Blake) after what elective operation has
shown benefit in preventing anastomotic and other complications besides seromas?

(Select 1)( 1pts extra credit)

appendectomies

parathyroid

hepatic

thyroid

all of the above operations have shown benefit

none of the above operations have shown benefit

You didn't make any selections

The answer is none of the above. Multiple randomized clinical trials have now
demonstrated that routine use of drains after elective operations—including
appendectomies and colorectal, hepatic, thyroid, and parathyroid procedures—does not
prevent anastomotic and other complications (although it does reduce seroma formation).
Drains and tubes are placed in a wide variety of locations for a number of different
purposes—in particular, drainage of purulent materials, serum, or blood from body
cavities. Several types are commonly used, including soft gravity drains (e.g., Penrose),
closed-suction drains (e.g., Hemovac, Jackson-Pratt, and Blake), and sump drains, which
draw air into one lumen and extract fluid via a companion lumen. Traditionally, surgeons
have often made the decision to place a drain on the basis of their surgical training and
practice habits rather than of any firm evidence that drainage is warranted. Multiple
randomized clinical trials have now demonstrated that routine use of drains after elective
operations—including appendectomies and colorectal, hepatic, thyroid, and parathyroid
procedures—does not prevent anastomotic and other complications (although it does
reduce seroma formation). Consequently, it is recommended that drains, like NG tubes,
be employed selectively.3-5Once a drain is in place, specific orders must be issued for its
maintenance. These include use of gravity or suction (and the means by which suction is
to be provided if ordered), management and measurement of output, stripping, and care
around the drain exit site.
Biliary tract drains include T tubes, cholecystostomy tubes, percutaneous drains of the
biliary tree, and nasobiliary drains. Daily site maintenance, flushing, and output recording
are performed by the nursing staff. Most biliary tract drains are removed by the
practitioner or other trained midlevel staff members.
T tubes are generally placed after operative exploration or repair of the common bile duct
(CBD). The long phalanges are left within the CBD, and the long portion of the tube is
brought out to the skin for drainage. The tube is left in place until the CBD is properly
healing and there is evidence of adequate distal drainage (signaled by a decrease in
external drainage of bile). Before the T tube is removed, a cholangiogram is
recommended to document distal patency and rule out retained gallstones or leakage.6
Cholecystostomy tubes are placed percutaneously—typically under ultrasonographic
guidance and with local anesthesia—to decompress the gallbladder. Generally, they are
used either (1) when cholecystectomy cannot be performed, because concomitant medical
problems make anesthesia or the stress of operation intolerable, or (2) when the presence
of severe inflammation leads the surgeon to conclude that dissection poses too high an
operative risk. Particularly in the latter setting, delayed elective cholecystectomy may be
appropriate; if so, the cholecystostomy tube may be removed at the time of the operation.
Nasobiliary tubes are placed endoscopically in the course of biliary endoscopy. They are
used to decompress the CBD in some settings. They are usually placed to gravity and
otherwise are managed in much the same way as NG tubes. Read more

Feedback on your answer


Collapse
15)
0/1
You are seeing a 63-year old obese black male who presented with left lower quadrant
abdominal pain. His CT scan showed mild sigmoid diverticulitis and you decide to treat
him with outpatient antibiotics. You also suggested increasing his fiber intake and
recommend outpatient colonscopy as he denies prior in the past. What percentage of
patients will go on to have another episode of acute diverticulitis after their first episode?
(Select 1)( 1pts extra credit)

15%

30%
50%

75%

You didn't make any selections

The answer is B. 30%. After successful medical treatment of an episode of acute


diverticulitis, careful judgment is required concerning whether to proceed with
subsequent elective colon resection. After one attack, about a third of patients will have a
later second attack of acute diverticulitis, and after a second episode, a further third will
have yet another attack. Read more

Feedback on your answer


Collapse
Unit 3
16)
0/1
You are seeing a 31-year old male who presents with 24 hours of progressively
worsening lower abdominal pain, vomiting and chills. He denies past medical history and
denies prior surgery.

On exam his heart rate is 95 bpm, BP is 129/80, oral temp 100.2 F and O2 sat is 100% on
room air. Upon palpation he has right lower quadrant pain when you palpated his left
lower quadrant. His labs show a WBC of 9,700mm3 with no bands.

True or False: A normal white blood cell count effectively rules out appendicitis in this
patient.
(1pts)
You didn't answer this question
False: Appropriate diagnostic testing varies based on the clinical situation. A complete
blood count is appropriate if infection or blood loss is suspected. One study of patients 15
to 83 years of age with suspected appendicitis found that a white blood cell count greater
than 10,000 per mm3 (10 × 10 9 per L) was 77 percent sensitive and 63 percent specific
for the diagnosis (LR+ = 2.1, LR– = 0.37). Thus, nearly one in four patients with
appendicitis does not have an elevated white blood cell count. Read More
Feedback on your answer
Collapse
17)
0/1
You are seeing a 24-year old male who presents with fever, right lower quadrant pain
(RLQ), and vomiting over the last 12 hours. On exam he has severe RLQ tenderness
without rebound. His current oral temp is 102.1 F, heart rate 110 beats/min, RR 16, blood
pressure 134/90. His WBC comes back at 18,000mm3 with 10% bands. What is the most
common underlying cause of the disease process in this patient?
(Select 1)(1pts)
Fecalith

Lymphoid Hyperplasia

Foreign Bodies

Tumors

You didn't make any selections

The answer is B. Lymphoid Hyperplasia. The most common causes of appendicits are as
follows Lymphoid Hyperplasia 60% , Fecalith 35% , Foreign Bodies 4%,
Tumors 1% The incidence of appendicitis gradually rises from birth, peaks in the late
teen years, and gradually declines in the geriatric years. The mean age when appendicitis
occurs in the pediatric population is 6-10 years. Lymphoid hyperplasia is observed more
often among infants and adults and is responsible for the increased incidence of
appendicitis in these age groups. Younger children have a higher rate of perforation, with
reported rates of 50-85%. The median age at appendectomy is 22 years. Although rare,
neonatal and even prenatal appendicitis have been reported. Clinicians must maintain a
high index of suspicion in all age groups. Read more from Medscape

Feedback on your answer


Collapse
18)
0/1
Neoplastic lesions of the appendix are found in as many as 5% of specimens obtained
with routine appendectomy for acute appendicitis. Most are benign. Preoperative
detection of such conditions is rare, and intraoperative diagnosis is made in fewer than
50% of cases. Which of the following statement regarding neoplastic lesions of the
appendix is true?
(Select 1)(1pts)

Metastasis to the appendix is common

Adenocarcinoma is the most common type represting >80%

The incidence of malignancy in the appendix is 1.35%.

If the tumor is less than 8 cm in diameter, is located within the body or


the tip of the appendix, and has not metastasized, appendectomy is the
treatment of choice.
All of the above

None of the above

You didn't make any selections


Answer C is the only true statement. Neoplastic lesions of the appendix are found in as
many as 5% of specimens obtained with routine appendectomy for acute appendicitis.18-
21 Most are benign. Preoperative detection of such conditions is rare, and intraoperative
diagnosis is made in fewer than 50% of cases. Appendectomy alone may be curative for
appendiceal mucocele, localized pseudomyxoma peritonei, most appendiceal carcinoids,
and other benign tumors. Definitive management of an appendiceal mass unexpectedly
encountered during exploration for clinically suspected acute appendicitis depends on
whether the tumor is carcinoid, its size and location, the presence or absence of metastatic
disease, and histologic and immunohistochemical findings.
Benign neoplasms of the appendix include mucosal hyper-plasia or metaplasia,
leiomyomas, neuromas, lipomas, angiomas, and other rare lesions. Appendiceal
adenomas tend to be diffuse and to have a predominant villous character. Mucus-
producing cystadenomas predispose to appendiceal mucocele, sometimes accompanied
by localized pseudomyxoma peritonei. These lesions are rarely symptomatic and are
often encountered incidentally during operation; however, they may also be clinically
manifested as acute appendicitis, torsion, intussusception, ureteral obstruction, or another
acute condition. If the base of the appendix is free of disease, appendectomy alone is
sufficient treatment.
Malignant tumors of the appendix primarily consist of carcinoids and adenocarcinomas;
all together, they account for 0.5% of all gastrointestinal malignancies. The incidence of
malignancy in the appendix is 1.35%. Metastasis to the appendix is rare. Carcinoids are
substantially more common than adenocarcinomas in the appendix: as many as 80% of
all appendiceal masses are carcinoid tumors. Overall, carcinoid tumors are found in 0.5%
of all appendiceal specimens, and appendiceal carcinoid tumors account for 18.9% of all
carcinoid lesions.23 These tumors are predominantly of neural cellular origin and have a
better prognosis than all other intestinal carcinoid tumors, which typically are of mucosal
cellular origin. If the tumor is less than 2 cm in diameter, is located within the body or the
tip of the appendix, and has not metastasized, appendectomy is the treatment of choice. If
the lesion is at the base of the appendix, is larger than 2 cm in diameter, or has
metastasized, right hemicolectomy is indicated. In addition, secondary right
hemicolectomy is indicated if the tumor is invasive, if mucin production is noted, or if the
tumor is found to be of mucosal cellular origin at the final pathologic examination.24,25
Patients with metastatic appendiceal carcinoid tumors appear to have a far better
prognosis than those with other types of metastatic cancers.24 Therefore, hepatic
debulking for symptomatic control is indicated and justified in cases of liver metastasis.
Figure 8. Laparoscopic appendectomy. The mesoappendix having been divided (a), the
base of the appendix is cleared circumferentially and divided with a gastrointestinal
anastomosis stapler (b). Read more
Feedback on your answer
Collapse
19)
0/1
Which sign on physical examination is MOST predictive in diagnosing appendicitis?

(Select 1)(1pts)

Presence of a Rovsing sign

RLQ tenderness and rigidity

Presence of a psoas sign

Temperature above 38.3°C (101°F)

Presence of a obturator sign

You didn't make any selections

The answer is B, RLQ tenderness and rigidity. Signs on physical examination that are
most predictive of appendicitis are RLQ tenderness and rigidity. Signs that demonstrated
a small effect to discriminate appendicitis were other peritoneal signs (rebound
tenderness, guarding, percussion tenderness), temperature above 38.3°C (101°F), and the
presence of a psoas sign.11,44,45 Signs that do not help in diagnosing appendicitis
include tenderness on rectal examination, increased local skin temperature of RLQ, and
Rovsing sign (ie, pain in the RLQ upon palpation of the LLQ).Read more

Feedback on your answer


Collapse
20)
0/1
You are seeing a 32-year-old white female who is currently 16 weeks'
pregnant and is presenting with right lower quadrant pain that started
abruptly last night along with nausea, vomiting, and a low grade fever.
Which one of the following imaging studies would be most appropriate
for initial evaluation of this patient?
(Select 1)(1pts)

CT of the abdomen

MRI of the abdomen

Ultrasonography of the abdomen and Right lower quadrant


A small bowel series

You didn't make any selections


The answer is C, Ultrasonography of the abdomen and Right lower quadrant. CT has
demonstrated superiority over transabdominal ultrasonography for identifying
appendicitis, associated abscess, and alternative diagnoses. However, ultrasonography is
indicated for the evaluation of women who are pregnant and women in whom there is a
high degree of suspicion for gynecologic disease. Ref: Paulson EK, Kalady MF, Pappas
TN: Suspected appendicitis. N Engl J Med 2003;348(3):236-241.
Feedback on your answer
Collapse
Unit 4
21)
0/1
You are seeing a 49-year old obese female who presents with recurrent worsening left
lower quadrant abdominal pain since last night. Her PCP diagnosed her with diverticulitis
5 days prior and started her on oral cefazolin and flagyl. She denies past medical history
and takes no other medications at home.
Vital signs show an oral temperature of 100.8 F, heart rate 101 beats/min, respirations
16/min, blood pressure 154/87, O2 saturation on room air 97%. Physical exam is normal
except for moderate left lower quadrant tenderness with no guarding.
IV and oral contrasted CT scan shows acute sigmoid diverticulitis with a 4.5cm peri-
sigmoid fluid collection consistent with abscess formation. What is the next best step in
management?
(Select 1)(1pts)

Surgical Laparotomy for sigmoid resection

IV antibiotics and Percutaneous Abscess Drainage

IV antibiotics and admission for observation

Continue current management with oral antibiotics with recheck in 24


hours

You didn't make any selections


The answer is B. This patient has acute complicated diverticulitis with an abscess, no
signs of pertionitis and should be treated with IV antibiotics and a percutaneous drain.
The size of the abscess is an important determinant of the need for percutaneous
drainage. Small peridiverticular absceses (4cm or less) without peritonitis (Hinchey stage
1) can be treated conservatively with bowel rest and broad-spectrum antibiotics. For
patients with peridiverticular abscesses that are larger than 4 cm in diameter (Hinchey
stage 2), observational studies indicate that CT-guided percutaneous drainage can be
beneficial. This procedure typically eliminates or reduces the size of the abscess,with a
reduction in pain, resolution of leukocytosis, and defervescence usually seen within
several days. Percutaneous drainage mayallow for elective rather than emergency
surgery, increasing the likelihood of a successful one-stage procedure.
Patients whose abscess cavities contain gross feculent material tend to respond poorly,
and early surgical intervention is usually required.
Outpatient Therapy: Most patients (i.e., immunocompetent patients who have a mild
attack and can tolerate oral intake),outpatient therapy is reasonable. Rx: 7 to 10 days of
oral broad-spectrum antimicrobial therapy, including coverageagainst anaerobic
microorganisms. A combination of ciprofloxacin and metronidazole is often used, but
other regimens are also effective.
Inpatient Therapy: Immunocompromised, age > 65, diabetes and any patient in whom
diverticulitis is complicated by peridiverticular abscess formation.
Jacobs DO. Diverticulitis. N Engl J Med 2007:357:2057-2066. The decision to
hospitalize a patient for diverticulitis dependson the patient's clinical status.
Abscesses occur in 16 percent of patients with acute diverticulitis without peritonitis [46]
and in 31 to 56 percent of those requiring surgery for diverticulitis [47]. Prior to
interventional radiologic techniques, abscesses mandated operative intervention, often
with two-stage procedures. Percutaneous drainage now permits elective single stage
surgery in 60 to 80 percent of patients [48-50]; furthermore, in selected patients with
contraindications to surgery, catheter drainage may be sufficient to relieve symptoms
[49]. Drainage is usually performed through the anterior abdominal wall; abscesses deep
in the pelvis or obscured by other organs may be accessed transgluteally, or through the
rectum or vagina, although this approach is rarely performed [51,52].
The catheter is left in place until drainage is less than 10 mL in 24 hours; this may take as
long as 30 days [49]. Catheter sinograms during this period can show persistent
communication between fistula and bowel, and permit assessment of resolution of the
abscess cavity. Surgical intervention is mandatory if improvement does not occur.
Until recently, all abscesses have been considered to represent complicated diverticulitis,
and surgery has typically been recommended once resolution has occurred. However,
with continued improvements in CT scan technology, smaller abscesses can be identified.
Some of these are so small (ie, less than 3 cm) that they are not amenable to CT guided
drainage [53-55]. In addition to size limitations, the presence of small bowel loops
adjacent to the collection may preclude drainage. Abscess recurrence and/or fistula
formation after percutaneous drainage may also complicate the patient's course and
compromise the surgical approach. Accordingly, there has been interest in treating small
abscesses with systemic antibiotics alone.
In a retrospective study of 22 patients with 23 diverticular abscesses less than 3 cm in
diameter and no evidence of free air, all resolved the acute episode with antibiotic
treatment alone [53]. In subsequent follow up, 36 percent required eventual surgery, 41
percent reported full resolution of symptoms, and 23 percent had mild recurrent or
persistent symptoms that did not warrant surgery. Limitations of the study are the small
sample size and the lack of proof that these collections were actually abscesses as
opposed to localized reactive peritoneal fluid collections. Patients with acute diverticulitis
should be evaluated by a surgeon. CT scan findings should be reviewed with the
interventional radiologist to determine if any collections are amenable to percutaneous
drainage. Indications for immediate surgery include free intraperitoneal air. A patient
with a fluid collection less than 3 cm and not amenable to catheter drainage can be treated
with intravenous antibiotics and monitored carefully for any worsening of symptoms.
Patients with worsening symptoms require surgical intervention.
For patients who are treated by CT-guided percutaneous drainage,
surgery is indicated if the drain does not control symptoms by the
third or fifth day. For patients successfully treated by percutaneous
drainage, a clinical assessment is performed to determine if the patient
can be safely observed, or as in the case of high risk patients (eg,
advanced age, debility, and chronic immunosuppressive therapy), an
operative intervention is recommended. More From UptoDate
Feedback on your answer
Collapse
22)
0/1
You are seeing a 64-year old female who is presenting with painless bright red rectal
bleeding for the last two days. She notes bright red blood coating the stool and on the
toilet paper when she wipes. He vitals are normal and her physical exam shows a soft non
tender abdomen. Stool is brown with red streaks. What is the most likely cause of
bleeding in this patient and the most common overall cause of lower GI bleeding in
adults?
(Select 1)(1pts)

Inflammatory Bowel Disease

Diverticulosis

Neoplasm

Angiodysplasia

You didn't make any selections

The answer is B. Diverticulosis results in 60% of cases of lower GI bleeding followed by


IBD(13%), benign anorectal disease (11%), neoplasia (10%) and angiodysplasia (<5%).
The causes of lower gastrointestinal bleeding (LGIB) may be grouped into several
categories (table 1): From Uptodate
 Anatomic (diverticulosis)
 Vascular (angiodysplasia, ischemic, radiation-induced)
 Inflammatory (inflammatory bowel disease, infectious)
 Neoplastic
In most series, diverticulosis is the most common source of LGIB, accounting for
approximately 15 to 55 percent of cases [9,10]. Angiodysplasia may be the most
frequent cause in patients over the age of 65 years [11,12], though more recent data
suggest that angiodysplasia may be a less common cause of LGIB than once thought.
Hemorrhoids are the most common cause of rectal bleeding in patients under the age of
50 years [13]. However, hemorrhoidal bleeding is usually minor. Similarly, bloody
diarrhea due to inflammatory causes can sometimes be distinguished from other causes of
LGIB because of the clinical setting. In general, anatomic and vascular causes of
bleeding present with painless, large-volume blood loss, whereas inflammatory sources
are associated with diarrhea and abdominal pain [14].
In a review of several large studies that included 1559 patients with acute hematochezia,
the following bleeding sources were identified [10]:
 Diverticulosis – 5 to 42 percent
 Ischemia – 6 to 18 percent
 Anorectal (hemorrhoids, anal fissures, rectal ulcers) – 6 to 16 percent
 Neoplasia (polyps and cancers) – 3 to 11 percent
 Angiodysplasia – 0 to 3 percent
 Postpolypectomy – 0 to 13 percent
 Inflammatory bowel disease – 2 to 4 percent
 Radiation colitis – 1 to 3 percent
 Other colitis (infectious, antibiotic associated, colitis of unclear etiology)
– 3 to 29 percent
 Small bowel/upper GI bleed – 3 to 13 percent
 Other causes – 1 to 9 percent
 Unknown cause – 6 to 23 percent
The vast majority of colonic diverticula are actually false diverticula (pseudodiverticula)
that contain only serosa and mucosa. They occur at weak points in the colonic wall where
the vasa recta penetrate the muscularis to supply the mucosa; as the diverticulum
expands, these vessels are displaced. A 1976 anatomic study of colonic specimens from
patients with diverticular bleeding used angiography to demonstrate that in all cases, the
vasa recta overlying the diverticulum ruptured into the lumen of the diverticulum, not
into the peritoneum.100

It has been estimated that approximately 17% of patients with colonic diverticulosis
experience bleeding, which may range from minor to severe and life-threatening.
Endoscopic treatment of diverticular hemorrhage can be difficult because of the high
bleeding rate and the location of the bleeding point within the diverticulum. In 2000, one
group of investigators reported their experience with endoscopic therapy for severe
hematochezia and diverticulosis in a prospective series of 121 patients.101 In this series,
none of the patients treated endoscopically with epinephrine injections, bipolar
coagulation, or both required surgery and none experienced recurrent bleeding episodes.
A 2001 study from another group, however, reported high rates of recurrent bleeding
episodes in both the early and the late posttreatment periods.102 In the absence of
prospective, randomized trials, it is difficult to draw definitive conclusions about the
utility of endoscopic therapy in treating diverticular hemorrhage.

Fortunately, most diverticular hemorrhages stop spontaneously. In one series, surgery


was unlikely to be necessary if fewer than four units of packed RBCs were transfused in a
24-hour period, whereas 60% of patients receiving more than four units of packed RBCs
in a 24-hour period required surgical intervention.103 Semielective surgical therapy is
usually offered after a second diverticular bleeding episode because once a second such
episode has occurred, the risk that a third will follow exceeds 50%. In a series of 83
conservatively managed cases of diverticular disease, the predicted yearly recurrence
rates were 9% at 1 year, 10% at 2 years, 19% at 3 years, and 25% at 4 years.104

In general, patients who require more than four units of blood in a 24-hour period to
remain hemodynamically stable, who have not stopped bleeding after 72 hours, or who
experience rebleeding within 1 week after an initial episode should undergo surgery.

2014. Scientific American Surgery. Hamilton, Ontario & Philadelphia, PA. Decker
Intellectual Properties Inc. ISSN 2368-2744. STAT!Ref Online Electronic Medical
Library. http://online.statref.com/Document.aspx?fxId=61&docId=848. 10/22/2014
7:03:07 PM CDT (UTC -05:00).

Feedback on your answer


Collapse
23)
0/1
You are seeing a 58-year-old female who is presenting with two days of a low-grade
fever and increasing abdominal cramps that are exacerbated by bowel movements. She
began a course of augmentin 2 days ago that her PCP gave her but has had no relief of
her symptoms and is getting worse. She has not had nausea, vomiting, urinary symptoms,
or bloody stools. She has a past medical history of hypertension.

Her vitals show a temperature is 100.6°F, pulse is 75/min, and blood pressure is 150/80
mm Hg. Her lungs are clear to auscultation. Cardiac examination shows no
abnormalities. Her abdomen is soft, and there is tenderness to palpation of the left lower
quadrant with guarding but no rebound. Bowel sounds are normal. Her stool is brown,
and test for occult blood is negative. Her hemoglobin concentration is 14.5 g/dL,
leukocyte count is 15,800/mm3, and platelet count is 280,000/mm3; serum studies and
urinalysis show no abnormalities. Which of the following is the most appropriate next
step in diagnosis?
(Select 1)(1pts)

Examination of the stool for ova and parasites

Test of the stool for Clostridium difficile toxin


Xray Abdomen Series

CT scan of the abdomen/pelvis with IV and oral contrast

You didn't make any selections


The answer is D, CT scan of the abdomen with IV and oral contrast. This patient is being
treated for diverticulitis and is not getting better. She needs a CT scan to evaluate for a
possible abscess. Read more

Feedback on your answer


Collapse
24)
0/1
You are seeing a 64-year old white male who is presenting with left lower abdominal
pain since last night. His WBC is 14,000mm3 and his CT scan of the abdomen and pelvis
of the abdomen/pelvis shows acute uncomplicated sigmoid diverticulitis. What bacteria is
usually the target in the treatment of acute diverticulitis?
(Select 1)(1pts)

Gram positive cocci

Gram positive rods

Gram negative cocci

Gram negative rods

You didn't make any selections

The answer is D, gram negative Rods. The most common choices would be either
augmentin as single coverage or cipro plus flagyl. Conservative treatment of acute
uncomplicated diverticulitis is successful in 70 to 100 percent of patients. Uncomplicated
diverticulitis may be managed as an outpatient (dietary modification and oral antibiotics)
for those without appreciable fever, excessive vomiting, or marked peritonitis, as long as
there is the opportunity for follow-up. The patient should be able to take liquids and
antibiotics by mouth. Hospitalization for treatment (dietary modification and intravenous
antibiotics) is usually best if the above conditions are not met, or if the patient fails to
improve with outpatient therapy. Antibiotics should be selected to treat the most common
bacteria found in the colon: gram-negative rods and anaerobic bacteria. 13 Single and
multiple antibiotic regimens are equally effective, as long as both groups of organisms
are covered. Nonoperative treatment will resolve acute diverticulitis in 85 percent of
patients, but approximately one-third will have a recurrent attack often within one
year. Read More
Feedback on your answer
Collapse
25)
0/1
You are seeing a 45-year old obese male with type II diabetes who presents with acute
onset left lower quadrant abdominal pain, vomiting, pain with defecation, and
fever. Physical exam is normal except for his abdominal exam that shows left lower
quadrant tenderness without signs of peritonitis. His labs are remarkable for a WBC of
16,000 mm3. Which of the following choices is most likely to reduce repeat incidences of
this patients condition?
(Select 1)(1pts)

Routine daily stool softners or laxatives

Biannual colonoscopy

High fiber diet

Avoidance of nuts and legumes

You didn't make any selections


The answer is C, High fiber diet. Environmental and lifestyle factors are important risk
factors for diverticular disease. Out of the following options only increasing fiber intake
has been shown to reduce episodes of diverticulitis. The role of fiber in the development
of diverticulosis is unclear. Several early studies suggested that low dietary fiber
predisposes to the development of diverticular disease, but other studies have been
conflicting [3,30-39]. Fiber also does not reduce symptoms in patients with
symptomatic uncomplicated diverticular disease [37,38,40,41]. However, dietary fiber
and a vegetarian diet may reduce the incidence of symptomatic diverticular disease by
decreasing intestinal inflammation and altering the intestinal microbiota [4,31,42].
In a cohort study that included over 47,000 men, after adjustment for age, energy-
adjusted total fat intake, and physical activity, total dietary fiber intake was noted to be
inversely associated with the risk of symptomatic diverticular disease (RR 0.58 highest
quintile versus lowest quintile for fiber intake) [31].
Fat and red meat — In the same cohort study, the risk of diverticular disease was
significantly increased with diets that were low in fiber and were high in total fat or red
meat as compared with diets that were low in both fiber and total fat or red meat (RR
2.35 and 3.32, respectively) [31].
Seeds and nuts — Nut, corn, and popcorn consumption arenot associated with an
increase in risk of diverticulosis, diverticulitis or diverticular bleeding. In a large
observational study (The Health Professionals Follow-up Study) that included 47,228
men between the ages of 40 and 75 years, there was an inverse association between the
amount of nut and popcorn consumption and the risk of diverticulitis (HR nuts 0.8, 95%
CI 0.63-1.01; HR popcorn 0.72, 95% CI 0.56-0.92) [43]. In addition, no association was
found between consumption of corn and diverticulitis or between nut, popcorn, or corn
consumption and diverticular bleeding or uncomplicated diverticulosis.
Physical activity — It is unclear if lack of vigorous exercise is a risk factor for
diverticular disease. However, vigorous physical activity appears to reduce the risk of
diverticulitis and diverticular bleeding. In a prospective study of approximately 48,000
men aged 40 to 75 who were free of known colonic disease at baseline, the risk of
developing symptomatic diverticular disease was inversely related to overall physical
activity (RR 0.63 for highest versus lowest extremes) after adjustment for age and dietary
fat and fiber [44]. Most of the decrease in risk with exercise was associated with
vigorous activity such as jogging and running. Men in the lowest quintile for both dietary
fiber and physical activity had an increased risk of symptomatic diverticular disease as
compared with men in the highest quintile for both (RR 2.56, 95% CI 1.36-4.82).
Obesity — Obesity has been associated with an increase in risk of both diverticulitis
and diverticular bleeding. In a large, prospective cohort study of 47,228 male health
professionals, there were 801 incident cases of diverticulitis and 383 cases of diverticular
bleeding during 18 years of follow-up [45]. The risk of diverticulitis and diverticular
bleeding was significantly higher in those with the highest quintile of waist
circumference as compared with the lowest (RR diverticulitis 1.56, 95% CI 1.18-2.07;
RR diverticular bleeding 1.96, 95% CI 1.30-2.97).
Other — Current smokers appear to be at increased risk for perforated diverticulitis and
a diverticular abscess as compared with nonsmokers (OR 1.89, 95% CI 1.15-3.10) [46].
Caffeine and alcohol are not associated with an increased risk for symptomatic
diverticular disease. Read more

1)
0/1
Which sign on physical examination is MOST predictive in diagnosing appendicitis?

(Select 1)(1pts)

Presence of a Rovsing sign

RLQ tenderness and rigidity

Presence of a psoas sign

Temperature above 38.3°C (101°F)


Presence of a obturator sign

The answer is B, RLQ tenderness and rigidity. Signs on physical examination that are
most predictive of appendicitis are RLQ tenderness and rigidity. Signs that demonstrated
a small effect to discriminate appendicitis were other peritoneal signs (rebound
tenderness, guarding, percussion tenderness), temperature above 38.3°C (101°F), and the
presence of a psoas sign.11,44,45 Signs that do not help in diagnosing appendicitis
include tenderness on rectal examination, increased local skin temperature of RLQ, and
Rovsing sign (ie, pain in the RLQ upon palpation of the LLQ).Read more

Feedback on your answer


Collapse
2)
0/1
Treatment for all patients with acute nonperforated appendicitis must include:

(Select 1)(1pts)

Witholding analgesia

Immediate antibiotics

Blood cultures

Surgery within 12 hours

All of the above

The answer is B, Immediate antibiotics. The treatment of acute appendicitis in the ED


consists of 3 major aspects: 1) supportive care, 2) antibiotic therapy, and 3) definitive
treatment as decided upon by a surgical consultation. Supportive care is to be tailored to
the patient’s symptoms. Most often, patients with appendicitis should receive IV fluids
(lactated Ringer or normal saline), should receive symptomatic treatment (eg, antiemetics
and analgesia), and should remain NPO. Choice of analgesia will depend on the personal
preference of the provider; however, IV opiates are often considered first-line treatment
in possible appendicitis cases.120 Multiple prospective randomized trials and a Cochrane
Database review on analgesia in patients with acute abdominal pain have all concluded
that opiates improve patients’ cooperation, relieve their pain, and do not mask the
physical signs of appendicitis. Unless contraindicated, it is unacceptable to withhold
analgesia from patients presenting with acute abdominal pain.121-124 In the rare case of
acute appendicitis causing septic shock, treatment is guided by the Surviving Sepsis
Campaign International Guidelines of Severe Sepsis and Septic Shock.
Administer antibiotics promptly upon making the diagnosis of appendicitis.126 A
Cochrane database review of 45 studies involving 9576 patients found that antibiotic
administration in appendicitis reduces the incidence of both postoperative wound
infection and intra-abdominal abscess formation when compared to placebo.127
Interestingly, these studies demonstrated no difference in the postoperative histologic
findings of appendicitis in the antibiotic versus placebo groups.
Recommendations on definitive management for acute appendicitis depend on the
presence or absence of complications. Treatment for nonperforated appendicitis with a
well-circumscribed abscess is nonoperative and consists of IV antibiotics and
percutaneous drainage. The management recommendations for nonperforated
appendicitis without abscess or perforated appendicitis is urgent operative intervention.
Conservative management for appendicitis without abscess has become a controversial
topic recently. Conservative therapy consists of hospitalization for IV antibiotics rather
than emergent surgical intervention. Although Dr. Coldrey published a protocol in 1956
involving exclusive antibiotic treatment of appendicitis lasting more than 24 hours with
great personal success, only in the last 20 years has the idea of conservative treatment
been highlighted by multiple prospective trials and meta-analyses. The benefit of
conservative treatment is rooted in the fact that there is significant perioperative
morbidity associated with an appendectomy. 28,135 On average, the rate of
complications from appendectomies is approximately 4.6% and includes small bowel
obstruction, wound infection, abscess formation, and adhesions. Read more
Feedback on your answer
Collapse
3)
1/1
You are seeing a 31-year old male who presents with 24 hours of progressively
worsening lower abdominal pain, vomiting and chills. He denies past medical history and
denies prior surgery.

On exam his heart rate is 95 bpm, BP is 129/80, oral temp 100.2 F and O2 sat is 100% on
room air. Upon palpation he has right lower quadrant pain when you palpated his left
lower quadrant. His labs show a WBC of 9,700mm3 with no bands.

True or False: A normal white blood cell count effectively rules out appendicitis in this
patient.
(1pts)

True

False Correct

False: Appropriate diagnostic testing varies based on the clinical situation. A complete
blood count is appropriate if infection or blood loss is suspected. One study of patients 15
to 83 years of age with suspected appendicitis found that a white blood cell count greater
than 10,000 per mm3 (10 × 10 9 per L) was 77 percent sensitive and 63 percent specific
for the diagnosis (LR+ = 2.1, LR– = 0.37). Thus, nearly one in four patients with
appendicitis does not have an elevated white blood cell count. Read More
Feedback on your answer
Collapse
4)
0/1
Neoplastic lesions of the appendix are found in as many as 5% of specimens obtained
with routine appendectomy for acute appendicitis. Most are benign. Preoperative
detection of such conditions is rare, and intraoperative diagnosis is made in fewer than
50% of cases. Which of the following statement regarding neoplastic lesions of the
appendix is true?
(Select 1)(1pts)

Metastasis to the appendix is common

Adenocarcinoma is the most common type represting >80%

The incidence of malignancy in the appendix is 1.35%.

If the tumor is less than 8 cm in diameter, is located within the body or


the tip of the appendix, and has not metastasized, appendectomy is the
treatment of choice.

All of the above

None of the above

Answer C is the only true statement. Neoplastic lesions of the appendix are found in as
many as 5% of specimens obtained with routine appendectomy for acute appendicitis.18-
21 Most are benign. Preoperative detection of such conditions is rare, and intraoperative
diagnosis is made in fewer than 50% of cases. Appendectomy alone may be curative for
appendiceal mucocele, localized pseudomyxoma peritonei, most appendiceal carcinoids,
and other benign tumors. Definitive management of an appendiceal mass unexpectedly
encountered during exploration for clinically suspected acute appendicitis depends on
whether the tumor is carcinoid, its size and location, the presence or absence of metastatic
disease, and histologic and immunohistochemical findings.
Benign neoplasms of the appendix include mucosal hyper-plasia or metaplasia,
leiomyomas, neuromas, lipomas, angiomas, and other rare lesions. Appendiceal
adenomas tend to be diffuse and to have a predominant villous character. Mucus-
producing cystadenomas predispose to appendiceal mucocele, sometimes accompanied
by localized pseudomyxoma peritonei. These lesions are rarely symptomatic and are
often encountered incidentally during operation; however, they may also be clinically
manifested as acute appendicitis, torsion, intussusception, ureteral obstruction, or another
acute condition. If the base of the appendix is free of disease, appendectomy alone is
sufficient treatment.
Malignant tumors of the appendix primarily consist of carcinoids and adenocarcinomas;
all together, they account for 0.5% of all gastrointestinal malignancies. The incidence of
malignancy in the appendix is 1.35%. Metastasis to the appendix is rare. Carcinoids are
substantially more common than adenocarcinomas in the appendix: as many as 80% of
all appendiceal masses are carcinoid tumors. Overall, carcinoid tumors are found in 0.5%
of all appendiceal specimens, and appendiceal carcinoid tumors account for 18.9% of all
carcinoid lesions.23 These tumors are predominantly of neural cellular origin and have a
better prognosis than all other intestinal carcinoid tumors, which typically are of mucosal
cellular origin. If the tumor is less than 2 cm in diameter, is located within the body or the
tip of the appendix, and has not metastasized, appendectomy is the treatment of choice. If
the lesion is at the base of the appendix, is larger than 2 cm in diameter, or has
metastasized, right hemicolectomy is indicated. In addition, secondary right
hemicolectomy is indicated if the tumor is invasive, if mucin production is noted, or if the
tumor is found to be of mucosal cellular origin at the final pathologic examination.24,25
Patients with metastatic appendiceal carcinoid tumors appear to have a far better
prognosis than those with other types of metastatic cancers.24 Therefore, hepatic
debulking for symptomatic control is indicated and justified in cases of liver metastasis.
Figure 8. Laparoscopic appendectomy. The mesoappendix having been divided (a), the
base of the appendix is cleared circumferentially and divided with a gastrointestinal
anastomosis stapler (b). Read more
Feedback on your answer
Collapse
5)
1/1
You are seeing a 7-year old boy in the ER accompanied by his mother. She notes that he
has been complaining of a fever aand lower abdominal pain for the last 24 hours. She
also notes that he has not eaten anything since yesterday morning as he vomits everything
he attempts to eat. His vitals show a temp is 101.2 F, HR 120, BP 98/70, RR 18 and O2
sat of 99% on room air. On exam he has moderate periumbilical tenderness to light
palpation as well as in his right lower quadrant. His lab work is pending and the surgeon
on call tells you to get some radiology study for confirmation. What is the recommended
initial imaging modality for this patient?
(Select 1)(1pts)

CT scan abdomen/pelvis with IV and PO contrast only

Ultrasound of the Right lower quadrant

MRI abdomen/pelvis

CT scan abdomen/pelvis without IV and PO contrast only


The answer is B, Ultrasound with its lack of ionizing radiation should be the investigation
of choice in young patients, and is effective in competent hands in identifying abnormal
appendixes, especially in thin patients. However, the identification of a normal appendix
is more problematic, and in many instances, appendicitis cannot be ruled out.
The technique used is known as graded compression, and uses the linear probe over
the site of maximal thickness, with gradual increasing pressure exerted to displace normal
overlying bowel gas.
Findings supportive of the diagnosis of appendicitis include 4:
 aperistaltic, noncompressible, dilated appendix ( >6mm outer diameter)
 distinct appendiceal wall layers
 target appearance (axial section)
 appendicolith
 periappendiceal fluid collection
 echogenic prominent pericaecal fat
 confirming that the structure visualised is the appendix is clearly essential
and requires demonstration of it being blind ending and arising from the
base of the caecum
Read more

Feedback on your answer


Collapse
Unit 2
6)
0/1
You are consulted to a 38-year-old black male for evaluation of his preoperative
cardiovascular risk. He has a 20-year history of type 1 diabetes mellitus and hypertension
and is undergoing preoperative evaluation for renal transplantation secondary to
uncontrolled hypertension and diabetic nephropathy. His current blood pressure is 142/85
mm Hg. He currently smokes one half of a pack of cigarettes daily. His
electrocardiogram is consistent with left ventricular hypertrophy and left atrial
enlargement. Which of the following is the most appropriate recommendation at this
time?
(1pts)

No further evaluation is needed, he is cleared for surgery without further


testing

Pharmacologic stress nuclear study

24-hour electrocardiographic holter monitor

Coronary angiography
The answer is B, Pharmacologic stress nuclear study. In general, the indications for
noninvasive cardiac testing are the same for patients undergoing surgery as for those who
are not. All symptomatic patients should be evaluated for cardiac ischemia, usually with
cardiac stress testing, even if their symptoms are atypical. Patients undergoing a low- or
intermediate risk surgery who have at most minor clinical risk predictors (advanced age,
abnormal ECG, rhythm other than sinus, low functional capacity, history of stroke, or
uncontrolled hyper tension) may proceed to surgery without preoperative cardiac testing.
Likewise, patients with intermediate clinical risk predictors (mild angina, diabetes
mellitus, compensated heart failure, previous myocardial infarction, or renal
insufficiency) who have good exercise capacity also do not need further preoperative
cardiac testing before a low- or intermediate-risk procedure. However, in the absence of
high-quality data, some experts recommend that those with intermediate clinical
predictors who have poor functional status and are undergoing an intermediate-risk
procedure, as well as those with intermediate clinical predictors and good functional
status who are undergoing a high-risk procedure, should have noninvasive testing
performed before surgery.
Coronary revascularization is rarely needed just to get a patient through surgery, but it
may be warranted in selected patients with CAD undergoing noncardiac surgery if they
would have required the procedure anyway. However, the only large randomized trial on
the subject showed no benefit for revascularization (40). Avoid preoperative coronary
sten ting to avoid the risk for perioperative stent thrombosis. Patients with recently placed
coronary stents are at risk for perioperative stent thrombosis. For bare-metal stents, the
risk diminishes 4 to 6 weeks after implantation; for drug-eluting stents, an elevated risk
for rethrombosis persists for at least 12 months after placement because of delayed
endothelialization. Because these patients require antiplatelet treatment to mitigate this
risk, purely elective procedures should be delayed accordingly, and the interval without
antiplatelet therapy should be minimized when surgery is required within the window of
vulnerability. Read more

Feedback on your answer


Collapse
7)
1/1
There is a high risk of pathogen transfer during surgery. This is a risk from which both
the patient and the surgical team must be protected. The risk can be reduced by using
protective barriers, such as surgical gloves. A 2002 Cochrane Review concluded that
wearing two pairs of latex gloves can further reduce the risk of contamination to what %?
(Select 1)(1pts)

20%

15%
10%

5%

The answer is D, There is a high risk of pathogen transfer during surgery. This is a risk
from which both the patient and the surgical team must be protected. The risk can be
reduced by using protective barriers, such as surgical gloves. A 2002 Cochrane Review
concluded that wearing two pairs of latex gloves significantly reduced the risk of hand
exposure. Wearing two pairs of surgical gloves rather than a single pair provides an
additional barrier and further reduces the risk of contamination. The rate of single-glove
perforations is approximately 15%. The rate of inner glove perforation when two pairs of
gloves are worn is approximately 5%. Read more
Feedback on your answer
Collapse
8)
0/1
What is the risk for serious medical complications from surgery in healthy patients?
(Select 1)(1pts)

<0.1%

<0.5-1%

2-3%

5-6%

The answer is A, <0.1%. Healthy patients have significant physiologic reserves and
tolerate major and minor surgical procedures well. The risk for serious medical
complications from surgery is less than 0.1% overall in healthy patients. When evaluating
healthy patients before surgery, physicians should use a focused history and physical
examination to predict the risk for serious medical complications. Researchers collected
information at 16 and 72 hours and 30 days for 38 598 patients undergoing 45 090
consecutive procedures and found that only 33 (0.08%) had major morbidity or mortality.
Of 4 deaths that occurred within 30 days of the procedure, 2 were due to myocardial
infarction and 2 were due to automobile accidents unlikely to be related to surgery. More
than one third of major morbidity occurred 48 hours or more after surgery. Read more
Feedback on your answer
Collapse
9)
0/1
A 72-year-old African-American male comes to your office for surgical clearance for an
elective left hemicolectomy for recurrent diverticulitis. His past medical history includes
an inferior-wall myocardial infarction approximately 12 months ago. Since then he had a
stress test that was normal 6 months ago. Currently, the patient feels well, walks while
playing nine holes of golf weekly, and is able to walk up a flight of stairs without chest
pain or significant dyspnea. Findings are normal on a physical examination.
Which one of the following would be most appropriate for this patient prior to surgery?
(1pts)

A 12-lead resting EKG

Another Nuclear Stress test

A dobutamine stress echocardiogram

A persantine stressed nuclear tracer study (technetium or thallium)

The answer is A, A 12-lead resting EKG. The current recommendations from the
American College of Cardiology and the American Heart Association on preoperative
clearance for noncardiac surgery state that preoperative intervention is rarely needed to
lower surgical risk. Patients who are not currently experiencing unstable coronary
syndrome, severe valvular disease, uncompensated congestive heart failure, or a
significant arrhythmia are not considered at high risk, and should be evaluated for most
surgery primarily on the basis of their functional status. If these patients are capable of
moderate activity (greater than 4 METs) without cardiac symptoms, they can be cleared
with no stress testing or coronary angiography for an elective minor or intermediate-risk
operation such as the one this patient is to undergo. A resting 12-lead EKG is
recommended for males over 45, females over 55, and patients with diabetes, symptoms
of chest pain, or a previous history of cardiac disease. Ref: ACC/AHA guideline update
for perioperative cardiovascular evaluation for noncardiac surgery--Executive summary:
A report of the American College of Cardiology/American Heart Association Task Force
on Practice Guidelines (Committee to Update the 1996 Guidelines on Perioperative
Cardiovascular Evaluation for Noncardiac Surgery). Circulation 2002;105(10):1257-
1267. Read more here
Feedback on your answer
Collapse
10)
0/1
Where is the surgical amphitheaters known as "The Ether Dome" located?
(Select 1)(1pts)

Beaumont Hospital in Dublin, Ireland

Guy's Hospital in London, England


Massachusetts General Hospital

Padua Hospital in Padua, Italy

The answer is C, Massachusetts General Hospital. The configuration and design of the
modern OR were derived from the anatomic amphitheater of late-Renaissance Italy.
Following the publication of Andreas Vesalius's De Fabrica Humani Corporis in 1543,
the study of human anatomy and dissection emerged as central elements in medical and
particularly in surgical education. This innovation reflected an estrangement from the
previous galenic curriculum, the beginnings of the rejection of scholasticism as the
dominating heuristic in medical investigation, and a sentinel movement toward
empiricism and what we now recognize as the beginnings of modern science. The first
documented dissecting amphitheater, constructed after the classical Greek style of tiered
seats or stands surrounding the stage, was built in Padua in the mid-16th century.1 Others
in Europe soon followed. Although early surgeons generally operated in the patient's
home or on the battlefield, and in hospitals only infrequently, by the first decade of the
18th century, the anatomic amphitheater could serve as an operating theater. In 1707,
Pierre Dionis, a Parisian anatomist and surgeon, included an etching of his surgical
amphitheater in his textbook of surgery. In England, at about the same time, the new
hospitals built to replace the Catholic institutions destroyed by Henry VIII and serve the
needs of the growing urban areas included operating theaters. Guy's Hospital built its
theater on the top floor so that it could be lit by skylights.2-4 A magnificently preserved
example of premodern design can be seen in Dublin, at the Royal College of Surgeons, in
the anatomic building and dissecting rooms that remain in use today. The effective
deployment of natural lighting is remarkable and instructive. American examples of 19th
century surgical amphitheaters also survive. The Ether Dome at the Massachusetts
General Hospital, which derived its name from the effective demonstration of ether as an
anesthetic, may be the best known, but there are others of equal historical
significance. Read more
Feedback on your answer
Collapse
Unit 5
11)
0/1
Initial wound management after an operative procedure generally entails placement of a
sterile dressing to cover the incision. The traditional recommendation has been to keep
this dressing in place and dry for what period of time?

(Select 1)(1pts)

12 hours

24 hours
48 hours

72 hours

All of the above

None of the above

The answer is C, 48 hours. Initial wound management after an operative procedure


generally entails placement of a sterile dressing to cover the incision. The traditional
recommendation has been to keep this dressing in place and dry for the first 48 hours
after operation; because epithelialization is known to take place within approximately this
period, the assumption is that this measure will reduce the risk of wound infection.
Although most surgeons still follow this practice, especially in general surgical cases,
supporting data from randomized clinical studies are lacking. In addition, several small
studies that evaluated early showering with closed surgical incisions found no increases
in the rate of infection or dehiscence.8,9 It should be kept in mind, however, that these
small studies looked primarily at soft tissue and other minor skin incisions that did not
involve fascia. Thus, even though the traditional approach to initial dressing management
is not strongly supported, the data currently available are insufficient to indicate that it
should be changed.
Wet-to-dry dressings are used in a variety of settings. In a surgical context, they are most
often applied to a wound that cannot be closed primarily as a consequence of
contamination or inability to approximate the skin edges. Wet-to-dry dressings provide a
moist environment that promotes granulation and wound closure by secondary intention.
Moreover, their removal and replacement cause debridement of excess exudate or
unhealthy superficial tissue. Postoperative orders should specify the frequency of
dressing changes and the solution used to provide dampness. For most clean open
incisions, twice-daily dressing changes using normal saline solution represent the most
common approach. If there is excess wound exudate to be debrided, dressing changes
may be performed more frequently. If there is particular concern about wound
contamination or superficial colonization of organisms, substitution of dilute Dakin
solution for normal saline may be considered. Read more

Feedback on your answer


Collapse
12)
1/1
Which of the following is NOT true regarding post operative placement of foley
catheters?
(1pts)
As many as 40% of all hospital infections are UTIs

Only 10% of hospital acquired UTIs are associated with urinary catheters

After many procedures, patients initially find it extremely difficult or


impossible to mobilize for urination, and a urinary catheter may be quite
helpful during this time

In the setting of bladder or genitourinary surgery, they are often


employed to decompress the system so that it will heal more readily

Answer B is false. Urinary catheters can serve a large variety of purposes. In the setting
of bladder or genitourinary surgery, they are often employed to decompress the system so
that it will heal more readily. After general surgical procedures—and many other surgical
procedures as well—they are used to provide accurate measurements of volume output
and thus, indirectly, to give some indication of the patient's overall volume and
resuscitation status. Furthermore, after many procedures, patients initially find it
extremely difficult or impossible to mobilize for urination, and a urinary catheter may be
quite helpful during this time.
Their utility and importance notwithstanding, urinary catheters are
associated with the development of nosocomial urinary tract infections
(UTIs). As many as 40% of all hospital infections are UTIs, and
80 to 90% of these UTIs are associated with urinary
catheters.Accordingly, when catheterization is no longer deemed
necessary, prompt removal is indicated. As a rule, orders specifically
pertaining to urinary catheter care are few, typically including gravity
drainage, flushing to maintain patency (if warranted), and removal
when appropriate. At times, irrigation is employed after urologic
procedures or for the management of certain infectious agents. Read
more
Feedback on your answer
Collapse
13)
0/1
Same-day surgery is appropriate for patients who (1) have few or no comorbid medical
conditions and (2) are undergoing a procedure that involves short-duration anesthesia or
local anesthesia plus sedation and that carries a low likelihood of urgent complications.
Operations commonly performed on a same-day basis include which of the following?
(Select 1)(1pts)

inguinal or umbilical hernia repair


simple laparoscopic cholecystectomy

breast biopsy

small subcutaneous procedures

All of the above

None of the above

The answer is all of the above. Same-day surgery is appropriate for patients who (1) have
few or no comorbid medical conditions and (2) are undergoing a procedure that involves
short-duration anesthesia or local anesthesia plus sedation and that carries a low
likelihood of urgent complications. Operations commonly performed on a same-day basis
include inguinal or umbilical hernia repair, simple laparoscopic cholecystectomy, breast
biopsy, and small subcutaneous procedures. The growth in the performance of minor and
same-day procedures has led to the development of various types of short-stay units or
wards. The level of care provided by a short-stay ward is generally equivalent to that
provided by a regular nursing ward; however, the anticipated duration of care is
substantially shorter, typically ranging from several hours to a maximum of 48 hours.
Short-stay wards also undergo some modifications to facilitate the use of streamlined
teaching protocols designed to prepare patients for home care. Many hospitals now have
short-stay units, as do some independent surgery centers. Read more
Feedback on your answer
Collapse
14)
0/1
The most common method of monitoring patients' nutritional status with nutritional
supplementation is to measure the serum albumin and prealbumin (transthyretin)
concentrations. What is the half life of prealbumin?
(Select 1)(1pts)

1-2 days

3-5 days

8-10 days

14-20 days

The answer is A, 1-2 days. Patients who require assistance with nutritional intake should
be monitored to determine whether the interventions being carried out are having the
desired effect. The most common method of monitoring patients' nutritional status with
nutritional supplementation is to measure the serum albumin and prealbumin
(transthyretin) concentrations. Albumin has a half-life of approximately 14 to 20 days
and thus serves as a marker of longer-term nutritional status. A value lower than 2.2 g/dL
is generally considered to represent severe malnutrition, but even somewhat higher values
(< 3.0 g/dL) have been associated with poorer outcomes after elective surgery. Although
the serum albumin concentration is a commonly used marker, it is not always a reliable
one. Because of albumin's relatively long half-life, the serum concentration does not
reflect the patient's more recent nutritional status. In addition, the measured concentration
can change quickly in response to the infusion of exogenous albumin or to the
development of dehydration, sepsis, and liver disease despite adequate nutrition. Pre
albumin is a separate serum protein that has a halflife of approximately 24 to 48 hours
and thus can serve as a marker of current and more recent nutritional status. Like the
albumin concentration, the prealbumin concentration can be affected by liver and renal
disease. Overall, however, it is more immediately reliable in following the effects of
nutritional intervention. Read more
Feedback on your answer
Collapse
15)
1/1
Multiple randomized clinical trials have now demonstrated that routine use of drains after
elective operations—including appendectomies and colorectal, hepatic, thyroid, and
parathyroid procedures-- reduces which of the following?
(Select 1)(1pts)

anastomotic complications

infection rates

seroma formation

bleeding

All of the above

None of the above

The answer is C, Drains and tubes are placed in a wide variety of locations for a number
of different purposes—in particular, drainage of purulent materials, serum, or blood from
body cavities. Several types are commonly used, including soft gravity drains (e.g.,
Penrose), closedsuction drains (e.g., Hemovac, Jackson-Pratt, and Blake), and sump
drains, which draw air into one lumen and extract fluid via a companion lumen.
Traditionally, surgeons have often made the decision to place a drain on the basis of their
surgical training and practice habits rather than of any firm evidence that drainage is
warranted. Multiple randomized clinical trials have now demonstrated that routine use of
drains after elective operations—including appendectomies and colorectal, hepatic,
thyroid, and parathyroid procedures—does not prevent anastomotic and other
complications (although it does reduce seroma formation). Consequently, it is
recommended that drains, like NG tubes, be employed selectively.3- 5Once a drain is in
place, specific orders must be issued for its maintenance. These include use of gravity or
suction (and the means by which suction is to be provided if ordered), management and
measurement of output, stripping, and care around the drain exit site. Read more
Feedback on your answer
Collapse
Unit 1
16)
0/1
You seeing a 68-year-old white male in your office for a pre-operative evaluation before
cataract replacement and intraocular lens implantation that is scheduled two weeks from
now. He has been your patient for about 12 years now and has a past medical history of
osteoarthritis. His medications include alleve as needed and a multivitamin. He denies
new complaints. He notes being able to climb the stairs in his house and mow the grass
without getting short of breath. Which of the following laboratory evaluations is
indicated in preoperative evaluation of this patient?

( 1pts extra credit)

No laboratory tests are needed

Basic Metabolic Panel (electrolytes)

Prothrombin time

Urinalysis

Complete blood count with platelet count

The answer is A. Preoperative laboratory testing should be limited in healthy patients,


especially those undergoing minor procedures. For minor surgical procedures, especially
cataract surgery, routine testing is not indicated in the presence of a normal history and
physical examination. In a multicenter, randomized, controlled trial comparing routine
with selective testing before cataract surgery, the combined event rate of death,
hospitalization, or other complications was identical (3.13%) in both groups (17). All
patients scheduled for surgery be should be considered for preoperative evaluation. For
very low-risk procedures, such as dental extractions or cataract surgery, the evaluation
may only involve the oral surgeon or ophthalmologist confirming the lack of significant
risk factors. For more complex procedures, evaluation by a physician experienced in
preoperative assessment may be judicious. Low-risk (<1% perioperative risk for death or
MI): endoscopic and superficial procedures, cataract surgery, breast surgery, dental
procedures, and ambulatory surgery. Etrocardiography (ECG) is recommended to test for
the presence of asymptomatic cardiac disease in men older than age 40 and women older
than age 50 having major surgery. For patients undergoing minor surgery in whom the
history and physical examination is normal, a routine ECG is unlikely to improve
outcomes and can be omitted. Read more on Preoperative Evaluation
Feedback on your answer
Collapse
17)
1/1
All patients with at least one uncomplicated diverticulitis are recommened for operative
management with colectomy to prevent worse recurrent attacks in the future.
( 1pts extra credit)

True

False Correct

False: This would be true if it was an episode of complicated diverticulitis. The decision
to recommend elective sigmoid colectomy after recovery from acute diverticulitis should
be made on a case-by-case basis. Level of Evidence: III; Grade of Recommendation: B.
After successful medical treatment of an episode of acute diverticulitis, careful judgment
is required concerning whether to proceed with subsequent elective colon resection. After
one attack, about a third of patients will have a later second attack of acute diverticulitis,
and after a second episode, a further third will have yet another attack. The decision to
recommend surgery should be influenced by the age and medical condition of the patient,
the frequency and severity of the attack(s), and whether there are persistent symptoms
after the acute episode. Most patients who present with complicated diverticulitis do so at
the time of their first attack, therefore, a policy of elective colon resection after recovery
from uncomplicated acute diverticulitis might not decrease the likelihood of later
emergency surgery or overall mortality. Therefore, the number of attacks of
uncomplicated diverticulitis is not necessarily an overriding factor in defining the
appropriateness of surgery. As noted earlier, CT graded severity of a first attack is a
predictor of an adverse natural history and may be helpful in determining the need for
surgery. 18 Inability to exclude carcinoma is another appropriate indication for
colectomy. There is no clear consensus regarding whether younger patients (younger than
aged 50 years) treated for diverticulitis are at increased risk of complications or recurrent
attacks. Nevertheless, because of their longer life span, younger patients will have a
higher cumulative risk for recurrent diverticulitis, even if the virulence of their disease is
no different than that of older patients. Elective colon resection should typically be
advised if an episode of complicated diverticulitis is treated nonoperatively. Level of
Evidence: III; Grade of Recommendation: B. After percutaneous drainage of a
diverticular abscess, a later colectomy usually should be planned, because 41 percent of
patients will otherwise develop severe recurrent sepsis. Read more

Feedback on your answer


Collapse
18)
0/1
Following abdominal surgery, how many days does a postoperative ileus usually last?
( 1pts extra credit)

1 day

3 days

6 days

9 days

The answer is B. Ileus, or the cessation of bowel function and transit in the absence of
mechanical obstruction, is a common postoperative phenomenon particularly after
abdominal surgery and occurs in virtually all patients undergoing bowel resection. Its
causes are multifactorial, including an overall increase in sympathetic tone, the
inflammatory response to bowel as a result of direct manipulation, and the well-known
effects of narcotic analgesics on GI motility. Although the typical duration of
postoperative ileus averages around 3 days, in up to 32% of patients, return of bowel
function may be particularly delayed.Read more

Feedback on your answer


Collapse
19)
0/1

Numerous complications may arise in the postoperative period especially fevers,


pneumonia, urinary tract infections, and wound infections. Which of the following
statements is FALSE regarding post operative fevers/infections?

( 1pts extra credit)

Postoperative temperature elevations are quite common, occurring in


nearly one third of patients after surgery
Given the high mortality associated with post operative infections, a
shotgun approach to the workup of postoperative fever is usually
warranted

Studies have found the incidence of postoperative pneumonia to be


approximately 1.5%

Noninfectious causes of postoperative fevers include components of the


inflammatory response to surgical intervention, reabsorption of
hematomas, and (possibly) atelectasis

Answer B is False as a shotgun approach is not recommended.Numerous complications


may arise in the postoperative period. Many of these are specific to particular operative
procedures and hence are best discussed in connection with those procedures. Many
others, however, may develop after virtually any operation and thus warrant a general
discussion in this chapter (see below). Prompt discovery and treatment of these latter
complications rely heavily on a sufficiently high index of suspicion.
Postoperative Fever
Postoperative temperature elevations are quite common, occurring in nearly one third of
patients after surgery. Only a relatively small number of these are actually caused by
infection. Fevers that are caused by infections (e.g., pneumonias, wound infections, or
UTIs) tend to reach higher temperatures (> 38.5°C [101.3°F]), usually are associated with
moderate elevation of the white blood cell (WBC) count 3 or more days after operation,
and typically extend over consecutive days. Noninfectious causes of postoperative fevers
include components of the inflammatory response to surgical intervention, reabsorption
of hematomas, and (possibly) atelectasis.28
Beyond checking the WBC count, a shotgun approach to the workup of postoperative
fever probably is not warranted. A focused approach based on well-directed questioning
and a careful physical examination is more likely to obtain the highest diagnostic yield.
Coughing, sputum production, and respiratory effort should be noted, and the lungs
should be
auscultated for rales. All incisions should be inspected for erythema and drainage, and
current and recent IV sites should be checked for evidence of cellulitis. If a central line
has been placed, particularly if it has been in place for several days, the possibility of a
line infection should be considered. Patients who have undergone prolonged NG
intubation may have sinusitis, which is most readily diagnosed through computed
tomography (CT) of the sinuses. Further workup for fever may include, as indicated,
chest x-ray, sputum cultures, urinalysis, blood cultures, or CT of the abdomen (after
procedures involving laparotomy—especially bowel resections—where intra-abdominal
abscess is a possible complication).
Pneumonia
Respiratory infections in the postoperative period are generally considered nosocomial
pneumonias and, as such, are potentially serious complications. The estimated incidence
of postoperative pneumonia varies significantly, with many estimates tending to run high.
A 2001 study of more than 160,000 patients undergoing major noncardiac surgery
provided what may be a reasonable overall figure, finding the incidence of postoperative
pneumonia to be approximately 1.5%.29 In the 2,466 patients with pneumonia, the 30-
day mortality was 21%. Thoracic procedures, upper abdominal procedures, abdominal
aortic aneurysm repair, peripheral vascular procedures, and neurosurgical procedures
were all identified as placing patients at significantly increased risk for pneumonia.
Patient-specific risk factors included age greater than 60 years, recent alcohol use,
dependent functional status, long-term steroid use, and a 10% weight loss in the 6 months
preceding the operation.29
The diagnosis of postoperative pneumonia is based on the usual combination of index of
suspicion, findings from the history and physical examination (e.g., fever, shortness of
breath, hypoxia, productive cough, and rales on lung auscultation), imaging, and
laboratory evaluation. Appropriate workup, directed by the clinical findings, typically
starts with chest x-rays (preferably in both posteroanterior and lateral views, if possible)
and sputum cultures, sometimes accompanied by CT scanning of the chest and, possibly,
bronchoscopy with bronchoalveolar lavage (which may be useful in directing antibiotic
therapy when sputum cultures are non-diagnostic). Empirical broad-spectrum antibiotic
therapy is typically initiated before the causative organism is identified; this practice has
been shown to reduce mortality. Piperacillintazobactam, which is effective
againstPseudomonas aeruginosa, is commonly used for this purpose; however,
when Gram staining of the sputum identifies gram-positive cocci, vancomycin or
linezolid may be used initially instead.30 Once the causative organism is identified,
specific antibiotic therapy directed at that organism is indicated, as in treatment of other
infectious processes. Drainage of parapneumonic effusions may also be necessary, and
this measure may be helpful in diagnosing or preventing the development of
empyema. Read more
Feedback on your answer
Collapse
20)
0/1
Maximizing which the following lung parameters is the most important factor in
prevention of postoperative pulmonary complications?
( 1pts extra credit)

Tital volume

Inspiratory reserve volume

Vital capacity

Functional reserve capacity

Inspiratory capacity
Functional reserve capacity (FRC) is composed of expiratory reserve volume plus
residual volume. It is essential to maximize FRC in the post op period to ensure that it
will be greater than closing volume. Closing volume is the lung volume at which small
airway closure occurs. Maximizing FRC reduces atelectasis and lessens the incidence of
hypoxemia and pneumonia. Maneuvers to increase FRC include ambulation, incentive
spirometry, deep breathing, and intermittent positive pressure breathing (IPPB). During
anesthesia, the reduced FRC can contribute to airway closure during expiration and to a
compression atelectasis that in turn precipitates hypoxemia and infection.

Postoperative pulmonary complications follow logically as an extension of normal


perioperative pulmonary physiology. Reduced lung volume after surgery is a major factor
contributing to the development of postoperative pulmonary complications.
Thoracic and upper-abdominal surgery are associated with a reduction in lung volumes in
a restrictive pattern as follows:
●Vital capacity (VC) is reduced by 50 to 60 percent and may remain decreased for up to
one week.
●Functional residual capacity (FRC) is reduced by about 30 percent.
Diaphragmatic dysfunction appears to play the most important role in these changes;
postoperative pain and splinting are also factors. Reduction of the FRC below closing
volumes contributes to the risk of atelectasis, pneumonia,
andventilation/perfusion (V/Q) mismatching. Microatelectasis results in areas of
the lung that are perfused but not ventilated, leading to impaired gas exchange with
consequent postoperative hypoxemia.
A decrease in tidal volume, loss of sighing breaths, and increase in respiratory rate occur
after abdominal and thoracic surgery and contribute to the risk of complications. In
addition, residual effects of anesthetic agents and postoperative opioids both depress the
respiratory drive. Inhibition of cough and impairment of mucociliary clearance of
respiratory secretions are factors that contribute to the risk of postoperative
infection. Read more
Feedback on your answer
Collapse
Unit 4
21)
0/1
You are seeing a 45-year old obese male with type II diabetes who presents with acute
onset left lower quadrant abdominal pain, vomiting, pain with defecation, and
fever. Physical exam is normal except for his abdominal exam that shows left lower
quadrant tenderness without signs of peritonitis. His labs are remarkable for a WBC of
16,000 mm3. Which of the following choices is most likely to reduce repeat incidences of
this patients condition?
(Select 1)(1pts)
Routine daily stool softners or laxatives

Biannual colonoscopy

High fiber diet

Avoidance of nuts and legumes

The answer is C, High fiber diet. Environmental and lifestyle factors are important risk
factors for diverticular disease. Out of the following options only increasing fiber intake
has been shown to reduce episodes of diverticulitis. The role of fiber in the development
of diverticulosis is unclear. Several early studies suggested that low dietary fiber
predisposes to the development of diverticular disease, but other studies have been
conflicting [3,30-39]. Fiber also does not reduce symptoms in patients with
symptomatic uncomplicated diverticular disease [37,38,40,41]. However, dietary fiber
and a vegetarian diet may reduce the incidence of symptomatic diverticular disease by
decreasing intestinal inflammation and altering the intestinal microbiota [4,31,42].
In a cohort study that included over 47,000 men, after adjustment for age, energy-
adjusted total fat intake, and physical activity, total dietary fiber intake was noted to be
inversely associated with the risk of symptomatic diverticular disease (RR 0.58 highest
quintile versus lowest quintile for fiber intake) [31].
Fat and red meat — In the same cohort study, the risk of diverticular disease was
significantly increased with diets that were low in fiber and were high in total fat or red
meat as compared with diets that were low in both fiber and total fat or red meat (RR
2.35 and 3.32, respectively) [31].
Seeds and nuts — Nut, corn, and popcorn consumption arenot associated with an
increase in risk of diverticulosis, diverticulitis or diverticular bleeding. In a large
observational study (The Health Professionals Follow-up Study) that included 47,228
men between the ages of 40 and 75 years, there was an inverse association between the
amount of nut and popcorn consumption and the risk of diverticulitis (HR nuts 0.8, 95%
CI 0.63-1.01; HR popcorn 0.72, 95% CI 0.56-0.92) [43]. In addition, no association was
found between consumption of corn and diverticulitis or between nut, popcorn, or corn
consumption and diverticular bleeding or uncomplicated diverticulosis.
Physical activity — It is unclear if lack of vigorous exercise is a risk factor for
diverticular disease. However, vigorous physical activity appears to reduce the risk of
diverticulitis and diverticular bleeding. In a prospective study of approximately 48,000
men aged 40 to 75 who were free of known colonic disease at baseline, the risk of
developing symptomatic diverticular disease was inversely related to overall physical
activity (RR 0.63 for highest versus lowest extremes) after adjustment for age and dietary
fat and fiber [44]. Most of the decrease in risk with exercise was associated with
vigorous activity such as jogging and running. Men in the lowest quintile for both dietary
fiber and physical activity had an increased risk of symptomatic diverticular disease as
compared with men in the highest quintile for both (RR 2.56, 95% CI 1.36-4.82).
Obesity — Obesity has been associated with an increase in risk of both diverticulitis
and diverticular bleeding. In a large, prospective cohort study of 47,228 male health
professionals, there were 801 incident cases of diverticulitis and 383 cases of diverticular
bleeding during 18 years of follow-up [45]. The risk of diverticulitis and diverticular
bleeding was significantly higher in those with the highest quintile of waist
circumference as compared with the lowest (RR diverticulitis 1.56, 95% CI 1.18-2.07;
RR diverticular bleeding 1.96, 95% CI 1.30-2.97).
Other — Current smokers appear to be at increased risk for perforated diverticulitis and
a diverticular abscess as compared with nonsmokers (OR 1.89, 95% CI 1.15-3.10) [46].
Caffeine and alcohol are not associated with an increased risk for symptomatic
diverticular disease. Read more
Feedback on your answer
Collapse
22)
0/1
You are seeing a 58-year-old female who is presenting with two days of a low-grade
fever and increasing abdominal cramps that are exacerbated by bowel movements. She
began a course of augmentin 2 days ago that her PCP gave her but has had no relief of
her symptoms and is getting worse. She has not had nausea, vomiting, urinary symptoms,
or bloody stools. She has a past medical history of hypertension.

Her vitals show a temperature is 100.6°F, pulse is 75/min, and blood pressure is 150/80
mm Hg. Her lungs are clear to auscultation. Cardiac examination shows no
abnormalities. Her abdomen is soft, and there is tenderness to palpation of the left lower
quadrant with guarding but no rebound. Bowel sounds are normal. Her stool is brown,
and test for occult blood is negative. Her hemoglobin concentration is 14.5 g/dL,
leukocyte count is 15,800/mm3, and platelet count is 280,000/mm3; serum studies and
urinalysis show no abnormalities. Which of the following is the most appropriate next
step in diagnosis?
(Select 1)(1pts)

Examination of the stool for ova and parasites

Test of the stool for Clostridium difficile toxin

Xray Abdomen Series

CT scan of the abdomen/pelvis with IV and oral contrast

The answer is D, CT scan of the abdomen with IV and oral contrast. This patient is being
treated for diverticulitis and is not getting better. She needs a CT scan to evaluate for a
possible abscess. Read more

Feedback on your answer


Collapse
23)
1/1
You are seeing a 49 -ear old obese male who presents with two days of worsening left
sided abdominal pain. He denies fever or chills but does note painful bowel movements
that have streaks of blood. On exam you note moderate left lower quadrant tenderness to
palpation without rebound or guarding. His labs show a white blood cell count of
14,000mm3 and his CT scan of his abdomen/pelvis shows acute uncomplicated sigmoid
diverticulitis. Nonoperative treatment with oral antibiotics will resolve acute
uncomplicated diverticulitis in what percent of patients?
(Select 1)(1pts)

<25%

50%

80%

>95%

The best answer is B, 80%. Although most patients with acute diverticulitis can be treated
medically, approximately 15-20% will require surgery. Uncomplicated diverticulitis may
be managed as an outpatient (dietary modification and oral antibiotics) for those without
appreciable fever, excessive vomiting, or marked peritonitis, as long as there is the
opportunity for follow-up. The patient should be able to take liquids and antibiotics by
mouth. Hospitalization for treatment (dietary modification and intravenous antibiotics) is
usually best if the above conditions are not met, or if the patient fails to improve with
outpatient therapy. Antibiotics should be selected to treat the most common bacteria
found in the colon: gram-negative rods and anaerobic bacteria. Single and multiple
antibiotic regimens are equally effective, as long as both groups of organisms are
covered. Nonoperative treatment will resolve acute diverticulitis in 80
percent of patients, but approximately one-third will have a recurrent attack often
within one year. approximately 20% of patients with diverticulitis require surgical
treatment.30 Most surgical procedures are reserved for patients who experience recurrent
episodes of acute diverticulitis that necessitate treatment (inpatient or out-patient) or who
have complicated diverticulitis. The most common indication for elective resection is
recurrent attacks—that is, several episodes of acute diverticulitis documented by studies
such as CT. Rerecurrences may be more common than recurrences. In 2000, a task force
of the American Society of Colon and Rectal Surgeons recommended sigmoid resection
after two attacks of diverticulitis. A subsequent cost analysis using a Markov model
suggested that cost savings could be achieved if resection was done after three
attacks.33 There is a growing tendency to question arbitrary guidelines for surgical
management of recurrent attacks, with the exception of certain groups, such as
immunocompromised patients. Current practice guidelines state that the recommendation
to perform elective sigmoid resection after recovery from uncomplicated acute
diverticulitis should be made on a case-by-case basis. The decision-making process
should be influenced by the age and medical condition of the patient, the frequency and
severity of attacks, and the presence of symptoms after the acute attack. Elective
resection is generally recommended after an episode of complicated
diverticulitis.36 Efforts are made to time surgical treatment so that it takes place during a
quiescent period 8 to 10 weeks after the last attack. Barium enema or colonoscopy may
be employed to evaluate the diverticular disease and rule out carcinoma. The bowel can
then be prepared mechanically and with antibiotics (e.g., oral neomycin and
metronidazole on the day before operation).
2014. Scientific American Surgery. Hamilton, Ontario & Philadelphia, PA. Decker
Intellectual Properties Inc. ISSN 2368-2744. STAT!Ref Online Electronic Medical
Library. http://online.statref.com/Document.aspx?fxId=61&docId=1368. 10/20/2014
12:41:34 PM CDT (UTC -05:00).

Feedback on your answer


Collapse
24)
0/1
What percentage of diveriticulosis affects the sigmoid colon?
(Select 1)(1pts)

50%

65%

75%

95%

The answer is D. The prevalence of diverticulosis is age-dependent, increasing from less


than 20 percent at age 40 to 60 percent by age 60. Approximately 95 percent of patients
with diverticula have sigmoid diverticula. Diverticula are limited to the sigmoid colon in
65 percent of patients; in 24 percent of patients diverticula predominantly involve the
sigmoid, but are also present in other parts of the colon (image 2); in 7 percent of patients
diverticula are equally distributed throughout the colon; and in 4 percent diverticula are
limited to a segment proximal to the sigmoid colon. Approximately 4 to 15 percent of
patients with diverticulosis develop diverticulitis [7,20-22]. The incidence of
diverticulitis increases with age. The mean age at admission for acute diverticulitis is 63
years [23]. While the incidence of acute diverticulitis is lower in younger individuals,
approximately 16 percent of admissions for acute diverticulitis are in patients under 45
years of age [24]. In contrast to Asia, diverticular disease is predominantly left-sided in
western countries, with right-sided diverticulitis being present in only 1.5 percent of cases
[25]. Thirty-five percent of patients with sigmoid diverticulosis also have disease in the
more proximal colon. Diverticula are rare below the pelvic peritoneal reflection.
Prevalence correlates with age; approximately 30 percent of the population has acquired
diverticular change by age 60 years,whereas almost 60 percent of those aged 80 years and
older are affected. Ten to 25 percent of patients with diverticulosis will develop
diverticulitis. Read More
Feedback on your answer
Collapse
25)
0/1
You are seeing a 49-year old obese female who presents with recurrent worsening left
lower quadrant abdominal pain since last night. Her PCP diagnosed her with diverticulitis
5 days prior and started her on oral cefazolin and flagyl. She denies past medical history
and takes no other medications at home.
Vital signs show an oral temperature of 100.8 F, heart rate 101 beats/min, respirations
16/min, blood pressure 154/87, O2 saturation on room air 97%. Physical exam is normal
except for moderate left lower quadrant tenderness with no guarding.
IV and oral contrasted CT scan shows acute sigmoid diverticulitis with a 4.5cm peri-
sigmoid fluid collection consistent with abscess formation. What is the next best step in
management?
(Select 1)(1pts)

Surgical Laparotomy for sigmoid resection

IV antibiotics and Percutaneous Abscess Drainage

IV antibiotics and admission for observation

Continue current management with oral antibiotics with recheck in 24


hours

The answer is B. This patient has acute complicated diverticulitis with an abscess, no
signs of pertionitis and should be treated with IV antibiotics and a percutaneous drain.
The size of the abscess is an important determinant of the need for percutaneous
drainage. Small peridiverticular absceses (4cm or less) without peritonitis (Hinchey stage
1) can be treated conservatively with bowel rest and broad-spectrum antibiotics. For
patients with peridiverticular abscesses that are larger than 4 cm in diameter (Hinchey
stage 2), observational studies indicate that CT-guided percutaneous drainage can be
beneficial. This procedure typically eliminates or reduces the size of the abscess,with a
reduction in pain, resolution of leukocytosis, and defervescence usually seen within
several days. Percutaneous drainage mayallow for elective rather than emergency
surgery, increasing the likelihood of a successful one-stage procedure.
Patients whose abscess cavities contain gross feculent material tend to respond poorly,
and early surgical intervention is usually required.
Outpatient Therapy: Most patients (i.e., immunocompetent patients who have a mild
attack and can tolerate oral intake),outpatient therapy is reasonable. Rx: 7 to 10 days of
oral broad-spectrum antimicrobial therapy, including coverageagainst anaerobic
microorganisms. A combination of ciprofloxacin and metronidazole is often used, but
other regimens are also effective.
Inpatient Therapy: Immunocompromised, age > 65, diabetes and any patient in whom
diverticulitis is complicated by peridiverticular abscess formation.
Jacobs DO. Diverticulitis. N Engl J Med 2007:357:2057-2066. The decision to
hospitalize a patient for diverticulitis dependson the patient's clinical status.
Abscesses occur in 16 percent of patients with acute diverticulitis without peritonitis [46]
and in 31 to 56 percent of those requiring surgery for diverticulitis [47]. Prior to
interventional radiologic techniques, abscesses mandated operative intervention, often
with two-stage procedures. Percutaneous drainage now permits elective single stage
surgery in 60 to 80 percent of patients [48-50]; furthermore, in selected patients with
contraindications to surgery, catheter drainage may be sufficient to relieve symptoms
[49]. Drainage is usually performed through the anterior abdominal wall; abscesses deep
in the pelvis or obscured by other organs may be accessed transgluteally, or through the
rectum or vagina, although this approach is rarely performed [51,52].
The catheter is left in place until drainage is less than 10 mL in 24 hours; this may take as
long as 30 days [49]. Catheter sinograms during this period can show persistent
communication between fistula and bowel, and permit assessment of resolution of the
abscess cavity. Surgical intervention is mandatory if improvement does not occur.
Until recently, all abscesses have been considered to represent complicated diverticulitis,
and surgery has typically been recommended once resolution has occurred. However,
with continued improvements in CT scan technology, smaller abscesses can be identified.
Some of these are so small (ie, less than 3 cm) that they are not amenable to CT guided
drainage [53-55]. In addition to size limitations, the presence of small bowel loops
adjacent to the collection may preclude drainage. Abscess recurrence and/or fistula
formation after percutaneous drainage may also complicate the patient's course and
compromise the surgical approach. Accordingly, there has been interest in treating small
abscesses with systemic antibiotics alone.
In a retrospective study of 22 patients with 23 diverticular abscesses less than 3 cm in
diameter and no evidence of free air, all resolved the acute episode with antibiotic
treatment alone [53]. In subsequent follow up, 36 percent required eventual surgery, 41
percent reported full resolution of symptoms, and 23 percent had mild recurrent or
persistent symptoms that did not warrant surgery. Limitations of the study are the small
sample size and the lack of proof that these collections were actually abscesses as
opposed to localized reactive peritoneal fluid collections. Patients with acute diverticulitis
should be evaluated by a surgeon. CT scan findings should be reviewed with the
interventional radiologist to determine if any collections are amenable to percutaneous
drainage. Indications for immediate surgery include free intraperitoneal air. A patient
with a fluid collection less than 3 cm and not amenable to catheter drainage can be treated
with intravenous antibiotics and monitored carefully for any worsening of symptoms.
Patients with worsening symptoms require surgical intervention.
For patients who are treated by CT-guided percutaneous drainage,
surgery is indicated if the drain does not control symptoms by the
third or fifth day. For patients successfully treated by percutaneous
drainage, a clinical assessment is performed to determine if the patient
can be safely observed, or as in the case of high risk patients (eg,
advanced age, debility, and chronic immunosuppressive therapy), an
operative intervention is recommended. More From UptoDate
Unit 1
1)
0/1
You are seeing a 74-year old female who presents with acute onset of left lower
abdominal pain along with fever and vomiting. She has a prior history of diverticular
disease as well as type II diabetes. You are suspicious for diverticulitis and order a CT
abdomen/pelvis with IV and PO contrast to confirm. Which of
the following incorrectly defines acute complicated diverticulitis?
(Select 1)( 1pts extra credit)

Divericulitis with abscess

Divericulitis with fistula

Divericulitis with colon obstruction

Divericulitis with elevated WBC and bandemia

All options correctly define complicated diverticulitis

Answer D incorrectly define complicated diverticulitis. For the purposes of this


discussion, complicated diverticulitis is defined as acute diverticulitis accompanied by
abscess, fistula, obstruction, or free intra-abdominal perforation. An elevated WBC with
bandemia does not define this.
Some cases of diverticulitis are classified as complicated, meaning that the disease
process has progressed to obstruction, abscess or fistula formation, or free perforation.
Complicated diverticulitis may be particularly challenging to manage, especially because
patients may have no known history of diverticular disease. Lower gastrointestinal (GI)
bleeding is also a complication of diverticular disease in 30 to 50% of cases; in fact,
diverticula are the most common colonic cause of lower GI bleeding. When diverticular
hemorrhage occurs [see 5:5 Lower Gastrointestinal Bleeding], it is usually
associated with diverticulosis rather than with diverticulitis. Approximately 50% of
diverticular bleeding originates in the right colon, despite the low incidence of diverticula
in this segment of the colon. Patients tend to be elderly and to have cardiovascular
disease and hypertension. Regular intake of NSAIDs may increase the risk of this
complication. Although patients may lose 1 to 2 units of blood, the bleeding usually
ceases spontaneously,13 and expeditious operative treatment generally is not necessary.
The most common form of complicated diverticulitis involves the development of a
pericolic abscess, typically signaled by high fever, chills, and lassitude. Such abscesses
may be small and localized or may extend to more distant sites (e.g., the pelvis). They
may be categorized according to the Hinchey classification of diverticular
perforations,14 in which stage I refers to a localized pericolic abscess and stage II to a
larger mesenteric abscess spreading toward the pelvis On rare occasions, an abscess
forms in the retroperitoneal tissues, subsequently extending to distant sites such as the
thigh or the flank. The location of the abscess can be defined precisely by means of
computed tomography (CT) with contrast.
Some abscesses rupture into adjacent tissues or viscera, resulting in the formation of
fistulas. The fistulas most commonly seen in this setting (50 to 65% of cases) are
colovesical fistulas. This complication is less common in women because of the
protection afforded by the uterus. Symptoms of colovesical fistulas tend to involve the
urinary tract (e.g., pneumaturia, hematuria, and urinary frequency). Fecaluria is
diagnostic of colovesical or enterovesical fistulas. Colovaginal fistulas (which account
for 25% of all diverticular fistulas) are usually seen in women who have undergone
hysterectomies. The diseased colon is adherent to the vaginal cuff. Most commonly,
patients complain of a foul vaginal discharge; however, some patients present with stool
emanating from the vagina.

About 10% of colon obstructions are attributable to diverticulitis. Acute diverticulitis can
cause colonic edema and a functional obstruction that usually resolves with antibiotic
infusion and bowel rest. Stricture formation is more common, usually occurring as a
consequence of recurrent attacks of diverticulitis. Circumferential pericolic fibrosis is
noted, and marked angulation of the pelvic colon with adherence to the pelvic sidewall
may be seen. Patients complain of constipation and narrowed stools. Colonoscopy can be
difficult and potentially dangerous in this setting. Differentiating a diverticular stricture
from carcinoma may be impossible by any means short of resection.

The term malignant diverticulitis has been employed to describe an extreme form of
sigmoid diverticulitis that is characterized by an extensive phlegmon and inflammatory
reaction extending below the peritoneal reflection, with a tendency toward obstruction
and fistula formation.15 Malignant diverticulitis is seen in fewer than 5% of patients older
than 50 years who are operated on for diverticulitis.15 The process is reminiscent of
Crohn disease, and CT scans demonstrate extensive inflammation. In this setting, a
staged resection might be preferable to attempting a primary resection through the pelvic
phlegmon. The degree of pelvic inflammation may subside significantly after
diversion.15
A dangerous but rare complication of acute diverticulitis (occurring in 1 to 2% of cases)
is free perforation,16 which includes both perforation of a diverticular abscess throughout
the abdomen leading to generalized peritonitis (purulent peritonitis; Hinchey stage III)
and free spillage of stool thorough an open diverticulum into the peritoneal cavity (fecal
peritonitis; Hinchey stage IV). The incidence of free perforations may be increasing, at
least in the southwestern United States. The overall mortality in this group is between 20
and 30%, that for purulent peritonitis is approximately 13%, and that for fecal peritonitis
is about 43%.

2014. Scientific American Surgery. Hamilton, Ontario & Philadelphia, PA. Decker
Intellectual Properties Inc. ISSN 2368-2744. STAT!Ref Online Electronic Medical
Library. http://online.statref.com/Document.aspx?fxId=61&docId=1368. 10/22/2014
12:22:12 PM CDT (UTC -05:00).

Feedback on your answer


Collapse
2)
1/1

Numerous complications may arise in the postoperative period especially fevers,


pneumonia, urinary tract infections, and wound infections. Which of the following
statements is FALSE regarding post operative fevers/infections?

( 1pts extra credit)

Postoperative temperature elevations are quite common, occurring


in nearly one third of patients after surgery

Given the high mortality associated with post operative infections, a


shotgun approach to the workup of postoperative fever is usually
warranted

Studies have found the incidence of postoperative pneumonia to be


approximately 1.5%

Noninfectious causes of postoperative fevers include components of


the inflammatory response to surgical intervention, reabsorption of
hematomas, and (possibly) atelectasis

Answer B is False as a shotgun approach is not recommended.Numerous complications


may arise in the postoperative period. Many of these are specific to particular operative
procedures and hence are best discussed in connection with those procedures. Many
others, however, may develop after virtually any operation and thus warrant a general
discussion in this chapter (see below). Prompt discovery and treatment of these latter
complications rely heavily on a sufficiently high index of suspicion.
Postoperative Fever
Postoperative temperature elevations are quite common, occurring in nearly one third of
patients after surgery. Only a relatively small number of these are actually caused by
infection. Fevers that are caused by infections (e.g., pneumonias, wound infections, or
UTIs) tend to reach higher temperatures (> 38.5°C [101.3°F]), usually are associated with
moderate elevation of the white blood cell (WBC) count 3 or more days after operation,
and typically extend over consecutive days. Noninfectious causes of postoperative fevers
include components of the inflammatory response to surgical intervention, reabsorption
of hematomas, and (possibly) atelectasis.28
Beyond checking the WBC count, a shotgun approach to the workup of postoperative
fever probably is not warranted. A focused approach based on well-directed questioning
and a careful physical examination is more likely to obtain the highest diagnostic yield.
Coughing, sputum production, and respiratory effort should be noted, and the lungs
should be
auscultated for rales. All incisions should be inspected for erythema and drainage, and
current and recent IV sites should be checked for evidence of cellulitis. If a central line
has been placed, particularly if it has been in place for several days, the possibility of a
line infection should be considered. Patients who have undergone prolonged NG
intubation may have sinusitis, which is most readily diagnosed through computed
tomography (CT) of the sinuses. Further workup for fever may include, as indicated,
chest x-ray, sputum cultures, urinalysis, blood cultures, or CT of the abdomen (after
procedures involving laparotomy—especially bowel resections—where intra-abdominal
abscess is a possible complication).
Pneumonia
Respiratory infections in the postoperative period are generally considered nosocomial
pneumonias and, as such, are potentially serious complications. The estimated incidence
of postoperative pneumonia varies significantly, with many estimates tending to run high.
A 2001 study of more than 160,000 patients undergoing major noncardiac surgery
provided what may be a reasonable overall figure, finding the incidence of postoperative
pneumonia to be approximately 1.5%.29 In the 2,466 patients with pneumonia, the 30-
day mortality was 21%. Thoracic procedures, upper abdominal procedures, abdominal
aortic aneurysm repair, peripheral vascular procedures, and neurosurgical procedures
were all identified as placing patients at significantly increased risk for pneumonia.
Patient-specific risk factors included age greater than 60 years, recent alcohol use,
dependent functional status, long-term steroid use, and a 10% weight loss in the 6 months
preceding the operation.29
The diagnosis of postoperative pneumonia is based on the usual combination of index of
suspicion, findings from the history and physical examination (e.g., fever, shortness of
breath, hypoxia, productive cough, and rales on lung auscultation), imaging, and
laboratory evaluation. Appropriate workup, directed by the clinical findings, typically
starts with chest x-rays (preferably in both posteroanterior and lateral views, if possible)
and sputum cultures, sometimes accompanied by CT scanning of the chest and, possibly,
bronchoscopy with bronchoalveolar lavage (which may be useful in directing antibiotic
therapy when sputum cultures are non-diagnostic). Empirical broad-spectrum antibiotic
therapy is typically initiated before the causative organism is identified; this practice has
been shown to reduce mortality. Piperacillintazobactam, which is effective
againstPseudomonas aeruginosa, is commonly used for this purpose; however,
when Gram staining of the sputum identifies gram-positive cocci, vancomycin or
linezolid may be used initially instead.30 Once the causative organism is identified,
specific antibiotic therapy directed at that organism is indicated, as in treatment of other
infectious processes. Drainage of parapneumonic effusions may also be necessary, and
this measure may be helpful in diagnosing or preventing the development of
empyema. Read more
Feedback on your answer
Collapse
3)
0/1
All patients with at least one uncomplicated diverticulitis are recommened for operative
management with colectomy to prevent worse recurrent attacks in the future.
( 1pts extra credit)

True Incorrect

False

False: This would be true if it was an episode of complicated diverticulitis. The decision
to recommend elective sigmoid colectomy after recovery from acute diverticulitis should
be made on a case-by-case basis. Level of Evidence: III; Grade of Recommendation: B.
After successful medical treatment of an episode of acute diverticulitis, careful judgment
is required concerning whether to proceed with subsequent elective colon resection. After
one attack, about a third of patients will have a later second attack of acute diverticulitis,
and after a second episode, a further third will have yet another attack. The decision to
recommend surgery should be influenced by the age and medical condition of the patient,
the frequency and severity of the attack(s), and whether there are persistent symptoms
after the acute episode. Most patients who present with complicated diverticulitis do so at
the time of their first attack, therefore, a policy of elective colon resection after recovery
from uncomplicated acute diverticulitis might not decrease the likelihood of later
emergency surgery or overall mortality. Therefore, the number of attacks of
uncomplicated diverticulitis is not necessarily an overriding factor in defining the
appropriateness of surgery. As noted earlier, CT graded severity of a first attack is a
predictor of an adverse natural history and may be helpful in determining the need for
surgery. 18 Inability to exclude carcinoma is another appropriate indication for colectomy.
There is no clear consensus regarding whether younger patients (younger than aged 50
years) treated for diverticulitis are at increased risk of complications or recurrent attacks.
Nevertheless, because of their longer life span, younger patients will have a higher
cumulative risk for recurrent diverticulitis, even if the virulence of their disease is no
different than that of older patients. Elective colon resection should typically be advised
if an episode of complicated diverticulitis is treated nonoperatively. Level of Evidence:
III; Grade of Recommendation: B. After percutaneous drainage of a diverticular abscess,
a later colectomy usually should be planned, because 41 percent of patients will
otherwise develop severe recurrent sepsis. Read more

Feedback on your answer


Collapse
4)
0/1
You are seeing a 74-year old male who is post op following colectomy for ischemic
colitis. He has a past medical history of hypertension and type II diabetes. During the
surgery he received 2 units of PRBCs and 1 L of normal saline. The nurses say that he
developed a fever of 101.5 F about 45 minutes after the surgery. His BP is 135/67, HR
90, RR 16. His abdominal wound looks intact without redness and his lungs are clear. He
has a foley catheter and a right subclavian triple lumen catheter that were both placed in
pre operative holding. What is the most likely source of his fever?

( 1pts extra credit)

Urinary tract infection

Pneumonia

Catheter associated infection

Wound infection

Transfusion reaction

Pulmonary embolism

Malignant hyperthermia

The answer is E. The most common transfusion reaction is a febrile, nonhemolytic


transfusion reaction (FNHTR). The clinical manifestations of this reaction include fever,
often a chill, and sometimes mild dyspnea within one to six hours after transfusion of red
cells or platelets. FNHTRs are benign, causing no lasting sequelae, but are uncomfortable
and sometimes frightening to the patient. Furthermore, since fever, with or without a
chill, also may be the sign of a severe, acute hemolytic transfusion reaction (see 'Acute
hemolytic reactions' below) or infection, FNHTRs cannot be ignored. More From
UptoDate
Feedback on your answer
Collapse
5)
0/1
Which one of the following is most predictive of increased perioperative cardiovascular
events associated with noncardiac surgery in the elderly?

( 1pts extra credit)

Age > 80 years

Left bundle-branch block on ECG

Atrial fibrillation at a rate of 80 beats/min

Renal insufficiency (creatinine 2.9 mg/dL)

The answer is D. Renal insufficiency (creatinine 2.9 mg/dL) is considered intermediate


risk, all other options are considered low risk. Clinical predictors of increased
perioperative cardiovascular risk for elderly patients include major risk factors such as
unstable coronary syndrome (acute or recent myocardial infarction, unstable angina),
decompensated congestive heart failure, significant arrhythmia (high-grade AV block,
symptomatic ventricular arrhythmia, supraventricular arrhythmias with uncontrolled
ventricular rate), and severe valvular disease. Intermediate predictors are mild angina,
previous myocardial infarction, compensated congestive heart failure, diabetes mellitus,
and renal insufficiency. Minor predictors are advanced age, an abnormal EKG, left
ventricular hypertrophy, left bundle-branch block, ST and T-wave abnormalities, rhythm
other than sinus, low functional capacity, history of stroke, and uncontrolled
hypertension. Ref: Schroeder BM: Updated guidelines for perioperative cardiovascular
evaluation for noncardiac surgery. Am Fam Physician 2002;66(6):1096-1107.

Feedback on your answer


Collapse
Unit 4
6)
0/1
You are seeing a 58-year-old female who is presenting with two days of a low-grade
fever and increasing abdominal cramps that are exacerbated by bowel movements. She
began a course of augmentin 2 days ago that her PCP gave her but has had no relief of
her symptoms and is getting worse. She has not had nausea, vomiting, urinary symptoms,
or bloody stools. She has a past medical history of hypertension.

Her vitals show a temperature is 100.6°F, pulse is 75/min, and blood pressure is 150/80
mm Hg. Her lungs are clear to auscultation. Cardiac examination shows no
abnormalities. Her abdomen is soft, and there is tenderness to palpation of the left lower
quadrant with guarding but no rebound. Bowel sounds are normal. Her stool is brown,
and test for occult blood is negative. Her hemoglobin concentration is 14.5 g/dL,
leukocyte count is 15,800/mm3, and platelet count is 280,000/mm3; serum studies and
urinalysis show no abnormalities. Which of the following is the most appropriate next
step in diagnosis?
(Select 1)(1pts)

Examination of the stool for ova and parasites

Test of the stool for Clostridium difficile toxin

Xray Abdomen Series

CT scan of the abdomen/pelvis with IV and oral contrast

The answer is D, CT scan of the abdomen with IV and oral contrast. This patient is being
treated for diverticulitis and is not getting better. She needs a CT scan to evaluate for a
possible abscess. Read more

Feedback on your answer


Collapse
7)
0/1
You are seeing a 64-year old white male who is presenting with left lower abdominal
pain since last night. His WBC is 14,000mm3 and his CT scan of the abdomen and pelvis
of the abdomen/pelvis shows acute uncomplicated sigmoid diverticulitis. What bacteria is
usually the target in the treatment of acute diverticulitis?
(Select 1)(1pts)

Gram positive cocci

Gram positive rods


Gram negative cocci

Gram negative rods

The answer is D, gram negative Rods. The most common choices would be either
augmentin as single coverage or cipro plus flagyl. Conservative treatment of acute
uncomplicated diverticulitis is successful in 70 to 100 percent of patients. Uncomplicated
diverticulitis may be managed as an outpatient (dietary modification and oral antibiotics)
for those without appreciable fever, excessive vomiting, or marked peritonitis, as long as
there is the opportunity for follow-up. The patient should be able to take liquids and
antibiotics by mouth. Hospitalization for treatment (dietary modification and intravenous
antibiotics) is usually best if the above conditions are not met, or if the patient fails to
improve with outpatient therapy. Antibiotics should be selected to treat the most common
bacteria found in the colon: gram-negative rods and anaerobic bacteria. 13 Single and
multiple antibiotic regimens are equally effective, as long as both groups of organisms
are covered. Nonoperative treatment will resolve acute diverticulitis in 85 percent of
patients, but approximately one-third will have a recurrent attack often within one
year. Read More

Feedback on your answer


Collapse
8)
1/1
You are seeing a 49-year old obese female who presents with recurrent worsening left
lower quadrant abdominal pain since last night. Her PCP diagnosed her with diverticulitis
5 days prior and started her on oral cefazolin and flagyl. She denies past medical history
and takes no other medications at home.
Vital signs show an oral temperature of 100.8 F, heart rate 101 beats/min, respirations
16/min, blood pressure 154/87, O2 saturation on room air 97%. Physical exam is normal
except for moderate left lower quadrant tenderness with no guarding.
IV and oral contrasted CT scan shows acute sigmoid diverticulitis with a 4.5cm peri-
sigmoid fluid collection consistent with abscess formation. What is the next best step in
management?
(Select 1)(1pts)

Surgical Laparotomy for sigmoid resection

IV antibiotics and Percutaneous Abscess Drainage


IV antibiotics and admission for observation

Continue current management with oral antibiotics with recheck in


24 hours

The answer is B. This patient has acute complicated diverticulitis with an abscess, no
signs of pertionitis and should be treated with IV antibiotics and a percutaneous drain.
The size of the abscess is an important determinant of the need for percutaneous
drainage. Small peridiverticular absceses (4cm or less) without peritonitis (Hinchey stage
1) can be treated conservatively with bowel rest and broad-spectrum antibiotics. For
patients with peridiverticular abscesses that are larger than 4 cm in diameter (Hinchey
stage 2), observational studies indicate that CT-guided percutaneous drainage can be
beneficial. This procedure typically eliminates or reduces the size of the abscess,with a
reduction in pain, resolution of leukocytosis, and defervescence usually seen within
several days. Percutaneous drainage mayallow for elective rather than emergency
surgery, increasing the likelihood of a successful one-stage procedure.
Patients whose abscess cavities contain gross feculent material tend to respond poorly,
and early surgical intervention is usually required.
Outpatient Therapy: Most patients (i.e., immunocompetent patients who have a mild
attack and can tolerate oral intake),outpatient therapy is reasonable. Rx: 7 to 10 days of
oral broad-spectrum antimicrobial therapy, including coverageagainst anaerobic
microorganisms. A combination of ciprofloxacin and metronidazole is often used, but
other regimens are also effective.
Inpatient Therapy: Immunocompromised, age > 65, diabetes and any patient in whom
diverticulitis is complicated by peridiverticular abscess formation.
Jacobs DO. Diverticulitis. N Engl J Med 2007:357:2057-2066. The decision to
hospitalize a patient for diverticulitis dependson the patient's clinical status.
Abscesses occur in 16 percent of patients with acute diverticulitis without peritonitis [46]
and in 31 to 56 percent of those requiring surgery for diverticulitis [47]. Prior to
interventional radiologic techniques, abscesses mandated operative intervention, often
with two-stage procedures. Percutaneous drainage now permits elective single stage
surgery in 60 to 80 percent of patients [48-50]; furthermore, in selected patients with
contraindications to surgery, catheter drainage may be sufficient to relieve symptoms
[49]. Drainage is usually performed through the anterior abdominal wall; abscesses deep
in the pelvis or obscured by other organs may be accessed transgluteally, or through the
rectum or vagina, although this approach is rarely performed [51,52].
The catheter is left in place until drainage is less than 10 mL in 24 hours; this may take as
long as 30 days [49]. Catheter sinograms during this period can show persistent
communication between fistula and bowel, and permit assessment of resolution of the
abscess cavity. Surgical intervention is mandatory if improvement does not occur.
Until recently, all abscesses have been considered to represent complicated diverticulitis,
and surgery has typically been recommended once resolution has occurred. However,
with continued improvements in CT scan technology, smaller abscesses can be identified.
Some of these are so small (ie, less than 3 cm) that they are not amenable to CT guided
drainage [53-55]. In addition to size limitations, the presence of small bowel loops
adjacent to the collection may preclude drainage. Abscess recurrence and/or fistula
formation after percutaneous drainage may also complicate the patient's course and
compromise the surgical approach. Accordingly, there has been interest in treating small
abscesses with systemic antibiotics alone.
In a retrospective study of 22 patients with 23 diverticular abscesses less than 3 cm in
diameter and no evidence of free air, all resolved the acute episode with antibiotic
treatment alone [53]. In subsequent follow up, 36 percent required eventual surgery, 41
percent reported full resolution of symptoms, and 23 percent had mild recurrent or
persistent symptoms that did not warrant surgery. Limitations of the study are the small
sample size and the lack of proof that these collections were actually abscesses as
opposed to localized reactive peritoneal fluid collections. Patients with acute diverticulitis
should be evaluated by a surgeon. CT scan findings should be reviewed with the
interventional radiologist to determine if any collections are amenable to percutaneous
drainage. Indications for immediate surgery include free intraperitoneal air. A patient
with a fluid collection less than 3 cm and not amenable to catheter drainage can be treated
with intravenous antibiotics and monitored carefully for any worsening of symptoms.
Patients with worsening symptoms require surgical intervention.
For patients who are treated by CT-guided percutaneous drainage,
surgery is indicated if the drain does not control symptoms by the
third or fifth day. For patients successfully treated by percutaneous
drainage, a clinical assessment is performed to determine if the patient
can be safely observed, or as in the case of high risk patients (eg,
advanced age, debility, and chronic immunosuppressive therapy), an
operative intervention is recommended. More From UptoDate

Feedback on your answer


Collapse
9)
0/1
You are seeing a 49 -ear old obese male who presents with two days of worsening left
sided abdominal pain. He denies fever or chills but does note painful bowel movements
that have streaks of blood. On exam you note moderate left lower quadrant tenderness to
palpation without rebound or guarding. His labs show a white blood cell count of
14,000mm3 and his CT scan of his abdomen/pelvis shows acute uncomplicated sigmoid
diverticulitis. Nonoperative treatment with oral antibiotics will resolve acute
uncomplicated diverticulitis in what percent of patients?
(Select 1)(1pts)

<25%
50%

80%

>95%

The best answer is B, 80%. Although most patients with acute diverticulitis can be treated
medically, approximately 15-20% will require surgery. Uncomplicated diverticulitis may
be managed as an outpatient (dietary modification and oral antibiotics) for those without
appreciable fever, excessive vomiting, or marked peritonitis, as long as there is the
opportunity for follow-up. The patient should be able to take liquids and antibiotics by
mouth. Hospitalization for treatment (dietary modification and intravenous antibiotics) is
usually best if the above conditions are not met, or if the patient fails to improve with
outpatient therapy. Antibiotics should be selected to treat the most common bacteria
found in the colon: gram-negative rods and anaerobic bacteria. Single and multiple
antibiotic regimens are equally effective, as long as both groups of organisms are
covered. Nonoperative treatment will resolve acute diverticulitis in 80
percent of patients, but approximately one-third will have a recurrent attack often
within one year. approximately 20% of patients with diverticulitis require surgical
treatment.30 Most surgical procedures are reserved for patients who experience recurrent
episodes of acute diverticulitis that necessitate treatment (inpatient or out-patient) or who
have complicated diverticulitis. The most common indication for elective resection is
recurrent attacks—that is, several episodes of acute diverticulitis documented by studies
such as CT. Rerecurrences may be more common than recurrences. In 2000, a task force
of the American Society of Colon and Rectal Surgeons recommended sigmoid resection
after two attacks of diverticulitis. A subsequent cost analysis using a Markov model
suggested that cost savings could be achieved if resection was done after three
attacks.33 There is a growing tendency to question arbitrary guidelines for surgical
management of recurrent attacks, with the exception of certain groups, such as
immunocompromised patients. Current practice guidelines state that the recommendation
to perform elective sigmoid resection after recovery from uncomplicated acute
diverticulitis should be made on a case-by-case basis. The decision-making process
should be influenced by the age and medical condition of the patient, the frequency and
severity of attacks, and the presence of symptoms after the acute attack. Elective
resection is generally recommended after an episode of complicated
diverticulitis.36 Efforts are made to time surgical treatment so that it takes place during a
quiescent period 8 to 10 weeks after the last attack. Barium enema or colonoscopy may
be employed to evaluate the diverticular disease and rule out carcinoma. The bowel can
then be prepared mechanically and with antibiotics (e.g., oral neomycin and
metronidazole on the day before operation).
2014. Scientific American Surgery. Hamilton, Ontario & Philadelphia, PA. Decker
Intellectual Properties Inc. ISSN 2368-2744. STAT!Ref Online Electronic Medical
Library. http://online.statref.com/Document.aspx?fxId=61&docId=1368. 10/20/2014
12:41:34 PM CDT (UTC -05:00).
Feedback on your answer
Collapse
10)
1/1
You are seeing a 64-year old female who is presenting with painless bright red rectal
bleeding for the last two days. She notes bright red blood coating the stool and on the
toilet paper when she wipes. He vitals are normal and her physical exam shows a soft non
tender abdomen. Stool is brown with red streaks. What is the most likely cause of
bleeding in this patient and the most common overall cause of lower GI bleeding in
adults?
(Select 1)(1pts)

Inflammatory Bowel Disease

Diverticulosis

Neoplasm

Angiodysplasia

The answer is B. Diverticulosis results in 60% of cases of lower GI bleeding followed by


IBD(13%), benign anorectal disease (11%), neoplasia (10%) and angiodysplasia (<5%).
The causes of lower gastrointestinal bleeding (LGIB) may be grouped into several
categories (table 1): From Uptodate
 Anatomic (diverticulosis)
 Vascular (angiodysplasia, ischemic, radiation-induced)
 Inflammatory (inflammatory bowel disease, infectious)
 Neoplastic
In most series, diverticulosis is the most common source of LGIB, accounting for
approximately 15 to 55 percent of cases [9,10]. Angiodysplasia may be the most
frequent cause in patients over the age of 65 years [11,12], though more recent data
suggest that angiodysplasia may be a less common cause of LGIB than once thought.
Hemorrhoids are the most common cause of rectal bleeding in patients under the age of
50 years [13]. However, hemorrhoidal bleeding is usually minor. Similarly, bloody
diarrhea due to inflammatory causes can sometimes be distinguished from other causes of
LGIB because of the clinical setting. In general, anatomic and vascular causes of
bleeding present with painless, large-volume blood loss, whereas inflammatory sources
are associated with diarrhea and abdominal pain [14].
In a review of several large studies that included 1559 patients with acute hematochezia,
the following bleeding sources were identified [10]:
 Diverticulosis – 5 to 42 percent
 Ischemia – 6 to 18 percent
 Anorectal (hemorrhoids, anal fissures, rectal ulcers) – 6 to 16 percent
 Neoplasia (polyps and cancers) – 3 to 11 percent
 Angiodysplasia – 0 to 3 percent
 Postpolypectomy – 0 to 13 percent
 Inflammatory bowel disease – 2 to 4 percent
 Radiation colitis – 1 to 3 percent
 Other colitis (infectious, antibiotic associated, colitis of unclear etiology) – 3 to
29 percent
 Small bowel/upper GI bleed – 3 to 13 percent
 Other causes – 1 to 9 percent
 Unknown cause – 6 to 23 percent
The vast majority of colonic diverticula are actually false diverticula (pseudodiverticula)
that contain only serosa and mucosa. They occur at weak points in the colonic wall where
the vasa recta penetrate the muscularis to supply the mucosa; as the diverticulum expands,
these vessels are displaced. A 1976 anatomic study of colonic specimens from patients
with diverticular bleeding used angiography to demonstrate that in all cases, the vasa
recta overlying the diverticulum ruptured into the lumen of the diverticulum, not into the
peritoneum.100

It has been estimated that approximately 17% of patients with colonic diverticulosis
experience bleeding, which may range from minor to severe and life-threatening.
Endoscopic treatment of diverticular hemorrhage can be difficult because of the high
bleeding rate and the location of the bleeding point within the diverticulum. In 2000, one
group of investigators reported their experience with endoscopic therapy for severe
hematochezia and diverticulosis in a prospective series of 121 patients.101 In this series,
none of the patients treated endoscopically with epinephrine injections, bipolar
coagulation, or both required surgery and none experienced recurrent bleeding episodes.
A 2001 study from another group, however, reported high rates of recurrent bleeding
episodes in both the early and the late posttreatment periods.102 In the absence of
prospective, randomized trials, it is difficult to draw definitive conclusions about the
utility of endoscopic therapy in treating diverticular hemorrhage.

Fortunately, most diverticular hemorrhages stop spontaneously. In one series, surgery


was unlikely to be necessary if fewer than four units of packed RBCs were transfused in a
24-hour period, whereas 60% of patients receiving more than four units of packed RBCs
in a 24-hour period required surgical intervention.103 Semielective surgical therapy is
usually offered after a second diverticular bleeding episode because once a second such
episode has occurred, the risk that a third will follow exceeds 50%. In a series of 83
conservatively managed cases of diverticular disease, the predicted yearly recurrence
rates were 9% at 1 year, 10% at 2 years, 19% at 3 years, and 25% at 4 years.104

In general, patients who require more than four units of blood in a 24-hour period to
remain hemodynamically stable, who have not stopped bleeding after 72 hours, or who
experience rebleeding within 1 week after an initial episode should undergo surgery.
2014. Scientific American Surgery. Hamilton, Ontario & Philadelphia, PA. Decker
Intellectual Properties Inc. ISSN 2368-2744. STAT!Ref Online Electronic Medical
Library. http://online.statref.com/Document.aspx?fxId=61&docId=848. 10/22/2014
7:03:07 PM CDT (UTC -05:00).

Feedback on your answer


Collapse
Unit 2
11)
0/1
With regards to positioning patients in the operating room, which of the following
statements if FALSE?
(Select 1)(1pts)

The lithotomy position is used in approximately 9% of cases and is


the next most commonly used position

Approximately 80% of surgical procedures are performed with


patients in the supine position.

Two major preventable consequences of malpositioning are


neuropathy or plexopathy and skin burns or ulceration.

Ulnar neuropathy and brachial plexopathy are the two most


common complications in this position and constitute around 90%
of claims.

Damage to the obturator, sciatic, lateral femoral cutaneous, and


peroneal nerves from being placed in lithotomy position account for
5% of medical liability claims for nerve damage

Answer D is the only false statement as Ulnar neuropathy and brachial plexopathy are
the two most common complications in this position and constitute 28% and 20% of
claims, respectively. The risks associated with malpositioning deserve further comment.
The skin over any bony prominence is subject to injury from excessive pressure. The
fragile skin of infants and older patients may be injured by dragging rather than lifting
into position. The American Society of Anesthesiologists (ASA) practice advisory on the
prevention of perioperative peripheral neuropathies recommends that when practical,
patients should be placed in the intended position before induction of anesthesia to test
for comfort.78 Uncomfortable positions should be modified. It is essential to listen to the
patient. This is especially important with older patients or patients with degenerative
spine and joint disease or skeletal instability. Approximately 80% of surgical procedures
are performed with patients in the supine position. Two major preventable consequences
of malpositioning are neuropathy or plexopathy and skin burns or ulceration. Ulnar
neuropathy and brachial plexopathy are the two most common complications in this
position and constitute 28% and 20% of claims, respectively, in recent closed claims
studies.79-84 Padding of the precondylar groove of the humerus and tucking of the arms
or, at the least, restriction of abduction to less than 90° help prevent this problem.
The lithotomy position is used in approximately 9% of cases and is the next most
commonly used position.80,82 Damage to the obturator, sciatic, lateral femoral
cutaneous, and peroneal nerves has been observed. These injuries account for 5% of
medical liability claims for nerve damage in the Closed Claims Data Base. They
represent a small but not insignificant risk.79 Other patient positions have also been
associated with malpositioning injuries. As would be expected, longer procedures in
sicker or mal-nourished patients are more likely to result in complications of
positioning. Read more
Feedback on your answer
Collapse
12)
0/1
There is a high risk of pathogen transfer during surgery. This is a risk from which both
the patient and the surgical team must be protected. The risk can be reduced by using
protective barriers, such as surgical gloves. A 2002 Cochrane Review concluded that
wearing two pairs of latex gloves can further reduce the risk of contamination to what %?
(Select 1)(1pts)

20%

15%

10%

5%

The answer is D, There is a high risk of pathogen transfer during surgery. This is a risk
from which both the patient and the surgical team must be protected. The risk can be
reduced by using protective barriers, such as surgical gloves. A 2002 Cochrane Review
concluded that wearing two pairs of latex gloves significantly reduced the risk of hand
exposure. Wearing two pairs of surgical gloves rather than a single pair provides an
additional barrier and further reduces the risk of contamination. The rate of single-glove
perforations is approximately 15%. The rate of inner glove perforation when two pairs of
gloves are worn is approximately 5%. Read more
Feedback on your answer
Collapse
13)
0/1
What is the risk for serious medical complications from surgery in healthy patients?
(Select 1)(1pts)
<0.1%

<0.5-1%

2-3%

5-6%

The answer is A, <0.1%. Healthy patients have significant physiologic reserves and
tolerate major and minor surgical procedures well. The risk for serious medical
complications from surgery is less than 0.1% overall in healthy patients. When evaluating
healthy patients before surgery, physicians should use a focused history and physical
examination to predict the risk for serious medical complications. Researchers collected
information at 16 and 72 hours and 30 days for 38 598 patients undergoing 45 090
consecutive procedures and found that only 33 (0.08%) had major morbidity or mortality.
Of 4 deaths that occurred within 30 days of the procedure, 2 were due to myocardial
infarction and 2 were due to automobile accidents unlikely to be related to surgery. More
than one third of major morbidity occurred 48 hours or more after surgery. Read more
Feedback on your answer
Collapse
14)
0/1
A 72-year-old African-American male comes to your office for surgical clearance for an
elective left hemicolectomy for recurrent diverticulitis. His past medical history includes
an inferior-wall myocardial infarction approximately 12 months ago. Since then he had a
stress test that was normal 6 months ago. Currently, the patient feels well, walks while
playing nine holes of golf weekly, and is able to walk up a flight of stairs without chest
pain or significant dyspnea. Findings are normal on a physical examination.
Which one of the following would be most appropriate for this patient prior to surgery?
(1pts)

A 12-lead resting EKG

Another Nuclear Stress test

A dobutamine stress echocardiogram

A persantine stressed nuclear tracer study (technetium or thallium)


The answer is A, A 12-lead resting EKG. The current recommendations from the
American College of Cardiology and the American Heart Association on preoperative
clearance for noncardiac surgery state that preoperative intervention is rarely needed to
lower surgical risk. Patients who are not currently experiencing unstable coronary
syndrome, severe valvular disease, uncompensated congestive heart failure, or a
significant arrhythmia are not considered at high risk, and should be evaluated for most
surgery primarily on the basis of their functional status. If these patients are capable of
moderate activity (greater than 4 METs) without cardiac symptoms, they can be cleared
with no stress testing or coronary angiography for an elective minor or intermediate-risk
operation such as the one this patient is to undergo. A resting 12-lead EKG is
recommended for males over 45, females over 55, and patients with diabetes, symptoms
of chest pain, or a previous history of cardiac disease. Ref: ACC/AHA guideline update
for perioperative cardiovascular evaluation for noncardiac surgery--Executive summary:
A report of the American College of Cardiology/American Heart Association Task Force
on Practice Guidelines (Committee to Update the 1996 Guidelines on Perioperative
Cardiovascular Evaluation for Noncardiac Surgery). Circulation 2002;105(10):1257-
1267. Read more here
Feedback on your answer
Collapse
15)
0/1
Which of the following statements regarding the pre-operative care of patients is false?
(Select 1)(1pts)

When a preoperative antibiotic is indicated, a single dose of


therapeutic strength, administered shortly before incision, usually
suffices

Cooling patients during colorectal surgery have been shown to


reduce infection rates

Supplemental perioperative oxygen (i.e., an FIO2 of 80% instead of


30%) significantly overcomes the decrease in phagocytosis and
bacterial killing usually associated with anesthesia and surgery (and
significantly reduces postoperative nausea and vomiting)

Intraoperative transfusion of packed red blood cells was an


independent risk factor for perioperative infection

Answer B is the only false statement. A 1996 study showed that warming patients during
colorectal surgery reduced infection rates.173 This finding was confirmed in a
subsequent observational cohort study that reported a significantly increased incidence of
SSI with hypothermia.174 A randomized controlled trial of 421 patients, published in
2001, determined that warming patients before short-duration clean procedures (breast,
varicose vein, or hernia) reduced infection (p = .001) and reduced wound scores (p =
.007) in this setting as well.175 The safest and most effective way of protecting patients
from hypothermia is to use forced-air warmers with specialized blankets placed over the
upper or lower body. Alternatives include placing a warming water mattress under the
patient and draping the patient with an aluminized blanket. Second-line therapy involves
warming all IV fluids. Any irrigation fluids used in a surgical procedure should be at or
slightly above body temperature before use. Radiant heating devices placed above the
operative field may be especially useful during operations on infants. Use of a warmer on
the inhalation side of the anesthetic gas circuit can also help maintain the patient's body
temperature during an operation.
The association between blood transfusion and increased perioperative infection rates is
well documented. In a 1997 study, geriatric hip fracture patients undergoing surgical
repair who received blood transfusion had significantly higher rates of perioperative
infection than those who did not (27% versus 15%), and this effect was confirmed on
multivariate analysis.177 Another study yielded similar findings in colorectal cancer: the
relative risk of infection was 1.6 for transfusion of one to three units of blood and 3.6 for
transfusion of more than three units.178 A large prospective cohort study published in
2002 evaluated the association between anemia, blood transfusion, and perioperative
infection.179 Regression analysis confirmed that intraoperative transfusion of packed red
blood cells was an independent risk factor for perioperative infection (odds ratio 1.06;
95% confidence interval 1.01 to 1.11; p > .0001). Furthermore, transfusion of more than
four units of packed red blood cells was associated with a ninefold increased risk of
infection (95% confidence interval 5.74 to 15.00; p < .0001).
In brief, the principles of optimal surgical antimicrobial prophylaxis include (1)
appropriate choice of an antimicrobial agent, (2) proper timing of antibiotic
administration before incision, and (3) limited duration of antibiotic administration after
operation. SSIs are established several hours after contamination.161 There is a well-
recognized "golden period" during which prophylactic antibiotics can be effective.
Administration of antibiotics before contamination reduces the risk of infection but
subsequently proves to be of little prophylactic value.162 The selective use of short-
duration, narrow-spectrum antibiotic agents chosen to reflect (1) the patient's individual
risk factors and (2) the recognized epidemiology of SSI, modified for local conditions
and experience, should be considered where deemed appropriate [see Table 9 for the
usual pathogens isolated from SSIs].163 When a preoperative antibiotic is indicated, a
single dose of therapeutic strength, administered shortly before incision, usually
suffices.164 The routine dose may have to be increased to achieve adequate serum levels
if the patient is morbidly obese.165 A second dose is indicated if the procedure is longer
than two half-lives of the drug or if extensive blood loss occurs. Continuation of
prophylaxis beyond 24 hours is not recommended. The provision of prophylactic
antibiotics, where indicated, has been accepted as a measure of surgical quality aimed at a
reduction in SSIs.166,167 A strong economic case has been made for infection control,
including prophylactic antibiotics, benefiting the hospital and the patient.168 The
perioperative nursing team has a central role, together with the anesthesiologist, in
monitoring the administration of preoperative prophylactic antibiotics.
Destruction by oxidation or by oxidative killing is the body's most important defense
against surgical pathogens. This defensive response depends on adequate levels of
oxygen tension in contaminated tissue. An easy method of improving oxygen tension in
adequately perfused tissue is to increase the FiO2. Supplemental perioperative oxygen
(i.e., an FIO2 of 80% instead of 30%) significantly overcomes the decrease in
phagocytosis and bacterial killing usually associated with anesthesia and surgery (and
significantly reduces postoperative nausea and vomiting). Low oxygen tension in wound
tissue has been found to be a good predictor of SSI risk.176 With that said, raised oxygen
tension is also an important risk factor in the outbreak of surgical fires. A balance must
be achieved between the adequacy of PO2 in the tissue and the restriction of O2 flow in
head and neck surgery and in other high-risk fire settings. Read more
Feedback on your answer
Collapse
Unit 5
16)
0/1
The most common method of monitoring patients' nutritional status with nutritional
supplementation is to measure the serum albumin and prealbumin (transthyretin)
concentrations. What is the half life of prealbumin?
(Select 1)(1pts)

1-2 days

3-5 days

8-10 days

14-20 days

The answer is A, 1-2 days. Patients who require assistance with nutritional intake should
be monitored to determine whether the interventions being carried out are having the
desired effect. The most common method of monitoring patients' nutritional status with
nutritional supplementation is to measure the serum albumin and prealbumin
(transthyretin) concentrations. Albumin has a half-life of approximately 14 to 20 days
and thus serves as a marker of longer-term nutritional status. A value lower than 2.2 g/dL
is generally considered to represent severe malnutrition, but even somewhat higher values
(< 3.0 g/dL) have been associated with poorer outcomes after elective surgery. Although
the serum albumin concentration is a commonly used marker, it is not always a reliable
one. Because of albumin's relatively long half-life, the serum concentration does not
reflect the patient's more recent nutritional status. In addition, the measured concentration
can change quickly in response to the infusion of exogenous albumin or to the
development of dehydration, sepsis, and liver disease despite adequate nutrition. Pre
albumin is a separate serum protein that has a halflife of approximately 24 to 48 hours
and thus can serve as a marker of current and more recent nutritional status. Like the
albumin concentration, the prealbumin concentration can be affected by liver and renal
disease. Overall, however, it is more immediately reliable in following the effects of
nutritional intervention. Read more
Feedback on your answer
Collapse
17)
0/1
You are seeing a 78-year old female who is post op day 1 following a colon resection
following a bout of recurrent diverticulitis with perforation. She has a past medical
history of cirrhosis from hepatitis C. On exam you note chronic lower extremity swelling
and 2+ pitting edema. Her albumin level was 2.7 g/dL. You are wanting to give her a liter
of normal saline (NS) suspecting dehydration as she has not eaten anything in 48 hours
and her heart rate has increased to around 105 beats/min. Out of the 1 Liter given of NS,
what volume administered stays in the intravascular space?
(Select 1)(1pts)

50-100 cc

250-330cc

500-650 cc

750-800cc

The answer is B, 250-330cc. Crystalloid solutions are water-based solutions to which


electrolytes (and, sometimes, organic molecules such as dextrose) have been added. The
crystalloid solutions used for resuscitation are generally isotonic to blood plasma and
include such common examples as 0.9% sodium chloride, lactated Ringer solution, and
Plasma-Lyte (Baxter Healthcare, Round Lake, IL). The choice to use one solution over
another is usually inconsequential, but there are a few notable exceptions. For example,
in the setting of renal dysfunction, there is a risk of hyperkalemia when potassium-
containing solutions such as lactated Ringer solution and Plasma-Lyte are used. As
another example, the administration of large volumes of 0.9% sodium chloride, which
has a pH of 5.0 and a chloride content of 154 mmol/L, can lead to hyperchloremic
metabolic acidosis. Regardless of which crystalloid solution is used, large volumes may
have to be infused to achieve a significant increase in the circulating intravascular
volume. Only one third to one quarter (250 to 330 mL/L) of the fluid administered stays
in the intravascular space; the rest migrates by osmosis into the interstitial tissues,
producing edema and potential impairment of tissue perfusion (the latter is a theoretical
consequence whose existence has not yet been directly demonstrated). Read more
Feedback on your answer
Collapse
18)
1/1
Initial wound management after an operative procedure generally entails placement of a
sterile dressing to cover the incision. The traditional recommendation has been to keep
this dressing in place and dry for what period of time?

(Select 1)(1pts)

12 hours

24 hours

48 hours

72 hours

All of the above

None of the above

The answer is C, 48 hours. Initial wound management after an operative procedure


generally entails placement of a sterile dressing to cover the incision. The traditional
recommendation has been to keep this dressing in place and dry for the first 48 hours
after operation; because epithelialization is known to take place within approximately this
period, the assumption is that this measure will reduce the risk of wound infection.
Although most surgeons still follow this practice, especially in general surgical cases,
supporting data from randomized clinical studies are lacking. In addition, several small
studies that evaluated early showering with closed surgical incisions found no increases
in the rate of infection or dehiscence.8,9 It should be kept in mind, however, that these
small studies looked primarily at soft tissue and other minor skin incisions that did not
involve fascia. Thus, even though the traditional approach to initial dressing management
is not strongly supported, the data currently available are insufficient to indicate that it
should be changed.
Wet-to-dry dressings are used in a variety of settings. In a surgical context, they are most
often applied to a wound that cannot be closed primarily as a consequence of
contamination or inability to approximate the skin edges. Wet-to-dry dressings provide a
moist environment that promotes granulation and wound closure by secondary intention.
Moreover, their removal and replacement cause debridement of excess exudate or
unhealthy superficial tissue. Postoperative orders should specify the frequency of
dressing changes and the solution used to provide dampness. For most clean open
incisions, twice-daily dressing changes using normal saline solution represent the most
common approach. If there is excess wound exudate to be debrided, dressing changes
may be performed more frequently. If there is particular concern about wound
contamination or superficial colonization of organisms, substitution of dilute Dakin
solution for normal saline may be considered. Read more

Feedback on your answer


Collapse
19)
0/1
Urinary catheters can serve a large variety of purposes. In the setting of bladder or
genitourinary surgery, they are often employed to decompress the system so that it will
heal more readily. After general surgical procedures—and many other surgical
procedures as well—they are used to provide accurate measurements of volume output
and thus, indirectly, to give some indication of the patient's overall volume and
resuscitation status.
Their utility and importance notwithstanding, urinary catheters are associated with the
development of nosocomial urinary tract infections (UTIs). As many as 40% of all
hospital infections are UTIs. What % of these UTI's are associated with urinary catheters?
(Select 1)(1pts)

20%

40%

60%

80%

The answer is D, 80%. Urinary catheters can serve a large variety of purposes. In the
setting of bladder or genitourinary surgery, they are often employed to decompress the
system so that it will heal more readily. After general surgical procedures—and many
other surgical procedures as well—they are used to provide accurate measurements of
volume output and thus, indirectly, to give some indication of the patient's overall volume
and resuscitation status. Furthermore, after many procedures, patients initially find it
extremely difficult or impossible to mobilize for urination, and a urinary catheter may be
quite helpful during this time. Their utility and importance notwithstanding, urinary
catheters are associated with the development of nosocomial urinary tract infections
(UTIs). As many as 40% of all hospital infections are UTIs, and 80 to 90% of these UTIs
are associated with urinary catheters [see Postoperative Complications,below].
Accordingly, when catheterization is no longer deemed necessary, prompt removal is
indicated. As a rule, orders specifically pertaining to urinary catheter care are few,
typically including gravity drainage, flushing to maintain patency (if warranted), and
removal when appropriate. At times, irrigation is employed after urologic procedures or
for the management of certain infectious agents. Read more
Feedback on your answer
Collapse
20)
1/1
Which of the following statements regarding peri and post-operative MI is true?
(Select 1)(1pts)

plaque rupture is the most common cause of MI

The majority of cardiac ischemic events occur after the 4th day

95% of patients who experience this complication will present with


classic symptoms

Perioperative beta blockade for patients at risk for MI is now


routine as this practice yields significant risk reductions in terms of
both cardiac morbidity and mortality

Answer D is the only true statement. Approximately one third of patients who undergo
noncardiac surgery in the United States have some degree of coronary artery disease and
thus are at increased risk for perioperative MI. The incidence of coronary artery disease is
even higher in certain subpopulations, such as patients who undergo major vascular
procedures.40,41 In the perioperative setting, however, the pathophysiology of coronary
ischemia is different from that in nonsurgical settings, where plaque rupture is the most
common cause of MI. Approximately 50% of all MIs occurring in surgical patients are
caused by increased myocardial oxygen demand in the face of inadequate supply
resulting from factors such as fluid shifts, physiologic stress, hypotension, and the effects
of anesthesia. The majority of cardiac ischemic events occur in the first 4 days of the
postoperative period.
Perioperative beta blockade for patients at risk for MI is now routine. Multiple trials and
meta-analyses have demonstrated that this practice yields significant risk reductions in
terms of both cardiac morbidity and mortality42,43 and that these risk reductions are
achieved regardless of the type of surgery being performed. Al though there has been
some variation in the protocols used by these trials and the results reported, there is
general agreement that beta blockade should be initiated preoperatively, delivered at the
time of surgery, and continued postoperatively for up to 1 week.
Diagnosis of postoperative MI is complicated by the fact that as many as 95% of patients
who experience this complication may not present with classic symptoms (e.g., chest
pain). Identification of MI may be further hindered by the ECG changes brought on by
the stress of the perioperative period (including dysrhythmias). Ultimately, the most
useful signal of an ischemic cardiac event in the postoperative period is a rise in the
levels of cardiac enzymes, particularly troponin I. Accordingly, cardiac enzyme activity
should be assessed whenever there is a high index of suspicion for MI or a patient is
considered to be at significant perioperative risk for MI.
Treatment of postoperative MI focuses on correcting any factors contributing to or
exacerbating the situation that led to the event (e.g., hypovolemia or hypotension).
Typically, although antiplatelet agents (e.g., aspirin) are sometimes given, thrombolytic
therapy is avoided because of concerns about postoperative bleeding. Acute percutaneous
coronary intervention is also associated with an increased risk of bleeding but has
nonetheless been used successfully in the peri-operative setting and is recommended by
some physicians.44 Beta blockade is often advocated as a means of treating postoperative
MI, although it is probably more effective when used both preoperatively and
perioperatively as a means of preventing MI.Read more
Feedback on your answer
Collapse
Unit 3
21)
0/1
Which sign on physical examination is MOST predictive in diagnosing appendicitis?

(Select 1)(1pts)

Presence of a Rovsing sign

RLQ tenderness and rigidity

Presence of a psoas sign

Temperature above 38.3°C (101°F)

Presence of a obturator sign

The answer is B, RLQ tenderness and rigidity. Signs on physical examination that are
most predictive of appendicitis are RLQ tenderness and rigidity. Signs that demonstrated
a small effect to discriminate appendicitis were other peritoneal signs (rebound
tenderness, guarding, percussion tenderness), temperature above 38.3°C (101°F), and the
presence of a psoas sign.11,44,45 Signs that do not help in diagnosing appendicitis
include tenderness on rectal examination, increased local skin temperature of RLQ, and
Rovsing sign (ie, pain in the RLQ upon palpation of the LLQ).Read more

Feedback on your answer


Collapse
22)
0/1
You are seeing a 7-year old boy in the ER accompanied by his mother. She notes that he
has been complaining of a fever aand lower abdominal pain for the last 24 hours. She
also notes that he has not eaten anything since yesterday morning as he vomits everything
he attempts to eat. His vitals show a temp is 101.2 F, HR 120, BP 98/70, RR 18 and O2
sat of 99% on room air. On exam he has moderate periumbilical tenderness to light
palpation as well as in his right lower quadrant. His lab work is pending and the surgeon
on call tells you to get some radiology study for confirmation. What is the recommended
initial imaging modality for this patient?
(Select 1)(1pts)

CT scan abdomen/pelvis with IV and PO contrast only

Ultrasound of the Right lower quadrant

MRI abdomen/pelvis

CT scan abdomen/pelvis without IV and PO contrast only

The answer is B, Ultrasound with its lack of ionizing radiation should be the investigation
of choice in young patients, and is effective in competent hands in identifying abnormal
appendixes, especially in thin patients. However, the identification of a normal appendix
is more problematic, and in many instances, appendicitis cannot be ruled out.
The technique used is known as graded compression, and uses the linear probe over
the site of maximal thickness, with gradual increasing pressure exerted to displace normal
overlying bowel gas.
Findings supportive of the diagnosis of appendicitis include 4:
 aperistaltic, noncompressible, dilated appendix ( >6mm outer diameter)
 distinct appendiceal wall layers
 target appearance (axial section)
 appendicolith
 periappendiceal fluid collection
 echogenic prominent pericaecal fat
 confirming that the structure visualised is the appendix is clearly essential and
requires demonstration of it being blind ending and arising from the base of the
caecum
Read more

Feedback on your answer


Collapse
23)
0/1
Neoplastic lesions of the appendix are found in as many as 5% of specimens obtained
with routine appendectomy for acute appendicitis. Most are benign. Preoperative
detection of such conditions is rare, and intraoperative diagnosis is made in fewer than
50% of cases. Which of the following statement regarding neoplastic lesions of the
appendix is true?
(Select 1)(1pts)

Metastasis to the appendix is common

Adenocarcinoma is the most common type represting >80%

The incidence of malignancy in the appendix is 1.35%.

If the tumor is less than 8 cm in diameter, is located within the


body or the tip of the appendix, and has not metastasized,
appendectomy is the treatment of choice.

All of the above

None of the above

Answer C is the only true statement. Neoplastic lesions of the appendix are found in as
many as 5% of specimens obtained with routine appendectomy for acute appendicitis.18-
21 Most are benign. Preoperative detection of such conditions is rare, and intraoperative
diagnosis is made in fewer than 50% of cases. Appendectomy alone may be curative for
appendiceal mucocele, localized pseudomyxoma peritonei, most appendiceal carcinoids,
and other benign tumors. Definitive management of an appendiceal mass unexpectedly
encountered during exploration for clinically suspected acute appendicitis depends on
whether the tumor is carcinoid, its size and location, the presence or absence of metastatic
disease, and histologic and immunohistochemical findings.
Benign neoplasms of the appendix include mucosal hyper-plasia or metaplasia,
leiomyomas, neuromas, lipomas, angiomas, and other rare lesions. Appendiceal
adenomas tend to be diffuse and to have a predominant villous character. Mucus-
producing cystadenomas predispose to appendiceal mucocele, sometimes accompanied
by localized pseudomyxoma peritonei. These lesions are rarely symptomatic and are
often encountered incidentally during operation; however, they may also be clinically
manifested as acute appendicitis, torsion, intussusception, ureteral obstruction, or another
acute condition. If the base of the appendix is free of disease, appendectomy alone is
sufficient treatment.
Malignant tumors of the appendix primarily consist of carcinoids and adenocarcinomas;
all together, they account for 0.5% of all gastrointestinal malignancies. The incidence of
malignancy in the appendix is 1.35%. Metastasis to the appendix is rare. Carcinoids are
substantially more common than adenocarcinomas in the appendix: as many as 80% of
all appendiceal masses are carcinoid tumors. Overall, carcinoid tumors are found in 0.5%
of all appendiceal specimens, and appendiceal carcinoid tumors account for 18.9% of all
carcinoid lesions.23 These tumors are predominantly of neural cellular origin and have a
better prognosis than all other intestinal carcinoid tumors, which typically are of mucosal
cellular origin. If the tumor is less than 2 cm in diameter, is located within the body or the
tip of the appendix, and has not metastasized, appendectomy is the treatment of choice. If
the lesion is at the base of the appendix, is larger than 2 cm in diameter, or has
metastasized, right hemicolectomy is indicated. In addition, secondary right
hemicolectomy is indicated if the tumor is invasive, if mucin production is noted, or if the
tumor is found to be of mucosal cellular origin at the final pathologic examination.24,25
Patients with metastatic appendiceal carcinoid tumors appear to have a far better
prognosis than those with other types of metastatic cancers.24 Therefore, hepatic
debulking for symptomatic control is indicated and justified in cases of liver metastasis.
Figure 8. Laparoscopic appendectomy. The mesoappendix having been divided (a), the
base of the appendix is cleared circumferentially and divided with a gastrointestinal
anastomosis stapler (b). Read more
Feedback on your answer
Collapse
24)
0/1
You are seeing a 32-year-old white female who is currently 16 weeks'
pregnant and is presenting with right lower quadrant pain that started
abruptly last night along with nausea, vomiting, and a low grade fever.
Which one of the following imaging studies would be most appropriate
for initial evaluation of this patient?
(Select 1)(1pts)

CT of the abdomen

MRI of the abdomen

Ultrasonography of the abdomen and Right lower quadrant

A small bowel series

The answer is C, Ultrasonography of the abdomen and Right lower quadrant. CT has
demonstrated superiority over transabdominal ultrasonography for identifying
appendicitis, associated abscess, and alternative diagnoses. However, ultrasonography is
indicated for the evaluation of women who are pregnant and women in whom there is a
high degree of suspicion for gynecologic disease. Ref: Paulson EK, Kalady MF, Pappas
TN: Suspected appendicitis. N Engl J Med 2003;348(3):236-241.
Feedback on your answer
Collapse
25)
1/1
You are seeing a 31-year old male who presents with 24 hours of progressively
worsening lower abdominal pain, vomiting and chills. He denies past medical history and
denies prior surgery.

On exam his heart rate is 95 bpm, BP is 129/80, oral temp 100.2 F and O2 sat is 100% on
room air. Upon palpation he has right lower quadrant pain when you palpated his left
lower quadrant. His labs show a WBC of 9,700mm3 with no bands.

True or False: A normal white blood cell count effectively rules out appendicitis in this
patient.
(1pts)

True

False Correct

False: Appropriate diagnostic testing varies based on the clinical situation. A complete
blood count is appropriate if infection or blood loss is suspected. One study of patients 15
to 83 years of age with suspected appendicitis found that a white blood cell count greater
than 10,000 per mm3 (10 × 10 9 per L) was 77 percent sensitive and 63 percent specific
for the diagnosis (LR+ = 2.1, LR– = 0.37). Thus, nearly one in four patients with
appendicitis does not have an elevated white blood cell count. Read More

Unit 1
1)
0/1
According to multiple sources, what is the most common diagnosis of abdominal pain in
the adult?

( 1pts extra credit)

Acute appendicitis

Acute Cholecystitis

Acute Pancreatitis

Non Diagnostic Abdominal Pain

The answer is D, Non Diagnostic Abdominal Pain. This is in contrast to the most
common surgical cause of abdominal pain in the adult, which is appendicitis. The value
of detailed epidemiologic knowledge notwithstanding, it is worthwhile to keep in mind
the truism that common things are common. Regarding which things are common, the
most extensive information currently available comes from the ongoing survey begun in
1977 by the Research Committee of the OMGE. As of the last progress report on this
survey, which was published in 1988, more than 200 physicians at 26 centers in 17
countries had accumulated data on 10,320 patients with acute abdominal pain
[see Table 3].23 The most common diagnosis in these patients was nonspecific
abdominal pain (NSAP)—that is, the retrospective diagnosis of exclusion in which no
cause for the pain can be identified.24,25 NSAP accounted for 34% of all patients seen;
the four most common diagnoses accounted for more than 75%. The most common
surgical diagnosis in the OMGE survey was acute appendicitis, followed by acute
cholecystitis, small bowel obstruction, and gynecologic disorders. Relatively few patients
had perforated peptic ulcer, a finding that confirms the current downward trend in the
incidence of this condition. Cancer was found to be a significant cause of acute
abdominal pain. There was little variation in the geographic distribution of surgical
causes of acute abdominal pain (i.e., conditions necessitating operation) among
developed countries. In patients who required surgery, the most common causes were
acute appendicitis (42.6%), acute cholecystitis (14.7%), small bowel obstruction (6.2%),
perforated peptic ulcer (3.7%), and acute pancreatitis (4.5%).23 The OMGE survey's
finding that NSAP was the most common diagnosis in patients with acute abdominal pain
has been confirmed by several srudies12,13,25; the finding that acute appendicitis,
cholecystitis, and intestinal obstruction were the three most common diagnoses in
patients with acute abdominal pain who require operation has also been amply confirmed
[see Table 3].1,12,13

Table 3. Frequency of Specific Diagnoses in Patients with Acute Abdominal Pain

Frequency in Individual Studies (% of Patients)

Wilso Brew
n et er et
OMGE23( Irvin13( de
Diagno al82(N al12(N Hawthorn83
N= N= Dombal1(
sis = = (N = 496)
10,320) 1,190) N = 552)
1,196 1,000
) )

Nonspecifi
c
34.0 45.6 34.9 41.3 50.5 36.0
abdominal
pain

Acute
appendicit 28.1 15.6 16.8 4.3 26.3 14.9
is
Acute
cholecysti 9.7 5.8 5.1 2.5 7.6 5.9
tis

Small
bowel
4.1 2.6 14.8 2.5 3.6 8.6
obstructio
n

Acute
gynecolog 4.0 4.0 1.1 8.5 — —
ic disease

Acute
pancreatiti 2.9 1.3 2.4 — 2.9 2.1
s

Urologic
2.9 4.7 5.9 11.4 — 12.8
disorders

Perforated
pepticulce 2.5 2.3 2.5 2.0 3.1 —
r

Cancer 1.5 — 3.0 — — —

Diverticula
1.5 1.1 3.9 — 2.0 3.0
r disease

Dyspepsia 1.4 7.6 1.4 1.4 — —

Gastroenter
— — 0.3 6.9 — 5.1
itis

Inflammato
ry bowel — — 0.8 — — 2.1
disease

Mesenteric
— 3.6 — — — 1.5
adenitis

Gastritis — 2.1 — 1.4 — —

Constipatio
— 2.4 — 2.3 — —
n

Amebic 1.2 — 1.9 — — —


hepatic
abscess

Miscellane
6.3 1.3 5.2 15.5 4.0 8.0
ous

OMGE = World Organization of Gastroenterology.


2014. Scientific American Surgery. Hamilton, Ontario & Philadelphia, PA. Decker
Intellectual Properties Inc. ISSN 2368-2744. STAT!Ref Online Electronic Medical
Library. http://online.statref.com/Document.aspx?fxId=61&docId=803. 10/16/2014
1:15:18 PM CDT (UTC -05:00).

Feedback on your answer


Collapse
2)
0/1
You are seeing a 26-year old college student in the ER following a roll over MVC. He
was wearing a seat belt and was not ejected. Upon arrival his complaining of abdominal
pain. His vital signs show a heart rate of 120/min, blood pressure is 134/98, RR 20/min,
O2 98% on room air. Physical exam shows he has a seat belt sign across his abdomen and
has tenderness greatest in his upper abdomen with no guarding. His CT scan of the
abdomen/pelvis with IV contrast only shows a liver laceration.

True or False: Most liver lacerations/injuries are managed operatively as they


often fail non-operative management.
( 1pts extra credit)

True Incorrect

False

False: The current approach to hepatic trauma has evolved to nonoperative management
in more than 80% of cases. Several contributing factors have been recognized: realization
that more than 50% of liver injuries stop bleeding spontaneously, the precedent of
successful nonoperative management in pediatric patients, knowledge that the liver has
tremendous capacity to heal after injury, and improvements in liver imaging with CT.
Criteria for nonoperative management include foremost, hemodynamic stability, absence
of other abdominal injuries that require laparotomy, immediate availability of resources
including a fully staffed operating room, and a vigilant surgeon. In general, any patient
who is stable enough to have a CT scan performed is likely to be successfully managed
nonoperatively. Grade I and II hepatic injuries should be observed in a monitored setting
with serial hematocrit evaluations and bed rest. Higher-grade injuries in stable patients
should be observed in an intensive care unit setting with optimization of all coagulation
factors. Read More
Feedback on your answer
Collapse
3)
0/1
A 32-year-old male presents to your office with persistent symptoms of nausea, vomiting,
and non bloody watery diarrhea which began about 48 hours ago on the last day of a 5-
day Caribbean cruise to the Bahamas. His wife was sick during the first 2 days of the
cruise with similar symptoms. On the ship, they both ate the “usual foods” in addition to
raw oysters. His vitals are normal and his physical exam in unremarkable. A stool
specimen is negative for white blood cells. Which one of the following is the most likely
cause of his illness?

(Select 1)( 1pts extra credit)

Escherichia coli

Rotavirus

Norwalk virus

Hepatitis A

The answer is C, Norwalk virus. Recent reports of epidemics of gastroenteritis on cruise


ships are consistent with Norwalk virus infections due to waterborne or foodborne
spread. In the United States, these viruses are responsible for about 90% of all epidemics
of nonbacterial gastroenteritis. The Norwalk-like viruses are common causes of
waterborne epidemics of gastroenteritis, and have been shown to be responsible for
outbreaks in nursing homes, on cruise ships, at summer camps, and in schools.
Symptomatic treatment is usually appropriate. Ref: Braunwald E, Fauci AS, Kasper DL,
et al (eds): Harrison’s Principles of Internal Medicine, ed 15. McGraw-Hill, 2001, p
1138.
Feedback on your answer
Collapse
4)
1/1
You are seeing an otherwise-healthy 28-year-old man who is presenting with a 4-month
history of epigastric discomfort and heartburn symptoms that are usually exacerbated
after meals, especially after eating spicy foods. He denies dysphagia, weight loss, or
decreased appetite. He has an active lifestyle and takes no medications. His vital signs or
normal and physical examination is normal except for mild epigastric tenderness to deep
palpation. Routine laboratory studies are normal including liver function studies and
lipase. Bedside ultrasound shows no evidence of gall stones. What is most appropriate
treatment at this time?

(Select 1)( 1pts extra credit)


Upper Endoscopy

Esophageal manometry

Ambulatory 24-hour esophageal pH monitoring

Trial of acid-suppression therapy

None of the above

The answer is D, Typical GERD symptoms include chest discomfort (heartburn) and
regurgitation. Symptoms occur most often after meals, especially fatty meals. Lying
down, bending, or physical exertion often aggravate symptoms, and antacids provide
relief. Patients with classic symptoms rarely require testing to confirm the diagnosis
because of the high positive predictive value of classic symptoms (1). When heartburn
(89% specificity, 81% positive predictive value) and regurgitation (95% specificity, 57%
positive predictive value) occur together, a physician can diagnose GERD with greater
than 90% accuracy.
Performing diagnostic tests for all patients presenting with symptoms that might indicate
GERD would be costly and is not necessary to arrive at a sufficiently accurate diagnosis.
Response to an empirical trial of acid-suppression therapy is considered a sufficiently
sensitive and specific method for establishing a GERD diagnosis among patients with
classic symptoms of heartburn or regurgitation. read more
Feedback on your answer
Collapse
5)
0/1
You have just intubated a 43-year old male after following a cardiac arrest using a 7.0
mm tube. At what depth should you secure the tube?

(Select 1)( 1pts extra credit)

15 cm at the lip

18 cm at the lip

21 cm at the lip

24 cm at the lip

The formula for tube depth following intubation is tube size x 3. In this case its 7.0 x 3
which is 21.
Feedback on your answer
Collapse
Unit 2
6)
0/1
What is the most common surgical cause of abdominal pain in adults?
(Select 1)(1pts)

Appendicitis

Cholecystitis

Small bowel obstruction

Perforated peptic ulcer

All of the above

None of the above

The answer is A, appendicitis. The value of detailed epidemiologic knowledge


notwithstanding, it is worthwhile to keep in mind the truism that common things are
common. Regarding which things are common, the most extensive information currently
available comes from the ongoing survey begun in 1977 by the Research Committee of
the OMGE. As of the last progress report on this survey, which was published in 1988,
more than 200 physicians at 26 centers in 17 countries had accumulated data on 10,320
patients with acute abdominal pain [see Table 3].23 The most common diagnosis in
these patients was nonspecific abdominal pain (NSAP)—that is, the retrospective
diagnosis of exclusion in which no cause for the pain can be identified.24,25 NSAP
accounted for 34% of all patients seen; the four most common diagnoses accounted for
more than 75%. The most common surgical diagnosis in the OMGE survey was acute
appendicitis, followed by acute cholecystitis, small bowel obstruction, and gynecologic
disorders. Relatively few patients had perforated peptic ulcer, a finding that confirms the
current downward trend in the incidence of this condition. Cancer was found to be a
significant cause of acute abdominal pain. There was little variation in the geographic
distribution of surgical causes of acute abdominal pain (i.e., conditions necessitating
operation) among developed countries. In patients who required surgery, the most
common causes were acute appendicitis (42.6%), acute cholecystitis (14.7%), small
bowel obstruction (6.2%), perforated peptic ulcer (3.7%), and acute pancreatitis
(4.5%).23 The OMGE survey's finding that NSAP was the most common diagnosis in
patients with acute abdominal pain has been confirmed by several srudies12,13,25; the
finding that acute appendicitis, cholecystitis, and intestinal obstruction were the three
most common diagnoses in patients with acute abdominal pain who require operation has
also been amply confirmed [see Table 3].1,12,13
Table 3. Frequency of Specific Diagnoses in Patients with Acute Abdominal Pain

Frequency in Individual Studies (% of Patients)

Wilso Brew
n et er et
OMGE23( Irvin13( de
Diagno al82(N al12(N Hawthorn83
N= N= Dombal1(
sis = = (N = 496)
10,320) 1,190) N = 552)
1,196 1,000
) )

Nonspecifi
c
34.0 45.6 34.9 41.3 50.5 36.0
abdominal
pain

Acute
appendicit 28.1 15.6 16.8 4.3 26.3 14.9
is

Acute
cholecysti 9.7 5.8 5.1 2.5 7.6 5.9
tis

Small
bowel
4.1 2.6 14.8 2.5 3.6 8.6
obstructio
n

Acute
gynecolog 4.0 4.0 1.1 8.5 — —
ic disease

Acute
pancreatiti 2.9 1.3 2.4 — 2.9 2.1
s

Urologic
2.9 4.7 5.9 11.4 — 12.8
disorders

Perforated
pepticulce 2.5 2.3 2.5 2.0 3.1 —
r

Cancer 1.5 — 3.0 — — —

Diverticula 1.5 1.1 3.9 — 2.0 3.0


r disease

Dyspepsia 1.4 7.6 1.4 1.4 — —

Gastroenter
— — 0.3 6.9 — 5.1
itis

Inflammato
ry bowel — — 0.8 — — 2.1
disease

Mesenteric
— 3.6 — — — 1.5
adenitis

Gastritis — 2.1 — 1.4 — —

Constipatio
— 2.4 — 2.3 — —
n

Amebic
hepatic 1.2 — 1.9 — — —
abscess

Miscellane
6.3 1.3 5.2 15.5 4.0 8.0
ous

OMGE = World Organization of Gastroenterology.


2014. Scientific American Surgery. Hamilton, Ontario & Philadelphia, PA. Decker
Intellectual Properties Inc. ISSN 2368-2744. STAT!Ref Online Electronic Medical
Library. http://online.statref.com/Document.aspx?fxId=61&docId=803. 10/16/2014
1:15:18 PM CDT (UTC -05:00).

Feedback on your answer


Collapse
7)
0/1
Which of the following is not an example of referred pain?
(Select 1)(1pts)

Ureteral stone causes ipsilateral testicular pain


Ruptured hemorrhagic cyst causing lower back pain

Acute MI causing jaw pain

Acute Appendicitis causing periumbilical pain

All of the above

None of the above

The answer is D, as Acute Appendicitis causing periumbilical pain is an example of


visceral pain.
Visceral Pain
•Caused by the stretching of fibers innervating the walls or capsules of hollow or solid
organs
•A steady ache or vague discomfort to excruciating or colicky
•Visceral afferents follow a segmental distribution and pain is localized by the sensory
cortex to an approximate spinal cord level determined by embryological organ of the
organ involved.
•Biliary, duodenum, stomach produce epigastric pains
•Most small bowel, appendix, cecum cause periumbilical pain
•Hindgut structures, such as the bladder, distal two-thirds of the colon, and pelvic
genitourinary (GU) organs, usually cause pain in the suprapubic region.
•Pain is often reported in the back for retroperitoneal structures, such as the aorta and
kidneys
•Deep musculoskeletal structures (especially of the back) are innervated by visceral
sensory fibers with similar qualities to those arising from intra-abdominal organs.
•Intraperitoneal organs are bilaterally innervated causing pain to be located in the midline
regardless of which side it is coming from. Classic example is early appendicitis causing
periumbilical pain

Visceral Pain
•Caused by the stretching of fibers innervating the walls or capsules of hollow or solid
organs
•A steady ache or vague discomfort to excruciating or colicky
•Visceral afferents follow a segmental distribution and pain is localized by the sensory
cortex to an approximate spinal cord level determined by embryological organ of the
organ involved.
•Biliary, duodenum, stomach produce epigastric pains
•Most small bowel, appendix, cecum cause periumbilical pain
•Hindgut structures, such as the bladder, distal two-thirds of the colon, and pelvic
genitourinary (GU) organs, usually cause pain in the suprapubic region.
•Pain is often reported in the back for retroperitoneal structures, such as the aorta and
kidneys
•Deep musculoskeletal structures (especially of the back) are innervated by visceral
sensory fibers with similar qualities to those arising from intra-abdominal organs.
•Intraperitoneal organs are bilaterally innervated causing pain to be located in the midline
regardless of which side it is coming from. Classic example is early appendicitis causing
periumbilical pain
Feedback on your answer
Collapse
8)
0/1
You are seeing a 28-year old white male who was brought in by EMS after he was
assaulted while at a night club. On exam his GCS is 7, he is breathing spontaneously and
localizes pain. He has a large scalp laceration to his right parietal region. He is rushed to
the CT scanner and the image is shown below. What is the diagnosis?

(Select 1)(1pts)

Subdural Hematoma

Epidural Hematoma
Intraparenchymal Hematoma with extension

Subarachnoid Hemorrhage

The answer is B. Epidural hematoma (EDH) is a traumatic accumulation of blood


between the inner table of the skull and the stripped-off dural membrane. The inciting
event often is a focused blow to the head, such as that produced by a hammer or baseball
bat. In 85-95% of patients, this type of trauma results in an overlying fracture of the skull.
Blood vessels in close proximity to the fracture are the sources of the hemorrhage in the
formation of an epidural hematoma. Because the underlying brain has usually been
minimally injured, prognosis is excellent if treated aggressively. Outcome from surgical
decompression and repair is related directly to patient's preoperative neurologic
condition. more from Read more
Feedback on your answer

Collapse
Unit 3
9)
0/1
You are seeing a 44-year-old female in your outpatient GI clinic for persistent acid reflux
symptoms. She was on zantac for 4 weeks following her last EGD and then switched to
protonix daily for 2 months which did not seem to help. Her records indicate a normal
upper endoscopy 4 months ago as she was complaining of some mild dysphagia and
biopsies were negative for h. pylori. She otherwise has no other past medical history and
takes no other medications, denies alcohol ingestion and is not a smoker. Vital signs and
physical examination are normal. Which of the following studies should be done next to
confirm this patient's most likely diagnosis?
(Select 1)(1pts)

Repeat H. pylori testing

Barium swallow radiography

Esophageal manometry

Ambulatory esophageal pH-impedance monitoring

The answer is Ambulatory esophageal pH-impedance monitoring, This patient most


likely has gastroesophageal reflux disease (GERD), and the gold standard for diagnosing
GERD is 24-hour esophageal pH-impedance monitoring, which establishes whether
increased esophageal exposure to acid is present. This study is most helpful in patients
who have GERD-like symptoms that fail to respond to an adequate trial of a proton pump
inhibitor (PPI) to correlate the presence of acid in the esophagus at the time the patient is
having symptoms, such as the patient described. Esophageal pH monitoring identifies the
reflux of acid, and impedance monitoring detects reflux of other gastric contents in a
small percentage of patients who have symptoms related to non-acid reflux.
GERD is definitively diagnosed by symptoms, upper endoscopy, or ambulatory
esophageal pH/impedance monitoring. However, the presence of heartburn, regurgitation,
or both is sufficient to presumptively diagnose GERD if symptoms are of sufficient
severity and frequency to be troublesome for the patient. A favorable response to empiric
therapy with PPIs is supportive evidence for a diagnosis of GERD and is a reasonable
first step in a patient without alarm symptoms (dysphagia, anemia, vomiting, or weight
loss). Kumar AR, Katz PO. Functional esophageal disorders: a review of diagnosis and
management. Expert Rev Gastroenterol Hepatol. 2013;7:453-61. PMID: 23899284

Feedback on your answer

Collapse
10)
0/1
You are consulted to see a 61-year-old male because of continuing symptoms due to
refractory gastroesophageal reflux that have not improved despite lifestyle
modifications and treatment with a twice daily proton pump inhibitor (omeprazole 40 mg
daily). He notes that he continues to have occasional chest pain and epigastric burning
with eating. He denies dysphagia, regurgitation, weight loss, or a change in bowel habits.
He denies other past medical history and has no cardiac risk factors. Vital signs and
physical examination are normal. His upper endoscopy shows no ulcers and is otherwise
unremarkable. Which of the following is the most appropriate treatment at this time?
(Select 1)(1pts)

Schedule consultation for evaluation for antireflux surgery

Increase the proton pump inhibitor to 3 times daily

Change to a different proton pump inhibitor

Add ranitidine at bedtime to the current regimen

The answer is A. Consider surgery as an option for patients with well-documented GERD
who require long-term PPI maintenance therapy but show satisfactory relief of symptoms
and who:
 Are older than 50 years
 Consider long-term medication a financial burden
 Are noncompliant with drug therapy
 Prefer a single surgical intervention to long-term drug treatment
 Experience prominent symptoms of regurgitation, even with medical
control of heartburn symptoms
Nissen fundoplication is the most common surgical intervention for GERD. This
procedure aims to restore the physiology and anatomy of the gastroesophageal junction
by wrapping the gastric fundus around the distal esophagus. The FDA has also approved
several endoscopic procedures for treatment of GERD, including endoscopic suturing and
radiofrequency ablation of the lower esophageal sphincter. Read More

Feedback on your answer

Collapse
11)
0/1
Which of the following is NOT associated with an increased risk of esophageal
adenocarcinoma?
(Select 1)(1pts)

Cigarette smoking

Gastroesophageal reflux disease

Obesity

Helibactor pylori infection

All of the above

None of the above

Answer D is not correct. Esophageal cancer has two main subtypes — esophageal
squamous-cell carcinoma and esophageal adenocarcinoma; their precursor lesions are
esophageal squamous dysplasia and Barrett’s esophagus, respectively. Although
squamous-cell carcinoma accounts for about 90% of cases of esophageal cancer
worldwide, the incidence of and mortality rates associated with esophageal
adenocarcinoma are rising and have surpassed those of esophageal squamous-cell
carcinoma in several regions in North America and Europe. In the United States, more
than 18,000 new cases of esophageal cancer and more than 15,000 deaths from
esophageal cancer were expected in 2014. Esophageal carcinoma is rare in young people
and increases in incidence with age, peaking in the seventh and eighth decades of life.
The main risk factors for esophageal adenocarcinoma are gastroesophageal reflux
disease, obesity, and cigarette smoking; H. pyloriinfection is associated
with a reduced risk. Cigarette smoking and alcohol consumption constitute the
main risk factors for esophageal squamous-cell carcinoma. High intake of red meats, fats,
and processed foods is associated with an increased risk of both types of esophageal
cancer, whereas high intake of fiber, fresh fruit, and vegetables is associated with a lower
risk.
N Engl J Med 2014; 371:2499-2509December 25, 2014DOI:
10.1056/NEJMra1314530

Feedback on your answer

Collapse
Unit 4
12)
0/1
A 48-year old black male presents to you with sudden onset of vomiting and diffuse
abdominal pain that hurts to breathe two hours prior to arrival. He denies past medical
history and his only medications are ibuprofen for chronic right shoulder pain. He smokes
1 pack per day and occasionally drinks alcohol.
His vitals show an oral temperature of 99.3 F, heart rate of 129 beats/min, respiratory rate
20/min, blood pressure 151/100, O2 saturation of 98% on room air.
Physical exam shows him to be uncomfotable. Cardiac exam shows tachycardia, his
lungs are clear to auscultation, and abdominal exam shows diffuse tenderness to light
palpation greatest in the right upper quadrant along with guarding. Rectal exam shows
heme positive brown stool. An upright chest x-ray which is shown below. What is the
diagnosis?
(Select 1)(1pts)

Pneumomediastinum

Pneumoperitoneum

Intestinal Malrotation

Small Bowel Obstruction

Pneumothorax

The answer is B, Pneumoperitoneum which describes gas within the peritoneal cavity,
and is often the harbinger of a critical illness. The most common cause of a
pneumoperitoneum is from the disruption of the wall of a hollow viscus. Read more
An erect chest x-ray is probably the most sensitive plain radiograph for the detection of
free intraperitoneal gas. If a large volume pneumoperitoneum is present, it may be
superimposed over normal aerated lung with normal lung markings.
 subdiaphragmatic free gas
 cupola sign (in supine film)
Free gas within the peritoneal cavity can be detected on an abdominal radiograph. These
signs can be further divided by anatomical compartments in relation to the
pneumoperitoneum:
 bowel related signs
 double wall sign (also known as Rigler's sign or bas-
relief sign)
 telltale triangle sign (also known as triangle sign)
 peritoneal ligament related signs
 football sign
 falciform ligament sign
 lateral umbilical ligament sign (also known as inverted
"V" sign)
 urachus sign
 right upper quadrant signs
 lucent liver sign
 hepatic edge sign
 fissure for ligamentum teres sign
 Morison's pouch sign
 cupola sign
The patient above likely has a perforated ulcer from underlying NSAID use. Ulcer
perforation should be suspected in patients who suddenly develop severe, diffuse
abdominal pain. Perforations complicate 2 to 10 percent of peptic ulcers [70]. Duodenal,
antral, and gastric body ulcers account for 60, 20, and 20 percent of perforations due to
peptic ulcer disease (PUD), respectively. If imaging is required, plain x-rays are typically
obtained first. Careful interpretation of upright chest and abdominal films can detect
diagnostic free air in many cases of perforated gastric and duodenal ulcers [76]. The
presence of free air on abdominal imaging is highly indicative of a perforated viscus
(image 1 and image 2 and image 3), although about 10 to 20 percent of patients
with a perforated duodenal ulcer will not have free air [76]. If free air is found, no other
diagnostic studies are necessary. Leakage of water soluble oral contrast may be useful in
selected cases. Once the oral contrast is given, the patient should be rotated 360 degrees
and placed on the right side to fill the antrum and duodenum with contrast. However,
many perforations have already sealed spontaneously by the time of presentation [77], so
the absence of a leak does not exclude the diagnosis of a perforated ulcer. Read More
Feedback on your answer
Collapse
13)
0/1
You are seeing a 44-year old female who presents with worsening epigastric pain
following meals and "heartburn" that has been getting more persistent over the last few
days. She has a past medical history of hypertension. Her only medication is naprosyn
500 mg BID and HCTZ 25mg daily. On exam she has mild epigastric tenderness to
palpation. Her rectal exam shows her stool is brown but guaiac positive for blood. Her
portable upright chest xray shows no signs of free air or perforation.Based on her history
of current NSAID use, where would you likely expect the location of her ulcer?
(Select 1)(1pts)

Gastric

Duodenal

Jejunal

Esophageal

NSAID use confers around a 40 fold increase in the development of gastric ulcers and an
8 fold increase risk for the development of duodenol ulcers and . Therefore in this patient
you would expect gastric ulcer more likely than a duodenal ulcer.
The use of NSAIDs is the most commonly identified risk factor for peptic ulcer bleeding,
especially in the elderly. Studies have found relative risks for bleeding ranging from 2.7
to 33.9 [11]. Studies have also shown that the risk is drug-specific and dose-dependent.
As an example, in a study of 2777 patients, the overall relative risk (RR) of bleeding
associated with NSAID use was 5.3 (95% CI 4.5-6.2). However, the risk varied by drug
and was lowest for aceclofenac (RR 3.1, 95% CI 2.3-4.2) and was highest for ketorolac
(RR 14.4, 95% CI 5.2-39.9) [18]. The risk was higher in patients taking high-dose
NSAIDs compared with those taking medium- or low-dose NSAIDs (RR 6.8, 95% CI
5.3-8.8 versus 4.0, 95% CI 3.2-5.0). There was also an increased risk of bleeding with
aspirin use (RR 5.3) that again was dose-dependent (RR 7.5 with 500 mg per day versus
2.7 with 100 mg per day). The concurrent use of aspirin and NSAIDs conferred an even
greater risk of bleeding than was seen with either agent alone (RR 12.7). Finally, the risk
was highest in the first 30 days of NSAID use, with a RR of 7.6 (95% CI 6.0-9.5). The
risk remained high between days 31 and 90 days (RR 7.3, 95% CI 4.0-13.2), but dropped
after 91 days (RR 2.6, 95% CI 1.6-4.1).
NSAID use has also been identified as a risk factor for ulcer perforation [16,19]. In a
study of 176 patients from Spain, NSAID use was the only risk factor that was
significantly associated with perforation (odds ratio [OR] 3.6). Read more
Feedback on your answer
Collapse
14)
0/1
You are seeing a 45-year old male in clinic who has a past medical history of
osteoarthritis for which he takes daily alleve. He denies tobacco use or alcohol use. He
has been having recent early satiety, bloating and epigastric discomfort over the last
month. You suspect peptic ulcer disease. Which of the following ulcer locations is more
associated with H. Pylori infection?
(Select 1)(1pts)

Duodenal

Gastric

Esophageal

Jejunal

None of the above

The answer is A, duodenal ulcers. H. pylori, a gram-negative, helical, rod-


shaped bacterium, colonizes the gastric mucosa of approximately one-half of the world
population and an estimated 30% to 40% of the U.S. population. H. pylori is present in
95% of patients with duodenal ulcers and in 70% of those with gastric ulcers. It is
typically transmitted via the fecal-oral route during early childhood and persists for
decades. The bacterium is a known cause of gastric and duodenal ulcers and is a risk
factor for mucosa-associated lymphoid tissue (MALT) lymphoma and gastric
adenocarcinoma. Am Fam Physician. 2015 Feb 15;91(4):236-242.

Feedback on your answer


Collapse
15)
1/1
What is the most common complication of peptic ulcer disease?

(Select 1)(1pts)

Gastric outlet obstruction

Perforation

Bleeding

Cancer

The answer is B, Bleeding. Bleeding is the most common complication of peptic ulcer
disease requiring hospitalization. In a study of the National Inpatient Sample (NIS),
bleeding occurred in 73 percent, perforation in 9 percent, and obstruction in 3 percent
[4]. H. pylori, nonsteroidal antiinflammatory drugs (NSAIDs), and the use of low dose
aspirin are the most common etiologies of ulcer bleeding [7] and ulcer perforation [8-
10]. In observational studies, duodenal, antral/pyloric, and gastric body ulcers
account for 60, 20, and 20 percent of perforations, respectively [8,9]. Among these
complications, perforation had the highest mortality rate, followed by obstruction, then
hemorrhage.
Bleeding peptic ulcer — Upper gastrointestinal bleeding due to peptic ulcer disease
is a common indication for emergency management of peptic ulcer disease [11]. Most
patients with acute bleeding can be managed with fluid resuscitation and transfusion, acid
suppression therapy, and endoscopic intervention. For those who fail these efforts,
surgery may become necessary. The specific indications for surgery for patients with
bleeding peptic ulcer are reviewed separately.
Perforated peptic ulcer — Ulcer perforation may be suspected in patients with a
history consistent with peptic ulcer disease who develop the sudden onset of severe,
diffuse abdominal pain. Once a diagnosis of perforation is established, surgical
intervention is indicated.
Gastric outlet obstruction — Gastric outlet obstruction is the least frequent ulcer
complication in developed countries. Most cases are associated with duodenal or pyloric
channel ulceration; gastric ulceration accounts for only five percent of cases.
Surgical consultation should be obtained for patients with chronic partial gastric outlet
obstruction that is refractory to medical treatment, and those found to have complete
gastric outlet obstruction, or those readmitted with gastric outlet obstruction after recent
“successful treatment.” Surgery is indicated if the patient fails to respond to conservative
medical management and endoscopic therapy.
Surgical treatment of gastric outlet obstruction is almost never an emergency. Because
most patients present with some degree of malnutrition and frequently have electrolyte
imbalances (ie, hypokalemic, hypochloremic metabolic alkalosis secondary to vomiting
or nasogastric suctioning), it is important to correct any derangements and optimize the
patient’s medical status prior to proceeding with surgery. Failure to identify and correct
these issues increases perioperative morbidity.
It may also be reasonable to try and improve the patient’s overall nutritional status with
nutritional support, as these patients are frequently malnourished and can have significant
postoperative delayed gastric emptying due to their atonic stomach.
Many experienced gastric surgeons advocate preoperative nasogastric sump
decompression to decrease gastric dilation and, hopefully, gastric atony. Read more

Feedback on your answer


Collapse
16)
0/1
What % of benign appearing gastric ulcers are found to be harbor malignancy?
(Select 1)(1pts)
<1%

5-11%

15-21%

24-30%

The answer is B, 5-11%. Given the low predictive value of the signs and symptoms of
PUD, the diagnosis of uncomplicated peptic ulcers is difficult to make solely on a clinical
basis. The diagnosis of PUD is typically made by endoscopy and upper gastrointestinal
(UGI) radiography. Endoscopy is the most accurate method of establishing the diagnosis
of peptic ulcers, with a reported sensitivity of 92% and specificity of 100%.7 In addition
to identifying the ulcer and its features, location, and size, endoscopy provides an
opportunity for biopsies to test for H. pylori and exclude malignancy and for therapeutic
interventions for bleeding ulcers. Duodenal ulcers are most often benign and do not
require routine biopsy. Multiple biopsies are indicated for all gastric ulcers as even
lesions with a benign appearance harbor malignancy in 5 to 11% of cases. Read more
Feedback on your answer
Collapse
Unit 5
17)
0/1
You are seeing a 42-year-old female following in the trauma bay following a high-speed
motor vehicle collision. She was was a restrained passenger and her car flipped over after
she lost control. She had to be cut out of the car and is complaining of shortness of breath
and upper abdominal pain.
She is awake and alert and her GCS is 15. Her pulse is 120/min, respirations are 28/min,
and blood pressure is 80/40 mm Hg. Breath sounds are decreased at the left lung base. An
x-ray of the chest shows opacification of the left lower lung field. You decide to place an
emergent chest tube that yields a small amount of air followed by greenish fluid. Which
of the following is the most appropriate next step in management?
(Select 1)(1pts)

CT scan of the abdomen and pelvis

CT scan of the chest

Thoracoscopy
Laparotomy

The answer is D. This patient suffered a ruptured diaphragm and chest tube placement
appears to have ruptured the bowel as evidence from the greenish fluid that was returned.
She needs an emergent laparotomy to control bleeding and repair her diaphragm. Read
More
Feedback on your answer
Collapse
18)
0/1
You are seeing a 27-year old male in the trauma bay shortly after being stabbed in this
back with a 6 cm buck knife by his disgruntled neighbor after an argument.
He arrived via EMS sitting up, awake and alert with a GCS of 15. His BP is 140/90, HR
120 and O2 sat is 85% on a non re-breather.
On physical exam there is a 3 cm non bleeding wound just left of his mid thoracic spine.
Upon auscultation there is decreased breath sounds on his left side. There are no signs of
tracheal deviation or JVD.
What is the next best step in management?
(1pts)

Immediately to the OR for surgical exploration

Upright Chest Xray followed by a chest tube

Immediate Needle thoracotomy on the left

Pericardiocentesis under US guidance

The answer is A, Upright Chest Xray followed by a chest tube. This patient is awake and
talking. He is stable enough for a chest xray followed by chest tube placement. A needle
thoracotomy to should be done if he has evidence of tension pneumothorax which would
be respiratory distress and hypotension. He will likely then need a chest tube and can be
further managed. Read More
Feedback on your answer
Collapse
19)
0/1
What is the most common abdominal organ injured in the setting of blunt abdominal
trauma?
(Select 1)(1pts)
Spleen

Liver

Kidneys

Small Intestines

The liver is the most commonly injured solid organ in blunt trauma, comprising 5% of all
trauma admissions, and because of its size is frequently involved in penetrating trauma.
Following blunt trauma, the most commonly injured structures are the parenchyma and
hepatic veins. Blunt forces dissipate along segments of the liver and along the fibrous
coverings of the portal triad structures; the hepatic veins, however, are not so insulated.
Given its size and location within the abdomen, the liver is also commonly involved in
penetrating trauma. Stab wounds typically result in direct linear tears, whereas gunshot
wounds or shotgun wounds result in significant cavitary injuries attributable to blast
effect and the "tumbling" of the missile within the liver parenchyma. Thus, arterial injury
is more common with penetrating trauma.
Over the past 20 years, nonoperative management (NOM) of liver injuries has evolved to
become the prevailing therapeutic strategy for blunt hepatic trauma. Several concurrent
changes resulted in this paradigm change. First was the realization that diagnostic
peritoneal lavage (DPL) was sensitive but not specific for identifying intraperitoneal
hemorrhage that necessitated operative management. Surgeons recognized that many
laparotomies undertaken for a positive DPL were associated with liver injuries that did
not require intervention for bleeding.1Second, trauma surgeons noted that nonbleeding
hepatic venous injuries, if manipulated at laparotomy, often resulted in more hemorrhage
and sometimes even death.2 Furthermore, it became conspicuous that with hemostasis
achieved in the operating room, recurrent postoperative bleeding was rare. Therefore,
surgeons queried whether hepatic venous injuries, which are low-pressure system
injuries, could heal without intervention. Finally, computed tomography (CT) provided a
reliable method for diagnosing and grading liver injuries.

The spleen is the second most commonly injured abdominal organ in blunt trauma
patients. Historical studies have reported a 10% mortality with all splenic injuries;
however, isolated splenic injury mortality is less than 1%. The mechanisms of injury are
similar to those seen with liver injuries: motor vehicle collisions, autopedestrian
accidents, and falls. Similar to penetrating trauma to the liver, stab wounds to the spleen
typically result in direct linear tears, whereas gunshot wounds result in significant
cavitary injuries.

Until the 1970s, splenectomy was considered mandatory for all splenic injuries.
Recognition of the immune function of the spleen refocused efforts on splenic salvage in
the 1980s.38,39Following success in pediatric patients, NOM of splenic injuries was
adopted in the adult population and has become the prevailing strategy for blunt splenic
trauma.
Duodenal and pancreatic injury continues to challenge the trauma surgeon. The relatively
rare occurrence of these injuries, the difficulty in making a timely diagnosis, and high
morbidity and mortality rates justify the anxiety these unforgiving injuries invoke.
Mortality rates for pancreatic trauma range from 9 to 34%, with a mean rate of 19%.
Duodenal injuries are similarly lethal, with mortality rates ranging from 6 to 25%.
Complications following duodenal or pancreatic injuries are alarmingly frequent,
occurring in 30 to 60% of patients.1-3 Recognized early, the operative treatment of most
duodenal and pancreatic injuries is straightforward, with low morbidity and mortality.

2014. Scientific American Surgery. Hamilton, Ontario & Philadelphia, PA. Decker
Intellectual Properties Inc. ISSN 2368-2744. STAT!Ref Online Electronic Medical
Library. http://online.statref.com/Document.aspx?fxId=61&docId=2329. 10/27/2014
1:39:15 PM CDT (UTC -05:00).

Feedback on your answer


Collapse
Unit 6
20)
0/1
You are seeing a 64-year old black female who is presenting with 2 days of worsening
upper abdominal pain that has become persistent. She denies fever at home but has a n
oral temp of 38.0 (100.4 F) in the ER. Her BP is 94/50, HR is 122 bpm and respiratory
rate 16/min. On exam she appears slightly jaundiced in her sclera, she has a positive
murphy's sign with some involuntary guarding. Her WBC comes back at 20k with 11%
bands. Her BMP is unremarkable. What triad of symptoms is classically associated with
cholangitis?
(Select 1)(1pts)

RUQ pain, fever, bandemia

Epigastric pain, fever, leukocystosis

Bandemia, fever, jaundice

RUQ pain, fever, jaundice

The answer is D. The classic presentation of acute cholangitis is fever, abdominal pain,
and jaundice (Charcot's triad), though only 50 to 75 percent of patients with acute
cholangitis have all three findings. Confusion and hypotension can occur in patients with
suppurative cholangitis, producing Reynolds pentad, which is associated with significant
morbidity and mortality.
 Fever is present in approximately 90% of cases.
 Abdominal pain and jaundice is thought to occur in 70% and 60% of
patients, respectively.
 Patients present with altered mental status 10-20% of the time and
hypotension approximately 30% of the time. These signs, combined
with Charcot's triad, constitute Reynolds pentad.
 Consequently, many patients with ascending cholangitis do not present
with the classic signs and symptoms.[4]
 Most patients complain of RUQ pain; however, some patients (ie,
elderly persons) are too ill to localize the source of infection.
Read more from Up to Date

Feedback on your answer


Collapse
21)
0/1
Which of the following complications of cholelithiasis can present with fever, persistent
tachycardia, hypotension, or jaundice?
(Select 1)(1pts)

Cholecystitis

Cholangitis

Pancreatitis

All of the above

None of the above

The presence of fever, persistent tachycardia, hypotension, or jaundice necessitate a


search for complications of cholelithiasis, including cholecystitis, cholangitis,
pancreatitis, or other systemic causes.
For more on the physical examination of patients with gallstones, read here.
Feedback on your answer
Collapse
22)
0/1

Which of following best describes a patient with Courvoisier sign?

(1pts)
a palpable tender gallbladder in a patient with jaundice

a palpable non-tender gallbladder in a patient with jaundice

a palpable non-tender gallbladder in a patient without jaundice

a palpable tender gallbladder in a patient without jaundice

The answer is B, A positive Courvoisier Sign is defined as a palpable non-tender


gallbladder in a patient with jaundice.
Over a century ago, Courvoisier observed that a palpable gallbladder in a patient with
obstructive jaundice is often caused by a non-calculus abnormality of the biliary system,
such as pancreatic cancer or cholangiocarcinoma, distal to the insertion of the cystic
duct.1–4 He attributed his findings to a higher likelihood of fibrosis of the gallbladder,
with stone disease rendering it less distensible.4
Although often associated with malignancy, the Courvoisier sign can also be seen in
benign processes causing obstruction of the common bile duct.5
For decades after its initial description, the Courvoisier sign was used as an important
sign for the differential diagnosis of jaundice, but advances in diagnostic imaging have
led to a more accurate and earlier diagnosis with less reliance on this sign.CCJM
Feedback on your answer
Collapse
Unit 7
23)
0/1
In the absence of a major surgical insult or concomitant coagulopathy, what platelet count
is required for normal coagulation?
(1pts)

10,000/μL

20,000/μL

50,000/μL

100,000/μL

The answer is B, Patients with a normal INR and a normal aPTT who exhibit unexpected
bleeding may have impaired platelet activity. Inadequate platelet activity is frequently
manifested as persistent oozing from wound edges or as low-volume bleeding. Such
bleeding is rarely the cause of exsanguinating hemorrhage, although it may be life-
threatening when it occurs in certain locations (e.g., inside the cranium or the
pericardium). Inadequate platelet activity may be attributable to an insufficient number of
platelets, platelet dysfunction, or a platelet inhibitor. In the absence of a major surgical
insult or concomitant coagulopathy, a platelet count of 20,000/μL or higher is usually
adequate for normal coagulation.14,15 There is some disagreement regarding the
threshold for platelet transfusion in the absence of active bleeding. Patients undergoing
procedures in which even small-vessel oozing is potentially life-threatening (e.g.,
craniotomy) should be maintained at a higher platelet count (i.e., > 20,000/μL). Patients
without ongoing bleeding who are not specifically at increased risk for major
complications from low-volume bleeding may be safely watched with platelet counts as
low as 10,000/μL. Read more
Feedback on your answer
Collapse
24)
1/1
Healthy young patients can usually maintain a normal blood pressure until their blood
loss exceeds what % of their blood volume (or roughly __ L)?
(Select 1)(1pts)

20%/3L

40%/2L

10%/1L

60%/4L

The answer is B, 40%/2L. It is critical to emphasize that the most common causes of
postoperative bleeding, even when it is not massive, are technical: an unligated vessel or
an unrecognized injury is much more likely to be the cause of a falling hematocrit than
either a drug effect or an endogenous hemostatic defect. Furthermore, if an unligated
vessel is treated as though it were an endogenous hemostatic defect (i.e., with
transfusions), the outcome is frequently disastrous. For these reasons, in all cases of
ongoing bleeding, the first consideration must always be to exclude a surgically
correctable cause. Ongoing bleeding may be surprisingly difficult to diagnose. Healthy
young patients can usually maintain a normal blood pressure until their blood loss
exceeds 40% of their blood volume (roughly 2 L). If the bleeding is from a laceration to
an extremity, it will be obvious; however, if the bleeding is occurring internally, there
may be few physiologic signs until the patient is at high risk for death. Even when a
technical cause of bleeding has seemingly been excluded, the possibility should be
reconsidered periodically throughout assessment. Patients who are either unresuscitated
or underresuscitated undergo vasospasm that results in decreased bleeding. As
resuscitation proceeds, the catecholamine-induced vasospasm subsides and bleeding may
recur. Only when the surgeon is confident that a missed injury or unligated vessel is not
the cause of the bleeding should other potential causes be investigated. Read more
Feedback on your answer
Collapse
25)
1/1
Which of the following statements about massive transfusion protocol is true?
(1pts)

Platelets and plasma are only given when the PT/INR or ptt are abnormal

The goal is to transfuse with a 4:1:1 ratio with one unit of plasma and one
of platelets along with each 4 units of red blood cells (RBCs)

The use of fresh whole blood transfusion in war zones were shown to
improve outcomes prompting military surgeons to advocate a different
transfusion ratio

Fresh whole blood is reduces the risk of transmitting transfusion-related


infections

Answer C is the only true statement. It is true that coagulopathy may promote massive
hemorrhage and that massive hemorrhage causes a coagulopathy. As a rule, the massively
hemorrhaging patient has a coagulopathy out of proportion to the abnormalities reflected
with in vitro laboratory tests such as the prothrombin time (PT, commonly expressed as
an international normalized ratio [INR]) or activated partial thromboplastin time (aPTT).
Coagulopathy most typically occurs when there is massive hemorrhage in concert with
injury to a large volume of tissue. Although the nature of this coagulopathy is not
precisely defined, it probably involves a number of factors: (1) platelet dysfunction and
diminished availability of platelets at the interface between the vascular endothelium and
luminal flow; (2) dysfunctional formation of thrombi in the setting of acidosis and
hypothermia; (3) accelerated thrombolysis (attributable to dysfunctional thrombosis);
and, eventually, (4) consumption of clotting factors.
Increased recognition of this coagulopathy has prompted enthusiasm for administration of
plasma and platelets early in the treatment of the patient with massive hemorrhage. This
practice, frequently incorporated into a massive transfusion protocol, is described as
"1:1:1 transfusion" because the goal is to transfuse one unit of plasma and one of platelets
along with each unit of red blood cells (RBCs) such that the ratio stays balanced
throughout the resuscitation phase. (Adjustment of these ratios is necessary for centers
that use pooled quantities of plasma and/or platelets.) The rationale is that patients are
already extensively coagulopathic by the time they manifest thrombocytopenia or
prolongation of the INR or aPTT. Thus, during the first several hours during and after
massive hemorrhage, plasma and platelets are necessary independent of the laboratory
tests and help address a significant contributing factor for persistent hemorrhage. The
concept of liberal, early use of plasma and platelets developed in large part from the
recent US-led military campaigns in Iraq and Afghanistan. Initially in those conflicts, the
lack of a reliable supply of blood products near the scene of injury—and platelets
especially—led to the use of fresh whole blood transfusion. Although fresh whole blood
would be impractical in the civilian setting because of logistical issues and the risk of
transmitting transfusion-related infections, the perception of improved outcomes
associated with its use prompted military surgeons to advocate 1:1:1 transfusion.
Proponents of this approach argue that it restores hemostatic capacity and helps limit
hemorrhage, leading to improved survival. Indeed, observational studies suggest that
mortality is reduced.3 However, the ratios examined in these studies typically have been
calculated retrospectively over relatively long (e.g., 12- or 24-hour) time periods after
hemorrhage. Critics emphasize that patients who do not survive long enough to receive
early plasma and platelets bias such observational comparisons (i.e., they are classified as
receiving a large ratio of RBCs to plasma or platelets because they died early) and that
plasma and platelets carry theoretical harms of transfusion-related acute lung injury and
bacterial infection, respectively. Despite some uncertainty about the actual merits of the
1:1:1 approach, given that military conflicts have frequently spurred advances in
transfusion medicine, it appears likely that the approach will be adopted more widely if
the availability of blood products does not limit its application. In the absence of
sufficient availability of blood products to allow a 1:1:1 approach, aggressive correction
of abnormalities of the INR, aPTT, and platelet count are warranted for patients with
massive hemorrhage. read more
Feedback on your answer
Collapse

A 40-year old male presents with severe upper abdominal pain that has worsened over the
last 2 weeks. It is described as an intermittent, gnawing pain that is made worse shortly
after eating and drinking. He has a past medical history chronic lower back pain
following an motor vehicle accident about five years ago and takes hydrocodone and
ibuprofen daily. What is the most likely diagnosis?

(Select 1)( 1pts extra credit)

Duodenal Ulcer

Gastric Ulcer

Pancreatitis

Cholecystitis

None of the above


The answer is B, gastric ulcer. The use of NSAIDs (40 fold increased risk for gastric
ulcer) and the worsening of pain after ingestion of food are both classic of gastric ulcers
in this case.
NSAID use confers around a 40 fold increase in the development of gastric ulcers and an
8 fold increase risk for the development of duodenol ulcers and . Therefore in this patient
you would expect gastric ulcer more likely than a duodenal ulcer. From Uptodate:
Peptic Ulcer Bleeding and NSAIDs:
The use of NSAIDs is the most commonly identified risk factor for peptic ulcer bleeding,
especially in the elderly. Studies have found relative risks for bleeding ranging from 2.7
to 33.9 [11]. Studies have also shown that the risk is drug-specific and dose-dependent.
As an example, in a study of 2777 patients, the overall relative risk (RR) of bleeding
associated with NSAID use was 5.3 (95% CI 4.5-6.2). However, the risk varied by drug
and was lowest for aceclofenac (RR 3.1, 95% CI 2.3-4.2) and was highest
for ketorolac (RR 14.4, 95% CI 5.2-39.9) [18]. The risk was higher in patients taking
high-dose NSAIDs compared with those taking medium- or low-dose NSAIDs (RR 6.8,
95% CI 5.3-8.8 versus 4.0, 95% CI 3.2-5.0). There was also an increased risk of bleeding
with aspirin use (RR 5.3) that again was dose-dependent (RR 7.5 with 500 mg per day
versus 2.7 with 100 mg per day). The concurrent use of aspirin and NSAIDs conferred an
even greater risk of bleeding than was seen with either agent alone (RR 12.7). Finally, the
risk was highest in the first 30 days of NSAID use, with a RR of 7.6 (95% CI 6.0-9.5).
The risk remained high between days 31 and 90 days (RR 7.3, 95% CI 4.0-13.2), but
dropped after 91 days (RR 2.6, 95% CI 1.6-4.1).
NSAID use has also been identified as a risk factor for ulcer perforation [16,19]. In a
study of 176 patients from Spain, NSAID use was the only risk factor that was
significantly associated with perforation (odds ratio [OR] 3.6). Read more
Feedback on your answer
Collapse
2)
0/1
A 42-year-old woman comes to you with an 8-week history of intermittent nausea and abdominal pain that
occurs 20 to 30 minutes after eating. The pain extends from the epigastrium to the right upper quadrant and
is sometimes felt in the right scapula; it usually lasts about 30 minutes to and hour and is not relieved by
antacids. The last episode occurred after she ate a hamburger and french fries. She has not had vomiting.
She is currently asymptomatic. Examination shows no other abnormalities. Which of the following imaging
studeies is the most appropriate next step ?
( 1pts extra credit)

Abdominal ultrasonography of the right upper quadrant

Upper gastrointestinal series with small bowel follow-through

HIDA scan
CT scan of the abdomen and pelvis with IV and PO contrast

This patient has evidence of intermittent bouts of gall bladder colic and the best initial study for
evaluation is always a ultrasound of the abdomen to look for gallbladder wall thickening, size of
the common bile duct and fluid around the gall bladder.

Imaging

Ultrasonography

Classically, ultrasonography has been the diagnostic test of choice for the diagnosis of gallstones.
Ultrasonography provides diagnostic accuracy (sensitivity and specificity > 95%) while being
noninvasive, readily available, low cost, and radiation free.9,10 Ultrasonography can detect the
number and size of stones, the presence of sludge or polyps, and the thickness of the gallbladder
wall. Ultrasonography can also determine if the ductal system is dilated (associated with
obstruction of the biliary tree). Although ultrasonography has many advantages, factors such as
operator dependence and challenges imposed by patient body habitus represent potential
limitations. It is important for the clinician to be familiar with the skill of the radiologist
interpreting ultrasound examinations that form the basis for clinical decision making.

Magnetic Resonance Cholangiopancreatography

Magnetic resonance cholangiopancreatography (MRCP) and endoscopic ultrasonography (EUS)


are highly accurate for the diagnosis of gallstones, with sensitivities of 100% and 98%,
respectively. Although these modalities are not commonly required for the diagnosis of
gallstones, they may be helpful in certain situations. For example, in the presence of high
suspicion but negative ultrasound findings, magnetic resonance imaging (MRI) and EUS may help
detect very small stones missed by ultrasonography. In the setting of suspected
choledocholithiasis, these modalities appear to be superior to ultrasonography because they
provide more detailed information about the extrahepatic biliary tree. 1

2014. Scientific American Surgery. Hamilton, Ontario & Philadelphia, PA. Decker Intellectual
Properties Inc. ISSN 2368-2744. STAT!Ref Online Electronic Medical Library.
http://online.statref.com/Document.aspx?fxId=61&docId=930. 10/17/2014 9:39:41 PM CDT
(UTC -05:00).
Feedback on your answer
Collapse
3)
0/1
What best describes the risk rate of receiving blood with HCV or HIV?

( 1pts extra credit)

< 1:10,000

< 1:100,000

< 1:500,000
< 1:1,000,000

< 1:10,000,000

< 1:100,000,000

The answer is < 1:1,000,000. Physicians should be aware that the FDA mandates that all
blood components are tested for hepatitis B virus (HBV), hepatitis C virus (HCV), HIV
types 1 and 2, human T-lymphotropic virus Types 1 and 2, syphilis, Trypanosoma cruzi,
and West Nile virus. Blood is not currently tested for other transfusion-transmissible
agents (such as babesiosis, malaria, and variant Creutzfeldt-Jakob [prion] disease),
because of extremely low rates of infection and, in some cases, donor screening and
exclusion based on travel to endemic areas.105 In the case of transfusion-transmissible
viruses (eg, HBV, HCV, HIV, etc.), testing is done via sensitive amplification of viral
genetic material (nucleic acid testing), which helps to eliminate the “window” period in
which donors may be infected but not yet antibody positive.106 With the addition of
nucleic acid testing to donor screening and serologic testing, the risk of receiving HCV or
HIV positive blood in the United States is now significantly < 1:1,000,000 units
transfused. Read more
Transmission of human immunodeficiency virus (HIV) through transfusion of
contaminated blood components was documented in the United States in 1982 (1). Since
then, the risk for transfusion-transmitted HIV infection has been almost eliminated by the
use of questionnaires to exclude donors at higher risk for HIV infection and the use of
highly sensitive laboratory screening tests to identify infected blood donations. The risk
for acquiring HIV infection through blood transfusion today is estimated conservatively
to be one in 1.5 million, based on 2007--2008 data (2). This report describes the first U.S.
case of transfusion-transmitted HIV infection reported to CDC since 2002 (3). A blood
center in Missouri discovered that blood components from a donation in November 2008
tested positive for HIV infection. A lookback investigation determined that this donor
had last donated in June 2008, at which time he incorrectly reported no HIV risk factors
and his donation tested negative for the presence of HIV. One of the two recipients of
blood components from this donation, a patient undergoing kidney transplantation, was
found to be HIV infected, and an investigation determined that the patient's infection was
acquired from the donor's blood products. Even though such transmissions are rare,
health-care providers should consider the possibility of transfusion-transmitted HIV in
HIV-infected transfusion recipients with no other risk factors. Read more
Feedback on your answer
Collapse
4)
1/1
Regarding esophageal adenocarcinoma and esophageal squamous-cell carcinoma, which
of the following are true?

( 1pts extra credit)


Esophageal adenocarcinoma has become the predominant type of
esophageal cancer in North America and Europe

Adenocarcinoma is three to four times as common in men as it is in


women

The sex distribution is more equal for squamous-cell carcinoma

Approximately three quarters of all adenocarcinoma lesions are found in


the distal esophagus

All of the above

None of the above

The answer is all of the above. Esophageal adenocarcinoma has become the predominant
type of esophageal cancer in North America and Europe, while esophageal squamous-cell
carcinoma remains the predominant esophageal cancer in Asia, Africa, and South
America and among African Americans in North America. Adenocarcinoma is three to
four times as common in men as it is in women, whereas the sex distribution is more
equal for squamous-cell carcinoma. The endoscopic appearance is also similar, although
approximately three quarters of all adenocarcinoma lesions are found in the distal
esophagus, whereas squamous-cell carcinoma is more frequent in the proximal to middle
esophagus. The overall 5-year survival rate for patients with esophageal adenocarcinoma
in the United States is approximately 17%, which is slightly higher than the rate for
patients with squamous-cell carcinoma.
N Engl J Med 2014; 371:2499-2509 December 25, 2014DOI:
10.1056/NEJMra1314530

Feedback on your answer


Collapse
5)
0/1
A 45-year old male while male smoker presents with worsening epigastric pain over the
last day and radiates to his back. He says that this type of pain has been present for a few
months but comes and goes, is worse at night, and also about an hour after he eats.
However yesterday's episode did not subside and has been more persistent and intense.
His vitals show a heart rate of 128 beats/min, blood pressure 178/94, respiratory rate
of 20/min, oral temperature of 100.1 F.
On physical exam he appears in moderate distress and actively vomiting. His abdomen is
diffusely tender to palpation with involuntary guarding along with hypoactive bowel
sounds in all 4 quadrants. His stool is black and guaiac positive for blood. What is the
next best step in his management?

(Select 1)( 1pts extra credit)

CT scan of his abdomen and pelvis

Immiediate surgical exploration

GI Consult for emergent endoscopy

Upright chest xray

The answer is D. Upright chest x-ray. This patient most likely has a perforated duodenal
ulcer and has free air in his peritoneum. This pain is sometimes described as gnawing,
initially better with food and radiates to the back and worse at night. He has since
perforated and the best test initially is to look for free air under his diaphragm on xray.
He will eventually need surgery and you might have to order CT scan if the xrays are
normal but this is after plain films are obtained. It is also good to know that these ulcers
can cause erosion of the gastroduodenal artery and cause a patient to bleed becoming
hypotensive and very ill quickly. Perforations are most likely in elderly patients on
chronic NSAID therapy. The perforation site usually involves the anterior wall of the
duodenum (60%), antrum (20%), or lesser-curvature gastric ulcers (20%).
Perforation of peptic ulcer remains largely a clinical diagnosis. Initial symptoms include
severe abdominal pain, worse in the epigastrium, often accompanied by nausea and
vomiting. Patients typically present in distress with an acute abdomen. A chest x-ray that
demonstrates free intraperitoneal air [see Figure 3] is all that is needed before the
patient is taken to the operating room; the decision to proceed to operation should not be
delayed by waiting for further images (e.g., computed tomographic [CT] scans). Surgical
delay is a critical determinant of survival in perforated peptic ulcer; every hour of delay
from presentation to surgery is associated with an adjusted 2.4% decreased probability of
survival compared with the previous hour.23

When there is no free air on the plain film and diagnostic uncertainty persists, an UGI
series using water-soluble contrast material may be helpful. Perforation is a
contraindication for endoscopy because air insufflation may disrupt a sealed perforation
and increase spillage of gastrointestinal contents.

In well-selected cases, when there is evidence of contained perforation without free air in
a hemodynamically normal patient without peritonitis, nonoperative management with
nasogastric drainage, intravenous PPIs, antibiotics, and serial abdominal examinations
can be attempted. However, the decision to manage patients without surgery should
undergo frequent reevaluations as failure of nonsurgical management is an indication for
surgery.
PUD has an overall point prevalence in the US population of 1.8%, with a lifetime
incidence of 10%.1 The presence ofHelicobacter pylori increases the incidence of
ulcer to about 1% per year, a rate that is six- to 10-fold higher than for noninfected
subjects.
Most patients with PUD present with mild pain, burning discomfort, tenderness in the
epigastrium, or nausea. Classically, the pain from a duodenal ulcer occurs 2 to 3 hours
after a meal, is relieved temporarily with food or antacids, and at times awakens patients
at night between about 11 pm and 2 am, when the circadian stimulation of acid secretion
is maximal. Gastric ulcer pain is often aggravated by meals. However, the classic
symptoms described are neither sensitive nor specific and are present in only a small
number of patients. Furthermore, the nature of presenting symptoms alone does not
differentiate between benign ulcerations and gastric malignancy.

Advances in the medical management of PUD, including the use of effective acid-
suppressing medications (e.g., histamine receptor antagonists and proton pump inhibitors
[PPIs]) and the treatment of H. pylori, have led to a decrease in the incidence of PUD,
the rates of hospitalization, and mortality. Despite these trends, PUD remains a
significant cause of morbidity and health care costs, with estimates of annual
expenditures (excluding medication costs) of $5.65 billion in the United States.
The majority of gastric and duodenal ulcers are caused by three factors: H.
pylori infection, nonsteroidal antiinflammatory drug (NSAID) use, and acid
hypersecretory states (e.g., Zollinger-Ellison syndrome [ZES]). Most patients with
gastroduodenal ulcers will be colonized with H. pylori, and recurrence of ulcers is
common without treating the H. pylori infection. H. pylori, a gram-negative spiral
organism, is present in about half the world population, particularly in developing
countries, where up to 20% of healthy volunteers will demonstrate infection. Infection of
gastric epithelium is followed by gastric inflammation, and the relationship between H.
pylori and ulcer formation has been widely demonstrated.5 The presence of ulcers in just
a small fraction of individuals with infection suggests the action of other etiologic factors
causing ulceration.

NSAIDs cause injury via suppression of prostaglandin synthesis, which leads to impaired
mucosal defense. NSAID use can result in a range of lesions, from superficial erosions to
deeper ulcerations. Mucosal injury caused by NSAIDs is more common in the stomach
than the duodenum. Cigarette smoking may impair ulcer healing and may increase the
risk of H. pylori-related ulceration.
2014. Scientific American Surgery. Hamilton, Ontario & Philadelphia, PA. Decker
Intellectual Properties Inc. ISSN 2368-2744. STAT!Ref Online Electronic Medical
Library. http://online.statref.com/Document.aspx?fxId=61&docId=900. 10/20/2014
7:58:06 PM CDT (UTC -05:00).

Feedback on your answer


Collapse
Unit 2
6)
0/1
You are seeing a 28-year old white male who was brought in by EMS after he was
assaulted while at a night club. On exam his GCS is 7, he is breathing spontaneously and
localizes pain. He has a large scalp laceration to his right parietal region. He is rushed to
the CT scanner and the image is shown below. What is the diagnosis?

(Select 1)(1pts)

Subdural Hematoma

Epidural Hematoma

Intraparenchymal Hematoma with extension

Subarachnoid Hemorrhage

The answer is B. Epidural hematoma (EDH) is a traumatic accumulation of blood


between the inner table of the skull and the stripped-off dural membrane. The inciting
event often is a focused blow to the head, such as that produced by a hammer or baseball
bat. In 85-95% of patients, this type of trauma results in an overlying fracture of the skull.
Blood vessels in close proximity to the fracture are the sources of the hemorrhage in the
formation of an epidural hematoma. Because the underlying brain has usually been
minimally injured, prognosis is excellent if treated aggressively. Outcome from surgical
decompression and repair is related directly to patient's preoperative neurologic
condition. more from Read more
Feedback on your answer
Collapse
7)
0/1
Which of the following is not an example of referred pain?
(Select 1)(1pts)

Ureteral stone causes ipsilateral testicular pain

Ruptured hemorrhagic cyst causing lower back pain

Acute MI causing jaw pain

Acute Appendicitis causing periumbilical pain

All of the above

None of the above

The answer is D, as Acute Appendicitis causing periumbilical pain is an example of


visceral pain.
Visceral Pain
•Caused by the stretching of fibers innervating the walls or capsules of hollow or solid
organs
•A steady ache or vague discomfort to excruciating or colicky
•Visceral afferents follow a segmental distribution and pain is localized by the sensory
cortex to an approximate spinal cord level determined by embryological organ of the
organ involved.
•Biliary, duodenum, stomach produce epigastric pains
•Most small bowel, appendix, cecum cause periumbilical pain
•Hindgut structures, such as the bladder, distal two-thirds of the colon, and pelvic
genitourinary (GU) organs, usually cause pain in the suprapubic region.
•Pain is often reported in the back for retroperitoneal structures, such as the aorta and
kidneys
•Deep musculoskeletal structures (especially of the back) are innervated by visceral
sensory fibers with similar qualities to those arising from intra-abdominal organs.
•Intraperitoneal organs are bilaterally innervated causing pain to be located in the midline
regardless of which side it is coming from. Classic example is early appendicitis causing
periumbilical pain
Visceral Pain
•Caused by the stretching of fibers innervating the walls or capsules of hollow or solid
organs
•A steady ache or vague discomfort to excruciating or colicky
•Visceral afferents follow a segmental distribution and pain is localized by the sensory
cortex to an approximate spinal cord level determined by embryological organ of the
organ involved.
•Biliary, duodenum, stomach produce epigastric pains
•Most small bowel, appendix, cecum cause periumbilical pain
•Hindgut structures, such as the bladder, distal two-thirds of the colon, and pelvic
genitourinary (GU) organs, usually cause pain in the suprapubic region.
•Pain is often reported in the back for retroperitoneal structures, such as the aorta and
kidneys
•Deep musculoskeletal structures (especially of the back) are innervated by visceral
sensory fibers with similar qualities to those arising from intra-abdominal organs.
•Intraperitoneal organs are bilaterally innervated causing pain to be located in the midline
regardless of which side it is coming from. Classic example is early appendicitis causing
periumbilical pain
Feedback on your answer
Collapse
8)
0/1
What is the most common surgical cause of abdominal pain in adults?
(Select 1)(1pts)

Appendicitis

Cholecystitis

Small bowel obstruction

Perforated peptic ulcer

All of the above

None of the above

The answer is A, appendicitis. The value of detailed epidemiologic knowledge


notwithstanding, it is worthwhile to keep in mind the truism that common things are
common. Regarding which things are common, the most extensive information currently
available comes from the ongoing survey begun in 1977 by the Research Committee of
the OMGE. As of the last progress report on this survey, which was published in 1988,
more than 200 physicians at 26 centers in 17 countries had accumulated data on 10,320
patients with acute abdominal pain [see Table 3].23 The most common diagnosis in
these patients was nonspecific abdominal pain (NSAP)—that is, the retrospective
diagnosis of exclusion in which no cause for the pain can be identified.24,25 NSAP
accounted for 34% of all patients seen; the four most common diagnoses accounted for
more than 75%. The most common surgical diagnosis in the OMGE survey was acute
appendicitis, followed by acute cholecystitis, small bowel obstruction, and gynecologic
disorders. Relatively few patients had perforated peptic ulcer, a finding that confirms the
current downward trend in the incidence of this condition. Cancer was found to be a
significant cause of acute abdominal pain. There was little variation in the geographic
distribution of surgical causes of acute abdominal pain (i.e., conditions necessitating
operation) among developed countries. In patients who required surgery, the most
common causes were acute appendicitis (42.6%), acute cholecystitis (14.7%), small
bowel obstruction (6.2%), perforated peptic ulcer (3.7%), and acute pancreatitis
(4.5%).23 The OMGE survey's finding that NSAP was the most common diagnosis in
patients with acute abdominal pain has been confirmed by several srudies12,13,25; the
finding that acute appendicitis, cholecystitis, and intestinal obstruction were the three
most common diagnoses in patients with acute abdominal pain who require operation has
also been amply confirmed [see Table 3].1,12,13

Table 3. Frequency of Specific Diagnoses in Patients with Acute Abdominal Pain

Frequency in Individual Studies (% of Patients)

Wilso Brew
n et er et
OMGE23( Irvin13( de
Diagno al82(N al12(N Hawthorn83
N= N= Dombal1(
sis = = (N = 496)
10,320) 1,190) N = 552)
1,196 1,000
) )

Nonspecifi
c
34.0 45.6 34.9 41.3 50.5 36.0
abdominal
pain

Acute
appendicit 28.1 15.6 16.8 4.3 26.3 14.9
is

Acute
cholecysti 9.7 5.8 5.1 2.5 7.6 5.9
tis

Small 4.1 2.6 14.8 2.5 3.6 8.6


bowel
obstructio
n

Acute
gynecolog 4.0 4.0 1.1 8.5 — —
ic disease

Acute
pancreatiti 2.9 1.3 2.4 — 2.9 2.1
s

Urologic
2.9 4.7 5.9 11.4 — 12.8
disorders

Perforated
pepticulce 2.5 2.3 2.5 2.0 3.1 —
r

Cancer 1.5 — 3.0 — — —

Diverticula
1.5 1.1 3.9 — 2.0 3.0
r disease

Dyspepsia 1.4 7.6 1.4 1.4 — —

Gastroenter
— — 0.3 6.9 — 5.1
itis

Inflammato
ry bowel — — 0.8 — — 2.1
disease

Mesenteric
— 3.6 — — — 1.5
adenitis

Gastritis — 2.1 — 1.4 — —

Constipatio
— 2.4 — 2.3 — —
n

Amebic
hepatic 1.2 — 1.9 — — —
abscess

Miscellane
6.3 1.3 5.2 15.5 4.0 8.0
ous

OMGE = World Organization of Gastroenterology.


2014. Scientific American Surgery. Hamilton, Ontario & Philadelphia, PA. Decker
Intellectual Properties Inc. ISSN 2368-2744. STAT!Ref Online Electronic Medical
Library. http://online.statref.com/Document.aspx?fxId=61&docId=803. 10/16/2014
1:15:18 PM CDT (UTC -05:00).

Feedback on your answer


Collapse
Unit 3
9)
1/1
You are seeing a 44-year-old female in your outpatient GI clinic for persistent acid reflux
symptoms. She was on zantac for 4 weeks following her last EGD and then switched to
protonix daily for 2 months which did not seem to help. Her records indicate a normal
upper endoscopy 4 months ago as she was complaining of some mild dysphagia and
biopsies were negative for h. pylori. She otherwise has no other past medical history and
takes no other medications, denies alcohol ingestion and is not a smoker. Vital signs and
physical examination are normal. Which of the following studies should be done next to
confirm this patient's most likely diagnosis?
(Select 1)(1pts)

Repeat H. pylori testing

Barium swallow radiography

Esophageal manometry

Ambulatory esophageal pH-impedance monitoring

The answer is Ambulatory esophageal pH-impedance monitoring, This patient most


likely has gastroesophageal reflux disease (GERD), and the gold standard for diagnosing
GERD is 24-hour esophageal pH-impedance monitoring, which establishes whether
increased esophageal exposure to acid is present. This study is most helpful in patients
who have GERD-like symptoms that fail to respond to an adequate trial of a proton pump
inhibitor (PPI) to correlate the presence of acid in the esophagus at the time the patient is
having symptoms, such as the patient described. Esophageal pH monitoring identifies the
reflux of acid, and impedance monitoring detects reflux of other gastric contents in a
small percentage of patients who have symptoms related to non-acid reflux.
GERD is definitively diagnosed by symptoms, upper endoscopy, or ambulatory
esophageal pH/impedance monitoring. However, the presence of heartburn, regurgitation,
or both is sufficient to presumptively diagnose GERD if symptoms are of sufficient
severity and frequency to be troublesome for the patient. A favorable response to empiric
therapy with PPIs is supportive evidence for a diagnosis of GERD and is a reasonable
first step in a patient without alarm symptoms (dysphagia, anemia, vomiting, or weight
loss). Kumar AR, Katz PO. Functional esophageal disorders: a review of diagnosis and
management. Expert Rev Gastroenterol Hepatol. 2013;7:453-61. PMID: 23899284

Feedback on your answer


Collapse
10)
1/1
A 22-year old male presents with severe epigastric pain that radiates into his chest after
vomiting over a dozen times following a night of heavy drinking. Vital signs show an
oraltemperature of 100.7 F, blood pressure 147/97, heart rate 127beats/min, respirations
24/min. Physical exam shows crepitus in the suprasternal notch, clear lung exam
bilaterally, and severe epigastric tenderness on abdominal exam. What is most likely
causing this patients symptoms?
(Select 1)(1pts)

Mallory Weiss tear

Spontaneous Pneumothorax

Perforated Duodenal Ulcer

Esophageal Perforation

The answer is D. This patient has evidence subcutaneous air (crepitus) and has likely
ruptured his esophagus. Effort rupture of the esophagus or Boerhaave's syndrome is a
spontaneous perforation of the esophagus that most commonly results from a sudden
increase in intraesophageal pressure combined with negative intrathoracic pressure
caused by straining or vomiting. It was first described by Dr. Herman Boerhaave, a
physician from Leiden, the Netherlands.

In patients suspected to have an esophageal perforation based upon clinical presentation


and chest radiography, computed tomography (CT) scan or esophagram should be
obtained promptly.
Computed tomography — Computed tomography (CT) scan shows esophageal wall
edema and thickening, extraesophageal air, periesophageal fluid with or without gas
bubbles, mediastinal widening, and air and fluid in the pleural spaces, retroperitoneum or
lesser sac (image 2).
Esophagram — The diagnosis of esophageal perforation can also be confirmed by water-
soluble contrast esophagram (Gastrografin), which reveals the location and extent of
extravasation of contrast material. However, false negative results occur in as many as 10
percent of cases. The sensitivity of contrast studies depends upon the size and location of
the perforation, and the technique used for the study.
Although barium is superior in demonstrating small perforations, it causes an
inflammatory response in mediastinal or pleural cavities and is therefore not used as the
primary diagnostic study [13]. If, however, the water-soluble study is negative, a barium
study should be performed for better definition (image 3).
Upper endoscopy — The role of upper endoscopy in the diagnosis of spontaneous
perforation remains controversial. Both the endoscope and insufflation of air can extend
the perforation and introduce air into the mediastinum [14]. However, in the appropriate
setting, often in the operating room with a patient prepared for surgical intervention,
endoscopy performed by a skilled endoscopist can be a useful tool for diagnosis and
treatment planning.
TREATMENT — Boerhaave's syndrome is rare and there is little evidence to guide
treatment. As a general rule, treatment depends upon the size and location of the
perforation, whether it is a contained perforation in the mediastinum or between the
mediastinum and visceral lung pleura, how rapidly it is diagnosed, whether the esophagus
has underlying disease, and the patient’s underlying comorbidities (algorithm 1).
Surgery is generally required for thoracic perforations [15], while cervical perforations
can often be managed without surgery.More from UptoDate
Feedback on your answer
Collapse
11)
0/1
You are consulted to see a 61-year-old male because of continuing symptoms due to
refractory gastroesophageal reflux that have not improved despite lifestyle
modifications and treatment with a twice daily proton pump inhibitor (omeprazole 40 mg
daily). He notes that he continues to have occasional chest pain and epigastric burning
with eating. He denies dysphagia, regurgitation, weight loss, or a change in bowel habits.
He denies other past medical history and has no cardiac risk factors. Vital signs and
physical examination are normal. His upper endoscopy shows no ulcers and is otherwise
unremarkable. Which of the following is the most appropriate treatment at this time?
(Select 1)(1pts)

Schedule consultation for evaluation for antireflux surgery

Increase the proton pump inhibitor to 3 times daily

Change to a different proton pump inhibitor

Add ranitidine at bedtime to the current regimen


The answer is A. Consider surgery as an option for patients with well-documented GERD
who require long-term PPI maintenance therapy but show satisfactory relief of symptoms
and who:
 Are older than 50 years
 Consider long-term medication a financial burden
 Are noncompliant with drug therapy
 Prefer a single surgical intervention to long-term drug treatment
 Experience prominent symptoms of regurgitation, even with medical
control of heartburn symptoms
Nissen fundoplication is the most common surgical intervention for GERD. This
procedure aims to restore the physiology and anatomy of the gastroesophageal junction
by wrapping the gastric fundus around the distal esophagus. The FDA has also approved
several endoscopic procedures for treatment of GERD, including endoscopic suturing and
radiofrequency ablation of the lower esophageal sphincter. Read More

Feedback on your answer


Collapse
Unit 4
12)
1/1
Which of the following statements about H. pylori is not true?
(Select 1)(1pts)

The serologic determination of anti-H. pylori IgG antibodies in peripheral


blood has high sensitivity (90 to 100%)

Urea breath testing has a sensitivity of 88 to 95% and a specificity of 95


to 100% for establishing the presence of H. pylori infection1

Anti-H. pylori IgG antibodies is the test of choice for confirming successful
eradication of the organism 4 to 6 weeks after the cessation of antibiotic
treatment

Histologic examination of tissue samples remains the gold standard for


determining the presence of H. pylori

Routine culture of the gastric biopsy specimens for H. pylori is not


recommended

Answer C is the only false statement. Urea breath testing is the test of choice for
confirming successful eradication of the organism 4 to 6 weeks after the cessation of
antibiotic treatment. Routine laboratory tests are of limited value in evaluating patients
with uncomplicated PUD. When ZES is suspected, a fasting serum gastrin level may be
useful. However, laboratory tests do play an important role in the diagnosis of H. pylori
infection. The serologic determination of anti-H. pylori IgG antibodies in peripheral
blood has high sensitivity (90 to 100%) but variable specificity (76 to 96%). Serology is
preferred for initial diagnosis when endoscopy is not required. Urea breath testing has a
sensitivity of 88 to 95% and a specificity of 95 to 100% for establishing the presence of
H. pylori infection12 and is the test of choice for confirming successful eradication of the
organism 4 to 6 weeks after the cessation of antibiotic treatment. Although endoscopy is
not indicated solely for the purpose of establishing the presence of H. pylori, gastric
biopsy specimens can be used in the diagnosis of H. pylori by rapid urease test, histology,
and bacterial culture [see Figure 2]. Rapid urease testing of the biopsy specimen is the
preferred method when histopathology is not required because it is fast (results within
hours), is less expensive, and has a sensitivity of 90 to 95% and a specificity of 95 to
100%. Histologic examination of tissue samples remains the gold standard for
determining the presence of H. pylori and can also provide additional information
regarding the presence of gastritis, intestinal metaplasia, or mucosa-associated lymphoid
tissue. Figure 1. Upper gastrointestinal series in which double contrast (barium and air) is
used, showing rounded collection of barium in an ulcer in the duodenal bulb of a patient
presenting with dyspepsia (uncomplicated duodenal ulcer). Routine culture of the gastric
biopsy specimens for H. pylori is not recommended as it is relatively expensive, and
diagnosis requires more time (3 to 5 days). However, the incidence of antimicrobial
resistance is high in patients with refractory peptic ulcers, and they may benefit from
culture and antibiotic sensitivity testing. Read more
Feedback on your answer
Collapse
13)
0/1
A 48-year old black male presents to you with sudden onset of vomiting and diffuse
abdominal pain that hurts to breathe two hours prior to arrival. He denies past medical
history and his only medications are ibuprofen for chronic right shoulder pain. He smokes
1 pack per day and occasionally drinks alcohol.
His vitals show an oral temperature of 99.3 F, heart rate of 129 beats/min, respiratory rate
20/min, blood pressure 151/100, O2 saturation of 98% on room air.
Physical exam shows him to be uncomfotable. Cardiac exam shows tachycardia, his
lungs are clear to auscultation, and abdominal exam shows diffuse tenderness to light
palpation greatest in the right upper quadrant along with guarding. Rectal exam shows
heme positive brown stool. An upright chest x-ray which is shown below. What is the
diagnosis?
(Select 1)(1pts)

Pneumomediastinum

Pneumoperitoneum

Intestinal Malrotation

Small Bowel Obstruction

Pneumothorax

The answer is B, Pneumoperitoneum which describes gas within the peritoneal cavity,
and is often the harbinger of a critical illness. The most common cause of a
pneumoperitoneum is from the disruption of the wall of a hollow viscus. Read more
An erect chest x-ray is probably the most sensitive plain radiograph for the detection of
free intraperitoneal gas. If a large volume pneumoperitoneum is present, it may be
superimposed over normal aerated lung with normal lung markings.
 subdiaphragmatic free gas
 cupola sign (in supine film)
Free gas within the peritoneal cavity can be detected on an abdominal radiograph. These
signs can be further divided by anatomical compartments in relation to the
pneumoperitoneum:
 bowel related signs
 double wall sign (also known as Rigler's sign or bas-relief
sign)
 telltale triangle sign (also known as triangle sign)
 peritoneal ligament related signs
 football sign
 falciform ligament sign
 lateral umbilical ligament sign (also known as inverted
"V" sign)
 urachus sign
 right upper quadrant signs
 lucent liver sign
 hepatic edge sign
 fissure for ligamentum teres sign
 Morison's pouch sign
 cupola sign
The patient above likely has a perforated ulcer from underlying NSAID use. Ulcer
perforation should be suspected in patients who suddenly develop severe, diffuse
abdominal pain. Perforations complicate 2 to 10 percent of peptic ulcers [70]. Duodenal,
antral, and gastric body ulcers account for 60, 20, and 20 percent of perforations due to
peptic ulcer disease (PUD), respectively. If imaging is required, plain x-rays are typically
obtained first. Careful interpretation of upright chest and abdominal films can detect
diagnostic free air in many cases of perforated gastric and duodenal ulcers [76]. The
presence of free air on abdominal imaging is highly indicative of a perforated viscus
(image 1 and image 2 and image 3), although about 10 to 20 percent of patients
with a perforated duodenal ulcer will not have free air [76]. If free air is found, no other
diagnostic studies are necessary. Leakage of water soluble oral contrast may be useful in
selected cases. Once the oral contrast is given, the patient should be rotated 360 degrees
and placed on the right side to fill the antrum and duodenum with contrast. However,
many perforations have already sealed spontaneously by the time of presentation [77], so
the absence of a leak does not exclude the diagnosis of a perforated ulcer. Read More
Feedback on your answer
Collapse
14)
0/1
You are seeing a 45-year old male in clinic who has a past medical history of
osteoarthritis for which he takes daily alleve. He denies tobacco use or alcohol use. He
has been having recent early satiety, bloating and epigastric discomfort over the last
month. You suspect peptic ulcer disease. Which of the following ulcer locations is more
associated with H. Pylori infection?
(Select 1)(1pts)
Duodenal

Gastric

Esophageal

Jejunal

None of the above

You didn't make any selections


The answer is A, duodenal ulcers. H. pylori, a gram-negative, helical, rod-
shaped bacterium, colonizes the gastric mucosa of approximately one-half of the world
population and an estimated 30% to 40% of the U.S. population. H. pylori is present in
95% of patients with duodenal ulcers and in 70% of those with gastric ulcers. It is
typically transmitted via the fecal-oral route during early childhood and persists for
decades. The bacterium is a known cause of gastric and duodenal ulcers and is a risk
factor for mucosa-associated lymphoid tissue (MALT) lymphoma and gastric
adenocarcinoma. Am Fam Physician. 2015 Feb 15;91(4):236-242.

Feedback on your answer


Collapse
15)
0/1
What is the most common complication of peptic ulcer disease?

(Select 1)(1pts)

Gastric outlet obstruction

Perforation

Bleeding

Cancer

The answer is B, Bleeding. Bleeding is the most common complication of peptic ulcer
disease requiring hospitalization. In a study of the National Inpatient Sample (NIS),
bleeding occurred in 73 percent, perforation in 9 percent, and obstruction in 3 percent
[4]. H. pylori, nonsteroidal antiinflammatory drugs (NSAIDs), and the use of low dose
aspirin are the most common etiologies of ulcer bleeding [7] and ulcer perforation [8-
10]. In observational studies, duodenal, antral/pyloric, and gastric body ulcers
account for 60, 20, and 20 percent of perforations, respectively [8,9]. Among these
complications, perforation had the highest mortality rate, followed by obstruction, then
hemorrhage.
Bleeding peptic ulcer — Upper gastrointestinal bleeding due to peptic ulcer disease
is a common indication for emergency management of peptic ulcer disease [11]. Most
patients with acute bleeding can be managed with fluid resuscitation and transfusion, acid
suppression therapy, and endoscopic intervention. For those who fail these efforts,
surgery may become necessary. The specific indications for surgery for patients with
bleeding peptic ulcer are reviewed separately.
Perforated peptic ulcer — Ulcer perforation may be suspected in patients with a
history consistent with peptic ulcer disease who develop the sudden onset of severe,
diffuse abdominal pain. Once a diagnosis of perforation is established, surgical
intervention is indicated.
Gastric outlet obstruction — Gastric outlet obstruction is the least frequent ulcer
complication in developed countries. Most cases are associated with duodenal or pyloric
channel ulceration; gastric ulceration accounts for only five percent of cases.
Surgical consultation should be obtained for patients with chronic partial gastric outlet
obstruction that is refractory to medical treatment, and those found to have complete
gastric outlet obstruction, or those readmitted with gastric outlet obstruction after recent
“successful treatment.” Surgery is indicated if the patient fails to respond to conservative
medical management and endoscopic therapy.
Surgical treatment of gastric outlet obstruction is almost never an emergency. Because
most patients present with some degree of malnutrition and frequently have electrolyte
imbalances (ie, hypokalemic, hypochloremic metabolic alkalosis secondary to vomiting
or nasogastric suctioning), it is important to correct any derangements and optimize the
patient’s medical status prior to proceeding with surgery. Failure to identify and correct
these issues increases perioperative morbidity.
It may also be reasonable to try and improve the patient’s overall nutritional status with
nutritional support, as these patients are frequently malnourished and can have significant
postoperative delayed gastric emptying due to their atonic stomach.
Many experienced gastric surgeons advocate preoperative nasogastric sump
decompression to decrease gastric dilation and, hopefully, gastric atony. Read more

Feedback on your answer


Collapse
16)
1/1
You are seeing a 46-year old female smoker in the ER who presents tonight with acute
onset of epigastric pain, nausea and vomiting. She past medical history except for
right hip arthritis and only takes Alleve daily.
Her vitals show a blood pressure of 145/75, heart rate of 95/min, respiratory rate
of 14/min, O2 saturation of room air of 99%, oral temperature 98.7 F
Her physical exam shows mild upper abdominal tenderness that is greatest in the
epigsatric area along with no voluntary guarding in her upper abdomen. Upright chest
xray reveals free air under the right hemidiaphragm. True or False: Based on these
findings, surgery is always indicated for definitive treatment.
(1pts)

True

False Correct

This statement is False. Ulcer perforation should be suspected in patients who suddenly
develop severe, diffuse abdominal pain. Perforations complicate 2 to 10 percent of peptic
ulcers [70]. Duodenal, antral, and gastric body ulcers account for 60, 20, and 20 percent
of perforations due to peptic ulcer disease (PUD), respectively. Also there is never an
ALWAYS in medicine and with perforated peptic ulcers, a large majority of them are
found to be sealed upon upper GI study with soluble contrast. If this is found then these
patients can be observed safely as long as peritonitis does not develop.
A major decision when treating patients with ulcer perforation is whether and when to
operate. After resuscitation, emergent operation and closure with a piece of omentum is
the standard of care for patients with an acute perforation and a rigid abdomen with free
intraperitoneal air. If the patient is stable or improving, especially if spontaneous sealing
of the perforation has been demonstrated, nonoperative management with close
monitoring is a reasonable option. With any free perforation, regardless of the presence
or size of the leak, if the patient's status is deteriorating, urgent surgery is indicated.
Prolonged efforts to establish a diagnosis or pursue non operative care despite worsening
status can be counterproductive, since a needed operation will be delayed. In addition,
surgery is indicated in circumstances where the cause of an acute abdomen has not been
established or the patient's status cannot be closely monitored.Read More from up to
date

Feedback on your answer


Collapse
Unit 5
17)
0/1
Roughly what percentage of patients with blunt liver injury initially managed non-
operatively will go on to need surgical intervention?
(1pts)
<10%

25%

50%

75%

The answer is A, During the last century, the management of blunt force trauma to the
liver has changed from observation and expectant management in the early part of the
1900s to mainly operative intervention, to the current practice of selective operative and
nonoperative management. A 2008 study by Tinkoff et al.4 showed that 86.3% of hepatic
injuries are now managed without operative intervention. The current reported success
rate of nonoperative management of hepatic trauma ranges from 82% to 100%. Most
blunt liver trauma (80% in adults, 97% in children) patients are currently treated
conservatively. The success of non-operative management depends upon proper selection
of the patient. The patients, who are managed non-operatively, usually have grade I and
II liver injuries, hemoperitoneum less than 900 ml and blood transfusion of less than 3
units. The contraindications to non-operative management include refractory
hypotension, signinficant fall in haematocrit, the extravasations of intravenous contrast
agent, expanding haematoma and grade IV and V liver injury on CECT abdomen. The
patients of grade III liver injuries need very close observation as they may require
surgical intervention during first 24 hours. The failure rate of non-operative management
is not more than 5% inmost studies. It seems that patients with grade VI injuries rarely
reaches to the hospital alive and are not salvageable. Therefore, such injuries are usually
documented on autopsy. Mortality from blunt hepatic trauma is about 5% and is related
to uncontrolled hemorrhage.
Interventional radiology may be needed to perform an angiogram and embolization for
bleeding or to percutaneously drain an abscess or b iloma. An endoscopic retrograde
cholangiopancreatogram (ERCP) and stent placement may be required for biliary leak.
Even when such complications of the liver injury develop, only 15% require operative
intervention. Hepatic artery angiography with embolization is an important tool for the
stable patient with contrast extravasation who is being managed nonoperatively. It can
also be invaluable for the postoperative patient who has been stabilized by perihepatic
packing or who has rebled after an initial period of stability. Angioembolization has a
greater than 90% success rate in the control of bleeding with a low risk of rebleeding and
a reduction in required volume of transfusion. Read More
Feedback on your answer
Collapse
18)
0/1
What is the most common abdominal organ injured in the setting of blunt abdominal
trauma?
(Select 1)(1pts)

Spleen

Liver

Kidneys

Small Intestines

The liver is the most commonly injured solid organ in blunt trauma, comprising 5% of all
trauma admissions, and because of its size is frequently involved in penetrating trauma.
Following blunt trauma, the most commonly injured structures are the parenchyma and
hepatic veins. Blunt forces dissipate along segments of the liver and along the fibrous
coverings of the portal triad structures; the hepatic veins, however, are not so insulated.
Given its size and location within the abdomen, the liver is also commonly involved in
penetrating trauma. Stab wounds typically result in direct linear tears, whereas gunshot
wounds or shotgun wounds result in significant cavitary injuries attributable to blast
effect and the "tumbling" of the missile within the liver parenchyma. Thus, arterial injury
is more common with penetrating trauma.
Over the past 20 years, nonoperative management (NOM) of liver injuries has evolved to
become the prevailing therapeutic strategy for blunt hepatic trauma. Several concurrent
changes resulted in this paradigm change. First was the realization that diagnostic
peritoneal lavage (DPL) was sensitive but not specific for identifying intraperitoneal
hemorrhage that necessitated operative management. Surgeons recognized that many
laparotomies undertaken for a positive DPL were associated with liver injuries that did
not require intervention for bleeding.1Second, trauma surgeons noted that nonbleeding
hepatic venous injuries, if manipulated at laparotomy, often resulted in more hemorrhage
and sometimes even death.2 Furthermore, it became conspicuous that with hemostasis
achieved in the operating room, recurrent postoperative bleeding was rare. Therefore,
surgeons queried whether hepatic venous injuries, which are low-pressure system
injuries, could heal without intervention. Finally, computed tomography (CT) provided a
reliable method for diagnosing and grading liver injuries.

The spleen is the second most commonly injured abdominal organ in blunt trauma
patients. Historical studies have reported a 10% mortality with all splenic injuries;
however, isolated splenic injury mortality is less than 1%. The mechanisms of injury are
similar to those seen with liver injuries: motor vehicle collisions, autopedestrian
accidents, and falls. Similar to penetrating trauma to the liver, stab wounds to the spleen
typically result in direct linear tears, whereas gunshot wounds result in significant
cavitary injuries.

Until the 1970s, splenectomy was considered mandatory for all splenic injuries.
Recognition of the immune function of the spleen refocused efforts on splenic salvage in
the 1980s.38,39Following success in pediatric patients, NOM of splenic injuries was
adopted in the adult population and has become the prevailing strategy for blunt splenic
trauma.

Duodenal and pancreatic injury continues to challenge the trauma surgeon. The relatively
rare occurrence of these injuries, the difficulty in making a timely diagnosis, and high
morbidity and mortality rates justify the anxiety these unforgiving injuries invoke.
Mortality rates for pancreatic trauma range from 9 to 34%, with a mean rate of 19%.
Duodenal injuries are similarly lethal, with mortality rates ranging from 6 to 25%.
Complications following duodenal or pancreatic injuries are alarmingly frequent,
occurring in 30 to 60% of patients.1-3 Recognized early, the operative treatment of most
duodenal and pancreatic injuries is straightforward, with low morbidity and mortality.

2014. Scientific American Surgery. Hamilton, Ontario & Philadelphia, PA. Decker
Intellectual Properties Inc. ISSN 2368-2744. STAT!Ref Online Electronic Medical
Library. http://online.statref.com/Document.aspx?fxId=61&docId=2329. 10/27/2014
1:39:15 PM CDT (UTC -05:00).

Feedback on your answer


Collapse
19)
0/1
You are seeing a 20-year-old male in hospital 12 hours after he was admitted for
observation following an MVC. Initially he was complaining of left sided chest wall pain
after his car was t-boned on the passenger side. His initial physical examination showed
bruises on the anterior chest wall and upper abdominal wall. X-rays revealed fractures
of his sixth and seventh ribs on the left (but no pneumothorax or pleural effusion). A
FAST Exam did not show any free intraperitoneal fluid. Currently, he complains of
worsening epigastric pain, left shoulder pain and mild nausea. His current vitals show his
blood pressure is now 95/60 and pulse rate 115 beats/min, and O2 saturation is 96% on
room air. Which of the following is the next best step in treatment?
(Select 1)(1pts)

Repeat ECG and cardiac biomarkers

Repeat PA and lateral chest x-ray

Abdominal CT scan with intravenous contrast

Transesophageal echocardiogram

This patient presents with blunt abdominal trauma with the delayed onset of hypertension
and signs and symptoms worrisome for likely splenic injury. Most common injuries are
to the spleen, liver, and less common injuries are to the hollow viscous organs in the
abdomen. Symptoms and signs suggesting splenic injury include left upper quadrant pain,
abdominal wall contusion, left lower chest wall tenderness, hypotension, and left
shoulder. Pain referred from splenic hemorrhage, hitting the phrenic nerve and diaphragm
(Kehr sign). The initial examination after blunt abdominal trauma can be unremarkable
and the symptoms can occur hours later, indicating ongoing splenic injury. The best
choice here would be an abdominal CT scan with intravenous contrast, only (no oral
contrast is needed because as little utility). This will define organ injury, assess for
presence of bleeding in all abdominal compartments, determine the need for surgery.
The spleen is the second most commonly injured abdominal organ in blunt trauma
patients. Historical studies have reported a 10% mortality with all splenic injuries;
however, isolated splenic injury mortality is less than 1%. The mechanisms of injury are
similar to those seen with liver injuries: motor vehicle collisions, autopedestrian
accidents, and falls. Similar to penetrating trauma to the liver, stab wounds to the spleen
typically result in direct linear tears, whereas gunshot wounds result in significant
cavitary injuries.
Until the 1970s, splenectomy was considered mandatory for all splenic injuries.
Recognition of the immune function of the spleen refocused efforts on splenic salvage in
the 1980s.38,39Following success in pediatric patients, NOM of splenic injuries was
adopted in the adult population and has become the prevailing strategy for blunt splenic
trauma.40

INITIAL EVALUATION AND INJURY GRADING

Addressing the patient's ABC's, examining the patient's abdomen, and performing
adjunctive imaging with FAST and CT are the initial steps of diagnosing a patient's
splenic injury. Hypotension with a positive FAST scan should prompt emergent
laparotomy. For patients with an identified blunt splenic injury on a CT scan, the injury
should be graded according to the AAST injury grading scale [see Table 1].3

Similar to liver injuries, the grade of splenic injury predicts failure rates and complication
rates of NOM. Other findings that should be searched for on a CT scan include contrast
extravasation (is the contrast blush contained within the spleen, or does it spill into the
peritoneum?), the amount of intra-abdominal hemorrhage (is it isolated to the splenic
fossa, or does blood extend into the pelvis?), and the presence of pseudoaneurysms.

NONOPERATIVE MANAGEMENT

Indications for Nonoperative Management

NOM of solid-organ injuries is pursued in hemodynamically stable patients who do not


have overt peritonitis or other indications for laparotomy.41-45 There is no age cutoff for
patients for the NOM of solid-organ injuries.46,47 High-grade injuries, a large amount of
hemoperitoneum, contrast extravasation, and pseudoaneurysms are not absolute
contraindications for NOM; however, these patients are at high risk for failure.48-51 The
identification of contrast extravasation as a risk factor for failure of NOM led to liberal
use of angioembolization. The true value of angioembolization in splenic salvage has not
been rigorously evaluated. Patients with intraparenchymal splenic blushes who are
otherwise asymptomatic may be considered for a period of observation rather than
empiric angioembolization52; it is thought that the contained hemorrhage within the
splenic capsule may result in tamponade of the bleeding

It is clear, however, that 20 to 30% of patients with splenic trauma deserve early
splenectomy and that failure of NOM often represents poor patient selection.53,54 In
adults, indications for prompt laparotomy include initiation of blood transfusion within
the first 12 hours considered to be secondary to the splenic injury or hemodynamic
instability. In the pediatric population, blood transfusions up to half of the patient's blood
volume are used prior to operative intervention. Following the first 12 postinjury hours,
indications for laparotomy are not as black and white. Determination of the patient's age,
comorbidities, current physiology, degree of anemia, and associated injuries will
determine the use of transfusion alone versus intervention with either embolization or
operation. Unlike hepatic injuries, which rebleed in 24 to 48 hours, delayed hemorrhage
or rupture of the spleen can occur up to weeks following injury. Algorithms for the
management of pediatric splenic injuries exist,55 and the patient's physiologic status is
the key determinant. Rapid mobilization in patients who are hemodynamically stable with
a stable hematocrit and no abdominal pain is generally successful. Overall, nonoperative
treatment obviates laparotomy in more than 90% of cases.

Follow-up Imaging

Out of concern over the risk of delayed hemorrhage or other complications, follow-up CT
scans have often been recommended; unfortunately, there is no consensus as to when or
even whether they should be obtained. Patients with grade I or II splenic injuries rarely
show progression of the lesion or other complications on routine follow-up CT scans; it is
reasonable to omit such scans if patients' hematocrits remain stable and they are
otherwise well. Patients with more extensive injuries often have a less predictable course,
and CT may be necessary to evaluate possible complications. Routine CT before
discharge, however, is unwarranted. Outpatient CT, however, in patients who participate
in vigorous or contact sports should be performed at 6 weeks to document complete
healing before resuming those activities. A more convenient and less expensive
alternative to follow-up CT is ultrasonographic monitoring of lesions.

OPERATIVE EXPOSURE AND HEMORRHAGE CONTROL

In penetrating abdominal injuries not suitable for NOM and in blunt abdominal injuries
when NOM is contraindicated or has failed, exploratory laparotomy is performed.

To ensure safe removal or repair, the spleen should be mobilized to the point where it can
be brought to the surface of the abdominal wall without tension. An incision is made in
the peritoneum and the endoabdominal fascia, beginning at the white line of Toldt along
the descending colon and continuing cephalad 1 to 2 cm lateral to the posterior peritoneal
reflection of the spleen; this plane of dissection is continued superiorly until the
esophagus is encountered [see

Figure 15a]. Posteriorly, blunt dissection is performed to mobilize the spleen and
pancreas as a composite away from Gerota fascia and up and out of the retroperitoneum;
this posterior plane may be extended to the aorta if necessary [seeFigure 15b].
Additionally, the attachments between the spleen and the splenic flexure of the colon may
be divided to avoid avulsion of the inferior splenic capsule. Care must be taken not to
pull on the spleen; otherwise, it will tear along the posterior peritoneal reflection, causing
significant hemorrhage. It is often helpful to rotate the operating table 20° to the patient's
right so that the weight of the abdominal viscera facilitates viscera retraction. Any
ongoing hemorrhage from the splenic injury may be temporarily controlled with digital
occlusion of the splenic hilar vessels. Once mobilization is complete, the spleen can be
repaired or removed without any need to struggle to achieve adequate exposure.

DEFINITIVE MANAGEMENT OF INJURIES

Splenic injuries are treated operatively by splenectomy, partial splenectomy, or splenic


repair (splenorrhaphy), based on the extent of the injury and the physiologic condition of
the patient. Splenectomy is indicated for hilar injuries, pulverized splenic parenchyma, or
any grade II or higher injury in a coagulopathic or multiply injured patient. We employ
autotransplantation of splenic implants [see

Figure 16] for partial immunocompetence in younger patients.56 Drains are not used.
Partial splenectomy can be employed in patients in whom only the superior or inferior
pole has been injured. Hemorrhage from the raw splenic edge is controlled with a
horizontal mattress suture, with gentle compression of the parenchyma [see

Figure 17]. Similar to hepatic injuries, splenorrhaphy techniques to achieve hemostasis


include topical agents (electrocautery, argon beam coagulation, thrombin-soaked gelatin
foam sponge, fibrin glue, BioGlue), enveloping the injured spleen in absorbable mesh,
and pledgeted suture repair.

POSTOPERATIVE CARE AND COMPLICATIONS

Enthusiasm for splenic salvage was driven by the rare but often fatal complication of
overwhelming postsplenectomy sepsis (OPSS). OPSS is caused by encapsulated
bacteria,Streptococcus pneumoniae, Haemophilus influenzae,
and Neisseria meningitidis, which are resistant to antimicrobial treatment. In
patients undergoing splenectomy, prevention against these bacteria is provided via
vaccines administered optimally at 14 days but definitely prior to hospital
discharge.57 Vaccines to be administered include Pneumovax (Merck & Co., Inc.,
Whitehouse Station, NJ), Menactra (Sanofi Pasteur, Swiftwater, PA), and Fluvirin
(Novartis, East Hanover, NJ). Revaccination remains open to debate, but some argue for
revaccination every 6 years.
An immediate postsplenectomy increase in platelets and white blood cells (WBCs) is
normal; however, beyond postoperative day 5, a WBC count above 15,000/μL and a
platelet to WBC ratio less than 20 are highly associated with sepsis and should prompt a
thorough search for underlying infection.58,59 A common infectious complication
following splenectomy is a subphrenic abscess, which should be managed with
percutaneous drainage. Following splenectomy or splenorrhaphy, postoperative
hemorrhage may be attributable to loosening of a tie around the splenic vessels, a missed
short gastric artery, or recurrent bleeding from the spleen if splenic repair was used.
Additional sources of morbidity include a concurrent but unrecognized iatrogenic injury
to the pancreatic tail during rapid splenectomy, resulting in pancreatic ascites or fistula.

2014. Scientific American Surgery. Hamilton, Ontario & Philadelphia, PA. Decker
Intellectual Properties Inc. ISSN 2368-2744. STAT!Ref Online Electronic Medical
Library. http://online.statref.com/Document.aspx?fxId=61&docId=2297. 10/17/2014
4:45:39 AM CDT (UTC -05:00).

Feedback on your answer


Collapse
Unit 6
20)
1/1
You are seeing a 64-year old black female who is presenting with 2 days of worsening
upper abdominal pain that has become persistent. She denies fever at home but has a n
oral temp of 38.0 (100.4 F) in the ER. Her BP is 94/50, HR is 122 bpm and respiratory
rate 16/min. On exam she appears slightly jaundiced in her sclera, she has a positive
murphy's sign with some involuntary guarding. Her WBC comes back at 20k with 11%
bands. Her BMP is unremarkable. What triad of symptoms is classically associated with
cholangitis?
(Select 1)(1pts)

RUQ pain, fever,


bandemia

Epigastric pain,
fever,
leukocystosis

Bandemia, fever,
jaundice

RUQ pain, fever,


jaundice

Charcot's triad

http://www.ncbi.nlm.nih.gov/pmc/articles/PMC3094361/
The answer is D. The classic presentation of acute cholangitis is fever, abdominal pain,
and jaundice (Charcot's triad), though only 50 to 75 percent of patients with acute
cholangitis have all three findings. Confusion and hypotension can occur in patients with
suppurative cholangitis, producing Reynolds pentad, which is associated with significant
morbidity and mortality.
 Fever is present in approximately 90% of cases.
 Abdominal pain and jaundice is thought to occur in 70% and 60% of
patients, respectively.
 Patients present with altered mental status 10-20% of the time and
hypotension approximately 30% of the time. These signs, combined with
Charcot's triad, constitute Reynolds pentad.
 Consequently, many patients with ascending cholangitis do not present
with the classic signs and symptoms.[4]
 Most patients complain of RUQ pain; however, some patients (ie, elderly
persons) are too ill to localize the source of infection.
Read more from Up to Date

Feedback on your answer


Collapse
21)
0/1
Which of the following complications of cholelithiasis can present with fever, persistent
tachycardia, hypotension, or jaundice?
(Select 1)(1pts)

Cholecystitis

Cholangitis

Pancreatitis

All of the above

None of the above

The presence of fever, persistent tachycardia, hypotension, or jaundice necessitate a


search for complications of cholelithiasis, including cholecystitis, cholangitis,
pancreatitis, or other systemic causes.
For more on the physical examination of patients with gallstones, read here.
Feedback on your answer
Collapse
22)
0/1
A 41-year-old obese white female presents to the emergency room with severe epigastric
pain and right shoulder pain that has been on or off over the last month but now
constant over the last 2 hours associated with vomiting. She notes it comes on after eating
and normally goes away on its own after an hour but tonight it has persisted longer. Her
past medical history is significant for frequent heartburn for which she takes zantac and
tums. After examination and history the patient notes that her pain has resolved
completely. Which of the following answers best explains the source of her pain?
(Select 1)(1pts)

Acid hypersecretion

Viscus distention

Mucosal inflammation

Peritoneal irritation

Symptomatic, uncomplicated gallstone disease (biliary colic) is one of the most common
manifestations of gallstones and is due to contraction of the gallbladder against a
transiently obstructed cystic duct. Classic symptoms of biliary colic include right upper
quadrant or epigastric pain that occurs after eating (especially fatty meals) and lasts from
30 minutes to several hours. Abdominal pain may be accompanied by diaphoresis,
nausea, and vomiting. The most common complications related to gallstones include
acute cholecystitis, choledocholithiasis, and gallstone pancreatitis.
This is the classic presentation of biliary colic secondary to gallstones. Ingestion of a
fatty meal causes the gallbladder to contract, which can press gallstones against a cystic
duct opening, increasing intra-gallbladder pressure and causing distention colicky pain.
Subsequent gallbladder relaxation allows the stone to fall back from the duct, causing the
pain resolves completely. Biliary colic is often accompanied by nausea, vomiting, and
rate cited shoulder or sub scapular discomfort (referred pain). The pain of biliary colic is
distinguished that of acute cholecystitis by its intermittent nature in relation to meals as
well as the absence of fever.
2014. Scientific American Surgery. Hamilton, Ontario & Philadelphia, PA. Decker
Intellectual Properties Inc. ISSN 2368-2744. STAT!Ref Online Electronic Medical
Library. http://online.statref.com/Document.aspx?fxId=61&docId=930. 10/17/2014
4:39:25 AM CDT (UTC -05:00).
Feedback on your answer
Collapse
Unit 7
23)
0/1
In the absence of a major surgical insult or concomitant coagulopathy, what platelet count
is required for normal coagulation?
(1pts)

10,000/μL

20,000/μL

50,000/μL

100,000/μL

The answer is B, Patients with a normal INR and a normal aPTT who exhibit unexpected
bleeding may have impaired platelet activity. Inadequate platelet activity is frequently
manifested as persistent oozing from wound edges or as low-volume bleeding. Such
bleeding is rarely the cause of exsanguinating hemorrhage, although it may be life-
threatening when it occurs in certain locations (e.g., inside the cranium or the
pericardium). Inadequate platelet activity may be attributable to an insufficient number of
platelets, platelet dysfunction, or a platelet inhibitor. In the absence of a major surgical
insult or concomitant coagulopathy, a platelet count of 20,000/μL or higher is usually
adequate for normal coagulation.14,15 There is some disagreement regarding the
threshold for platelet transfusion in the absence of active bleeding. Patients undergoing
procedures in which even small-vessel oozing is potentially life-threatening (e.g.,
craniotomy) should be maintained at a higher platelet count (i.e., > 20,000/μL). Patients
without ongoing bleeding who are not specifically at increased risk for major
complications from low-volume bleeding may be safely watched with platelet counts as
low as 10,000/μL. Read more
Feedback on your answer
Collapse
24)
0/1
The incidence of TRALI (Transfusion-related acute lung injury) has declined in recent
years due to which of the following choices below?

(1pts)
The use of plasma from female donors only

The use of plasma from male donors only

The use of whole blood instead of plasma

Different storage techniques

The answer is B, The use of plasma from male donors only. Transfusion of blood
products is not without risk. According to the Annual Summary of the FDA for 2012,
there were 38 transfusion-related fatalities in the United States, as opposed to 46 in
2008. Since the number of adverse outcomes is small in comparison to the overall
number of transfusions (an estimated 24 million in 2008), many emergency physicians
are unfamiliar with the potential complications of blood product administration. As an
example, TRALI is unfamiliar to many physicians and not universally discussed with
patients during the consent process,63 despite the fact that it was the leading cause of
transfusion-related fatality in both 2008 and 2012.
As early as the 1950s, noncardiogenic pulmonary edema was recognized as a potential
complication of blood transfusion, although it was not until 1983 that Popovsky and
colleagues coined the term “transfusion-related acute lung injury.” These investigators
reported that transfusion of whole blood, RBCs, and plasma from multiparous women
was found to be associated with the rapid onset of hypoxemia and pulmonary infiltrates,
either during transfusion or within several hours of administration. The association with
multiparous donors suggested as the mechanism of lung injury the passive transfer of
antibodies in the transfused blood product, with subsequent activation of recipient white
blood cells. Just as pregnant women can become immunized to Rh antigen present on the
RBCs of the fetus during periods of feto-maternal hemorrhage, exposure to fetal white
blood cells can give rise to antileukocyte antibodies. This idea has been supported by the
identification of antileukocyte antibodies in transfused blood products derived from
multiparous women. Although other factors may contribute to the pathogenesis of
TRALI, exposure to plasma or plasma-containing blood products containing
antileukocyte antibodies is still believed to be the primary mechanism for this
transfusion-related syndrome. Both the American Association of Blood Banks and the
American Red Cross have implemented changes to reduce this risk, including the
preferential use of male plasma, especially in high-plasmacontent products (FFP and
platelets). A national survey in 2008 revealed that > 90% of blood banks had instituted
collection of male-only, malepredominant, or never-pregnant female plasma.
Retrospective studies support these practices, confirming a significant reduction in rates
of TRALI following their adoption.67 Unified definitions of TRALI have been proposed
by the National Institutes of Health (NIH) and a Canadian Consensus Panel. The
Consensus Panel defined 4 criteria, as well as a separate diagnostic category, “possible
TRALI,” for patients in which the development of acute lung injury is temporally related
to both transfusion and another potential cause of acute lung injury. Read more
Feedback on your answer
Collapse
25)
0/1
Which of the following statements about massive transfusion protocol is true?
(1pts)

Platelets and plasma are only given when the PT/INR or ptt are abnormal

The goal is to transfuse with a 4:1:1 ratio with one unit of plasma and one
of platelets along with each 4 units of red blood cells (RBCs)

The use of fresh whole blood transfusion in war zones were shown to
improve outcomes prompting military surgeons to advocate a different
transfusion ratio

Fresh whole blood is reduces the risk of transmitting transfusion-related


infections

Answer C is the only true statement. It is true that coagulopathy may promote massive
hemorrhage and that massive hemorrhage causes a coagulopathy. As a rule, the massively
hemorrhaging patient has a coagulopathy out of proportion to the abnormalities reflected
with in vitro laboratory tests such as the prothrombin time (PT, commonly expressed as
an international normalized ratio [INR]) or activated partial thromboplastin time (aPTT).
Coagulopathy most typically occurs when there is massive hemorrhage in concert with
injury to a large volume of tissue. Although the nature of this coagulopathy is not
precisely defined, it probably involves a number of factors: (1) platelet dysfunction and
diminished availability of platelets at the interface between the vascular endothelium and
luminal flow; (2) dysfunctional formation of thrombi in the setting of acidosis and
hypothermia; (3) accelerated thrombolysis (attributable to dysfunctional thrombosis);
and, eventually, (4) consumption of clotting factors.
Increased recognition of this coagulopathy has prompted enthusiasm for administration of
plasma and platelets early in the treatment of the patient with massive hemorrhage. This
practice, frequently incorporated into a massive transfusion protocol, is described as
"1:1:1 transfusion" because the goal is to transfuse one unit of plasma and one of platelets
along with each unit of red blood cells (RBCs) such that the ratio stays balanced
throughout the resuscitation phase. (Adjustment of these ratios is necessary for centers
that use pooled quantities of plasma and/or platelets.) The rationale is that patients are
already extensively coagulopathic by the time they manifest thrombocytopenia or
prolongation of the INR or aPTT. Thus, during the first several hours during and after
massive hemorrhage, plasma and platelets are necessary independent of the laboratory
tests and help address a significant contributing factor for persistent hemorrhage. The
concept of liberal, early use of plasma and platelets developed in large part from the
recent US-led military campaigns in Iraq and Afghanistan. Initially in those conflicts, the
lack of a reliable supply of blood products near the scene of injury—and platelets
especially—led to the use of fresh whole blood transfusion. Although fresh whole blood
would be impractical in the civilian setting because of logistical issues and the risk of
transmitting transfusion-related infections, the perception of improved outcomes
associated with its use prompted military surgeons to advocate 1:1:1 transfusion.
Proponents of this approach argue that it restores hemostatic capacity and helps limit
hemorrhage, leading to improved survival. Indeed, observational studies suggest that
mortality is reduced.3 However, the ratios examined in these studies typically have been
calculated retrospectively over relatively long (e.g., 12- or 24-hour) time periods after
hemorrhage. Critics emphasize that patients who do not survive long enough to receive
early plasma and platelets bias such observational comparisons (i.e., they are classified as
receiving a large ratio of RBCs to plasma or platelets because they died early) and that
plasma and platelets carry theoretical harms of transfusion-related acute lung injury and
bacterial infection, respectively. Despite some uncertainty about the actual merits of the
1:1:1 approach, given that military conflicts have frequently spurred advances in
transfusion medicine, it appears likely that the approach will be adopted more widely if
the availability of blood products does not limit its application. In the absence of
sufficient availability of blood products to allow a 1:1:1 approach, aggressive correction
of abnormalities of the INR, aPTT, and platelet count are warranted for patients with
massive hemorrhage. read more

Unit 1
1)
1/1
True or False: More generalized tenderness (vs more localized tenderness) on abdominal
examination is generally a reliable guide to underlying cause of the pain.

( 1pts extra credit)

True

False Correct

Localized tenderness is generally a reliable guide to the underlying cause of the pain and
more generalized tenderness presents a greater diagnostic challenge. Read More
Feedback on your answer
Collapse
2)
0/1
What is a primary goal in the management of patients with abdominal pain?

(Select 1)( 1pts extra credit)

To establish a differential diagnosis through obtaining a clinical history

To refine that differential diagnosis with a physical examination and


appropriate studies

To determine the role of operative intervention in the treatment or


refinement of the working diagnosis

Only A and C

Only B and C

A, B, and C are all primary goals

The answer is F. Along with the basic methods of clinical evaluation known since
antiquity, the modern surgeon has an armamentarium of sophisticated diagnostic studies
that aid in the detection, diagnosis, and appropriate treatment of these frequently
overlapping clinical entities. The primary goals in the management of patients with
abdominal pain and/or an abdominal mass are (1) to establish a differential diagnosis
through obtaining a clinical history, (2) to refine that differential diagnosis with a
physical examination and appropriate studies, and (3) to determine the role of operative
intervention in the treatment or refinement of the working diagnosis. Read more

Feedback on your answer


Collapse
3)
0/1
Regarding esophageal adenocarcinoma and esophageal squamous-cell carcinoma, which
of the following are true?

( 1pts extra credit)

Esophageal adenocarcinoma has become the predominant type of


esophageal cancer in North America and Europe

Adenocarcinoma is three to four times as common in men as it is in


women

The sex distribution is more equal for squamous-cell carcinoma

Approximately three quarters of all adenocarcinoma lesions are found in


the distal esophagus

All of the above

None of the above

The answer is all of the above. Esophageal adenocarcinoma has become the predominant
type of esophageal cancer in North America and Europe, while esophageal squamous-cell
carcinoma remains the predominant esophageal cancer in Asia, Africa, and South
America and among African Americans in North America. Adenocarcinoma is three to
four times as common in men as it is in women, whereas the sex distribution is more
equal for squamous-cell carcinoma. The endoscopic appearance is also similar, although
approximately three quarters of all adenocarcinoma lesions are found in the distal
esophagus, whereas squamous-cell carcinoma is more frequent in the proximal to middle
esophagus. The overall 5-year survival rate for patients with esophageal adenocarcinoma
in the United States is approximately 17%, which is slightly higher than the rate for
patients with squamous-cell carcinoma.
N Engl J Med 2014; 371:2499-2509 December 25, 2014DOI:
10.1056/NEJMra1314530

Feedback on your answer


Collapse
4)
0/1
You are seeing a 26-year old college student in the ER following a roll over MVC. He
was wearing a seat belt and was not ejected. Upon arrival his complaining of abdominal
pain. His vital signs show a heart rate of 120/min, blood pressure is 134/98, RR 20/min,
O2 98% on room air. Physical exam shows he has a seat belt sign across his abdomen and
has tenderness greatest in his upper abdomen with no guarding. His CT scan of the
abdomen/pelvis with IV contrast only shows a liver laceration.

True or False: Most liver lacerations/injuries are managed operatively as they


often fail non-operative management.
( 1pts extra credit)

True Incorrect

False

False: The current approach to hepatic trauma has evolved to nonoperative management
in more than 80% of cases. Several contributing factors have been recognized: realization
that more than 50% of liver injuries stop bleeding spontaneously, the precedent of
successful nonoperative management in pediatric patients, knowledge that the liver has
tremendous capacity to heal after injury, and improvements in liver imaging with CT.
Criteria for nonoperative management include foremost, hemodynamic stability, absence
of other abdominal injuries that require laparotomy, immediate availability of resources
including a fully staffed operating room, and a vigilant surgeon. In general, any patient
who is stable enough to have a CT scan performed is likely to be successfully managed
nonoperatively. Grade I and II hepatic injuries should be observed in a monitored setting
with serial hematocrit evaluations and bed rest. Higher-grade injuries in stable patients
should be observed in an intensive care unit setting with optimization of all coagulation
factors. Read More
Feedback on your answer
Collapse
5)
0/1
A 40-year old male presents with severe upper abdominal pain that has worsened over the
last 2 weeks. It is described as an intermittent, gnawing pain that is made worse shortly
after eating and drinking. He has a past medical history chronic lower back pain
following an motor vehicle accident about five years ago and takes hydrocodone and
ibuprofen daily. What is the most likely diagnosis?

(Select 1)( 1pts extra credit)

Duodenal Ulcer

Gastric Ulcer

Pancreatitis

Cholecystitis

None of the above

The answer is B, gastric ulcer. The use of NSAIDs (40 fold increased risk for gastric
ulcer) and the worsening of pain after ingestion of food are both classic of gastric ulcers
in this case.
NSAID use confers around a 40 fold increase in the development of gastric ulcers and an
8 fold increase risk for the development of duodenol ulcers and . Therefore in this patient
you would expect gastric ulcer more likely than a duodenal ulcer. From Uptodate:
Peptic Ulcer Bleeding and NSAIDs:
The use of NSAIDs is the most commonly identified risk factor for peptic ulcer bleeding,
especially in the elderly. Studies have found relative risks for bleeding ranging from 2.7
to 33.9 [11]. Studies have also shown that the risk is drug-specific and dose-dependent.
As an example, in a study of 2777 patients, the overall relative risk (RR) of bleeding
associated with NSAID use was 5.3 (95% CI 4.5-6.2). However, the risk varied by drug
and was lowest for aceclofenac (RR 3.1, 95% CI 2.3-4.2) and was highest
for ketorolac (RR 14.4, 95% CI 5.2-39.9) [18]. The risk was higher in patients taking
high-dose NSAIDs compared with those taking medium- or low-dose NSAIDs (RR 6.8,
95% CI 5.3-8.8 versus 4.0, 95% CI 3.2-5.0). There was also an increased risk of bleeding
with aspirin use (RR 5.3) that again was dose-dependent (RR 7.5 with 500 mg per day
versus 2.7 with 100 mg per day). The concurrent use of aspirin and NSAIDs conferred an
even greater risk of bleeding than was seen with either agent alone (RR 12.7). Finally, the
risk was highest in the first 30 days of NSAID use, with a RR of 7.6 (95% CI 6.0-9.5).
The risk remained high between days 31 and 90 days (RR 7.3, 95% CI 4.0-13.2), but
dropped after 91 days (RR 2.6, 95% CI 1.6-4.1).
NSAID use has also been identified as a risk factor for ulcer perforation [16,19]. In a
study of 176 patients from Spain, NSAID use was the only risk factor that was
significantly associated with perforation (odds ratio [OR] 3.6). Read more
Feedback on your answer
Collapse
Unit 2
6)
1/1
You are seeing a 28-year old white male who was brought in by EMS after he was
assaulted while at a night club. On exam his GCS is 7, he is breathing spontaneously and
localizes pain. He has a large scalp laceration to his right parietal region. He is rushed to
the CT scanner and the image is shown below. What is the diagnosis?

(Select 1)(1pts)

Subdural Hematoma

Epidural Hematoma

Intraparenchymal Hematoma with extension

Subarachnoid Hemorrhage
The answer is B. Epidural hematoma (EDH) is a traumatic accumulation of blood
between the inner table of the skull and the stripped-off dural membrane. The inciting
event often is a focused blow to the head, such as that produced by a hammer or baseball
bat. In 85-95% of patients, this type of trauma results in an overlying fracture of the skull.
Blood vessels in close proximity to the fracture are the sources of the hemorrhage in the
formation of an epidural hematoma. Because the underlying brain has usually been
minimally injured, prognosis is excellent if treated aggressively. Outcome from surgical
decompression and repair is related directly to patient's preoperative neurologic
condition. more from Read more
Feedback on your answer
Collapse
7)
0/1
You are seeing a 32-year old male who presents after being kicked multiple times in his
right flank. He is complaining of severe flank pain and gross hematuria. What is the gold
standard for imaging of renal trauma?
(Select 1)(1pts)

MRI abdomen/pelvis

Ultrasound

CT abdomen/pelvis with contrast

Intravenous pyelography

None of the above

You didn't make any selections

The answer is C, CT abdomen/pelvis with contrast. CT scanning with intravenous


contrast enhancement is the modality of choice for the identification and staging of renal
trauma in the hemodynamically stable patient (image 6). The initial CT images will
frequently miss injuries to the renal pelvis and ureters as sufficient contrast may not yet
be present in the collecting system. Additional delayed images are needed to assess for
contrast extravasation when these injuries are suspected (image 7). In patients
undergoing CT scanning of other organ systems, suspected bladder injuries may be
investigated with CT cystography after retrograde filling of the bladder, as described
above. Read more
 CT with IV contrast is the Gold standard, high sensitivity
 Immediate and delayed post-contrast images to view collecting
system
 Images abdomen and retroperitoneum
 Allows diagnosis and staging
 Intravenous pyelography doesnt allow you to image the abdomen or
retroperitoneum.

Feedback on your answer


Collapse
8)
0/1
Which of the following is not an example of referred pain?
(Select 1)(1pts)

Ureteral stone causes ipsilateral testicular pain

Ruptured hemorrhagic cyst causing lower back pain

Acute MI causing jaw pain

Acute Appendicitis causing periumbilical pain

All of the above

None of the above

The answer is D, as Acute Appendicitis causing periumbilical pain is an example of


visceral pain.
Visceral Pain
•Caused by the stretching of fibers innervating the walls or capsules of hollow or solid
organs
•A steady ache or vague discomfort to excruciating or colicky
•Visceral afferents follow a segmental distribution and pain is localized by the sensory
cortex to an approximate spinal cord level determined by embryological organ of the
organ involved.
•Biliary, duodenum, stomach produce epigastric pains
•Most small bowel, appendix, cecum cause periumbilical pain
•Hindgut structures, such as the bladder, distal two-thirds of the colon, and pelvic
genitourinary (GU) organs, usually cause pain in the suprapubic region.
•Pain is often reported in the back for retroperitoneal structures, such as the aorta and
kidneys
•Deep musculoskeletal structures (especially of the back) are innervated by visceral
sensory fibers with similar qualities to those arising from intra-abdominal organs.
•Intraperitoneal organs are bilaterally innervated causing pain to be located in the midline
regardless of which side it is coming from. Classic example is early appendicitis causing
periumbilical pain
Visceral Pain
•Caused by the stretching of fibers innervating the walls or capsules of hollow or solid
organs
•A steady ache or vague discomfort to excruciating or colicky
•Visceral afferents follow a segmental distribution and pain is localized by the sensory
cortex to an approximate spinal cord level determined by embryological organ of the
organ involved.
•Biliary, duodenum, stomach produce epigastric pains
•Most small bowel, appendix, cecum cause periumbilical pain
•Hindgut structures, such as the bladder, distal two-thirds of the colon, and pelvic
genitourinary (GU) organs, usually cause pain in the suprapubic region.
•Pain is often reported in the back for retroperitoneal structures, such as the aorta and
kidneys
•Deep musculoskeletal structures (especially of the back) are innervated by visceral
sensory fibers with similar qualities to those arising from intra-abdominal organs.
•Intraperitoneal organs are bilaterally innervated causing pain to be located in the midline
regardless of which side it is coming from. Classic example is early appendicitis causing
periumbilical pain
Feedback on your answer
Collapse
Unit 3
9)
0/1
You are consulted to see a 61-year-old male because of continuing symptoms due to
refractory gastroesophageal reflux that have not improved despite lifestyle
modifications and treatment with a twice daily proton pump inhibitor (omeprazole 40 mg
daily). He notes that he continues to have occasional chest pain and epigastric burning
with eating. He denies dysphagia, regurgitation, weight loss, or a change in bowel habits.
He denies other past medical history and has no cardiac risk factors. Vital signs and
physical examination are normal. His upper endoscopy shows no ulcers and is otherwise
unremarkable. Which of the following is the most appropriate treatment at this time?
(Select 1)(1pts)

Schedule consultation for evaluation for antireflux surgery

Increase the proton pump inhibitor to 3 times daily

Change to a different proton pump inhibitor

Add ranitidine at bedtime to the current regimen

The answer is A. Consider surgery as an option for patients with well-documented GERD
who require long-term PPI maintenance therapy but show satisfactory relief of symptoms
and who:
 Are older than 50 years
 Consider long-term medication a financial burden
 Are noncompliant with drug therapy
 Prefer a single surgical intervention to long-term drug treatment
 Experience prominent symptoms of regurgitation, even with medical
control of heartburn symptoms
Nissen fundoplication is the most common surgical intervention for GERD. This
procedure aims to restore the physiology and anatomy of the gastroesophageal junction
by wrapping the gastric fundus around the distal esophagus. The FDA has also approved
several endoscopic procedures for treatment of GERD, including endoscopic suturing and
radiofrequency ablation of the lower esophageal sphincter. Read More

Feedback on your answer


Collapse
10)
1/1
Which of the following is NOT associated with an increased risk of esophageal
adenocarcinoma?
(Select 1)(1pts)

Cigarette smoking

Gastroesophageal reflux disease

Obesity

Helibactor pylori infection

All of the above

None of the above

Answer D is not correct. Esophageal cancer has two main subtypes — esophageal
squamous-cell carcinoma and esophageal adenocarcinoma; their precursor lesions are
esophageal squamous dysplasia and Barrett’s esophagus, respectively. Although
squamous-cell carcinoma accounts for about 90% of cases of esophageal cancer
worldwide, the incidence of and mortality rates associated with esophageal
adenocarcinoma are rising and have surpassed those of esophageal squamous-cell
carcinoma in several regions in North America and Europe. In the United States, more
than 18,000 new cases of esophageal cancer and more than 15,000 deaths from
esophageal cancer were expected in 2014. Esophageal carcinoma is rare in young people
and increases in incidence with age, peaking in the seventh and eighth decades of life.
The main risk factors for esophageal adenocarcinoma are gastroesophageal reflux
disease, obesity, and cigarette smoking; H. pyloriinfection is associated
with a reduced risk. Cigarette smoking and alcohol consumption constitute the
main risk factors for esophageal squamous-cell carcinoma. High intake of red meats, fats,
and processed foods is associated with an increased risk of both types of esophageal
cancer, whereas high intake of fiber, fresh fruit, and vegetables is associated with a lower
risk.
N Engl J Med 2014; 371:2499-2509December 25, 2014DOI:
10.1056/NEJMra1314530

Feedback on your answer


Collapse
11)
0/1
A 22-year old male presents with severe epigastric pain that radiates into his chest after
vomiting over a dozen times following a night of heavy drinking. Vital signs show an
oraltemperature of 100.7 F, blood pressure 147/97, heart rate 127beats/min, respirations
24/min. Physical exam shows crepitus in the suprasternal notch, clear lung exam
bilaterally, and severe epigastric tenderness on abdominal exam. What is most likely
causing this patients symptoms?
(Select 1)(1pts)

Mallory Weiss tear

Spontaneous Pneumothorax

Perforated Duodenal Ulcer

Esophageal Perforation

The answer is D. This patient has evidence subcutaneous air (crepitus) and has likely
ruptured his esophagus. Effort rupture of the esophagus or Boerhaave's syndrome is a
spontaneous perforation of the esophagus that most commonly results from a sudden
increase in intraesophageal pressure combined with negative intrathoracic pressure
caused by straining or vomiting. It was first described by Dr. Herman Boerhaave, a
physician from Leiden, the Netherlands.

In patients suspected to have an esophageal perforation based upon clinical presentation


and chest radiography, computed tomography (CT) scan or esophagram should be
obtained promptly.
Computed tomography — Computed tomography (CT) scan shows esophageal wall
edema and thickening, extraesophageal air, periesophageal fluid with or without gas
bubbles, mediastinal widening, and air and fluid in the pleural spaces, retroperitoneum or
lesser sac (image 2).
Esophagram — The diagnosis of esophageal perforation can also be confirmed by water-
soluble contrast esophagram (Gastrografin), which reveals the location and extent of
extravasation of contrast material. However, false negative results occur in as many as 10
percent of cases. The sensitivity of contrast studies depends upon the size and location of
the perforation, and the technique used for the study.
Although barium is superior in demonstrating small perforations, it causes an
inflammatory response in mediastinal or pleural cavities and is therefore not used as the
primary diagnostic study [13]. If, however, the water-soluble study is negative, a barium
study should be performed for better definition (image 3).
Upper endoscopy — The role of upper endoscopy in the diagnosis of spontaneous
perforation remains controversial. Both the endoscope and insufflation of air can extend
the perforation and introduce air into the mediastinum [14]. However, in the appropriate
setting, often in the operating room with a patient prepared for surgical intervention,
endoscopy performed by a skilled endoscopist can be a useful tool for diagnosis and
treatment planning.
TREATMENT — Boerhaave's syndrome is rare and there is little evidence to guide
treatment. As a general rule, treatment depends upon the size and location of the
perforation, whether it is a contained perforation in the mediastinum or between the
mediastinum and visceral lung pleura, how rapidly it is diagnosed, whether the esophagus
has underlying disease, and the patient’s underlying comorbidities (algorithm 1).
Surgery is generally required for thoracic perforations [15], while cervical perforations
can often be managed without surgery.More from UptoDate
Feedback on your answer
Collapse
Unit 4
12)
1/1
You are seeing a 46-year old female smoker in the ER who presents tonight with acute
onset of epigastric pain, nausea and vomiting. She past medical history except for
right hip arthritis and only takes Alleve daily.
Her vitals show a blood pressure of 145/75, heart rate of 95/min, respiratory rate
of 14/min, O2 saturation of room air of 99%, oral temperature 98.7 F
Her physical exam shows mild upper abdominal tenderness that is greatest in the
epigsatric area along with no voluntary guarding in her upper abdomen. Upright chest
xray reveals free air under the right hemidiaphragm. True or False: Based on these
findings, surgery is always indicated for definitive treatment.
(1pts)

True

False Correct
This statement is False. Ulcer perforation should be suspected in patients who suddenly
develop severe, diffuse abdominal pain. Perforations complicate 2 to 10 percent of peptic
ulcers [70]. Duodenal, antral, and gastric body ulcers account for 60, 20, and 20 percent
of perforations due to peptic ulcer disease (PUD), respectively. Also there is never an
ALWAYS in medicine and with perforated peptic ulcers, a large majority of them are
found to be sealed upon upper GI study with soluble contrast. If this is found then these
patients can be observed safely as long as peritonitis does not develop.
A major decision when treating patients with ulcer perforation is whether and when to
operate. After resuscitation, emergent operation and closure with a piece of omentum is
the standard of care for patients with an acute perforation and a rigid abdomen with free
intraperitoneal air. If the patient is stable or improving, especially if spontaneous sealing
of the perforation has been demonstrated, nonoperative management with close
monitoring is a reasonable option. With any free perforation, regardless of the presence
or size of the leak, if the patient's status is deteriorating, urgent surgery is indicated.
Prolonged efforts to establish a diagnosis or pursue non operative care despite worsening
status can be counterproductive, since a needed operation will be delayed. In addition,
surgery is indicated in circumstances where the cause of an acute abdomen has not been
established or the patient's status cannot be closely monitored.Read More from up to
date

Feedback on your answer


Collapse
13)
0/1
A 48-year old black male presents to you with sudden onset of vomiting and diffuse
abdominal pain that hurts to breathe two hours prior to arrival. He denies past medical
history and his only medications are ibuprofen for chronic right shoulder pain. He smokes
1 pack per day and occasionally drinks alcohol.
His vitals show an oral temperature of 99.3 F, heart rate of 129 beats/min, respiratory rate
20/min, blood pressure 151/100, O2 saturation of 98% on room air.
Physical exam shows him to be uncomfotable. Cardiac exam shows tachycardia, his
lungs are clear to auscultation, and abdominal exam shows diffuse tenderness to light
palpation greatest in the right upper quadrant along with guarding. Rectal exam shows
heme positive brown stool. An upright chest x-ray which is shown below. What is the
diagnosis?
(Select 1)(1pts)

Pneumomediastinum

Pneumoperitoneum

Intestinal Malrotation

Small Bowel Obstruction

Pneumothorax

The answer is B, Pneumoperitoneum which describes gas within the peritoneal cavity,
and is often the harbinger of a critical illness. The most common cause of a
pneumoperitoneum is from the disruption of the wall of a hollow viscus. Read more
An erect chest x-ray is probably the most sensitive plain radiograph for the detection of
free intraperitoneal gas. If a large volume pneumoperitoneum is present, it may be
superimposed over normal aerated lung with normal lung markings.
 subdiaphragmatic free gas
 cupola sign (in supine film)
Free gas within the peritoneal cavity can be detected on an abdominal radiograph. These
signs can be further divided by anatomical compartments in relation to the
pneumoperitoneum:
 bowel related signs
 double wall sign (also known as Rigler's sign or bas-
relief sign)
 telltale triangle sign (also known as triangle sign)
 peritoneal ligament related signs
 football sign
 falciform ligament sign
 lateral umbilical ligament sign (also known as inverted
"V" sign)
 urachus sign
 right upper quadrant signs
 lucent liver sign
 hepatic edge sign
 fissure for ligamentum teres sign
 Morison's pouch sign
 cupola sign
The patient above likely has a perforated ulcer from underlying NSAID use. Ulcer
perforation should be suspected in patients who suddenly develop severe, diffuse
abdominal pain. Perforations complicate 2 to 10 percent of peptic ulcers [70]. Duodenal,
antral, and gastric body ulcers account for 60, 20, and 20 percent of perforations due to
peptic ulcer disease (PUD), respectively. If imaging is required, plain x-rays are typically
obtained first. Careful interpretation of upright chest and abdominal films can detect
diagnostic free air in many cases of perforated gastric and duodenal ulcers [76]. The
presence of free air on abdominal imaging is highly indicative of a perforated viscus
(image 1 and image 2 and image 3), although about 10 to 20 percent of patients
with a perforated duodenal ulcer will not have free air [76]. If free air is found, no other
diagnostic studies are necessary. Leakage of water soluble oral contrast may be useful in
selected cases. Once the oral contrast is given, the patient should be rotated 360 degrees
and placed on the right side to fill the antrum and duodenum with contrast. However,
many perforations have already sealed spontaneously by the time of presentation [77], so
the absence of a leak does not exclude the diagnosis of a perforated ulcer. Read More
Feedback on your answer
Collapse
14)
0/1
What % of benign appearing gastric ulcers are found to be harbor malignancy?
(Select 1)(1pts)

<1%
5-11%

15-21%

24-30%

The answer is B, 5-11%. Given the low predictive value of the signs and symptoms of
PUD, the diagnosis of uncomplicated peptic ulcers is difficult to make solely on a clinical
basis. The diagnosis of PUD is typically made by endoscopy and upper gastrointestinal
(UGI) radiography. Endoscopy is the most accurate method of establishing the diagnosis
of peptic ulcers, with a reported sensitivity of 92% and specificity of 100%.7 In addition
to identifying the ulcer and its features, location, and size, endoscopy provides an
opportunity for biopsies to test for H. pylori and exclude malignancy and for therapeutic
interventions for bleeding ulcers. Duodenal ulcers are most often benign and do not
require routine biopsy. Multiple biopsies are indicated for all gastric ulcers as even
lesions with a benign appearance harbor malignancy in 5 to 11% of cases. Read more
Feedback on your answer
Collapse
15)
0/1
You are seeing a 44-year old female who presents with worsening epigastric pain
following meals and "heartburn" that has been getting more persistent over the last few
days. She has a past medical history of hypertension. Her only medication is naprosyn
500 mg BID and HCTZ 25mg daily. On exam she has mild epigastric tenderness to
palpation. Her rectal exam shows her stool is brown but guaiac positive for blood. Her
portable upright chest xray shows no signs of free air or perforation.Based on her history
of current NSAID use, where would you likely expect the location of her ulcer?

(Select 1)(1pts)

Gastric

Duodenal

Jejunal

Esophageal

NSAID use confers around a 40 fold increase in the development of gastric ulcers and an
8 fold increase risk for the development of duodenol ulcers and . Therefore in this patient
you would expect gastric ulcer more likely than a duodenal ulcer.
The use of NSAIDs is the most commonly identified risk factor for peptic ulcer bleeding,
especially in the elderly. Studies have found relative risks for bleeding ranging from 2.7
to 33.9 [11]. Studies have also shown that the risk is drug-specific and dose-dependent.
As an example, in a study of 2777 patients, the overall relative risk (RR) of bleeding
associated with NSAID use was 5.3 (95% CI 4.5-6.2). However, the risk varied by drug
and was lowest for aceclofenac (RR 3.1, 95% CI 2.3-4.2) and was highest for ketorolac
(RR 14.4, 95% CI 5.2-39.9) [18]. The risk was higher in patients taking high-dose
NSAIDs compared with those taking medium- or low-dose NSAIDs (RR 6.8, 95% CI
5.3-8.8 versus 4.0, 95% CI 3.2-5.0). There was also an increased risk of bleeding with
aspirin use (RR 5.3) that again was dose-dependent (RR 7.5 with 500 mg per day versus
2.7 with 100 mg per day). The concurrent use of aspirin and NSAIDs conferred an even
greater risk of bleeding than was seen with either agent alone (RR 12.7). Finally, the risk
was highest in the first 30 days of NSAID use, with a RR of 7.6 (95% CI 6.0-9.5). The
risk remained high between days 31 and 90 days (RR 7.3, 95% CI 4.0-13.2), but dropped
after 91 days (RR 2.6, 95% CI 1.6-4.1).
NSAID use has also been identified as a risk factor for ulcer perforation [16,19]. In a
study of 176 patients from Spain, NSAID use was the only risk factor that was
significantly associated with perforation (odds ratio [OR] 3.6). Read more
Feedback on your answer
Collapse
16)
0/1
You are seeing a 45-year old male in clinic who has a past medical history of
osteoarthritis for which he takes daily alleve. He denies tobacco use or alcohol use. He
has been having recent early satiety, bloating and epigastric discomfort over the last
month. You suspect peptic ulcer disease. Which of the following ulcer locations is more
associated with H. Pylori infection?
(Select 1)(1pts)

Duodenal

Gastric

Esophageal

Jejunal

None of the above

The answer is A, duodenal ulcers. H. pylori, a gram-negative, helical, rod-


shaped bacterium, colonizes the gastric mucosa of approximately one-half of the world
population and an estimated 30% to 40% of the U.S. population. H. pylori is present in
95% of patients with duodenal ulcers and in 70% of those with gastric ulcers. It is
typically transmitted via the fecal-oral route during early childhood and persists for
decades. The bacterium is a known cause of gastric and duodenal ulcers and is a risk
factor for mucosa-associated lymphoid tissue (MALT) lymphoma and gastric
adenocarcinoma. Am Fam Physician. 2015 Feb 15;91(4):236-242.

Feedback on your answer


Collapse
Unit 5
17)
0/1
Roughly what percentage of patients with blunt liver injury initially managed non-
operatively will go on to need surgical intervention?
(1pts)

<10%

25%

50%

75%

The answer is A, During the last century, the management of blunt force trauma to the
liver has changed from observation and expectant management in the early part of the
1900s to mainly operative intervention, to the current practice of selective operative and
nonoperative management. A 2008 study by Tinkoff et al.4 showed that 86.3% of hepatic
injuries are now managed without operative intervention. The current reported success
rate of nonoperative management of hepatic trauma ranges from 82% to 100%. Most
blunt liver trauma (80% in adults, 97% in children) patients are currently treated
conservatively. The success of non-operative management depends upon proper selection
of the patient. The patients, who are managed non-operatively, usually have grade I and
II liver injuries, hemoperitoneum less than 900 ml and blood transfusion of less than 3
units. The contraindications to non-operative management include refractory
hypotension, signinficant fall in haematocrit, the extravasations of intravenous contrast
agent, expanding haematoma and grade IV and V liver injury on CECT abdomen. The
patients of grade III liver injuries need very close observation as they may require
surgical intervention during first 24 hours. The failure rate of non-operative management
is not more than 5% inmost studies. It seems that patients with grade VI injuries rarely
reaches to the hospital alive and are not salvageable. Therefore, such injuries are usually
documented on autopsy. Mortality from blunt hepatic trauma is about 5% and is related
to uncontrolled hemorrhage.
Interventional radiology may be needed to perform an angiogram and embolization for
bleeding or to percutaneously drain an abscess or b iloma. An endoscopic retrograde
cholangiopancreatogram (ERCP) and stent placement may be required for biliary leak.
Even when such complications of the liver injury develop, only 15% require operative
intervention. Hepatic artery angiography with embolization is an important tool for the
stable patient with contrast extravasation who is being managed nonoperatively. It can
also be invaluable for the postoperative patient who has been stabilized by perihepatic
packing or who has rebled after an initial period of stability. Angioembolization has a
greater than 90% success rate in the control of bleeding with a low risk of rebleeding and
a reduction in required volume of transfusion. Read More
Feedback on your answer
Collapse
18)
1/1
You are seeing a 20-year-old male in hospital 12 hours after he was admitted for
observation following an MVC. Initially he was complaining of left sided chest wall pain
after his car was t-boned on the passenger side. His initial physical examination showed
bruises on the anterior chest wall and upper abdominal wall. X-rays revealed fractures
of his sixth and seventh ribs on the left (but no pneumothorax or pleural effusion). A
FAST Exam did not show any free intraperitoneal fluid. Currently, he complains of
worsening epigastric pain, left shoulder pain and mild nausea. His current vitals show his
blood pressure is now 95/60 and pulse rate 115 beats/min, and O2 saturation is 96% on
room air. Which of the following is the next best step in treatment?
(Select 1)(1pts)

Repeat ECG and cardiac biomarkers

Repeat PA and lateral chest x-ray

Abdominal CT scan with intravenous contrast

Transesophageal echocardiogram

This patient presents with blunt abdominal trauma with the delayed onset of hypertension
and signs and symptoms worrisome for likely splenic injury. Most common injuries are
to the spleen, liver, and less common injuries are to the hollow viscous organs in the
abdomen. Symptoms and signs suggesting splenic injury include left upper quadrant pain,
abdominal wall contusion, left lower chest wall tenderness, hypotension, and left
shoulder. Pain referred from splenic hemorrhage, hitting the phrenic nerve and diaphragm
(Kehr sign). The initial examination after blunt abdominal trauma can be unremarkable
and the symptoms can occur hours later, indicating ongoing splenic injury. The best
choice here would be an abdominal CT scan with intravenous contrast, only (no oral
contrast is needed because as little utility). This will define organ injury, assess for
presence of bleeding in all abdominal compartments, determine the need for surgery.
The spleen is the second most commonly injured abdominal organ in blunt trauma
patients. Historical studies have reported a 10% mortality with all splenic injuries;
however, isolated splenic injury mortality is less than 1%. The mechanisms of injury are
similar to those seen with liver injuries: motor vehicle collisions, autopedestrian
accidents, and falls. Similar to penetrating trauma to the liver, stab wounds to the spleen
typically result in direct linear tears, whereas gunshot wounds result in significant
cavitary injuries.
Until the 1970s, splenectomy was considered mandatory for all splenic injuries.
Recognition of the immune function of the spleen refocused efforts on splenic salvage in
the 1980s.38,39Following success in pediatric patients, NOM of splenic injuries was
adopted in the adult population and has become the prevailing strategy for blunt splenic
trauma.40

INITIAL EVALUATION AND INJURY GRADING

Addressing the patient's ABC's, examining the patient's abdomen, and performing
adjunctive imaging with FAST and CT are the initial steps of diagnosing a patient's
splenic injury. Hypotension with a positive FAST scan should prompt emergent
laparotomy. For patients with an identified blunt splenic injury on a CT scan, the injury
should be graded according to the AAST injury grading scale [see Table 1].3

Similar to liver injuries, the grade of splenic injury predicts failure rates and complication
rates of NOM. Other findings that should be searched for on a CT scan include contrast
extravasation (is the contrast blush contained within the spleen, or does it spill into the
peritoneum?), the amount of intra-abdominal hemorrhage (is it isolated to the splenic
fossa, or does blood extend into the pelvis?), and the presence of pseudoaneurysms.

NONOPERATIVE MANAGEMENT

Indications for Nonoperative Management

NOM of solid-organ injuries is pursued in hemodynamically stable patients who do not


have overt peritonitis or other indications for laparotomy.41-45 There is no age cutoff for
patients for the NOM of solid-organ injuries.46,47 High-grade injuries, a large amount of
hemoperitoneum, contrast extravasation, and pseudoaneurysms are not absolute
contraindications for NOM; however, these patients are at high risk for failure.48-51 The
identification of contrast extravasation as a risk factor for failure of NOM led to liberal
use of angioembolization. The true value of angioembolization in splenic salvage has not
been rigorously evaluated. Patients with intraparenchymal splenic blushes who are
otherwise asymptomatic may be considered for a period of observation rather than
empiric angioembolization52; it is thought that the contained hemorrhage within the
splenic capsule may result in tamponade of the bleeding

It is clear, however, that 20 to 30% of patients with splenic trauma deserve early
splenectomy and that failure of NOM often represents poor patient selection.53,54 In
adults, indications for prompt laparotomy include initiation of blood transfusion within
the first 12 hours considered to be secondary to the splenic injury or hemodynamic
instability. In the pediatric population, blood transfusions up to half of the patient's blood
volume are used prior to operative intervention. Following the first 12 postinjury hours,
indications for laparotomy are not as black and white. Determination of the patient's age,
comorbidities, current physiology, degree of anemia, and associated injuries will
determine the use of transfusion alone versus intervention with either embolization or
operation. Unlike hepatic injuries, which rebleed in 24 to 48 hours, delayed hemorrhage
or rupture of the spleen can occur up to weeks following injury. Algorithms for the
management of pediatric splenic injuries exist,55 and the patient's physiologic status is
the key determinant. Rapid mobilization in patients who are hemodynamically stable with
a stable hematocrit and no abdominal pain is generally successful. Overall, nonoperative
treatment obviates laparotomy in more than 90% of cases.

Follow-up Imaging

Out of concern over the risk of delayed hemorrhage or other complications, follow-up CT
scans have often been recommended; unfortunately, there is no consensus as to when or
even whether they should be obtained. Patients with grade I or II splenic injuries rarely
show progression of the lesion or other complications on routine follow-up CT scans; it is
reasonable to omit such scans if patients' hematocrits remain stable and they are
otherwise well. Patients with more extensive injuries often have a less predictable course,
and CT may be necessary to evaluate possible complications. Routine CT before
discharge, however, is unwarranted. Outpatient CT, however, in patients who participate
in vigorous or contact sports should be performed at 6 weeks to document complete
healing before resuming those activities. A more convenient and less expensive
alternative to follow-up CT is ultrasonographic monitoring of lesions.

OPERATIVE EXPOSURE AND HEMORRHAGE CONTROL

In penetrating abdominal injuries not suitable for NOM and in blunt abdominal injuries
when NOM is contraindicated or has failed, exploratory laparotomy is performed.

To ensure safe removal or repair, the spleen should be mobilized to the point where it can
be brought to the surface of the abdominal wall without tension. An incision is made in
the peritoneum and the endoabdominal fascia, beginning at the white line of Toldt along
the descending colon and continuing cephalad 1 to 2 cm lateral to the posterior peritoneal
reflection of the spleen; this plane of dissection is continued superiorly until the
esophagus is encountered [see

Figure 15a]. Posteriorly, blunt dissection is performed to mobilize the spleen and
pancreas as a composite away from Gerota fascia and up and out of the retroperitoneum;
this posterior plane may be extended to the aorta if necessary [seeFigure 15b].
Additionally, the attachments between the spleen and the splenic flexure of the colon may
be divided to avoid avulsion of the inferior splenic capsule. Care must be taken not to
pull on the spleen; otherwise, it will tear along the posterior peritoneal reflection, causing
significant hemorrhage. It is often helpful to rotate the operating table 20° to the patient's
right so that the weight of the abdominal viscera facilitates viscera retraction. Any
ongoing hemorrhage from the splenic injury may be temporarily controlled with digital
occlusion of the splenic hilar vessels. Once mobilization is complete, the spleen can be
repaired or removed without any need to struggle to achieve adequate exposure.

DEFINITIVE MANAGEMENT OF INJURIES

Splenic injuries are treated operatively by splenectomy, partial splenectomy, or splenic


repair (splenorrhaphy), based on the extent of the injury and the physiologic condition of
the patient. Splenectomy is indicated for hilar injuries, pulverized splenic parenchyma, or
any grade II or higher injury in a coagulopathic or multiply injured patient. We employ
autotransplantation of splenic implants [see

Figure 16] for partial immunocompetence in younger patients.56 Drains are not used.
Partial splenectomy can be employed in patients in whom only the superior or inferior
pole has been injured. Hemorrhage from the raw splenic edge is controlled with a
horizontal mattress suture, with gentle compression of the parenchyma [see

Figure 17]. Similar to hepatic injuries, splenorrhaphy techniques to achieve hemostasis


include topical agents (electrocautery, argon beam coagulation, thrombin-soaked gelatin
foam sponge, fibrin glue, BioGlue), enveloping the injured spleen in absorbable mesh,
and pledgeted suture repair.

POSTOPERATIVE CARE AND COMPLICATIONS

Enthusiasm for splenic salvage was driven by the rare but often fatal complication of
overwhelming postsplenectomy sepsis (OPSS). OPSS is caused by encapsulated
bacteria,Streptococcus pneumoniae, Haemophilus influenzae,
and Neisseria meningitidis, which are resistant to antimicrobial treatment. In
patients undergoing splenectomy, prevention against these bacteria is provided via
vaccines administered optimally at 14 days but definitely prior to hospital
discharge.57 Vaccines to be administered include Pneumovax (Merck & Co., Inc.,
Whitehouse Station, NJ), Menactra (Sanofi Pasteur, Swiftwater, PA), and Fluvirin
(Novartis, East Hanover, NJ). Revaccination remains open to debate, but some argue for
revaccination every 6 years.

An immediate postsplenectomy increase in platelets and white blood cells (WBCs) is


normal; however, beyond postoperative day 5, a WBC count above 15,000/μL and a
platelet to WBC ratio less than 20 are highly associated with sepsis and should prompt a
thorough search for underlying infection.58,59 A common infectious complication
following splenectomy is a subphrenic abscess, which should be managed with
percutaneous drainage. Following splenectomy or splenorrhaphy, postoperative
hemorrhage may be attributable to loosening of a tie around the splenic vessels, a missed
short gastric artery, or recurrent bleeding from the spleen if splenic repair was used.
Additional sources of morbidity include a concurrent but unrecognized iatrogenic injury
to the pancreatic tail during rapid splenectomy, resulting in pancreatic ascites or fistula.
2014. Scientific American Surgery. Hamilton, Ontario & Philadelphia, PA. Decker
Intellectual Properties Inc. ISSN 2368-2744. STAT!Ref Online Electronic Medical
Library. http://online.statref.com/Document.aspx?fxId=61&docId=2297. 10/17/2014
4:45:39 AM CDT (UTC -05:00).

Feedback on your answer


Collapse
19)
0/1
You are seeing a 27-year old male in the trauma bay shortly after being stabbed in this
back with a 6 cm buck knife by his disgruntled neighbor after an argument.
He arrived via EMS sitting up, awake and alert with a GCS of 15. His BP is 140/90, HR
120 and O2 sat is 85% on a non re-breather.
On physical exam there is a 3 cm non bleeding wound just left of his mid thoracic spine.
Upon auscultation there is decreased breath sounds on his left side. There are no signs of
tracheal deviation or JVD.
What is the next best step in management?
(1pts)

Immediately to the OR for surgical exploration

Upright Chest Xray followed by a chest tube

Immediate Needle thoracotomy on the left

Pericardiocentesis under US guidance

The answer is A, Upright Chest Xray followed by a chest tube. This patient is awake and
talking. He is stable enough for a chest xray followed by chest tube placement. A needle
thoracotomy to should be done if he has evidence of tension pneumothorax which would
be respiratory distress and hypotension. He will likely then need a chest tube and can be
further managed. Read More
Feedback on your answer
Collapse
Unit 6
20)
1/1
You are seeing a 64-year old black female who is presenting with 2 days of worsening
upper abdominal pain that has become persistent. She denies fever at home but has a n
oral temp of 38.0 (100.4 F) in the ER. Her BP is 94/50, HR is 122 bpm and respiratory
rate 16/min. On exam she appears slightly jaundiced in her sclera, she has a positive
murphy's sign with some involuntary guarding. Her WBC comes back at 20k with 11%
bands. Her BMP is unremarkable. What triad of symptoms is classically associated with
cholangitis?
(Select 1)(1pts)

RUQ pain, fever,


bandemia

Epigastric pain,
fever,
leukocystosis

Bandemia, fever,
jaundice

RUQ pain, fever,


jaundice

Charcot's triad

http://www.ncbi.nlm.nih.gov/pmc/articles/PMC3094361/

The answer is D. The classic presentation of acute cholangitis is fever, abdominal pain,
and jaundice (Charcot's triad), though only 50 to 75 percent of patients with acute
cholangitis have all three findings. Confusion and hypotension can occur in patients with
suppurative cholangitis, producing Reynolds pentad, which is associated with significant
morbidity and mortality.
 Fever is present in approximately 90% of cases.
 Abdominal pain and jaundice is thought to occur in 70% and 60% of
patients, respectively.
 Patients present with altered mental status 10-20% of the time and
hypotension approximately 30% of the time. These signs, combined
with Charcot's triad, constitute Reynolds pentad.
 Consequently, many patients with ascending cholangitis do not present
with the classic signs and symptoms.[4]
 Most patients complain of RUQ pain; however, some patients (ie,
elderly persons) are too ill to localize the source of infection.
Read more from Up to Date

Feedback on your answer


Collapse
21)
1/1
Which of the following imaging studies is most sensitive for the diagnosis of acute
cholecystitis?

(1pts)

Ultrasound

HIDA scan

CT scan

MRI

The answer is B, HIDA scan. US is about 88% sensitive where as HIDA Scanning is
slightly better and more senstive but US is still the recommended as the first line
diagnostic imaging study. Cholescintigraphy using 99mTc-hepatic iminodiacetic acid
(generically referred to as a HIDA scan) is indicated if the diagnosis remains uncertain
following ultrasonography. Technetium labeled hepatic iminodiacetic acid (HIDA) is
injected intravenously and is then taken up selectively by hepatocytes and excreted into
bile. If the cystic duct is patent, the tracer will enter the gallbladder, leading to its
visualization without the need for concentration. The HIDA scan is also useful for
demonstrating patency of the common bile duct and ampulla. Normally, visualization of
contrast within the common bile duct, gallbladder, and small bowel occurs within 30 to
60 minutes (image 3). The test is positive if the gallbladder does not visualize. This
occurs because of cystic duct obstruction, usually from edema associated with acute
cholecystitis or an obstructing stone (image 4).
Cholescintigraphy has a sensitivity and specificity for acute cholecystitis of
approximately 90 to 97 percent and 71 to 90 percent, respectively [23,26,28,29].
Cystic duct obstruction with a stone or tumor in the absence of acute cholecystitis can
cause a false positive test. Conditions that can cause false positive results despite a non-
obstructed cystic duct include:
 Severe liver disease, which may lead to abnormal uptake and excretion
of the tracer.
 Fasting patients receiving total parenteral nutrition, in whom the
gallbladder is already maximally full due to prolonged lack of
stimulation.
 Biliary sphincterotomy, which may result in low resistance to bile
flow, leading to preferential excretion of the tracer into the duodenum
without filling of the gallbladder.
 Hyperbilirubinemia, which may be associated with impaired hepatic
clearance of iminodiacetic acid compounds. Newer agents commonly
used in cholescintigraphy (diisopropyl and m-bromotrimethyl
iminodiacetic acid) have generally overcome this limitation.
False negative results are uncommon since most patients with acute cholecystitis have
obstruction of the cystic duct. When they occur, they may be due to incomplete cystic
duct obstruction. Rear more

Feedback on your answer


Collapse
22)
0/1
A 41-year-old obese white female presents to the emergency room with severe epigastric
pain and right shoulder pain that has been on or off over the last month but now
constant over the last 2 hours associated with vomiting. She notes it comes on after eating
and normally goes away on its own after an hour but tonight it has persisted longer. Her
past medical history is significant for frequent heartburn for which she takes zantac and
tums. After examination and history the patient notes that her pain has resolved
completely. Which of the following answers best explains the source of her pain?
(Select 1)(1pts)

Acid hypersecretion

Viscus distention

Mucosal inflammation

Peritoneal irritation

Symptomatic, uncomplicated gallstone disease (biliary colic) is one of the most common
manifestations of gallstones and is due to contraction of the gallbladder against a
transiently obstructed cystic duct. Classic symptoms of biliary colic include right upper
quadrant or epigastric pain that occurs after eating (especially fatty meals) and lasts from
30 minutes to several hours. Abdominal pain may be accompanied by diaphoresis,
nausea, and vomiting. The most common complications related to gallstones include
acute cholecystitis, choledocholithiasis, and gallstone pancreatitis.
This is the classic presentation of biliary colic secondary to gallstones. Ingestion of a
fatty meal causes the gallbladder to contract, which can press gallstones against a cystic
duct opening, increasing intra-gallbladder pressure and causing distention colicky pain.
Subsequent gallbladder relaxation allows the stone to fall back from the duct, causing the
pain resolves completely. Biliary colic is often accompanied by nausea, vomiting, and
rate cited shoulder or sub scapular discomfort (referred pain). The pain of biliary colic is
distinguished that of acute cholecystitis by its intermittent nature in relation to meals as
well as the absence of fever.
2014. Scientific American Surgery. Hamilton, Ontario & Philadelphia, PA. Decker
Intellectual Properties Inc. ISSN 2368-2744. STAT!Ref Online Electronic Medical
Library. http://online.statref.com/Document.aspx?fxId=61&docId=930. 10/17/2014
4:39:25 AM CDT (UTC -05:00).

Feedback on your answer


Collapse
Unit 7
23)
1/1
In the setting of trauma, what is the recommended transfusion target goal?
(Select 1)(1pts)

5 g/dL

7 g/dl

9 g/dl

11 g/dL

The answer is B, Banked RBCs have been widely available since World War II, but the
indications for administration continue to be debated.15 Animals with hemorrhagic shock
were shown to have improved survival, with hemoglobin concentration ([Hb])
maintained in the range of 12 to 13 g/dL. However, animal models of isovolemic
hemodilution demonstrated that the optimum [Hb] for maintaining systemic oxygen
delivery (DO2) is 10 g/dL, but in healthy human volunteers, isovolemic hemodilution is
tolerated at concentrations as low as 5 g/dL. In the not so distant past, clinical studies in
critically ill patients concluded that 10 g/dL hemoglobin was optimum for shock
resuscitation, but more recent consensus panels have judged that a lower concentration is
adequate. The American Society of Anesthesiologists recommends maintaining
hemoglobin at greater than 6 g/dL, whereas the NIH recommends a concentration of
greater than 7 g/dL. A PRCT showed that ICU patients randomized to restrictive blood
transfusions (transfuse if [Hb] < 7 g/L and maintain between 7 and 9 g/dL) did as well
and possibly better than patients who were liberally transfused (transfuse if < 10 g/dL and
maintain between 10 and 12 g/dL).80 We and others have reported that transfusion of
PRBCs is a strong risk factor, independent of shock severity, for adverse outcomes (i.e.,
infections, MOF, and deaths) in trauma patients (a patient population most likely to
benefit from PRBC transfusion).81 The observation that transfusion of PRBCs is
associated with adverse outcomes has now been consistently observed across a broad
spectrum of high-risk hospitalized patients.82 The choice of transfusion trigger is
dependent on the clinical scenario. During active shock resuscitation with ongoing
bleeding, maintaining for a higher [Hb] is preferred. Blood is a good volume expander
and will thus limit isotonic crystalloid infusions, and the higher [Hb] provides a margin
of safety and a higher [Hb] level may promote hemostasis.83 Once bleeding is controlled,
the lower transfusion trigger should be used unless there is evidence of high oxygen
extraction (i.e., SaO2 minus mixed venous hemoglobin oxygen saturation < 60%). Read
more
Feedback on your answer
Collapse
24)
0/1
Oozing from a surgical wound in a patient who has an adequate platelet count and normal
coagulation parameters may be a signal of platelet dysfunction. How many days does it
take for sufficient numbers of new platelets to be formed such that the effect of aspirin
and clopidogrel lapses, respectively?
(Select 1)(1pts)

1 day/5 days

5 days/10 days

5 days/1 day

10 days/ 5 days

The answer is D, 10 days for aspirin/ 5 days for clopidogrel. Oozing in a patient who has
an adequate platelet count and normal coagulation parameters may be a signal of platelet
dysfunction. The now-routine administration of acetylsalicylic acid (aspirin) and
clopidogrel (Plavix) to reduce the risk of myocardial infarction and stroke has led to a
rise in the incidence of drug-induced platelet dysfunction. Aspirin causes irreversible
platelet dysfunction through the cyclooxygenase pathway. It takes approximately 10 days
for sufficient numbers of new platelets to be formed such that the effect of aspirin lapses;
however, because the half-life of aspirin is on the order of 2 to 5 hours, new platelet
transfusions should be unaffected if at least a day has passed since the last use of aspirin.
The platelet dysfunction caused by other nonsteroid al antiinflammatory drugs (e.g.,
ibuprofen) is reversible and consequently does not generally last as long as that caused by
aspirin.
Newer platelet-blocking agents have been found to be effective in improving outcome
after coronary angioplasty.17 These drugs function through different mechanisms. Some,
such as abciximab (ReoPro), eptifibatide (Integrilin), and tirofiban (Aggrastat), block the
platelet surface receptor glycoprotein (GP) IIb-IIIa, which binds platelets to fibrinogen.
Others, such as clopidogrel, ticlopidine (Ticlid), and prasugrel (Effient), target the P2Y12
receptor (also known as the adenosine diphosphate [ADP] receptor). Clopidogrel, the
most commonly used among this group, permanently prevents the association of P2Y12
receptors into functional oligomeric complexes. As a result, there is no way to inactivate
this medication if the patient experiences bleeding during its therapeutic window, which
typically lasts about 5 days from the last dose. In the face of life-threatening bleeding
during this interval, it is reasonable to administer platelets based on the rationale that, in
large-enough quantities, they may eventually bind all of the active metabolite. Some of
the newest drugs in the P2Y12 inhibitor class (which have not yet been evaluated for use
in the United States), for example, ticagrelor, are reversible, which may help avoid such
difficulties in the future. Read more
Feedback on your answer
Collapse
25)
0/1
When geographically and logistically feasible, critically injured patients should be taken
directly to a designated level I trauma center or to a level II trauma center if a level I
trauma center is more than 30 minutes away. The currently available field trauma scores,
however, are not reliable in making this triage decision. Because the potential harm
associated with undertriage is high and could result in death or substantial morbidity and
disability, criteria were initially chosen to err on the side of minimizing undertriage rather
than minimizing overtriage. What are the target levels for undertriage rates within a
trauma system to a level I or II trauma center?
(Select 1)(1pts)

<5%

5-10%

10-15%

15-20%

The answer is A, <5%. When geographically and logistically feasible, critically injured
patients should be taken directly to a designated level I trauma center or to a level II
trauma center if a level I trauma center is more than 30 minutes away. The currently
available field trauma scores, however, are not reliable in making this triage decision.
The development of field triage criteria has paralleled the development of trauma centers.
The decision is based on a practice algorithm called a "decision scheme." The first field
triage decision scheme was published by the American College of Surgeons in 1986, with
subsequent updates in 1990, 1993, 1999, 2006, and 2011.6 The decision scheme has four
steps that the emergency medical service (EMS) provider proceeds through after
measuring vital signs and level of consciousness to decide if the patient should be
transported to a higher-level trauma center [see Table 2]. Because the potential harm
associated with undertriage is high and could result in death or substantial morbidity and
disability, criteria were initially chosen to err on the side of minimizing undertriage rather
than minimizing overtriage. However, overtriage results in an overuse of financial and
human resources, overcrowding of trauma centers, and increased EMS transport times.
Target levels for undertriage rates within a trauma system range from 0 to 5% of patients
requiring level I or level II trauma center care, whereas levels of overtriage vary from 25
to 50%. As field triage continues to evolve on the basis of new research findings,
overtriage rates might be reduced while maintaining low undertriage rates. Read more
Feedback on your answer
Collapse

Unit 1
1)
0/1
You are seeing a 22-year-old marine in the emergency department following a motor
vehicle crash after his car slid off an icy road into a telephone pole. He was the restrained
driver, and the air bag inflated. He was awake and talking when EMS arrived after
suffering a brief loss of consciousness and need to be extracted from the vehicle. His is
currently complaining of mild right flank and lower back pain.

His current vitals show a BP of 120/71, HR 94, RR 14, O2 sat of 100% on room air. He is
awake, alert and his GCS is 15. On physical exam there are multiple contusions over the
anterior chest; there is also tenderness to palpation over the right lower chest wall. His
abdomen is flat, soft, and non-tender. There is no midline tenderness to his spine and no
evidence of flank ecchymosis. His labs show his complete blood count, serum
concentrations of electrolytes, urea nitrogen, and creatinine are all within the reference
range. Urine toxicology screening is negative. His urine is red and urinalysis shows
>180 RBC/hpf but no WBCs. Which of the following is the most appropriate next step in
management?
(Select 1)( 1pts extra credit)

CT scan of the abdomen and pelvis with IV contrast

Magnetic resonance arteriography of the renal arteries

To the OR for Exploratory laparotomy

DPL (Diagonostic Peritoneal Lavage)

The answer is A, CT scan of the abdomen and pelvis with IV contrast. This patient has
evidence of trauma to his kidneys and a CT scan in IV contrast is best to evaluate his
kidneys for trauma. An ultrasound Fast exam could also have been done initially as well
but the CT scan is a more definitive study and more prognostic. Read More
Feedback on your answer
Collapse
2)
1/1
You are rounding on 62-year-old male who status post an elective AAA repair. He was
given 3 units of packed red blood cells during surgery and about an hour after he
developed fever and chills. He received one dose of prophylactic antibiotics before the
surgery. Currently his oral temperature is 101.3°F, blood pressure is 124/80, pulse rate
88 beats/min, and respirations are 14/minute. Physical examination shows a mildly tender
wound without induration or redness and his lungs are clear. He has a Foley catheter that
was placed at the time of surgery. What is the most likely cause of his fever?

(Select 1)( 1pts extra credit)

Drug fever

Urinary Catheter associated infection

Deep Venous Thrombosis

Transfusion reaction

The answer is D. Acute transfusion reactions range from bothersome yet clinically benign
to life-threatening reactions. The nature of the reaction may not be immediately apparent,
because many reactions begin with nonspecific symptoms such as fever or chills. In
addition, patients receiving transfusions often have complex underlying clinical
conditions, the symptoms of which may mimic a transfusion reaction. Thus, a patient
experiencing symptoms or signs consistent with an acute transfusion reaction must be
evaluated promptly, with input from the transfusion service, and treated as expeditiously
as possible to minimize the impact of the reaction.
The time course of this fever is to acute to be attributable to DVT, infection, hematoma or
drugs. The patient is most likely suffering from an acute febrile nonhemolytic transfusion
reaction. Acute transfusion reactions classically occur during or within a few hours of
completion of the transfusion. Classically patients with acute febrile nonhemolytic
transfusion reactions experienced an increase in temperature or at least 1°C accompanied
by rigors. This is an immune mediated phenomenon mediated by host antibodies that
bind to donor cells causing the activation of complement components and release of
inflammatory cytokines. The condition worsens, with continued transfusion of the
offending blood product. Treatment is with discontinuation of the products and
administration of antipyretics. Read more from Up to date

Feedback on your answer


Collapse
3)
0/1
A 54-year-old white male presents to your clinic with a 3-month history of hoarseness
and nocturnal wheezing. On further questioning, he tells you that she has to clear his
throat repeatedly. What do these symptoms most likely represent?
(Select 1)( 1pts extra credit)

Thyroid nodule

Gastroesophageal reflux disease

Tracheal stenosis

Sinusitis

Allergies

Acid laryngitis is a group of respiratory symptoms related to gastroesophageal reflux


disease. The symptoms of hoarseness (especially in the morning), a repeated need to clear
the throat, and nocturnal or early morning wheezing may occur singly or in varying
combinations, and are believed to be caused by gastric contents irritating the larynx and
hypopharynx. Thyroid disease, sinusitis, and tracheal stenosis can produce one or more of
the symptoms described, but not all of them.

Ref: Feldman M, Friedman LS, Brandt LJ: Sleisenger and Fordtran’s


Gastrointestinal and Liver Disease, ed 9. Saunders, 2010, pp 179-181.

Feedback on your answer


Collapse
4)
0/1
You are seeing a 63-year old white female on morning rounds after she was admitted last
night for acute cholecystitis. She has a planned cholecystectomy later today but her
platelets this am are 20,000 k/mcL. You plan on giving her a transfusion prior to surgery
and are in discussion with the general surgeon. A transfusion of a six pack of platelets
can be expected to raise the platelet count by which of following?

(Select 1)( 1pts extra credit)

5,000-10,000

10,000-20,000

20,000-40,000
40,000-60,000

The answer is D. Platelet transfusions tend to be dosed as a “6-pack” of platelets, which


contains 6 units of platelet concentrate from multiple donors or a single apheresis unit; a
transfusion can be expected to raise the platelet count by 40–60,000 (Emerg Med Clin
North Am;32(3):727-38).
Feedback on your answer
Collapse
5)
0/1
You are seeing a 31-year old native American female who is currently 24 weeks pregnant
and has a history of chronic hypertension in your clinic with a complaint of upper
abdominal pain over the last few days. The pain is intermittent, worse after she eats and
generally lasts between 30 min to an hour. You are suspecting gall stones and discuss
with the patient about ordering an ultrasound to confirm. Which of the following
is NOT a risk factor for cholesterol gallstones?

(Select 1)( 1pts extra credit)

Obesity

Pregnancy

Hypertension

Native American ancestry

Hypertension is not a risk factor. Cholesterol gallstones are associated with female sex,
European or Native American ancestry, and increasing age. Other risk factors include
obesity, pregnancy, and gallbladder stasis.
For more on the risk factors for gallstones, read here.

Feedback on your answer


Collapse
Unit 2
6)
0/1
What is the most common surgical cause of abdominal pain in adults?
(Select 1)(1pts)
Appendicitis

Cholecystitis

Small bowel obstruction

Perforated peptic ulcer

All of the above

None of the above

The answer is A, appendicitis. The value of detailed epidemiologic knowledge


notwithstanding, it is worthwhile to keep in mind the truism that common things are
common. Regarding which things are common, the most extensive information currently
available comes from the ongoing survey begun in 1977 by the Research Committee of
the OMGE. As of the last progress report on this survey, which was published in 1988,
more than 200 physicians at 26 centers in 17 countries had accumulated data on 10,320
patients with acute abdominal pain [see Table 3].23 The most common diagnosis in
these patients was nonspecific abdominal pain (NSAP)—that is, the retrospective
diagnosis of exclusion in which no cause for the pain can be identified.24,25 NSAP
accounted for 34% of all patients seen; the four most common diagnoses accounted for
more than 75%. The most common surgical diagnosis in the OMGE survey was acute
appendicitis, followed by acute cholecystitis, small bowel obstruction, and gynecologic
disorders. Relatively few patients had perforated peptic ulcer, a finding that confirms the
current downward trend in the incidence of this condition. Cancer was found to be a
significant cause of acute abdominal pain. There was little variation in the geographic
distribution of surgical causes of acute abdominal pain (i.e., conditions necessitating
operation) among developed countries. In patients who required surgery, the most
common causes were acute appendicitis (42.6%), acute cholecystitis (14.7%), small
bowel obstruction (6.2%), perforated peptic ulcer (3.7%), and acute pancreatitis
(4.5%).23 The OMGE survey's finding that NSAP was the most common diagnosis in
patients with acute abdominal pain has been confirmed by several srudies12,13,25; the
finding that acute appendicitis, cholecystitis, and intestinal obstruction were the three
most common diagnoses in patients with acute abdominal pain who require operation has
also been amply confirmed [see Table 3].1,12,13

Table 3. Frequency of Specific Diagnoses in Patients with Acute Abdominal Pain

Frequency in Individual Studies (% of Patients)

Diagno OMGE23( Wilso Irvin13( Brew de Hawthorn83


sis N= n et N= er et Dombal1( (N = 496)
10,320) al82(N 1,190) al12(N N = 552)
= =
1,196 1,000
) )

Nonspecifi
c
34.0 45.6 34.9 41.3 50.5 36.0
abdominal
pain

Acute
appendicit 28.1 15.6 16.8 4.3 26.3 14.9
is

Acute
cholecysti 9.7 5.8 5.1 2.5 7.6 5.9
tis

Small
bowel
4.1 2.6 14.8 2.5 3.6 8.6
obstructio
n

Acute
gynecolog 4.0 4.0 1.1 8.5 — —
ic disease

Acute
pancreatiti 2.9 1.3 2.4 — 2.9 2.1
s

Urologic
2.9 4.7 5.9 11.4 — 12.8
disorders

Perforated
pepticulce 2.5 2.3 2.5 2.0 3.1 —
r

Cancer 1.5 — 3.0 — — —

Diverticula
1.5 1.1 3.9 — 2.0 3.0
r disease

Dyspepsia 1.4 7.6 1.4 1.4 — —

Gastroenter
— — 0.3 6.9 — 5.1
itis
Inflammato
ry bowel — — 0.8 — — 2.1
disease

Mesenteric
— 3.6 — — — 1.5
adenitis

Gastritis — 2.1 — 1.4 — —

Constipatio
— 2.4 — 2.3 — —
n

Amebic
hepatic 1.2 — 1.9 — — —
abscess

Miscellane
6.3 1.3 5.2 15.5 4.0 8.0
ous

OMGE = World Organization of Gastroenterology.


2014. Scientific American Surgery. Hamilton, Ontario & Philadelphia, PA. Decker
Intellectual Properties Inc. ISSN 2368-2744. STAT!Ref Online Electronic Medical
Library. http://online.statref.com/Document.aspx?fxId=61&docId=803. 10/16/2014
1:15:18 PM CDT (UTC -05:00).

Feedback on your answer


Collapse
7)
0/1
Which of the following is not an example of referred pain?
(Select 1)(1pts)

Ureteral stone causes ipsilateral testicular pain

Ruptured hemorrhagic cyst causing lower back pain

Acute MI causing jaw pain

Acute Appendicitis causing periumbilical pain

All of the above


None of the above

The answer is D, as Acute Appendicitis causing periumbilical pain is an example of


visceral pain.
Visceral Pain
•Caused by the stretching of fibers innervating the walls or capsules of hollow or solid
organs
•A steady ache or vague discomfort to excruciating or colicky
•Visceral afferents follow a segmental distribution and pain is localized by the sensory
cortex to an approximate spinal cord level determined by embryological organ of the
organ involved.
•Biliary, duodenum, stomach produce epigastric pains
•Most small bowel, appendix, cecum cause periumbilical pain
•Hindgut structures, such as the bladder, distal two-thirds of the colon, and pelvic
genitourinary (GU) organs, usually cause pain in the suprapubic region.
•Pain is often reported in the back for retroperitoneal structures, such as the aorta and
kidneys
•Deep musculoskeletal structures (especially of the back) are innervated by visceral
sensory fibers with similar qualities to those arising from intra-abdominal organs.
•Intraperitoneal organs are bilaterally innervated causing pain to be located in the midline
regardless of which side it is coming from. Classic example is early appendicitis causing
periumbilical pain

Visceral Pain
•Caused by the stretching of fibers innervating the walls or capsules of hollow or solid
organs
•A steady ache or vague discomfort to excruciating or colicky
•Visceral afferents follow a segmental distribution and pain is localized by the sensory
cortex to an approximate spinal cord level determined by embryological organ of the
organ involved.
•Biliary, duodenum, stomach produce epigastric pains
•Most small bowel, appendix, cecum cause periumbilical pain
•Hindgut structures, such as the bladder, distal two-thirds of the colon, and pelvic
genitourinary (GU) organs, usually cause pain in the suprapubic region.
•Pain is often reported in the back for retroperitoneal structures, such as the aorta and
kidneys
•Deep musculoskeletal structures (especially of the back) are innervated by visceral
sensory fibers with similar qualities to those arising from intra-abdominal organs.
•Intraperitoneal organs are bilaterally innervated causing pain to be located in the midline
regardless of which side it is coming from. Classic example is early appendicitis causing
periumbilical pain
Feedback on your answer
Collapse
8)
0/1
You are seeing a 32-year old male who presents after being kicked multiple times in his
right flank. He is complaining of severe flank pain and gross hematuria. What is the gold
standard for imaging of renal trauma?
(Select 1)(1pts)

MRI abdomen/pelvis

Ultrasound

CT abdomen/pelvis with contrast

Intravenous pyelography

None of the above

The answer is C, CT abdomen/pelvis with contrast. CT scanning with intravenous


contrast enhancement is the modality of choice for the identification and staging of renal
trauma in the hemodynamically stable patient (image 6). The initial CT images will
frequently miss injuries to the renal pelvis and ureters as sufficient contrast may not yet
be present in the collecting system. Additional delayed images are needed to assess for
contrast extravasation when these injuries are suspected (image 7). In patients
undergoing CT scanning of other organ systems, suspected bladder injuries may be
investigated with CT cystography after retrograde filling of the bladder, as described
above. Read more
 CT with IV contrast is the Gold standard, high sensitivity
 Immediate and delayed post-contrast images to view collecting system
 Images abdomen and retroperitoneum
 Allows diagnosis and staging
 Intravenous pyelography doesnt allow you to image the abdomen or
retroperitoneum.

Feedback on your answer


Collapse
Unit 3
9)
1/1
Which of the following is NOT associated with an increased risk of esophageal
adenocarcinoma?
(Select 1)(1pts)
Cigarette smoking

Gastroesophageal reflux disease

Obesity

Helibactor pylori infection

All of the above

None of the above

Answer D is not correct. Esophageal cancer has two main subtypes — esophageal
squamous-cell carcinoma and esophageal adenocarcinoma; their precursor lesions are
esophageal squamous dysplasia and Barrett’s esophagus, respectively. Although
squamous-cell carcinoma accounts for about 90% of cases of esophageal cancer
worldwide, the incidence of and mortality rates associated with esophageal
adenocarcinoma are rising and have surpassed those of esophageal squamous-cell
carcinoma in several regions in North America and Europe. In the United States, more
than 18,000 new cases of esophageal cancer and more than 15,000 deaths from
esophageal cancer were expected in 2014. Esophageal carcinoma is rare in young people
and increases in incidence with age, peaking in the seventh and eighth decades of life.
The main risk factors for esophageal adenocarcinoma are gastroesophageal reflux
disease, obesity, and cigarette smoking; H. pyloriinfection is associated
with a reduced risk. Cigarette smoking and alcohol consumption constitute the
main risk factors for esophageal squamous-cell carcinoma. High intake of red meats, fats,
and processed foods is associated with an increased risk of both types of esophageal
cancer, whereas high intake of fiber, fresh fruit, and vegetables is associated with a lower
risk.
N Engl J Med 2014; 371:2499-2509December 25, 2014DOI:
10.1056/NEJMra1314530

Feedback on your answer


Collapse
10)
0/1
You are consulted to see a 61-year-old male because of continuing symptoms due to
refractory gastroesophageal reflux that have not improved despite lifestyle
modifications and treatment with a twice daily proton pump inhibitor (omeprazole 40 mg
daily). He notes that he continues to have occasional chest pain and epigastric burning
with eating. He denies dysphagia, regurgitation, weight loss, or a change in bowel habits.
He denies other past medical history and has no cardiac risk factors. Vital signs and
physical examination are normal. His upper endoscopy shows no ulcers and is otherwise
unremarkable. Which of the following is the most appropriate treatment at this time?
(Select 1)(1pts)

Schedule consultation for evaluation for antireflux surgery

Increase the proton pump inhibitor to 3 times daily

Change to a different proton pump inhibitor

Add ranitidine at bedtime to the current regimen

The answer is A. Consider surgery as an option for patients with well-documented GERD
who require long-term PPI maintenance therapy but show satisfactory relief of symptoms
and who:
 Are older than 50 years
 Consider long-term medication a financial burden
 Are noncompliant with drug therapy
 Prefer a single surgical intervention to long-term drug treatment
 Experience prominent symptoms of regurgitation, even with medical
control of heartburn symptoms
Nissen fundoplication is the most common surgical intervention for GERD. This
procedure aims to restore the physiology and anatomy of the gastroesophageal junction
by wrapping the gastric fundus around the distal esophagus. The FDA has also approved
several endoscopic procedures for treatment of GERD, including endoscopic suturing and
radiofrequency ablation of the lower esophageal sphincter. Read More

Feedback on your answer


Collapse
11)
0/1
You are seeing a 44-year-old female in your outpatient GI clinic for persistent acid reflux
symptoms. She was on zantac for 4 weeks following her last EGD and then switched to
protonix daily for 2 months which did not seem to help. Her records indicate a normal
upper endoscopy 4 months ago as she was complaining of some mild dysphagia and
biopsies were negative for h. pylori. She otherwise has no other past medical history and
takes no other medications, denies alcohol ingestion and is not a smoker. Vital signs and
physical examination are normal. Which of the following studies should be done next to
confirm this patient's most likely diagnosis?
(Select 1)(1pts)

Repeat H. pylori testing


Barium swallow radiography

Esophageal manometry

Ambulatory esophageal pH-impedance monitoring

The answer is Ambulatory esophageal pH-impedance monitoring, This patient most


likely has gastroesophageal reflux disease (GERD), and the gold standard for diagnosing
GERD is 24-hour esophageal pH-impedance monitoring, which establishes whether
increased esophageal exposure to acid is present. This study is most helpful in patients
who have GERD-like symptoms that fail to respond to an adequate trial of a proton pump
inhibitor (PPI) to correlate the presence of acid in the esophagus at the time the patient is
having symptoms, such as the patient described. Esophageal pH monitoring identifies the
reflux of acid, and impedance monitoring detects reflux of other gastric contents in a
small percentage of patients who have symptoms related to non-acid reflux.
GERD is definitively diagnosed by symptoms, upper endoscopy, or ambulatory
esophageal pH/impedance monitoring. However, the presence of heartburn, regurgitation,
or both is sufficient to presumptively diagnose GERD if symptoms are of sufficient
severity and frequency to be troublesome for the patient. A favorable response to empiric
therapy with PPIs is supportive evidence for a diagnosis of GERD and is a reasonable
first step in a patient without alarm symptoms (dysphagia, anemia, vomiting, or weight
loss). Kumar AR, Katz PO. Functional esophageal disorders: a review of diagnosis and
management. Expert Rev Gastroenterol Hepatol. 2013;7:453-61. PMID: 23899284

Feedback on your answer


Collapse
Unit 4
12)
0/1
You are seeing a 45-year old male in clinic who has a past medical history of
osteoarthritis for which he takes daily alleve. He denies tobacco use or alcohol use. He
has been having recent early satiety, bloating and epigastric discomfort over the last
month. You suspect peptic ulcer disease. Which of the following ulcer locations is more
associated with H. Pylori infection?
(Select 1)(1pts)

Duodenal

Gastric

Esophageal
Jejunal

None of the above

The answer is A, duodenal ulcers. H. pylori, a gram-negative, helical, rod-


shaped bacterium, colonizes the gastric mucosa of approximately one-half of the world
population and an estimated 30% to 40% of the U.S. population. H. pylori is present in
95% of patients with duodenal ulcers and in 70% of those with gastric ulcers. It is
typically transmitted via the fecal-oral route during early childhood and persists for
decades. The bacterium is a known cause of gastric and duodenal ulcers and is a risk
factor for mucosa-associated lymphoid tissue (MALT) lymphoma and gastric
adenocarcinoma. Am Fam Physician. 2015 Feb 15;91(4):236-242.

Feedback on your answer


Collapse
13)
0/1
Which of the following statements about H. pylori is not true?
(Select 1)(1pts)

The serologic determination of anti-H. pylori IgG antibodies in peripheral


blood has high sensitivity (90 to 100%)

Urea breath testing has a sensitivity of 88 to 95% and a specificity of 95


to 100% for establishing the presence of H. pylori infection1

Anti-H. pylori IgG antibodies is the test of choice for confirming successful
eradication of the organism 4 to 6 weeks after the cessation of antibiotic
treatment

Histologic examination of tissue samples remains the gold standard for


determining the presence of H. pylori

Routine culture of the gastric biopsy specimens for H. pylori is not


recommended

Answer C is the only false statement. Urea breath testing is the test of choice for
confirming successful eradication of the organism 4 to 6 weeks after the cessation of
antibiotic treatment. Routine laboratory tests are of limited value in evaluating patients
with uncomplicated PUD. When ZES is suspected, a fasting serum gastrin level may be
useful. However, laboratory tests do play an important role in the diagnosis of H. pylori
infection. The serologic determination of anti-H. pylori IgG antibodies in peripheral
blood has high sensitivity (90 to 100%) but variable specificity (76 to 96%). Serology is
preferred for initial diagnosis when endoscopy is not required. Urea breath testing has a
sensitivity of 88 to 95% and a specificity of 95 to 100% for establishing the presence of
H. pylori infection12 and is the test of choice for confirming successful eradication of the
organism 4 to 6 weeks after the cessation of antibiotic treatment. Although endoscopy is
not indicated solely for the purpose of establishing the presence of H. pylori, gastric
biopsy specimens can be used in the diagnosis of H. pylori by rapid urease test, histology,
and bacterial culture [see Figure 2]. Rapid urease testing of the biopsy specimen is the
preferred method when histopathology is not required because it is fast (results within
hours), is less expensive, and has a sensitivity of 90 to 95% and a specificity of 95 to
100%. Histologic examination of tissue samples remains the gold standard for
determining the presence of H. pylori and can also provide additional information
regarding the presence of gastritis, intestinal metaplasia, or mucosa-associated lymphoid
tissue. Figure 1. Upper gastrointestinal series in which double contrast (barium and air) is
used, showing rounded collection of barium in an ulcer in the duodenal bulb of a patient
presenting with dyspepsia (uncomplicated duodenal ulcer). Routine culture of the gastric
biopsy specimens for H. pylori is not recommended as it is relatively expensive, and
diagnosis requires more time (3 to 5 days). However, the incidence of antimicrobial
resistance is high in patients with refractory peptic ulcers, and they may benefit from
culture and antibiotic sensitivity testing. Read more
Feedback on your answer
Collapse
14)
0/1
You are seeing a 44-year old female who presents with worsening epigastric pain
following meals and "heartburn" that has been getting more persistent over the last few
days. She has a past medical history of hypertension. Her only medication is naprosyn
500 mg BID and HCTZ 25mg daily. On exam she has mild epigastric tenderness to
palpation. Her rectal exam shows her stool is brown but guaiac positive for blood. Her
portable upright chest xray shows no signs of free air or perforation.Based on her history
of current NSAID use, where would you likely expect the location of her ulcer?

(Select 1)(1pts)

Gastric

Duodenal

Jejunal

Esophageal

NSAID use confers around a 40 fold increase in the development of gastric ulcers and an
8 fold increase risk for the development of duodenol ulcers and . Therefore in this patient
you would expect gastric ulcer more likely than a duodenal ulcer.
The use of NSAIDs is the most commonly identified risk factor for peptic ulcer bleeding,
especially in the elderly. Studies have found relative risks for bleeding ranging from 2.7
to 33.9 [11]. Studies have also shown that the risk is drug-specific and dose-dependent.
As an example, in a study of 2777 patients, the overall relative risk (RR) of bleeding
associated with NSAID use was 5.3 (95% CI 4.5-6.2). However, the risk varied by drug
and was lowest for aceclofenac (RR 3.1, 95% CI 2.3-4.2) and was highest for ketorolac
(RR 14.4, 95% CI 5.2-39.9) [18]. The risk was higher in patients taking high-dose
NSAIDs compared with those taking medium- or low-dose NSAIDs (RR 6.8, 95% CI
5.3-8.8 versus 4.0, 95% CI 3.2-5.0). There was also an increased risk of bleeding with
aspirin use (RR 5.3) that again was dose-dependent (RR 7.5 with 500 mg per day versus
2.7 with 100 mg per day). The concurrent use of aspirin and NSAIDs conferred an even
greater risk of bleeding than was seen with either agent alone (RR 12.7). Finally, the risk
was highest in the first 30 days of NSAID use, with a RR of 7.6 (95% CI 6.0-9.5). The
risk remained high between days 31 and 90 days (RR 7.3, 95% CI 4.0-13.2), but dropped
after 91 days (RR 2.6, 95% CI 1.6-4.1).
NSAID use has also been identified as a risk factor for ulcer perforation [16,19]. In a
study of 176 patients from Spain, NSAID use was the only risk factor that was
significantly associated with perforation (odds ratio [OR] 3.6). Read more
Feedback on your answer
Collapse
15)
0/1
A 48-year old black male presents to you with sudden onset of vomiting and diffuse
abdominal pain that hurts to breathe two hours prior to arrival. He denies past medical
history and his only medications are ibuprofen for chronic right shoulder pain. He smokes
1 pack per day and occasionally drinks alcohol.
His vitals show an oral temperature of 99.3 F, heart rate of 129 beats/min, respiratory rate
20/min, blood pressure 151/100, O2 saturation of 98% on room air.
Physical exam shows him to be uncomfotable. Cardiac exam shows tachycardia, his
lungs are clear to auscultation, and abdominal exam shows diffuse tenderness to light
palpation greatest in the right upper quadrant along with guarding. Rectal exam shows
heme positive brown stool. An upright chest x-ray which is shown below. What is the
diagnosis?
(Select 1)(1pts)

Pneumomediastinum

Pneumoperitoneum

Intestinal Malrotation

Small Bowel Obstruction

Pneumothorax

The answer is B, Pneumoperitoneum which describes gas within the peritoneal cavity,
and is often the harbinger of a critical illness. The most common cause of a
pneumoperitoneum is from the disruption of the wall of a hollow viscus. Read more
An erect chest x-ray is probably the most sensitive plain radiograph for the detection of
free intraperitoneal gas. If a large volume pneumoperitoneum is present, it may be
superimposed over normal aerated lung with normal lung markings.
 subdiaphragmatic free gas
 cupola sign (in supine film)
Free gas within the peritoneal cavity can be detected on an abdominal radiograph. These
signs can be further divided by anatomical compartments in relation to the
pneumoperitoneum:
 bowel related signs
 double wall sign (also known as Rigler's sign or bas-relief
sign)
 telltale triangle sign (also known as triangle sign)
 peritoneal ligament related signs
 football sign
 falciform ligament sign
 lateral umbilical ligament sign (also known as inverted
"V" sign)
 urachus sign
 right upper quadrant signs
 lucent liver sign
 hepatic edge sign
 fissure for ligamentum teres sign
 Morison's pouch sign
 cupola sign
The patient above likely has a perforated ulcer from underlying NSAID use. Ulcer
perforation should be suspected in patients who suddenly develop severe, diffuse
abdominal pain. Perforations complicate 2 to 10 percent of peptic ulcers [70]. Duodenal,
antral, and gastric body ulcers account for 60, 20, and 20 percent of perforations due to
peptic ulcer disease (PUD), respectively. If imaging is required, plain x-rays are typically
obtained first. Careful interpretation of upright chest and abdominal films can detect
diagnostic free air in many cases of perforated gastric and duodenal ulcers [76]. The
presence of free air on abdominal imaging is highly indicative of a perforated viscus
(image 1 and image 2 and image 3), although about 10 to 20 percent of patients
with a perforated duodenal ulcer will not have free air [76]. If free air is found, no other
diagnostic studies are necessary. Leakage of water soluble oral contrast may be useful in
selected cases. Once the oral contrast is given, the patient should be rotated 360 degrees
and placed on the right side to fill the antrum and duodenum with contrast. However,
many perforations have already sealed spontaneously by the time of presentation [77], so
the absence of a leak does not exclude the diagnosis of a perforated ulcer. Read More
Feedback on your answer
Collapse
16)
0/1
What is the most common complication of peptic ulcer disease?

(Select 1)(1pts)
Gastric outlet obstruction

Perforation

Bleeding

Cancer

The answer is B, Bleeding. Bleeding is the most common complication of peptic ulcer
disease requiring hospitalization. In a study of the National Inpatient Sample (NIS),
bleeding occurred in 73 percent, perforation in 9 percent, and obstruction in 3 percent
[4]. H. pylori, nonsteroidal antiinflammatory drugs (NSAIDs), and the use of low dose
aspirin are the most common etiologies of ulcer bleeding [7] and ulcer perforation [8-
10]. In observational studies, duodenal, antral/pyloric, and gastric body ulcers
account for 60, 20, and 20 percent of perforations, respectively [8,9]. Among these
complications, perforation had the highest mortality rate, followed by obstruction, then
hemorrhage.
Bleeding peptic ulcer — Upper gastrointestinal bleeding due to peptic ulcer disease
is a common indication for emergency management of peptic ulcer disease [11]. Most
patients with acute bleeding can be managed with fluid resuscitation and transfusion, acid
suppression therapy, and endoscopic intervention. For those who fail these efforts,
surgery may become necessary. The specific indications for surgery for patients with
bleeding peptic ulcer are reviewed separately.
Perforated peptic ulcer — Ulcer perforation may be suspected in patients with a
history consistent with peptic ulcer disease who develop the sudden onset of severe,
diffuse abdominal pain. Once a diagnosis of perforation is established, surgical
intervention is indicated.
Gastric outlet obstruction — Gastric outlet obstruction is the least frequent ulcer
complication in developed countries. Most cases are associated with duodenal or pyloric
channel ulceration; gastric ulceration accounts for only five percent of cases.
Surgical consultation should be obtained for patients with chronic partial gastric outlet
obstruction that is refractory to medical treatment, and those found to have complete
gastric outlet obstruction, or those readmitted with gastric outlet obstruction after recent
“successful treatment.” Surgery is indicated if the patient fails to respond to conservative
medical management and endoscopic therapy.
Surgical treatment of gastric outlet obstruction is almost never an emergency. Because
most patients present with some degree of malnutrition and frequently have electrolyte
imbalances (ie, hypokalemic, hypochloremic metabolic alkalosis secondary to vomiting
or nasogastric suctioning), it is important to correct any derangements and optimize the
patient’s medical status prior to proceeding with surgery. Failure to identify and correct
these issues increases perioperative morbidity.
It may also be reasonable to try and improve the patient’s overall nutritional status with
nutritional support, as these patients are frequently malnourished and can have significant
postoperative delayed gastric emptying due to their atonic stomach.
Many experienced gastric surgeons advocate preoperative nasogastric sump
decompression to decrease gastric dilation and, hopefully, gastric atony. Read more

Feedback on your answer


Collapse
Unit 5
17)
0/1
You are seeing a 27-year old male in the trauma bay shortly after being stabbed in this
back with a 6 cm buck knife by his disgruntled neighbor after an argument.
He arrived via EMS sitting up, awake and alert with a GCS of 15. His BP is 140/90, HR
120 and O2 sat is 85% on a non re-breather.
On physical exam there is a 3 cm non bleeding wound just left of his mid thoracic spine.
Upon auscultation there is decreased breath sounds on his left side. There are no signs of
tracheal deviation or JVD.
What is the next best step in management?
(1pts)

Immediately to the OR for surgical exploration

Upright Chest Xray followed by a chest tube

Immediate Needle thoracotomy on the left

Pericardiocentesis under US guidance

The answer is A, Upright Chest Xray followed by a chest tube. This patient is awake and
talking. He is stable enough for a chest xray followed by chest tube placement. A needle
thoracotomy to should be done if he has evidence of tension pneumothorax which would
be respiratory distress and hypotension. He will likely then need a chest tube and can be
further managed. Read More
Feedback on your answer
Collapse
18)
0/1
Roughly what percentage of patients with blunt liver injury initially managed non-
operatively will go on to need surgical intervention?
(1pts)
<10%

25%

50%

75%

The answer is A, During the last century, the management of blunt force trauma to the
liver has changed from observation and expectant management in the early part of the
1900s to mainly operative intervention, to the current practice of selective operative and
nonoperative management. A 2008 study by Tinkoff et al.4 showed that 86.3% of hepatic
injuries are now managed without operative intervention. The current reported success
rate of nonoperative management of hepatic trauma ranges from 82% to 100%. Most
blunt liver trauma (80% in adults, 97% in children) patients are currently treated
conservatively. The success of non-operative management depends upon proper selection
of the patient. The patients, who are managed non-operatively, usually have grade I and
II liver injuries, hemoperitoneum less than 900 ml and blood transfusion of less than 3
units. The contraindications to non-operative management include refractory
hypotension, signinficant fall in haematocrit, the extravasations of intravenous contrast
agent, expanding haematoma and grade IV and V liver injury on CECT abdomen. The
patients of grade III liver injuries need very close observation as they may require
surgical intervention during first 24 hours. The failure rate of non-operative management
is not more than 5% inmost studies. It seems that patients with grade VI injuries rarely
reaches to the hospital alive and are not salvageable. Therefore, such injuries are usually
documented on autopsy. Mortality from blunt hepatic trauma is about 5% and is related
to uncontrolled hemorrhage.
Interventional radiology may be needed to perform an angiogram and embolization for
bleeding or to percutaneously drain an abscess or b iloma. An endoscopic retrograde
cholangiopancreatogram (ERCP) and stent placement may be required for biliary leak.
Even when such complications of the liver injury develop, only 15% require operative
intervention. Hepatic artery angiography with embolization is an important tool for the
stable patient with contrast extravasation who is being managed nonoperatively. It can
also be invaluable for the postoperative patient who has been stabilized by perihepatic
packing or who has rebled after an initial period of stability. Angioembolization has a
greater than 90% success rate in the control of bleeding with a low risk of rebleeding and
a reduction in required volume of transfusion. Read More
Feedback on your answer
Collapse
19)
0/1
You are seeing a 42-year-old female following in the trauma bay following a high-speed
motor vehicle collision. She was was a restrained passenger and her car flipped over after
she lost control. She had to be cut out of the car and is complaining of shortness of breath
and upper abdominal pain.
She is awake and alert and her GCS is 15. Her pulse is 120/min, respirations are 28/min,
and blood pressure is 80/40 mm Hg. Breath sounds are decreased at the left lung base. An
x-ray of the chest shows opacification of the left lower lung field. You decide to place an
emergent chest tube that yields a small amount of air followed by greenish fluid. Which
of the following is the most appropriate next step in management?
(Select 1)(1pts)

CT scan of the abdomen and pelvis

CT scan of the chest

Thoracoscopy

Laparotomy

The answer is D. This patient suffered a ruptured diaphragm and chest tube placement
appears to have ruptured the bowel as evidence from the greenish fluid that was returned.
She needs an emergent laparotomy to control bleeding and repair her diaphragm. Read
More
Feedback on your answer
Collapse
Unit 6
20)
0/1
A 46-year obese black female presents to you with 2 days of worsening of right upper
abdominal pain and vomiting. She feels hot and cold chills and says that she sometimes
feels this pain after eating but now its much worse.
Vitals: Temp is 39.0 C, heart rate 125 beats/min, blood pressure 124/90. Physical exam
reveals inspiratory arrest upon palpation of her RUQ to deep palpation.
Her labs show a WBC of 17,000. Her ALT, AST, bilirubin, and lipase are within normal
limits. What is the most likely diagnosis?
(Select 1)(1pts)

Cholelithiasis

Choledocholithiasis
Klatskin Tumor

Acute Cholecystitis

The patient presents with the triad of acute cholecystitis which is fever, vomiting,
jaundice and RUQ pain. This triad has been reported to be present in as high 70% present
of cases to as little as 20% of cases. Acute cholecystitis is the result of cystic duct
obstruction leading to gallbladder edema, mucosal sloughing, and, potentially, ischemia.
Signs and symptoms of acute cholecystitis include right upper quadrant pain, nausea, and
vomiting. Unlike the self-limited pain associated with biliary colic, pain associated with
acute cholecystitis generally lasts more than 6 to 8 hours. In addition to the signs of local
inflammation (pain and tenderness in the right upper abdomen or epigastric area),
patients also exhibit signs of systemic inflammation, including fever, leukocytosis, and
elevated C-reactive protein. Mild elevations in bilirubin and liver enzymes may be
present. A positive Murphy sign on physical examination is suggestive of a diagnosis of
cholecystitis (likelihood ratio of 2.8). Necrosis of the gall-bladder wall can occur due to
prolonged inflammation and decreased gallbladder perfusion. Gallbladder perforation
occurs in 5 to 10% of patients with acute cholecystitis.
Renoylds pentad adds sepsis and altered mental status to the triad and is less common and
can be easily missed. A Klatskin tumor presents with painless jaundice. While she might
have cholelithiasis and choledocholithiasis her symptoms are more indicative of acute
cholecystitis. Some cases of acute cholecystitis have mildly elevated bilirubin but severe
elevation suggests either Choledocholithiasis, cholangitis, or obstruction of the CBD.
2014. Scientific American Surgery. Hamilton, Ontario & Philadelphia, PA. Decker
Intellectual Properties Inc. ISSN 2368-2744. STAT!Ref Online Electronic Medical
Library. http://online.statref.com/Document.aspx?fxId=61&docId=942. 10/20/2014
8:07:24 PM CDT (UTC -05:00).

Feedback on your answer


Collapse
21)
1/1
You are seeing a 64-year old black female who is presenting with 2 days of worsening
upper abdominal pain that has become persistent. She denies fever at home but has a n
oral temp of 38.0 (100.4 F) in the ER. Her BP is 94/50, HR is 122 bpm and respiratory
rate 16/min. On exam she appears slightly jaundiced in her sclera, she has a positive
murphy's sign with some involuntary guarding. Her WBC comes back at 20k with 11%
bands. Her BMP is unremarkable. What triad of symptoms is classically associated with
cholangitis?
(Select 1)(1pts)
RUQ pain, fever,
bandemia

Epigastric pain,
fever,
leukocystosis

Bandemia, fever,
jaundice

RUQ pain, fever,


jaundice

Charcot's triad

http://www.ncbi.nlm.nih.gov/pmc/articles/PMC3094361/

The answer is D. The classic presentation of acute cholangitis is fever, abdominal pain,
and jaundice (Charcot's triad), though only 50 to 75 percent of patients with acute
cholangitis have all three findings. Confusion and hypotension can occur in patients with
suppurative cholangitis, producing Reynolds pentad, which is associated with significant
morbidity and mortality.
 Fever is present in approximately 90% of cases.
 Abdominal pain and jaundice is thought to occur in 70% and 60% of
patients, respectively.
 Patients present with altered mental status 10-20% of the time and
hypotension approximately 30% of the time. These signs, combined with
Charcot's triad, constitute Reynolds pentad.
 Consequently, many patients with ascending cholangitis do not present
with the classic signs and symptoms.[4]
 Most patients complain of RUQ pain; however, some patients (ie, elderly
persons) are too ill to localize the source of infection.
Read more from Up to Date

Feedback on your answer


Collapse
22)
1/1
A 41-year-old obese white female presents to the emergency room with severe epigastric
pain and right shoulder pain that has been on or off over the last month but now
constant over the last 2 hours associated with vomiting. She notes it comes on after eating
and normally goes away on its own after an hour but tonight it has persisted longer. Her
past medical history is significant for frequent heartburn for which she takes zantac and
tums. After examination and history the patient notes that her pain has resolved
completely. Which of the following answers best explains the source of her pain?
(Select 1)(1pts)

Acid hypersecretion

Viscus distention

Mucosal inflammation

Peritoneal irritation

Symptomatic, uncomplicated gallstone disease (biliary colic) is one of the most common
manifestations of gallstones and is due to contraction of the gallbladder against a
transiently obstructed cystic duct. Classic symptoms of biliary colic include right upper
quadrant or epigastric pain that occurs after eating (especially fatty meals) and lasts from
30 minutes to several hours. Abdominal pain may be accompanied by diaphoresis,
nausea, and vomiting. The most common complications related to gallstones include
acute cholecystitis, choledocholithiasis, and gallstone pancreatitis.
This is the classic presentation of biliary colic secondary to gallstones. Ingestion of a
fatty meal causes the gallbladder to contract, which can press gallstones against a cystic
duct opening, increasing intra-gallbladder pressure and causing distention colicky pain.
Subsequent gallbladder relaxation allows the stone to fall back from the duct, causing the
pain resolves completely. Biliary colic is often accompanied by nausea, vomiting, and
rate cited shoulder or sub scapular discomfort (referred pain). The pain of biliary colic is
distinguished that of acute cholecystitis by its intermittent nature in relation to meals as
well as the absence of fever.
2014. Scientific American Surgery. Hamilton, Ontario & Philadelphia, PA. Decker
Intellectual Properties Inc. ISSN 2368-2744. STAT!Ref Online Electronic Medical
Library. http://online.statref.com/Document.aspx?fxId=61&docId=930. 10/17/2014
4:39:25 AM CDT (UTC -05:00).

Feedback on your answer


Collapse
Unit 7
23)
0/1
In the setting of trauma, what is the recommended transfusion target goal?
(Select 1)(1pts)
5 g/dL

7 g/dl

9 g/dl

11 g/dL

The answer is B, Banked RBCs have been widely available since World War II, but the
indications for administration continue to be debated.15 Animals with hemorrhagic shock
were shown to have improved survival, with hemoglobin concentration ([Hb])
maintained in the range of 12 to 13 g/dL. However, animal models of isovolemic
hemodilution demonstrated that the optimum [Hb] for maintaining systemic oxygen
delivery (DO2) is 10 g/dL, but in healthy human volunteers, isovolemic hemodilution is
tolerated at concentrations as low as 5 g/dL. In the not so distant past, clinical studies in
critically ill patients concluded that 10 g/dL hemoglobin was optimum for shock
resuscitation, but more recent consensus panels have judged that a lower concentration is
adequate. The American Society of Anesthesiologists recommends maintaining
hemoglobin at greater than 6 g/dL, whereas the NIH recommends a concentration of
greater than 7 g/dL. A PRCT showed that ICU patients randomized to restrictive blood
transfusions (transfuse if [Hb] < 7 g/L and maintain between 7 and 9 g/dL) did as well
and possibly better than patients who were liberally transfused (transfuse if < 10 g/dL and
maintain between 10 and 12 g/dL).80 We and others have reported that transfusion of
PRBCs is a strong risk factor, independent of shock severity, for adverse outcomes (i.e.,
infections, MOF, and deaths) in trauma patients (a patient population most likely to
benefit from PRBC transfusion).81 The observation that transfusion of PRBCs is
associated with adverse outcomes has now been consistently observed across a broad
spectrum of high-risk hospitalized patients.82 The choice of transfusion trigger is
dependent on the clinical scenario. During active shock resuscitation with ongoing
bleeding, maintaining for a higher [Hb] is preferred. Blood is a good volume expander
and will thus limit isotonic crystalloid infusions, and the higher [Hb] provides a margin
of safety and a higher [Hb] level may promote hemostasis.83 Once bleeding is controlled,
the lower transfusion trigger should be used unless there is evidence of high oxygen
extraction (i.e., SaO2 minus mixed venous hemoglobin oxygen saturation < 60%). Read
more
Feedback on your answer
Collapse
24)
0/1
Healthy young patients can usually maintain a normal blood pressure until their blood
loss exceeds what % of their blood volume (or roughly __ L)?
(Select 1)(1pts)
20%/3L

40%/2L

10%/1L

60%/4L

The answer is B, 40%/2L. It is critical to emphasize that the most common causes of
postoperative bleeding, even when it is not massive, are technical: an unligated vessel or
an unrecognized injury is much more likely to be the cause of a falling hematocrit than
either a drug effect or an endogenous hemostatic defect. Furthermore, if an unligated
vessel is treated as though it were an endogenous hemostatic defect (i.e., with
transfusions), the outcome is frequently disastrous. For these reasons, in all cases of
ongoing bleeding, the first consideration must always be to exclude a surgically
correctable cause. Ongoing bleeding may be surprisingly difficult to diagnose. Healthy
young patients can usually maintain a normal blood pressure until their blood loss
exceeds 40% of their blood volume (roughly 2 L). If the bleeding is from a laceration to
an extremity, it will be obvious; however, if the bleeding is occurring internally, there
may be few physiologic signs until the patient is at high risk for death. Even when a
technical cause of bleeding has seemingly been excluded, the possibility should be
reconsidered periodically throughout assessment. Patients who are either unresuscitated
or underresuscitated undergo vasospasm that results in decreased bleeding. As
resuscitation proceeds, the catecholamine-induced vasospasm subsides and bleeding may
recur. Only when the surgeon is confident that a missed injury or unligated vessel is not
the cause of the bleeding should other potential causes be investigated. Read more
Feedback on your answer
Collapse
25)
1/1
Which of the following statements about massive transfusion protocol is true?
(1pts)

Platelets and plasma are only given when the PT/INR or ptt are abnormal

The goal is to transfuse with a 4:1:1 ratio with one unit of plasma and one
of platelets along with each 4 units of red blood cells (RBCs)

The use of fresh whole blood transfusion in war zones were shown to
improve outcomes prompting military surgeons to advocate a different
transfusion ratio
Fresh whole blood is reduces the risk of transmitting transfusion-related
infections

Answer C is the only true statement. It is true that coagulopathy may promote massive
hemorrhage and that massive hemorrhage causes a coagulopathy. As a rule, the massively
hemorrhaging patient has a coagulopathy out of proportion to the abnormalities reflected
with in vitro laboratory tests such as the prothrombin time (PT, commonly expressed as
an international normalized ratio [INR]) or activated partial thromboplastin time (aPTT).
Coagulopathy most typically occurs when there is massive hemorrhage in concert with
injury to a large volume of tissue. Although the nature of this coagulopathy is not
precisely defined, it probably involves a number of factors: (1) platelet dysfunction and
diminished availability of platelets at the interface between the vascular endothelium and
luminal flow; (2) dysfunctional formation of thrombi in the setting of acidosis and
hypothermia; (3) accelerated thrombolysis (attributable to dysfunctional thrombosis);
and, eventually, (4) consumption of clotting factors.
Increased recognition of this coagulopathy has prompted enthusiasm for administration of
plasma and platelets early in the treatment of the patient with massive hemorrhage. This
practice, frequently incorporated into a massive transfusion protocol, is described as
"1:1:1 transfusion" because the goal is to transfuse one unit of plasma and one of platelets
along with each unit of red blood cells (RBCs) such that the ratio stays balanced
throughout the resuscitation phase. (Adjustment of these ratios is necessary for centers
that use pooled quantities of plasma and/or platelets.) The rationale is that patients are
already extensively coagulopathic by the time they manifest thrombocytopenia or
prolongation of the INR or aPTT. Thus, during the first several hours during and after
massive hemorrhage, plasma and platelets are necessary independent of the laboratory
tests and help address a significant contributing factor for persistent hemorrhage. The
concept of liberal, early use of plasma and platelets developed in large part from the
recent US-led military campaigns in Iraq and Afghanistan. Initially in those conflicts, the
lack of a reliable supply of blood products near the scene of injury—and platelets
especially—led to the use of fresh whole blood transfusion. Although fresh whole blood
would be impractical in the civilian setting because of logistical issues and the risk of
transmitting transfusion-related infections, the perception of improved outcomes
associated with its use prompted military surgeons to advocate 1:1:1 transfusion.
Proponents of this approach argue that it restores hemostatic capacity and helps limit
hemorrhage, leading to improved survival. Indeed, observational studies suggest that
mortality is reduced.3 However, the ratios examined in these studies typically have been
calculated retrospectively over relatively long (e.g., 12- or 24-hour) time periods after
hemorrhage. Critics emphasize that patients who do not survive long enough to receive
early plasma and platelets bias such observational comparisons (i.e., they are classified as
receiving a large ratio of RBCs to plasma or platelets because they died early) and that
plasma and platelets carry theoretical harms of transfusion-related acute lung injury and
bacterial infection, respectively. Despite some uncertainty about the actual merits of the
1:1:1 approach, given that military conflicts have frequently spurred advances in
transfusion medicine, it appears likely that the approach will be adopted more widely if
the availability of blood products does not limit its application. In the absence of
sufficient availability of blood products to allow a 1:1:1 approach, aggressive correction
of abnormalities of the INR, aPTT, and platelet count are warranted for patients with
massive hemorrhage. read more

Unit 1
1)
0/1
A 45-year old female presents with acute onset of epigastric pain after eating at a family
picnic. She notes the pain has fluctuated with intensity since it started. Since the pain
started she has vomited twice. Her vitals show an oral temperature of 99.1F, heart rate
110 beats/min, respirations 18/min, blood pressure 141/91, O2 sat on room air is
99%. Physical exam shows voluntary guarding in her right upper quadrant and
epigastrum. What is the next best step?
(Select 1)( 1pts extra credit)

CT scan to look for perforation

Abdominal Ultrasound to evaluate her gall bladder

Admit for an upper endoscopy

Give her a GI cocktail for her gastritis

None of the above

The answer is B, This patient most likely has gall bladder pain which can present as
either RUQ or sometimes with epigastric pain. An US should be your first step in
imaging followed by a HIDA scan if your think its still her GB despite a normal scan.
You still could treat her in the ED for gastritis with a GI cocktail but the US needs to be
done in this case. Obtaining a CT scan might show gallstones but it is not as sensitive and
it is difficult to evaluate her common bile duct which is very important in working up
these patients.
Feedback on your answer
Collapse
2)
0/1
A 45-year old male presents to you with recurrent chest pain over the last month as well
as globus and indigestion. He has a past medical history of hypertension and type II
diabetes. His stress test was normal and barium swallow is shown below. What is the
most likely diagnosis?
(Select 1)( 1pts extra credit)

Achalasia

Esophageal stricture

Diffuse esophageal spasm

Hiatal hernia

The answer is C, Diffuse esophageal spasm. His barium-swallow examination shows


marked corkscrew appearance of the distal esophagus, consistent with a diagnosis of
diffuse esophageal spasm. Read more on this topic
Feedback on your answer
Collapse
3)
0/1
A 45-year old male while male smoker presents with worsening epigastric pain over the
last day and radiates to his back. He says that this type of pain has been present for a few
months but comes and goes, is worse at night, and also about an hour after he eats.
However yesterday's episode did not subside and has been more persistent and intense.
His vitals show a heart rate of 128 beats/min, blood pressure 178/94, respiratory rate
of 20/min, oral temperature of 100.1 F.
On physical exam he appears in moderate distress and actively vomiting. His abdomen is
diffusely tender to palpation with involuntary guarding along with hypoactive bowel
sounds in all 4 quadrants. His stool is black and guaiac positive for blood. What is the
next best step in his management?

(Select 1)( 1pts extra credit)

CT scan of his abdomen and pelvis

Immiediate surgical exploration

GI Consult for emergent endoscopy

Upright chest xray

The answer is D. Upright chest x-ray. This patient most likely has a perforated duodenal
ulcer and has free air in his peritoneum. This pain is sometimes described as gnawing,
initially better with food and radiates to the back and worse at night. He has since
perforated and the best test initially is to look for free air under his diaphragm on xray.
He will eventually need surgery and you might have to order CT scan if the xrays are
normal but this is after plain films are obtained. It is also good to know that these ulcers
can cause erosion of the gastroduodenal artery and cause a patient to bleed becoming
hypotensive and very ill quickly. Perforations are most likely in elderly patients on
chronic NSAID therapy. The perforation site usually involves the anterior wall of the
duodenum (60%), antrum (20%), or lesser-curvature gastric ulcers (20%).
Perforation of peptic ulcer remains largely a clinical diagnosis. Initial symptoms include
severe abdominal pain, worse in the epigastrium, often accompanied by nausea and
vomiting. Patients typically present in distress with an acute abdomen. A chest x-ray that
demonstrates free intraperitoneal air [see Figure 3] is all that is needed before the
patient is taken to the operating room; the decision to proceed to operation should not be
delayed by waiting for further images (e.g., computed tomographic [CT] scans). Surgical
delay is a critical determinant of survival in perforated peptic ulcer; every hour of delay
from presentation to surgery is associated with an adjusted 2.4% decreased probability of
survival compared with the previous hour.23
When there is no free air on the plain film and diagnostic uncertainty persists, an UGI
series using water-soluble contrast material may be helpful. Perforation is a
contraindication for endoscopy because air insufflation may disrupt a sealed perforation
and increase spillage of gastrointestinal contents.

In well-selected cases, when there is evidence of contained perforation without free air in
a hemodynamically normal patient without peritonitis, nonoperative management with
nasogastric drainage, intravenous PPIs, antibiotics, and serial abdominal examinations
can be attempted. However, the decision to manage patients without surgery should
undergo frequent reevaluations as failure of nonsurgical management is an indication for
surgery.
PUD has an overall point prevalence in the US population of 1.8%, with a lifetime
incidence of 10%.1 The presence ofHelicobacter pylori increases the incidence of
ulcer to about 1% per year, a rate that is six- to 10-fold higher than for noninfected
subjects.
Most patients with PUD present with mild pain, burning discomfort, tenderness in the
epigastrium, or nausea. Classically, the pain from a duodenal ulcer occurs 2 to 3 hours
after a meal, is relieved temporarily with food or antacids, and at times awakens patients
at night between about 11 pm and 2 am, when the circadian stimulation of acid secretion
is maximal. Gastric ulcer pain is often aggravated by meals. However, the classic
symptoms described are neither sensitive nor specific and are present in only a small
number of patients. Furthermore, the nature of presenting symptoms alone does not
differentiate between benign ulcerations and gastric malignancy.

Advances in the medical management of PUD, including the use of effective acid-
suppressing medications (e.g., histamine receptor antagonists and proton pump inhibitors
[PPIs]) and the treatment of H. pylori, have led to a decrease in the incidence of PUD,
the rates of hospitalization, and mortality. Despite these trends, PUD remains a
significant cause of morbidity and health care costs, with estimates of annual
expenditures (excluding medication costs) of $5.65 billion in the United States.
The majority of gastric and duodenal ulcers are caused by three factors: H.
pylori infection, nonsteroidal antiinflammatory drug (NSAID) use, and acid
hypersecretory states (e.g., Zollinger-Ellison syndrome [ZES]). Most patients with
gastroduodenal ulcers will be colonized with H. pylori, and recurrence of ulcers is
common without treating the H. pylori infection. H. pylori, a gram-negative spiral
organism, is present in about half the world population, particularly in developing
countries, where up to 20% of healthy volunteers will demonstrate infection. Infection of
gastric epithelium is followed by gastric inflammation, and the relationship between H.
pylori and ulcer formation has been widely demonstrated.5 The presence of ulcers in just
a small fraction of individuals with infection suggests the action of other etiologic factors
causing ulceration.

NSAIDs cause injury via suppression of prostaglandin synthesis, which leads to impaired
mucosal defense. NSAID use can result in a range of lesions, from superficial erosions to
deeper ulcerations. Mucosal injury caused by NSAIDs is more common in the stomach
than the duodenum. Cigarette smoking may impair ulcer healing and may increase the
risk of H. pylori-related ulceration.
2014. Scientific American Surgery. Hamilton, Ontario & Philadelphia, PA. Decker
Intellectual Properties Inc. ISSN 2368-2744. STAT!Ref Online Electronic Medical
Library. http://online.statref.com/Document.aspx?fxId=61&docId=900. 10/20/2014
7:58:06 PM CDT (UTC -05:00).

Feedback on your answer


Collapse
4)
0/1
A 30-year old male presents to you with difficulty swallowing both solids and liquids
over the 3-months and has is now experiencing regurgitation 1-2 hours after meals that
appears to be undigested. You order a barium swallow which shows a dilated esophagus
with marked narrowing at the lower sphincter (see x-ray below). What is the most likely
diagnosis?

(Select 1)( 1pts extra credit)

Esophageal Web

Achalasia
Diffuse Esophageal Spasm

Lower Esophageal (Schatzki) Ring

This is a classic presentation of achalasia which is best described as impaired peristalsis


and relaxation of the lower sphincter related to an abnormality in the myenteric plexus. It
is most common in the 3-4th decade of life and treated with dilation of the lower
esophageal sphincter, botulism injection and myotomy if all other treatment options fail.
Short term symptom relief can be from sublingual nitroglycerin before meals and calcium
channel blockers. Read more

Feedback on your answer


Collapse
5)
0/1
What best describes the risk rate of receiving blood with HCV or HIV?

( 1pts extra credit)

< 1:10,000

< 1:100,000

< 1:500,000

< 1:1,000,000

< 1:10,000,000

< 1:100,000,000

The answer is < 1:1,000,000. Physicians should be aware that the FDA mandates that all
blood components are tested for hepatitis B virus (HBV), hepatitis C virus (HCV), HIV
types 1 and 2, human T-lymphotropic virus Types 1 and 2, syphilis, Trypanosoma cruzi,
and West Nile virus. Blood is not currently tested for other transfusion-transmissible
agents (such as babesiosis, malaria, and variant Creutzfeldt-Jakob [prion] disease),
because of extremely low rates of infection and, in some cases, donor screening and
exclusion based on travel to endemic areas.105 In the case of transfusion-transmissible
viruses (eg, HBV, HCV, HIV, etc.), testing is done via sensitive amplification of viral
genetic material (nucleic acid testing), which helps to eliminate the “window” period in
which donors may be infected but not yet antibody positive.106 With the addition of
nucleic acid testing to donor screening and serologic testing, the risk of receiving HCV or
HIV positive blood in the United States is now significantly < 1:1,000,000 units
transfused. Read more
Transmission of human immunodeficiency virus (HIV) through transfusion of
contaminated blood components was documented in the United States in 1982 (1). Since
then, the risk for transfusion-transmitted HIV infection has been almost eliminated by the
use of questionnaires to exclude donors at higher risk for HIV infection and the use of
highly sensitive laboratory screening tests to identify infected blood donations. The risk
for acquiring HIV infection through blood transfusion today is estimated conservatively
to be one in 1.5 million, based on 2007--2008 data (2). This report describes the first U.S.
case of transfusion-transmitted HIV infection reported to CDC since 2002 (3). A blood
center in Missouri discovered that blood components from a donation in November 2008
tested positive for HIV infection. A lookback investigation determined that this donor
had last donated in June 2008, at which time he incorrectly reported no HIV risk factors
and his donation tested negative for the presence of HIV. One of the two recipients of
blood components from this donation, a patient undergoing kidney transplantation, was
found to be HIV infected, and an investigation determined that the patient's infection was
acquired from the donor's blood products. Even though such transmissions are rare,
health-care providers should consider the possibility of transfusion-transmitted HIV in
HIV-infected transfusion recipients with no other risk factors. Read more
Feedback on your answer
Collapse
Unit 2
6)
0/1
Abdominal pain is traditionally divided into 3 categories. What are the 2 basic causes of
abdominal pain of the 3 categories?
(1pts)

Visceral and Autonomic

Visceral and Parietal

Parietal and Somatic

Referred and Somatic

Abdominal pain is traditionally divided into 3 categories: visceral, parietal and


referred. In general visceral (autonomic) and parietal (somatic) are the 2 basic causes of
abdominal pain. Referred pain can be considered separately as a cortical misperception of
either visceral or parietal afferent stumuli. The two are often not discrete as visceral pain
often blends with parietal pain as a pathological process evolves. Tintinalli's Emergency
Medicine: A Comprehensive Study Guide, 7th edition, Tintinalli et al. Read more
on Acute Abdominal Pain Power Point PDF
Feedback on your answer
Collapse
7)
0/1
You are seeing a 32-year old male who presents after being kicked multiple times in his
right flank. He is complaining of severe flank pain and gross hematuria. What is the gold
standard for imaging of renal trauma?
(Select 1)(1pts)

MRI abdomen/pelvis

Ultrasound

CT abdomen/pelvis with contrast

Intravenous pyelography

None of the above

The answer is C, CT abdomen/pelvis with contrast. CT scanning with intravenous


contrast enhancement is the modality of choice for the identification and staging of renal
trauma in the hemodynamically stable patient (image 6). The initial CT images will
frequently miss injuries to the renal pelvis and ureters as sufficient contrast may not yet
be present in the collecting system. Additional delayed images are needed to assess for
contrast extravasation when these injuries are suspected (image 7). In patients
undergoing CT scanning of other organ systems, suspected bladder injuries may be
investigated with CT cystography after retrograde filling of the bladder, as described
above. Read more
 CT with IV contrast is the Gold standard, high sensitivity
 Immediate and delayed post-contrast images to view collecting
system
 Images abdomen and retroperitoneum
 Allows diagnosis and staging
 Intravenous pyelography doesnt allow you to image the abdomen or
retroperitoneum.

Feedback on your answer


Collapse
8)
1/1
You are seeing a 28-year old white male who was brought in by EMS after he was
assaulted while at a night club. On exam his GCS is 7, he is breathing spontaneously and
localizes pain. He has a large scalp laceration to his right parietal region. He is rushed to
the CT scanner and the image is shown below. What is the diagnosis?
(Select 1)(1pts)

Subdural Hematoma

Epidural Hematoma

Intraparenchymal Hematoma with extension

Subarachnoid Hemorrhage

The answer is B. Epidural hematoma (EDH) is a traumatic accumulation of blood


between the inner table of the skull and the stripped-off dural membrane. The inciting
event often is a focused blow to the head, such as that produced by a hammer or baseball
bat. In 85-95% of patients, this type of trauma results in an overlying fracture of the skull.
Blood vessels in close proximity to the fracture are the sources of the hemorrhage in the
formation of an epidural hematoma. Because the underlying brain has usually been
minimally injured, prognosis is excellent if treated aggressively. Outcome from surgical
decompression and repair is related directly to patient's preoperative neurologic
condition. more from Read more
Feedback on your answer
Collapse
Unit 3
9)
0/1
Which of the following is NOT associated with an increased risk of esophageal
adenocarcinoma?
(Select 1)(1pts)

Cigarette smoking

Gastroesophageal reflux disease

Obesity

Helibactor pylori infection

All of the above

None of the above

Answer D is not correct. Esophageal cancer has two main subtypes — esophageal
squamous-cell carcinoma and esophageal adenocarcinoma; their precursor lesions are
esophageal squamous dysplasia and Barrett’s esophagus, respectively. Although
squamous-cell carcinoma accounts for about 90% of cases of esophageal cancer
worldwide, the incidence of and mortality rates associated with esophageal
adenocarcinoma are rising and have surpassed those of esophageal squamous-cell
carcinoma in several regions in North America and Europe. In the United States, more
than 18,000 new cases of esophageal cancer and more than 15,000 deaths from
esophageal cancer were expected in 2014. Esophageal carcinoma is rare in young people
and increases in incidence with age, peaking in the seventh and eighth decades of life.
The main risk factors for esophageal adenocarcinoma are gastroesophageal reflux
disease, obesity, and cigarette smoking; H. pyloriinfection is associated
with a reduced risk. Cigarette smoking and alcohol consumption constitute the
main risk factors for esophageal squamous-cell carcinoma. High intake of red meats, fats,
and processed foods is associated with an increased risk of both types of esophageal
cancer, whereas high intake of fiber, fresh fruit, and vegetables is associated with a lower
risk.
N Engl J Med 2014; 371:2499-2509December 25, 2014DOI:
10.1056/NEJMra1314530

Feedback on your answer


Collapse
10)
0/1
A 22-year old male presents with severe epigastric pain that radiates into his chest after
vomiting over a dozen times following a night of heavy drinking. Vital signs show an
oraltemperature of 100.7 F, blood pressure 147/97, heart rate 127beats/min, respirations
24/min. Physical exam shows crepitus in the suprasternal notch, clear lung exam
bilaterally, and severe epigastric tenderness on abdominal exam. What is most likely
causing this patients symptoms?
(Select 1)(1pts)

Mallory Weiss tear

Spontaneous Pneumothorax

Perforated Duodenal Ulcer

Esophageal Perforation

The answer is D. This patient has evidence subcutaneous air (crepitus) and has likely
ruptured his esophagus. Effort rupture of the esophagus or Boerhaave's syndrome is a
spontaneous perforation of the esophagus that most commonly results from a sudden
increase in intraesophageal pressure combined with negative intrathoracic pressure
caused by straining or vomiting. It was first described by Dr. Herman Boerhaave, a
physician from Leiden, the Netherlands.

In patients suspected to have an esophageal perforation based upon clinical presentation


and chest radiography, computed tomography (CT) scan or esophagram should be
obtained promptly.
Computed tomography — Computed tomography (CT) scan shows esophageal wall
edema and thickening, extraesophageal air, periesophageal fluid with or without gas
bubbles, mediastinal widening, and air and fluid in the pleural spaces, retroperitoneum or
lesser sac (image 2).
Esophagram — The diagnosis of esophageal perforation can also be confirmed by water-
soluble contrast esophagram (Gastrografin), which reveals the location and extent of
extravasation of contrast material. However, false negative results occur in as many as 10
percent of cases. The sensitivity of contrast studies depends upon the size and location of
the perforation, and the technique used for the study.
Although barium is superior in demonstrating small perforations, it causes an
inflammatory response in mediastinal or pleural cavities and is therefore not used as the
primary diagnostic study [13]. If, however, the water-soluble study is negative, a barium
study should be performed for better definition (image 3).
Upper endoscopy — The role of upper endoscopy in the diagnosis of spontaneous
perforation remains controversial. Both the endoscope and insufflation of air can extend
the perforation and introduce air into the mediastinum [14]. However, in the appropriate
setting, often in the operating room with a patient prepared for surgical intervention,
endoscopy performed by a skilled endoscopist can be a useful tool for diagnosis and
treatment planning.
TREATMENT — Boerhaave's syndrome is rare and there is little evidence to guide
treatment. As a general rule, treatment depends upon the size and location of the
perforation, whether it is a contained perforation in the mediastinum or between the
mediastinum and visceral lung pleura, how rapidly it is diagnosed, whether the esophagus
has underlying disease, and the patient’s underlying comorbidities (algorithm 1).
Surgery is generally required for thoracic perforations [15], while cervical perforations
can often be managed without surgery.More from UptoDate
Feedback on your answer
Collapse
11)
0/1
You are seeing a 44-year-old female in your outpatient GI clinic for persistent acid reflux
symptoms. She was on zantac for 4 weeks following her last EGD and then switched to
protonix daily for 2 months which did not seem to help. Her records indicate a normal
upper endoscopy 4 months ago as she was complaining of some mild dysphagia and
biopsies were negative for h. pylori. She otherwise has no other past medical history and
takes no other medications, denies alcohol ingestion and is not a smoker. Vital signs and
physical examination are normal. Which of the following studies should be done next to
confirm this patient's most likely diagnosis?
(Select 1)(1pts)

Repeat H. pylori testing

Barium swallow radiography

Esophageal manometry

Ambulatory esophageal pH-impedance monitoring

The answer is Ambulatory esophageal pH-impedance monitoring, This patient most


likely has gastroesophageal reflux disease (GERD), and the gold standard for diagnosing
GERD is 24-hour esophageal pH-impedance monitoring, which establishes whether
increased esophageal exposure to acid is present. This study is most helpful in patients
who have GERD-like symptoms that fail to respond to an adequate trial of a proton pump
inhibitor (PPI) to correlate the presence of acid in the esophagus at the time the patient is
having symptoms, such as the patient described. Esophageal pH monitoring identifies the
reflux of acid, and impedance monitoring detects reflux of other gastric contents in a
small percentage of patients who have symptoms related to non-acid reflux.
GERD is definitively diagnosed by symptoms, upper endoscopy, or ambulatory
esophageal pH/impedance monitoring. However, the presence of heartburn, regurgitation,
or both is sufficient to presumptively diagnose GERD if symptoms are of sufficient
severity and frequency to be troublesome for the patient. A favorable response to empiric
therapy with PPIs is supportive evidence for a diagnosis of GERD and is a reasonable
first step in a patient without alarm symptoms (dysphagia, anemia, vomiting, or weight
loss). Kumar AR, Katz PO. Functional esophageal disorders: a review of diagnosis and
management. Expert Rev Gastroenterol Hepatol. 2013;7:453-61. PMID: 23899284

Feedback on your answer


Collapse
Unit 4
12)
0/1
You are seeing a 45-year old male in clinic who has a past medical history of
osteoarthritis for which he takes daily alleve. He denies tobacco use or alcohol use. He
has been having recent early satiety, bloating and epigastric discomfort over the last
month. You suspect peptic ulcer disease. Which of the following ulcer locations is more
associated with H. Pylori infection?
(Select 1)(1pts)

Duodenal

Gastric

Esophageal

Jejunal

None of the above

The answer is A, duodenal ulcers. H. pylori, a gram-negative, helical, rod-


shaped bacterium, colonizes the gastric mucosa of approximately one-half of the world
population and an estimated 30% to 40% of the U.S. population. H. pylori is present in
95% of patients with duodenal ulcers and in 70% of those with gastric ulcers. It is
typically transmitted via the fecal-oral route during early childhood and persists for
decades. The bacterium is a known cause of gastric and duodenal ulcers and is a risk
factor for mucosa-associated lymphoid tissue (MALT) lymphoma and gastric
adenocarcinoma. Am Fam Physician. 2015 Feb 15;91(4):236-242.

Feedback on your answer


Collapse
13)
0/1
Which of the following statements about H. pylori is not true?
(Select 1)(1pts)

The serologic determination of anti-H. pylori IgG antibodies in peripheral


blood has high sensitivity (90 to 100%)

Urea breath testing has a sensitivity of 88 to 95% and a specificity of 95


to 100% for establishing the presence of H. pylori infection1

Anti-H. pylori IgG antibodies is the test of choice for confirming successful
eradication of the organism 4 to 6 weeks after the cessation of antibiotic
treatment

Histologic examination of tissue samples remains the gold standard for


determining the presence of H. pylori

Routine culture of the gastric biopsy specimens for H. pylori is not


recommended

Answer C is the only false statement. Urea breath testing is the test of choice for
confirming successful eradication of the organism 4 to 6 weeks after the cessation of
antibiotic treatment. Routine laboratory tests are of limited value in evaluating patients
with uncomplicated PUD. When ZES is suspected, a fasting serum gastrin level may be
useful. However, laboratory tests do play an important role in the diagnosis of H. pylori
infection. The serologic determination of anti-H. pylori IgG antibodies in peripheral
blood has high sensitivity (90 to 100%) but variable specificity (76 to 96%). Serology is
preferred for initial diagnosis when endoscopy is not required. Urea breath testing has a
sensitivity of 88 to 95% and a specificity of 95 to 100% for establishing the presence of
H. pylori infection12 and is the test of choice for confirming successful eradication of the
organism 4 to 6 weeks after the cessation of antibiotic treatment. Although endoscopy is
not indicated solely for the purpose of establishing the presence of H. pylori, gastric
biopsy specimens can be used in the diagnosis of H. pylori by rapid urease test, histology,
and bacterial culture [see Figure 2]. Rapid urease testing of the biopsy specimen is the
preferred method when histopathology is not required because it is fast (results within
hours), is less expensive, and has a sensitivity of 90 to 95% and a specificity of 95 to
100%. Histologic examination of tissue samples remains the gold standard for
determining the presence of H. pylori and can also provide additional information
regarding the presence of gastritis, intestinal metaplasia, or mucosa-associated lymphoid
tissue. Figure 1. Upper gastrointestinal series in which double contrast (barium and air) is
used, showing rounded collection of barium in an ulcer in the duodenal bulb of a patient
presenting with dyspepsia (uncomplicated duodenal ulcer). Routine culture of the gastric
biopsy specimens for H. pylori is not recommended as it is relatively expensive, and
diagnosis requires more time (3 to 5 days). However, the incidence of antimicrobial
resistance is high in patients with refractory peptic ulcers, and they may benefit from
culture and antibiotic sensitivity testing. Read more
Feedback on your answer
Collapse
14)
0/1
What % of benign appearing gastric ulcers are found to be harbor malignancy?
(Select 1)(1pts)

<1%

5-11%

15-21%

24-30%

The answer is B, 5-11%. Given the low predictive value of the signs and symptoms of
PUD, the diagnosis of uncomplicated peptic ulcers is difficult to make solely on a clinical
basis. The diagnosis of PUD is typically made by endoscopy and upper gastrointestinal
(UGI) radiography. Endoscopy is the most accurate method of establishing the diagnosis
of peptic ulcers, with a reported sensitivity of 92% and specificity of 100%.7 In addition
to identifying the ulcer and its features, location, and size, endoscopy provides an
opportunity for biopsies to test for H. pylori and exclude malignancy and for therapeutic
interventions for bleeding ulcers. Duodenal ulcers are most often benign and do not
require routine biopsy. Multiple biopsies are indicated for all gastric ulcers as even
lesions with a benign appearance harbor malignancy in 5 to 11% of cases. Read more
Feedback on your answer
Collapse
15)
0/1
You are seeing a 44-year old female who presents with worsening epigastric pain
following meals and "heartburn" that has been getting more persistent over the last few
days. She has a past medical history of hypertension. Her only medication is naprosyn
500 mg BID and HCTZ 25mg daily. On exam she has mild epigastric tenderness to
palpation. Her rectal exam shows her stool is brown but guaiac positive for blood. Her
portable upright chest xray shows no signs of free air or perforation.Based on her history
of current NSAID use, where would you likely expect the location of her ulcer?

(Select 1)(1pts)

Gastric
Duodenal

Jejunal

Esophageal

NSAID use confers around a 40 fold increase in the development of gastric ulcers and an
8 fold increase risk for the development of duodenol ulcers and . Therefore in this patient
you would expect gastric ulcer more likely than a duodenal ulcer.
The use of NSAIDs is the most commonly identified risk factor for peptic ulcer bleeding,
especially in the elderly. Studies have found relative risks for bleeding ranging from 2.7
to 33.9 [11]. Studies have also shown that the risk is drug-specific and dose-dependent.
As an example, in a study of 2777 patients, the overall relative risk (RR) of bleeding
associated with NSAID use was 5.3 (95% CI 4.5-6.2). However, the risk varied by drug
and was lowest for aceclofenac (RR 3.1, 95% CI 2.3-4.2) and was highest for ketorolac
(RR 14.4, 95% CI 5.2-39.9) [18]. The risk was higher in patients taking high-dose
NSAIDs compared with those taking medium- or low-dose NSAIDs (RR 6.8, 95% CI
5.3-8.8 versus 4.0, 95% CI 3.2-5.0). There was also an increased risk of bleeding with
aspirin use (RR 5.3) that again was dose-dependent (RR 7.5 with 500 mg per day versus
2.7 with 100 mg per day). The concurrent use of aspirin and NSAIDs conferred an even
greater risk of bleeding than was seen with either agent alone (RR 12.7). Finally, the risk
was highest in the first 30 days of NSAID use, with a RR of 7.6 (95% CI 6.0-9.5). The
risk remained high between days 31 and 90 days (RR 7.3, 95% CI 4.0-13.2), but dropped
after 91 days (RR 2.6, 95% CI 1.6-4.1).
NSAID use has also been identified as a risk factor for ulcer perforation [16,19]. In a
study of 176 patients from Spain, NSAID use was the only risk factor that was
significantly associated with perforation (odds ratio [OR] 3.6). Read more
Feedback on your answer
Collapse
16)
1/1
You are seeing a 46-year old female smoker in the ER who presents tonight with acute
onset of epigastric pain, nausea and vomiting. She past medical history except for
right hip arthritis and only takes Alleve daily.
Her vitals show a blood pressure of 145/75, heart rate of 95/min, respiratory rate
of 14/min, O2 saturation of room air of 99%, oral temperature 98.7 F
Her physical exam shows mild upper abdominal tenderness that is greatest in the
epigsatric area along with no voluntary guarding in her upper abdomen. Upright chest
xray reveals free air under the right hemidiaphragm. True or False: Based on these
findings, surgery is always indicated for definitive treatment.
(1pts)

True
False Correct

This statement is False. Ulcer perforation should be suspected in patients who suddenly
develop severe, diffuse abdominal pain. Perforations complicate 2 to 10 percent of peptic
ulcers [70]. Duodenal, antral, and gastric body ulcers account for 60, 20, and 20 percent
of perforations due to peptic ulcer disease (PUD), respectively. Also there is never an
ALWAYS in medicine and with perforated peptic ulcers, a large majority of them are
found to be sealed upon upper GI study with soluble contrast. If this is found then these
patients can be observed safely as long as peritonitis does not develop.
A major decision when treating patients with ulcer perforation is whether and when to
operate. After resuscitation, emergent operation and closure with a piece of omentum is
the standard of care for patients with an acute perforation and a rigid abdomen with free
intraperitoneal air. If the patient is stable or improving, especially if spontaneous sealing
of the perforation has been demonstrated, nonoperative management with close
monitoring is a reasonable option. With any free perforation, regardless of the presence
or size of the leak, if the patient's status is deteriorating, urgent surgery is indicated.
Prolonged efforts to establish a diagnosis or pursue non operative care despite worsening
status can be counterproductive, since a needed operation will be delayed. In addition,
surgery is indicated in circumstances where the cause of an acute abdomen has not been
established or the patient's status cannot be closely monitored.Read More from up to
date

Feedback on your answer


Collapse
Unit 5
17)
1/1
You are seeing a 27-year old male in the trauma bay shortly after being stabbed in this
back with a 6 cm buck knife by his disgruntled neighbor after an argument.
He arrived via EMS sitting up, awake and alert with a GCS of 15. His BP is 140/90, HR
120 and O2 sat is 85% on a non re-breather.
On physical exam there is a 3 cm non bleeding wound just left of his mid thoracic spine.
Upon auscultation there is decreased breath sounds on his left side. There are no signs of
tracheal deviation or JVD.
What is the next best step in management?
(1pts)
Immediately to the OR for surgical exploration

Upright Chest Xray followed by a chest tube

Immediate Needle thoracotomy on the left

Pericardiocentesis under US guidance

The answer is A, Upright Chest Xray followed by a chest tube. This patient is awake and
talking. He is stable enough for a chest xray followed by chest tube placement. A needle
thoracotomy to should be done if he has evidence of tension pneumothorax which would
be respiratory distress and hypotension. He will likely then need a chest tube and can be
further managed. Read More
Feedback on your answer
Collapse
18)
0/1
What is the most common abdominal organ injured in the setting of blunt abdominal
trauma?
(Select 1)(1pts)

Spleen

Liver

Kidneys

Small Intestines

The liver is the most commonly injured solid organ in blunt trauma, comprising 5% of all
trauma admissions, and because of its size is frequently involved in penetrating trauma.
Following blunt trauma, the most commonly injured structures are the parenchyma and
hepatic veins. Blunt forces dissipate along segments of the liver and along the fibrous
coverings of the portal triad structures; the hepatic veins, however, are not so insulated.
Given its size and location within the abdomen, the liver is also commonly involved in
penetrating trauma. Stab wounds typically result in direct linear tears, whereas gunshot
wounds or shotgun wounds result in significant cavitary injuries attributable to blast
effect and the "tumbling" of the missile within the liver parenchyma. Thus, arterial injury
is more common with penetrating trauma.
Over the past 20 years, nonoperative management (NOM) of liver injuries has evolved to
become the prevailing therapeutic strategy for blunt hepatic trauma. Several concurrent
changes resulted in this paradigm change. First was the realization that diagnostic
peritoneal lavage (DPL) was sensitive but not specific for identifying intraperitoneal
hemorrhage that necessitated operative management. Surgeons recognized that many
laparotomies undertaken for a positive DPL were associated with liver injuries that did
not require intervention for bleeding.1Second, trauma surgeons noted that nonbleeding
hepatic venous injuries, if manipulated at laparotomy, often resulted in more hemorrhage
and sometimes even death.2 Furthermore, it became conspicuous that with hemostasis
achieved in the operating room, recurrent postoperative bleeding was rare. Therefore,
surgeons queried whether hepatic venous injuries, which are low-pressure system
injuries, could heal without intervention. Finally, computed tomography (CT) provided a
reliable method for diagnosing and grading liver injuries.

The spleen is the second most commonly injured abdominal organ in blunt trauma
patients. Historical studies have reported a 10% mortality with all splenic injuries;
however, isolated splenic injury mortality is less than 1%. The mechanisms of injury are
similar to those seen with liver injuries: motor vehicle collisions, autopedestrian
accidents, and falls. Similar to penetrating trauma to the liver, stab wounds to the spleen
typically result in direct linear tears, whereas gunshot wounds result in significant
cavitary injuries.

Until the 1970s, splenectomy was considered mandatory for all splenic injuries.
Recognition of the immune function of the spleen refocused efforts on splenic salvage in
the 1980s.38,39Following success in pediatric patients, NOM of splenic injuries was
adopted in the adult population and has become the prevailing strategy for blunt splenic
trauma.

Duodenal and pancreatic injury continues to challenge the trauma surgeon. The relatively
rare occurrence of these injuries, the difficulty in making a timely diagnosis, and high
morbidity and mortality rates justify the anxiety these unforgiving injuries invoke.
Mortality rates for pancreatic trauma range from 9 to 34%, with a mean rate of 19%.
Duodenal injuries are similarly lethal, with mortality rates ranging from 6 to 25%.
Complications following duodenal or pancreatic injuries are alarmingly frequent,
occurring in 30 to 60% of patients.1-3 Recognized early, the operative treatment of most
duodenal and pancreatic injuries is straightforward, with low morbidity and mortality.

2014. Scientific American Surgery. Hamilton, Ontario & Philadelphia, PA. Decker
Intellectual Properties Inc. ISSN 2368-2744. STAT!Ref Online Electronic Medical
Library. http://online.statref.com/Document.aspx?fxId=61&docId=2329. 10/27/2014
1:39:15 PM CDT (UTC -05:00).

Feedback on your answer


Collapse
19)
0/1
You are seeing a 42-year-old female following in the trauma bay following a high-speed
motor vehicle collision. She was was a restrained passenger and her car flipped over after
she lost control. She had to be cut out of the car and is complaining of shortness of breath
and upper abdominal pain.
She is awake and alert and her GCS is 15. Her pulse is 120/min, respirations are 28/min,
and blood pressure is 80/40 mm Hg. Breath sounds are decreased at the left lung base. An
x-ray of the chest shows opacification of the left lower lung field. You decide to place an
emergent chest tube that yields a small amount of air followed by greenish fluid. Which
of the following is the most appropriate next step in management?
(Select 1)(1pts)

CT scan of the abdomen and pelvis

CT scan of the chest

Thoracoscopy

Laparotomy

The answer is D. This patient suffered a ruptured diaphragm and chest tube placement
appears to have ruptured the bowel as evidence from the greenish fluid that was returned.
She needs an emergent laparotomy to control bleeding and repair her diaphragm. Read
More
Feedback on your answer
Collapse
Unit 6
20)
0/1

Which of the following imaging studies is most sensitive for the diagnosis of acute
cholecystitis?

(1pts)

Ultrasound

HIDA scan

CT scan

MRI

The answer is B, HIDA scan. US is about 88% sensitive where as HIDA Scanning is
slightly better and more senstive but US is still the recommended as the first line
diagnostic imaging study. Cholescintigraphy using 99mTc-hepatic iminodiacetic acid
(generically referred to as a HIDA scan) is indicated if the diagnosis remains uncertain
following ultrasonography. Technetium labeled hepatic iminodiacetic acid (HIDA) is
injected intravenously and is then taken up selectively by hepatocytes and excreted into
bile. If the cystic duct is patent, the tracer will enter the gallbladder, leading to its
visualization without the need for concentration. The HIDA scan is also useful for
demonstrating patency of the common bile duct and ampulla. Normally, visualization of
contrast within the common bile duct, gallbladder, and small bowel occurs within 30 to
60 minutes (image 3). The test is positive if the gallbladder does not visualize. This
occurs because of cystic duct obstruction, usually from edema associated with acute
cholecystitis or an obstructing stone (image 4).
Cholescintigraphy has a sensitivity and specificity for acute cholecystitis of
approximately 90 to 97 percent and 71 to 90 percent, respectively [23,26,28,29].
Cystic duct obstruction with a stone or tumor in the absence of acute cholecystitis can
cause a false positive test. Conditions that can cause false positive results despite a non-
obstructed cystic duct include:
 Severe liver disease, which may lead to abnormal uptake and excretion
of the tracer.
 Fasting patients receiving total parenteral nutrition, in whom the
gallbladder is already maximally full due to prolonged lack of
stimulation.
 Biliary sphincterotomy, which may result in low resistance to bile
flow, leading to preferential excretion of the tracer into the duodenum
without filling of the gallbladder.
 Hyperbilirubinemia, which may be associated with impaired hepatic
clearance of iminodiacetic acid compounds. Newer agents commonly
used in cholescintigraphy (diisopropyl and m-bromotrimethyl
iminodiacetic acid) have generally overcome this limitation.
False negative results are uncommon since most patients with acute cholecystitis have
obstruction of the cystic duct. When they occur, they may be due to incomplete cystic
duct obstruction. Rear more

Feedback on your answer


Collapse
21)
0/1
Which of the following complications of cholelithiasis can present with fever, persistent
tachycardia, hypotension, or jaundice?
(Select 1)(1pts)

Cholecystitis

Cholangitis
Pancreatitis

All of the above

None of the above

The presence of fever, persistent tachycardia, hypotension, or jaundice necessitate a


search for complications of cholelithiasis, including cholecystitis, cholangitis,
pancreatitis, or other systemic causes.
For more on the physical examination of patients with gallstones, read here.
Feedback on your answer
Collapse
22)
0/1
A 46-year obese black female presents to you with 2 days of worsening of right upper
abdominal pain and vomiting. She feels hot and cold chills and says that she sometimes
feels this pain after eating but now its much worse.
Vitals: Temp is 39.0 C, heart rate 125 beats/min, blood pressure 124/90. Physical exam
reveals inspiratory arrest upon palpation of her RUQ to deep palpation.
Her labs show a WBC of 17,000. Her ALT, AST, bilirubin, and lipase are within normal
limits. What is the most likely diagnosis?
(Select 1)(1pts)

Cholelithiasis

Choledocholithiasis

Klatskin Tumor

Acute Cholecystitis

The patient presents with the triad of acute cholecystitis which is fever, vomiting,
jaundice and RUQ pain. This triad has been reported to be present in as high 70% present
of cases to as little as 20% of cases. Acute cholecystitis is the result of cystic duct
obstruction leading to gallbladder edema, mucosal sloughing, and, potentially, ischemia.
Signs and symptoms of acute cholecystitis include right upper quadrant pain, nausea, and
vomiting. Unlike the self-limited pain associated with biliary colic, pain associated with
acute cholecystitis generally lasts more than 6 to 8 hours. In addition to the signs of local
inflammation (pain and tenderness in the right upper abdomen or epigastric area),
patients also exhibit signs of systemic inflammation, including fever, leukocytosis, and
elevated C-reactive protein. Mild elevations in bilirubin and liver enzymes may be
present. A positive Murphy sign on physical examination is suggestive of a diagnosis of
cholecystitis (likelihood ratio of 2.8). Necrosis of the gall-bladder wall can occur due to
prolonged inflammation and decreased gallbladder perfusion. Gallbladder perforation
occurs in 5 to 10% of patients with acute cholecystitis.
Renoylds pentad adds sepsis and altered mental status to the triad and is less common and
can be easily missed. A Klatskin tumor presents with painless jaundice. While she might
have cholelithiasis and choledocholithiasis her symptoms are more indicative of acute
cholecystitis. Some cases of acute cholecystitis have mildly elevated bilirubin but severe
elevation suggests either Choledocholithiasis, cholangitis, or obstruction of the CBD.
2014. Scientific American Surgery. Hamilton, Ontario & Philadelphia, PA. Decker
Intellectual Properties Inc. ISSN 2368-2744. STAT!Ref Online Electronic Medical
Library. http://online.statref.com/Document.aspx?fxId=61&docId=942. 10/20/2014
8:07:24 PM CDT (UTC -05:00).

Feedback on your answer


Collapse
Unit 7
23)
0/1
When geographically and logistically feasible, critically injured patients should be taken
directly to a designated level I trauma center or to a level II trauma center if a level I
trauma center is more than 30 minutes away. The currently available field trauma scores,
however, are not reliable in making this triage decision. Because the potential harm
associated with undertriage is high and could result in death or substantial morbidity and
disability, criteria were initially chosen to err on the side of minimizing undertriage rather
than minimizing overtriage. What are the target levels for undertriage rates within a
trauma system to a level I or II trauma center?
(Select 1)(1pts)

<5%

5-10%

10-15%

15-20%

The answer is A, <5%. When geographically and logistically feasible, critically injured
patients should be taken directly to a designated level I trauma center or to a level II
trauma center if a level I trauma center is more than 30 minutes away. The currently
available field trauma scores, however, are not reliable in making this triage decision.
The development of field triage criteria has paralleled the development of trauma centers.
The decision is based on a practice algorithm called a "decision scheme." The first field
triage decision scheme was published by the American College of Surgeons in 1986, with
subsequent updates in 1990, 1993, 1999, 2006, and 2011.6 The decision scheme has four
steps that the emergency medical service (EMS) provider proceeds through after
measuring vital signs and level of consciousness to decide if the patient should be
transported to a higher-level trauma center [see Table 2]. Because the potential harm
associated with undertriage is high and could result in death or substantial morbidity and
disability, criteria were initially chosen to err on the side of minimizing undertriage rather
than minimizing overtriage. However, overtriage results in an overuse of financial and
human resources, overcrowding of trauma centers, and increased EMS transport times.
Target levels for undertriage rates within a trauma system range from 0 to 5% of patients
requiring level I or level II trauma center care, whereas levels of overtriage vary from 25
to 50%. As field triage continues to evolve on the basis of new research findings,
overtriage rates might be reduced while maintaining low undertriage rates. Read more
Feedback on your answer
Collapse
24)
0/1
In the setting of trauma, what is the recommended transfusion target goal?
(Select 1)(1pts)

5 g/dL

7 g/dl

9 g/dl

11 g/dL

The answer is B, Banked RBCs have been widely available since World War II, but the
indications for administration continue to be debated.15 Animals with hemorrhagic shock
were shown to have improved survival, with hemoglobin concentration ([Hb])
maintained in the range of 12 to 13 g/dL. However, animal models of isovolemic
hemodilution demonstrated that the optimum [Hb] for maintaining systemic oxygen
delivery (DO2) is 10 g/dL, but in healthy human volunteers, isovolemic hemodilution is
tolerated at concentrations as low as 5 g/dL. In the not so distant past, clinical studies in
critically ill patients concluded that 10 g/dL hemoglobin was optimum for shock
resuscitation, but more recent consensus panels have judged that a lower concentration is
adequate. The American Society of Anesthesiologists recommends maintaining
hemoglobin at greater than 6 g/dL, whereas the NIH recommends a concentration of
greater than 7 g/dL. A PRCT showed that ICU patients randomized to restrictive blood
transfusions (transfuse if [Hb] < 7 g/L and maintain between 7 and 9 g/dL) did as well
and possibly better than patients who were liberally transfused (transfuse if < 10 g/dL and
maintain between 10 and 12 g/dL).80 We and others have reported that transfusion of
PRBCs is a strong risk factor, independent of shock severity, for adverse outcomes (i.e.,
infections, MOF, and deaths) in trauma patients (a patient population most likely to
benefit from PRBC transfusion).81 The observation that transfusion of PRBCs is
associated with adverse outcomes has now been consistently observed across a broad
spectrum of high-risk hospitalized patients.82 The choice of transfusion trigger is
dependent on the clinical scenario. During active shock resuscitation with ongoing
bleeding, maintaining for a higher [Hb] is preferred. Blood is a good volume expander
and will thus limit isotonic crystalloid infusions, and the higher [Hb] provides a margin
of safety and a higher [Hb] level may promote hemostasis.83 Once bleeding is controlled,
the lower transfusion trigger should be used unless there is evidence of high oxygen
extraction (i.e., SaO2 minus mixed venous hemoglobin oxygen saturation < 60%). Read
more
Feedback on your answer
Collapse
25)
0/1
In the absence of a major surgical insult or concomitant coagulopathy, what platelet count
is required for normal coagulation?
(1pts)

10,000/μL

20,000/μL

50,000/μL

100,000/μL

The answer is B, Patients with a normal INR and a normal aPTT who exhibit unexpected
bleeding may have impaired platelet activity. Inadequate platelet activity is frequently
manifested as persistent oozing from wound edges or as low-volume bleeding. Such
bleeding is rarely the cause of exsanguinating hemorrhage, although it may be life-
threatening when it occurs in certain locations (e.g., inside the cranium or the
pericardium). Inadequate platelet activity may be attributable to an insufficient number of
platelets, platelet dysfunction, or a platelet inhibitor. In the absence of a major surgical
insult or concomitant coagulopathy, a platelet count of 20,000/μL or higher is usually
adequate for normal coagulation.14,15 There is some disagreement regarding the
threshold for platelet transfusion in the absence of active bleeding. Patients undergoing
procedures in which even small-vessel oozing is potentially life-threatening (e.g.,
craniotomy) should be maintained at a higher platelet count (i.e., > 20,000/μL). Patients
without ongoing bleeding who are not specifically at increased risk for major
complications from low-volume bleeding may be safely watched with platelet counts as
low as 10,000/μL. Read more
Feedback on your answer
Collapse

S-ar putea să vă placă și